You are on page 1of 570

REED'S

\SIC ELECTROTECHNOLOGY
FOR
ENGINEERS

BY

E D M U N D G. R. KRAAL
C E n g . . D F . H . (Hons.), M.I.E.E., M.1.Mar.E.

...K e.-,

. f -

Elecrrlcal Engmeering ond Radio Depvrtment


S;:~eldcMarme and Technical College

-T:-.c

t'irst Edition - 1965


- 1973 (SI u n ~ t s )
Reprinted - I

j'rrt, Edition

PREFACE
F IRST E DITION
This book is intended to cover the basic theoretical work in the
syllabuses for Electrotechnology in Part B of the Department of
Trade and Industry Examinations for Second and First Class
Engineers and also Principles of Electricity and Electrical
Engineering for Marine Engineer Cadets of the Alternative
Training Scheme for Marine Engineers.
It follows a similar pattern a s the previous yoiumes in this series
which has already proved so successful, giving emphasis on first
principles, referring to numerous illustrations. providing worked
examples within the text, and supplying many problems for the
student t o attempt on his own. The subject matter has been treated
in the order and in the manner in which it would be taught at a
college and the book is thuscomplementary to lecture notes taken
at such a college.
The typical examination questions at the end provide the
student with the opportunity of'fin;illy testing himself thoroughly
hcl'orc;~~lcrnplinp
r h c c x ; ~ m ~ n ; ~ t 1:ullyworkctl-or11
ion.
slcp-by-step
solutions are given to cvcry p1.0blc111.t11115 \ ) C I I I ~I ~ I I . I I C I I I ; I I . I ~ , I I S C ful to the engineer at sea without a college tutor a t hand.
The author wishes to acknowledge the assistance given by h ~ s
College colleagues, M r . J . W . Powell for drawing the diagrams
and Mr. T . E. Fox for assisting with the proof reading. Acknowledgement is also made to the Controller of Her Majesty's
Stationery Office for permission to reproduce and use the specimen questions from "Examination of Engineers in the Mercantile Marine" as are made available by the Department of
Trade and Industry.
E. G. R. KRAAL

PREFACE
This revision has been undertaken to meet the requirement of
metrication and the up-dating of the Department of Trade and
Industry examinations. In this connection, an additional chapter
No. 15 has been devoted to the introduction of electronics and
some extra material has been added to.preceding chapters.
As for the first edition, the author has been assisted by his
colleagues Messrs Powell and Fox. Their help is gratefully
acknowledged.
#-

E. G. R. KRAAL

CONTENTS
~I{APTF!W

I---THE

ELECTRIC

CIRC(JIT. EI~ECTRICAI

PAC;I.

'TERMS

Circuit conditions, Ohm's I ~ I w . Scsic:,


and parallcl circui~s.Kirchhull's laws.
Internal resistance of supply source.
Electromotive force and terminal p.d.
or voltage. The series-parallel circuit.
Ammeters and voltmeters. Range extension of ammeters and voltmeters
CHAPTER

1-1

2-THEELECTRIC CIRCUIT (CONTINUED).


ELECTRICAL UNITS

The SI system. Mechanical units of


force, work and energy, power. Electrical units of current, quantity, voltage and resistance. Examples relating
mechanical and elecirical eneigy. Efficiency. Grouping of e l l s
..
..
CHAPTER

18-36

3- CONDUCTORS AND INSULATORS

Resistance of a conductor,-variation
with dimensions and material. Variation of conductor resistance with temperature. Temperature coefficient of
resistance. Resistance of an insulator.
--variation with dimensions and
material. Variation 01' insulation resistance with temperature. Resistance
of a semi-conductor,-variation with
temperature. Heat and electrical
energy. Relations between mechanical
and heat energy. Relation between
..
electrical and heat energy . .

C H A PT E R

37-57
LELECTROCHEMISTRY

Electrolysis. Electrolytic cells. Voltammeters (water,and copper). Quantitative laws of electrolysis ( ~ a r a d a ~ ' ~ ) .
The electro-chemical equivalent, chemical equivalent, valency and atomic
weight. Back e.m.f. of electrolysis.
Resistance of electrolytes. Power expended during electrolysis. Primary
and secondary cells. The simple voltaic
cell,--cell e.m.f. Electromotive series.
Polarisation. The primary cell,-

CHAPTER

L C o n t inued
Daniell (energy and e.m.f.). The Leclanche cell (wet and dry types). The
secondary cell,--capacity and efficiency. Charging procedure . .
..

CHAPTER

%MAGNETISM.

PAGE

58-86

ELECTROMAGNETISM

Natural and artificial magnets. The


magnetic field.-flux and flux-density .
Molecular theory of magnetism Electromagnetism. Fields due to long,
straight, current-carrying conductor,
loop and solen~id,-introduction of
itn iron core. Force on a currentcarrylrtg conductor in a magnetic field,
units of ampere, flux-density and flux.
The magnetic circuit, magnetising
force or magnetic field strength. Magnetising force of a current-carrying
conductor. Permeability of free space

(Po>. . .
CHAPTER

..

..

..

..

87-110

~ E L E C T R O M A G N E ~INDUCTION
C

Flux-linkages. Faraday's and Lenz's


laws of electromagnetic induction.
Static induction,+.m.f. of self and
mutual induction. Dynamic induction,
-magnitude of e.m.f. The weber.
Direction of induced e.m.f.-Fleming's right-hand rule. The simple
magneto-dynamo. The simple d.c.
generator, commutation, and practical
requirements,-windings. A.C. and
d.c. theory,-introduction . .
. . 111-137
CHAPTER

CHAPTER

7- BASICA.C. THEORY

The a.c. waveform. Representati'on


o f sinusoidal alternating quantities,trigonometrical and phasor represent:ttion. Addition ilnd subtraction of i l l ternating quantities,-graphical and
m:~thcm;~tic;~l
mcthotls R o o t mc;ln
square and average values. 1;orlil
factor. Peak factor . .
..
. . 138--158
8-THE D.C. GENERATOR
D.C. machine construction,-field system and armature. D.C. armature
winding :\rr:lngcmcnts. The d.c. gcnerator,-4.rn.f. equation,
no-load

CHAPTER

('lIAPT1:R

characteristics. Associated rnuynctic


PAGE
circuit effects. Generator characteristics. Types of d.c, generator,-perIllilYldIlt mayscl ul~dscpurutcly-cxcitcd
types. The shunt-connected generator,
-theory of sclf-excitation, 7'he magnetisation curve or O.C.C. and crhical
resistance. Load characteristic. The
series-connected generator, self-excitation and load characteristic. The
compound-connected generator.Types
of connection. Load characteristic . . 158-1 9 1
9-THEA.C. CIRCUIT (CONTINUED)
Impedance, inductance, inductive reactance. Circuits with pure resistance,
pure inductance and resistance and inductance in series,-power factor,-true and apparent power. C a p ~ i t a n c e ,
capacitive reactance. Circuits with pure
capacitance, and resistance and capacitance in series. The series circuit,inductive impedances in series and inductive and capacitive impedances in
series. The general series circuit,resonance
..
..
..
. . 192-222
10 TFIE D.C. MOTOR

I>ircctinn of f;>rc.c, -l'lclil~rig's Icft-

hand rule. Magnitude of force. Back


e.m.f. of a motor. Voltage, current
and speed equations. Specd controlling factors. Types of d.c. motor,shunt, series and compound. The
power and torque equations. Torque
controlling factors. Motor characteristics. The shunt motor,--electrical
characteristics (speed and torque),
mechanical characteristic. The series
motor, - electrical
characteristics
(speed and torque), mechanical characteristic. The compound motor,-xlectrical characteristics (speed and
torque), mechanical characteristics.
Cumulative and differential connection of fields,--strength of shunt and
series fields. Motor starters. Speed
control,-field and voltage control . . 2 2 L 2 4 6

CHAPTER

I I A I - I I I(

1 ~ A . C . C ' I K C ~ I T S ( C O N T I N L E D ) A N I ~ S Y S T E M S PAGE
Power in the a.c. circuit. Act~veand
reactive components. The parallel circuit. Inductive impedances in parallel.
Inductive and capacitive impedances
in parallel. Parallel resonance. Powerfactor improvement, advantages of p.f.
improvement. k W , k V A and kVAr.
Power-factor improvement ( kV A
method). Polyphase working,-threephase systems. Star or Y connectiqn,
-use of the neutral. Balanced and unbalanced loads. Delta or A (mesh) connection. Three-phase power Threephase k V A , k W and k VAr . .
. . 247-275
I2

I,I-I;(,I ROMAC~NI~I.ISM
(CONTINLJI~)

Permeability of free space 01,). Magnetising force due to a long, straight,


current-carrying conductor, inside a
solenoid and inside a toroid. Ferromagnetism. Relative permeability 01,).
The B-H or magnetisation curve. Absolute permeability 01). Relilctance
(S). The composite magnetic circuit,series and parallel arrangement. Magnetic fringing and leakage. Iron iosses,
-the hysteresis loop. hysteresis and
eddy-current losses. Pull of an electremagnet
..
..
..
. . 276- 300
CHAPTER

1 3-THE ELECTRON THEORY, BASIC


ELECTRONICS AND ELECTROSTATICS

Constitution of matter. The structure


of the atom. Current flow as electron
movement, ionisation. Electric field.
The electroscope. Potential difference.
Electrostatic
charging,-induction.
Distribution 01' charge. Electrostatic
fields o f force. Electrostatic flux. ElecI ri(, lv)lrt\!i:~I 'l'lir (,:I p ; ~ ( . i t o r ( ' ; I ~ ; I ( . I
tor jystcn1s.- \crle\ ; ~ n dpar:~llcl corlncction. c;\p:~citor currcnr Encrg!
stored in an electr~cfield or dielectric
Relative and absolute permittivity
(s, and s). Permittivity of free space (e,,).
<';~p:~cit;~ncc
ol'pnr:~llcl-plnlcc:~pacito~-\ O 1

173

C H A P T ER

I LMISCELLANE~US
C IR C U I T C O N D IT IO N S
AND METHODS 01- SOLUTION. SPECIAL
APPLICATIONS

D.C. rrctwork s. Applic~tionuor Kirclihoffs laws. Maxwell's circulating current theorem. Thc super-position of
current theorem. Conductance, susceptance and admittance,-- the series and
parallel circuit. The Wheatstone
bridge. Measurement of resistance.
Temperature measurement. The potentiometer. The thermocouple. Temperature measurement, compensating
..
leads or cables, instrumentation
C H A PT E R

1 5--ELECTRONI(.S
Thermionic devices. Electron emission.
The vacuum diode. static characteristic, dynamic characteristic,-load line,
diode as a rectifier. Ionisation. Discharge lamps, The fluorescent lamp
(low-pressure). The fluorescent lamp
(high-pressure). Cold-cathode lamps.
The cathode-ray oscilloscope. The
cathode-ray tube (C.R.T.), operation,
focussing, deflection. Time-base. Solidstate devices. Semiconductors. Basic
tllcory, co-v;~lcrit13onding. ~ ' o n d i ~ c t i o t ~

control, in11insic conductivity, impurity (extrinsic) conductivity, N-type


germanium. P-type germanium, ionisation. The P-N junction. The junction
diode, forward bias, reverse bias,
diode characteristic. Rectifier operation, static and dynamic operation . .
SOLUTIONS TO PRACTICE EXAMPLES . .
SECOND CLASS EXAMINATION QUESTIONS
SOLUTIONS TO S1:COND CLASS QUESTIONS
F I R S T C L A SS E X A MI N A TI O N Q U E S T IO N S
SOLUTIONS TO FIRST CLASS QUESTIONS
INDEX
..
..
~.
..
..

THE S.I. SYSTEM


PREFIXES, SYMBOLS, MULTIPLES
A N D SUB-MULTIPLES
PREFIX

SYMBOL

tetra
T
gigs
G
mega
M
kilo
k
milli
m
micro
C1
nano
n
pica
P
Examples 1 megawatt (MW)

UNITS MULTIPLYING FACTOR

x 1012
x 109
x lo6
lo3
x
x 10- "
=

1 kilovolt (kV)
=
1 milliampere (mA) =

1 microfarad (pF) =

I x lo3 kilowatts (kW)


1 x lo6 watts (W)
1 x
1 x
1 x

lo3 volts (V)

ampere (A)
farad (F)

PHYSICAL, QUANTITIES (ELECTRICAL),


SYMBOLS A N D UNITS
The table has been compiled from recommendations in B.S.
1991 and the List of symbols and Abbreviations issued by the
I.E.E.
QUANTITY

SYMBOL

Force
Work
or
Energy
Torque
Power
'1-~nlc
Angular

vcloclt >

Speed

newton
joule
W
or
newton metre
T
newton metre
P
watt
I
jccond
to (omega) radians per

N
J
Nm
Nm
W
S

radis

wcond

N
n

lilectric chnrgc
Potential difference
(p.d.)

UNIT

ABBREVIATION OF
UNIT AFTER
NUMERICAL
VALUE

Q
C'

rcvolut~onsper
minute
revolutions per
second

coulomb
volt

revls

QUANTITY

SYMBOL

UNIT

ABBREVIATION OF
UNIT A R F R

NUMERICAL.
VALUE

volt

p (rho)

awpre
ohm
ohm metre

A
R (omega)
Rm

G
F

siemens
ampere-tun)

Magnetic field
strength

Magnetic flux
Magnetic flux
density
Reluctance

@ (phi)

ampere-turn
per metre or
ampere per
metre
weber
*
tesla

Electromotive form E
(e.m.f.)
Current
Resistance
Resistivity (specific
resistance)
Conductance
Magnetomotive force

(m.m.0

I
R

B
S

ampere-turn or
ampere per
weber
henry per metre

p, (mu)
Absolute permeaLility of free space
Absolute permeability
P
henry per metre
Relative
Pk
permeability
Self inductance
L
henry
henry
Mutual inductance M
Reactance
X
o h
Z
ohm
Impedance
f
hertz
Frequency
C
farad
Capacitance
Absolute permittivity 6,
farad per metre
of free space
(epsilon)
Absolute permittivity E
farad per metre
Relative permittivity E ,
(dielectric constant,
specific inductive
capacity)
Electric field strength, I
volt per metre
electric force
Electric flux
P
' (psi)
coulomb

Atlm

Electric flux density, D


electric displacement
Active power
P
Reactive power
Q
Apparent power
Phase difference
Power factor (p.f.)

4 (phi)

coulomb per
square metre
watt
volt ampere
reactive
volt ampere
degree

cos 4

C/m Z

W
VAr
VA
G

CHAPTER 1

THE ELECTRIC CIRCUIT


ELECTRICAL TERMS
A circuit can be defined as the path taken by an electric
current. Such a current will flow through a circuit if (i) a source

of electrical energy such as a battery or generator is connected


and (ii) the circuit is continuous or conducting throughout its
whole length. The diagram (Fig 1) represents a simple circuit in
which current is flowing. It shows the source, from which energy
is transmitted through the medium of the current, the conducting
path or cables along which the current flows and the 'load'. The
load is the point at which the energy is required to be released o r
work is to be done through the agency of the current flowing.
CABLE

-CURRENT-+
I

LO*.

4-CURRtNT

CABLE

Fig 1 '
The conditions of Fig 1 are better represented by a circuit
diagram as in Fig 2, which illustrates the energy source as a
chemical cell, the conducting path as the leads or wires and the
load as a heating element. A switch is shown as a pivoted link
which, when opened, interrupts the corUinuity of the circuit and
thus stops current flowing.
SWITCH

I-'-\

HEATING
ELEMENT

CELL

-AIROW
S H O W S D I REC TI O N
Of C U R R E N T F L O W W H E N

S W I T C H I S CLOSED

Fig 2

Consideration of the simple circuit introduces more fundamental terms and the practical units as used in electrical
engineering. Flow of electricity o r current is the result of a
pressure built u p within the energy source which manifests itself,
at the circuit connecting-points o r terminals, as a pressure
difference. One terminal, called the positive, is considered to be
at a higher pressure o r potential than the other terminal, called
the negative. A potential cli8erenc.e (p.d.) is said to exist between
these terminals. The direction of current is from the positive
( + ve) terminal through the circuit external to the energy source,
back to the ncgativc (- vc) tcrmin;~land thence through the
source to the +ve terminal. Thus for the load, current is from
+ve to - ve terminal, but for the energy source in the form of a
cell, battery or generator, current is I'rom the -vc to the +vc
terminal.
The electrical pressure generated by the energy source is
termed its electromotive force (e.rn.f.).The symbol used is E, and
e.m.f. is measured as a voltage. The unit is the Volt, which will be
defined later, but any voltage value can be represented by the
letter V attached to the numerical value. Thus a vo!tage of two
hundred and twenty volts would be written as 220V. For reasons
which will be explained when the mathematics of the circuit is
considered. the whole generated e.m.f. of 3 cell, battery o r
generator does not appear a t the terminals, when current is
flowing. The p.d. across the terminals is also measured in tcrms
of the potential o r voltage dropped round the external circuit.
The symbol used for the terminal p.d. is V and it is measured as
a voltage. ie it? volts.
CIRCUIT LAWS
1. For any circuit, current strength is found to be proportional
its- ends. Current strength- is
to the voltage appfied-%iosS
d ~ Y i h - e ~ S j r i i i 7 i a n measured
d~s
in Ainperes. The ampere
will be defined later by consideration of the electromagnetic
effect of current Row, but uny currcnt valuc can bc rcprcscntcd
by the letter A appended to the numerical value. Thus 200A
mcllnri two hundred nrnpcrcs.
Any electrical circuit is found to oll'er opposition to lhe Ilow
of curmit. This opposition is termed the rc,.vistrrrtc~t1
of the circuit
and is denoted by the symbol R. The priictical unit of resistance
is the Ohm, but any value is rcprcsentcd by thc Grcck Icttcr
cirpit;rl fJ(omega) appended to the numeric;il viill~e.Thus 10000
rncans one thousand ohms. The ohni can ht. dcfinetl in terms of
~ I I C volt

i111cl

i1111l~r'c
I~IUS:

il 1csi51ot

I\.\\

CI

L;II\IC' 01'

O I I C 011111

THE ELECTRIC CIRCUIT: ELECTRICAL TERMS

resistance, if a current of one ampere passes through it when a


potential difference of one volt is applied across I& ends. An
alternative definition appears in Chapter 2.
2. The current in any circuit, for a constant voltage, is found to
vary inversely with the resistance; for instance, the greater the
resistance, the smaller the current and vice versa.
OHM'S L AW

The relationships set out above, are summarised by the first


law of the electrical circbit, which is called Ohm's law and can
be expressed thus:-the current in a circuit is directly proportional to the voltage and inversely proportional to the
resistance. This can be written
Voltage
Current =
Resistance
V (volts)
or I (amperes) = R (ohms)
V
Other forms are V = IR or R = I
When using the Ohm's law formula it is essential to pay due
regard to the correct magnitudes of the units used. Reference
should be made to the appropriate table of conversions.
Example 1. A voltage of 6V is applied across a 300R resistor.
Find the current which will flow.
1'
/ = - =

= 20mA.
= 0.02A or 20 x
R
300
Example 2. A current of lOmA passes through a 60kR
resistor. Find the voltage drop across the ends of the resistor.
V = IR = (10 x
x (60 x lo3) = 600V.
SERIES A N D PARALLEL CIRCUITS

Study of the electrical circuit shows that in its simplest form it


may be built up as (i) a series circuit, (ii) a parallel circuit. For
converilence, reiis'lance 'is consihereh 10 b e concentrated ' ~ na
resistor, as for Fig 2, or in more than one resistor; while the
con~lecting leads are assumed to have negligible resistance,
unless a definite resistance value for these is stated. Similarly the
cell, battery o r generator can be assumed to have no resistance
unless otherwise stated.
The diagram (Fig 3) shows a Series Circuit. It will be seen that
only one current path is provided and that the same current
passes through all the resistors. The current is thus common for
such a circuit but the applied potential is dropped progressively
as the current flows along the circuit.

APPLIED P D

VVOLTS

I AMPS

Fig 3

The diagram (Fig 4) shows a Parallel Circuit. Here the main


current is made up of a number of branch currents, but the
;~pplicdp.d. 1s tllc s;\rnc o r common l o r all branches. It will bc
noted that at an), junction point there is no accumulation 01'
current. ir the tot;~lcurrent entering that point is the same as the
lotitI currcrll I C ~ I V I I I K I ~ I Cpotrll. S I I I ~li~ws
~ ~hitscd
C
o n lhc v o l l i ~ ~ c
conditions for the series circuit and the current conditions 1'0s
the parallel circuit have been evolved which allow the solution of
associated problems for these simple circuits, and also for those
of the more complicated series-parallel arrangements and
electrical networks.

-*
Fig 4
KIRCHHOFF'S L A W S
1. VOLTAGE L AW .

The sum of the potential or voltage drops


taken round a circuit must be equal to the applied potential
difference.
Thus for Fig 3 C', + V , + V:, = V

2.

The current flowing away from a junction


circuit must C ~ L I ; I Itllc curscnl flowing into that point.
Thus Ibr Fig 4 I , + I 2 -t l3 = I
'1'11~above I;Iw\ '11 c ~ 1 x 1 1L O C I C C I L I C C \ ~ I I I ~ T I C S ~ I . I I ~ L I1'~01. ~ I1C1 1 ~ '
series and parallel circuits in tcrms of' the equivalent resistances
of the circuits.
THE S ERIES C IR C U I T . For Flg 3. let I amperes be the common
current 'flowing round the clrcult. Then from Ohm's law, the
voltage dropped across resistor- R , is C', volts = I R , . Similarly
the voltage dropptxi :tc.~.orsK , is C', = I/?, and so on. It' R I S
C U RRE N T L A W .

point in

ii

THE ELECTRIC

IKC I I I T . L : F; TKIC A L rL.KMS

taken as the equivalent resistance of the \thole clrcult then as I '


is llic iipplicd volti~gci i ~ i i lI ( \ V I I I
L I I ~ o ~ ~ ~O ~VL ~~ Ic I L - ~ L I I \ , : I ~ C * I I I
resistance, we can write C' = IR.
llsirlg Kircl~hoITsvolt;\gc I;IW tlicn
C ' = C', + I.', 4- C',
or I R = I R , + I R , + I R , = I ( R , + R , + R,)
giving finally R = R , + R, + R ,
THE PARALLEL C I R C U I T . For Fig 4, let V volts be the common
voltage applied t o all the parallel branches and let i t cause a total
main current of I amperes. Voltage V would also cause a current
of I amperes through an equivalent circuit of resistance R ohms.

Thus I = - and using Kirchhoffs current law then


R
V = I l R l or I i

But for branch 1

Similarly for branch 2


Thus I = I,

+ I,

Rl

v
- - and so on
-

R,

+ 1; can

be written

1
R
R,
Rz
R,
N o r t , . 'l'hc reciprocal ol' rrsis~ancci \ I ' ~ ~ c q u c r ~
rcl'crrccl
~ly
I
Conducranc,e. Symbol G = -

1
-1+ . - +

10

:t\

R
The unit is the Sirtncns and the symbol used is S
So for a parallel circuit G = G , 4 G , + G , ere.
Example 3. Three resistors of values 2, 4 and 8 ohms are connected in series across a supply of 42 volts. Find the current
taken from the supply and the voltage dropped across each
resistor.
Here R = R , + R , + R , = 2 t 4 + 8 = 14R
42
So supply current = - = 3A
14
Voltage dropped across 2R resistor = 3 x 2 = 6V
,,
), 4Q
,
= 3 x 4 = 12V
Voltage
Voltage
,,
,
8R
= 3 x 8 = 24V
Check. 6V + 12V 24V = 4 2 i (the applied voltage).
Example 4. The above resistor: are connected In parallel
across the same supply voltage, fit1 the total current and the
current in each branch.
7

X
42
1.1452and I = -- = 36.75A
7
1.14
= 21A
The current in Branch I =
,,
,.
2 = -4: = 1 0 . 5 ~
The
The
,,
. 3 = = 5.25A
Chcck. 21A + 10.5A
5.25A = 36.75A (the total supply
current).

or H

= - =

INTI'KNAI. Kt:SISfAN('li 0 1 : SIJPIJ[.\' SOUK('li

l i p to now the energy supply source has been considered as


h:~vingnepligihlc rcsistancc. I n practicc a cell, hi~tteryor gcncri r ~ o r . , I I O rllallsr IIOW
wcll dcsrgnrcl, 11i1sall internal rcsistirncr
which results in an internal voltage drop when current is being
supplied. Thus the emf generated appears at the supply terminals,
shown as AR in the diagram (Fig 5), only when the circuit switch
is open, ir the cell is on open circuit (O.C.). When current I is
supplied. an internal voltage drop given by /Rioccurs, 'Ribeing
the internal resistance of the cell. The potential difference V
appearing at the energy source terminals is E - IR,.Thus V is
less than the generated e.m.f. E by the p.d. required to drive the
current through the resistance of the cell itself.

.K
Fig 5
I~~L~I:('TKOMOTIVli
FOKCE A N D TERMINAL PD OK VOLTAGE

In Fig 5. R i represents the internal resistance of the cell as


\liown crttcrn;~l l o ~ l i cccll itsclf. Tlii:; is di;lgri~mmiiticonly;

\ornctlmca
r'csistance is not shown, being written 2s a figure
O I ~ I ! hcaidc the ccl! e.ni.f. If given, however, internal resistance
I T I L I ~ (be taken into account. The foregoing paragraph describes
the l'undamental dill'erence between Electromotive Force E, and
Potcntiirl Difference 1'. On open-circuit (O.C.) the terminal p.d.
c>S;~n
energy source eclualsthe e.m.f.generated; but 'on load', ic.
W I I V I I L ' I I I I ' C I I ~ I \ \ I I I ) I ) I I C Y I , 1 1 1 ~~ C I ' I I I I I I ~l >I , ~( l . C C I I I ; I I Y I I I C c,III,I.,

THE ELECTRIC CIRCUIT : 1II.IiC:THIC:AL. TliKMS


-

-- - - ---. - ..-.. -

minus the internal voltage drop. This can bc surnrn:~risetl


mathematically thus:
On Open Circuit (O.C.) V = E
V = '5.- IR,
On Load
We can also deduce that since V = IR where R is the load
resistance
Then IR = E - I R i and E = IR + I R i or E = I(R + R , )
Expressed another way :
On O.C. Cell terminal voltage = E
On Load Cell terminal voltage = E - IRi
= Circuit terminal voltagc
= IR
Problems can be treated as a simple serle:, clrcult, if' E i h wed
as the circuit voltage and Ri is included in the series resistance.
Example 5. A battery of e.1n.f. 42V and internal resistance 7 0
is used to supply the series circuit of Example 3, ie three resistors
of 2 , 4 and 8R in series. Find the current and the terminal voltage
and by how much the cell voltage 'sits down' when supplying
the load,.
Note. I t is appropriate a t this stage to explain that a battery is
an arrangement of more than one cell. The methods of connecting the cells will be discussed in Chapter 2, but for the present a
battery can be considered to be an arrangement of cells in series.
Thus the battery c.m.f. is the sum of the ccll e.m.f.s and the
battcry intcrnal ~.cbistii~lcc
is thc sun1 ol' 111cccll intcrrlal rcbibtances.
4 2 VOLTS + . 7 O H M S
1 AMPS 1
,z+-----4

Fig 6

+ +

External resistance of the circuit = 2


4
8 = 14R
Battery resistance = 7R
Total resistance of circuit = 14 + 7 = 2 1 R
Circuit current =
= 2A
Terminal voltage = 14 x 2 7 28V
Voltage drop in cell = 7 x 2 = 14V
So terminal voltage = 42 - 14 = 28V
Cell voltage 'sits down' 14 volts.
THE S ERIES - PAR ALLEL CI'.CUIT. A S the ritle implies, circu~tsmay
be built up from series end parallel grouping of resistors. The

solution of' associated problems, though not straightforward,


f'ollow\ a logical sequence of operiition based on the methods
u5c.d I'or rllc sltnplc scrics ;rnd parallel arrangement of resistors.
I t cannot be too strongly urged that here is a case of 'practice
maklng perfect' and the reader should work a variety of appropriate problems. A method of solution for any particular problem will thus readily become apparent once its form is recognised
and both tlmc and labour will ultimately be saved. The more
dillicult ne~work problems are solved by reverting to fundament;~]principles such ils the direct application of Kirchhoff s
I;~ws01. using ;r 'I'licorctn or Mctliod ol' SoIu1ion b;~scdon IIic\c
laws. Such problems will be introduced at a very much later
st;~gcin this book. when the render should feel sufficiently
confident to appreciate the fincr points of theory and may cvcn
en.joy the methods and techniques necessary to allow solution.
The diagram (Fig 7a) shows a straightforward series-parallel
circuit. I t consists of a series circuit made up of two sections,
each comprising o f a group of resistors in parallel. Since the
main circuit o r supply current may require to be found together
with the current in 'each resistor, solutiori can only be obtained
by a simplification of the problem. It will be noted thzt the
parallel groups or banks of resistors have been called,sections A
and 3 respectively. The voltage dropped across these sections is
not known and since such knowledge is essential, procedure
would be as follows.

Fig 7a
' I llc crrcurt I:, si~l~plilicd
by linclrng tlic cquivi~lentresistance
\slues R , and R B of the parallel banks from

OnCr. K,, and

K,,h;~vcheen found the total supply current can

hc ohtnined sincc.

:IS

1s shdwn in the diagram (Fig 7b). thc

C L I U I V ~ I I ~ ~i,~rc~rit
I~I
is I I O W c ~ i!Ilr
'
sin~plcseries typc. 'Thus st~pply

'IHE ELECTRIC CIR(.IJI I

current i.; I -

I:L.I:('I~KI(.,'AL rEKMS

and the volt;~gcdrop.; ;+crowg r o t r p A

ftA 1

and B are
C',

I R A and L',,

-I

SUPPLY

IK,,

VOLTAGE

VVOLTS

Fig 7b
Once V A and V B are known, the Individual currents in each
resistor can be found by reverting to the original circuit.
r
V A and I, = vA Also I - etc.
Thus I, = -RI
R2
- R4
I t is not intended that the above method of solution should be
memorised. I t is given to explain the solution of Example 6 and
to illustrate the step-by-step procedure. There is no short-cut for
problems of the series-parallel type. The reader should work
only with the data given and should not make any assumptions.
Methods of solution using proportions for currents or voltages
;~croh\p:~r:~llcl
o r hcricb S C C ~ I O I I Sof' [tic circi~itiirc d i s c o ~ ~ r ; ~ ~ c ( l ~
slnctl seldo~nI Q pructlce art. !I,c resistance ratios s~mple.and only
good can come from adher~ngto and following the straightforward, though mmcrimc\ rnore teciious methods.
Example 6:A circuit IS built up from five resistors. Kesistors of
values 40,6R and 8R are connected in parallel to form a group,
whilst resistors of 3R and 6 0 are connected in parallel to form

rLT-'--j
I

6 OHMS

C--r

6 OHMS

OHMS

another group. The two parallel groups of resistors are connected in series across a 10V supply. Find the voltage dropped
iicross each parallel group, the main supply current and the
current in each resistor.
Let R, be the equivalent resistance of the first group.

Simil:trly Ict R, bc the equivalent resistance of the second


group.
1
2 + 1
3Tticn 1- - 1
0
6
Nu - 3 + 6
For the equivalent series circuit total resistance is R
o r R = RA
. - -t R,- = 1.85 + 2 = 3.85Q
v=10 = 2.6A
Main supply current I = R
3.85.
Voltage drop across RA o r the first parallel group = 1.85 x 2.6
= 4.8V
.,
,,
,, RE o r the second ,,
,,
= 2 x 2.6
= 5.2V
Check. Total supply voltage is (4.8 + 5-2) = 10V.
4-8
Current in BR resistor = - = 1.2A
4

Check. Total current is (1.2 + 0.8

+ 0.6) = 2.6A

Similarly: Current in 3R resistor

,, ,, 6R

,,

5.2
=

3 .
5'2
6

1.73A
0.87A

C ' h r c k . 'I'otal current IS (1.73 + 0.87) = 236A.


Example 7. A battery of e m f . 42V and internal resistance 7R
feeds a circuit consisting of three resistors connected in parallel.
Tlic resistors have values of 2R, 4R and 8R. Find the battery
current, the battery terminal voltage and the current in each

resistor.

-1 -

VOLTS

- - - - - -+
I-

70HMS

2 OHMS

"

I,

74O W

8 0tIHS

Fig 9

The circuit can now be considered to have a total resistance of


S.14R made up from 1.14R and 7R in series.
42
r
The battery current I is given by
= 5.16A
8.!"
The termlnal voltage will be 5.16 x 1.14 = 5.88V
Or ,,
,)
,,42 - ( 7 x 5.16) = 42 - 36.12
= 5.85V
5'88
=
2.94A
Current I, in 2Q resistor 1s -2
99

Current I, in 4 0

,.

Current I, in 8R

,,

'

51.88 = 1 . 4 7 ~
4

5,88 -0.74~

8
Check. Total current I = 5.16A.
"

AMMETERS A N D VOLTMETERS
These are the primary instruments used for electrical work and
the diagram (Fig lo), shows how they are connected into the
circuit. Ammeters are used for measuring current and voltmeters for measuring potential difference o r voltage. Both
instruments operate on the same principle, but ammeters must

Fig 10

be of very low resistance since they are in series with the load
and must be responsible for negligble voltage drop. Voltmeters
on the other hand must be of high resistance, since they may be
connected across points which may be at a high potential
difference. For most circuit purposes, the ammeter is considered
to have negligible resistance and the voltmeter to have infinite
resistance, ie to take n o current.
In Fig 10 a generator is shown as the energy source, S may
be a singk-pole or double-pole switch, as is shown here, and R
is the load resistance. .As ;I practical example, the generator may
li:ivc ;III 111tcr11:)lI . ~ ~ I S ~ ; I I I 01'C C 0 , 0 2 f l , t l ~ c c;~l)lcI C ; I ~ S I I L I )
have a total resistance of 0.03R and K may have a value of'
Sf2 I f ttlc gcllcr;~toris .;ct ((7 320V o n opcn-circuit. icz with
tllc switch open, then when the switcll I S closed a current 01'
220
-- 220 = 43.56A would flow round the circuit.
5 + 0.02 + 0.03 5.05
The terminal voltage of the generator would 'sit down' to
220 - (43.56 x 0.02) volts = 220 - 0.87 = 219.13V. T h s
would be shown by the voltmeter, while the ammeter would
show 43.56A. If the voltmeter w a s disconnected and then connected directly across R it would indicate 219.13 - (43.56 x
0-03) volts = 219.13 - 1.3 = 217.83V. The voltage drop in the
cables would be 1.3V. It will be seen that the example of a simple
lstribution system has been worked as a simple series circuit
and that the instrume~ltsperform their required functions. The
ammeter show2 the series circuit current, whilst the vollmeter
indicates the potential drop across any particular portion of the
circuit. I t also can record the e.m.f. built up by the generator
when the switch is open, since this is the only condition when the
e.m.f. appears at the terminals of the energy source.
RANGE EXTENSION OF AMMETERS A N D VOLTMETERS

For practical work it may not be possible to pass all the circuit
current through the ammeter. I t may be difficult to construct a
suit;iblc instrument bec:~uscof' size or otlicl. limitations, and in
ordcr to introduce a certain amount of standardisation. i t may
I>c caslrr. lo LISC tI1c ~ I I I ~ I I I C I C w
I . i [ l l ;I . V / I I I I I I in O I . C ~ C I .lo I I I C ; I S I I I . C
the circuit current. A shunt 1s a spec~allyconstructed resistor ol'
low ohmic value and, in order to make an ammeter capable of
measuring a current greater than that which can be passed
through it. a parallel arrangement of thc ammeter and the shunt
is used. The ammeter is designed to carry a definite but small
rr:~ctionof tiic main current : ~ n dthe rest of the current is made lo
by-pass the ammeter through thc shunt, which is accurately

made a n d set to :I dcfinitc l-cs~sl;lncc\ ; I I I I ( .


I t 15 C : I I I I ~ ~ : I I C \~4 I7 1 1 1
the ammeter instrument and r~iust;rl~iaghbc used w ~ t hi t Tlit'
calibrated leads between Instrument ;rnd .;ll~rntI'orrn p;irt of tlic
;Irrangcmenl ; ~ n dnluhl not hc cut 01.~ I . I ~ > S ~ I I I .1'01.I ~ C13)~ j71cces 0 1
ordinary copper wlre. T h e diagram (Fig I I ) shows tile normal
Lrrrangernent o f instrument and \bun! and tlie e.\ample hlious
the form of' cnlcul:itlon necessary I t will tx 5ct.n that tlic
calculation follow^ the piittern set I'or parallel rcsl<t;inccC I ~ . ~ L I I I \
SHUNT

Rw

RM IKLUOES RE518 TANCE OF


L LEAD S

I N S T R U M EN T

Fig I I
Example 8 . Calculate the reslstance of' a shunt r e q u ~ r e dto
operate with a milliammeter. ivh~chp i e s full-scale deflection for
a current of 15mA and which has a reslstance of 512. (,Vorc.. 5 R
can be taken to include the resistance of the c o n n e c t ~ n gleads.
since n o specific mcntion of lead I-eb~stanceh;is been made ) TI-ie
cornbin;ltion ot'mctcr ;lnd sI111ntis rcquirrcl 1 0 rc;lcl c l t r r c n l , ~1 1 1 1
to IOOA.

Voltage d r o p across instrument when g i ~ , i n gfull-scale det1c.ction = current causing full-scale deflection x resistance 01'
instrument circuit
= I,
R, = ( 1 5
5 = 75
= 0.075V o r 751nV
Now the voltage d r a p across the instru~nentis tlie hame a s the
voltage d r o p across the shunt o r Is, x R,, = 0.075 volta.
But the shunt current I,, would be 100 - meter current
= 100 - 0.015 = 99.985A

= 0.000 75112
I t is important to note the low resistance u l u e of the shunt
which is designed to carry the cur r ent without 'heating up' The
shunt I S usually mounted on the switchboard, behind the a n i meter and in the main current circuit. Tlie 'light' calibrated le;ldj
are coiled to take u p any 'slack' and then brought out to tllc
instrument. Thus the ammeter may be marked 0-100 arnyxrcs.
but in actual fact only a minute current, some 15mA. passes

through the instrument itself. The remainder and by far the


largest proportion of the current, passes through the shunt. The
reason for always using the instrument with its own calibrated
shunt and leads is thus obvious.
T o measure voltages higher than that for which the instrument
movement is designed a series or range resistor must be used.
This resistor is designed to drop the excess voltage and dissipates
a certain amount of heat; It consists of special fine-gauge wire
wound on a porcelain spool or on a mica card, the whole being
moi~ntedinside a ventilated case. Here again the arrangement
n l a y bc mounted behind the switchboard, if i t cannot be contained in the case of the instrument. Thin leads for carrying the
small instrument current connect the range resistor unit and the
instrument l o ~ h cnuin supply terminals, usually through fuses.
Thus the voltmeter may be scaled 0-250 volts, but in fact only
0.075V may be dropped across it, when full-scale deflection
occurs. By far the major voltage drop occurs across the range
resistor, which is always high in ohmic value: thousands of
ohms. This fact should be noted. The diagram (Fig 12) shows
the arrangement and Example 9 shows how the value of an
appropriate range resistor is calculated.

---j

VOLTAGE TO @E

t7Y2.

Fig 12
Example 9. Calculate the resistance of the range resistor
required to be placed in series with the instrument of Example 8
to make it into a voltmeter reading 0-250V.(The instrument has
a resistance of 5Q and gives full-scale deflection with a current
o f I5mA).

T h e current through the complete voltmeter circuit must be


limited to 15mA, otherwise the instrument will be 'burnt out'.
Thus resistance R of voltmeter circuit must be

the series or range resi\tor R,,


olllnh

muit h a \ ?

;I

\ : ~ l l l cof ( I h hhf,

5J

I0 OOlil

The actual 'movement' or working unlt 01' ;in amrnstcr 01.


voltmeter is much t h e same ;lnd i t is tllc usc 01' :I sliunl 01'rangc
resistor which decides whether current o r voltage can be
measured. Multi-purpose portable test instruments are available
which can make a range of measurements. A range switch is
provided which makes the appropriate connection of shunt or
range resistors.
Consider an instrument movement in which 15mA at a p.d. o f
75
10-~
75mV gives full-scale deflection. Its resistance =
15 x lo-'
= 5R.
If a voltage range &15V is required, the instrument circuit
I5
= lOOOR and a range resistor of
resistance must be
15 x 10,.
1000 - 5 = 9 9 5 a must be swltched in.
Similarly if a voltage range of 0-150V is required the range
150
- 5 = 10000 - 5 = 9995R.
resistor must be
15 x lo-'
If a current range 0-5A is required, a shunt is used whose
value can be obtained thus:
P.D. across shunt = p.d. across instrument movement for
full-scale deflection
0r
= 75111v -- 75 X 10 ' ~011s
The current through the shunt = 5 - 0.015 = 4.98%
and the resistance of the shunt would be

..

CHAPTER 1
PRACTICE EXAMPLES
1.

A circuit is made up from four resistors of value 2R, 4R,


5R and 10R connected in parallel. If the current is 8.6A, find
the voltage drop across the arrangement and the current in
each resistor.

2.

One resistor group consists of 452, 6R and 852 connected


in pi~~.;rllcl
; ~ n dit second group consists of 3R and 6R in
parallel. The two groups are connected in series across a 24V
supply. Calculate (a) the circuit current, (b) the p.d. across
d i t ~ l l~ I . L ) L I I ) , (c) fllc curre111 in cuch resistor.

3.

If the resistor arrangement of Q1 is connected to a 12V


battery of internal resistance 0.6552, find the circuit current
and the battery terminal voltage. Find also, the current in
the 5R resistor.

4.

A moving-coil instrument has a resistance of 10R and


requires a current of l5mA to give full-scale deflection.
Calculate the resistance value of the resistor necessary to
enable it to be used to measure (a) currents .up to 25A, (b)
voltages up to 500V.

5.

Two.+esistors of 60k52 and 40kR value are connected in


series acFoss a 240V supply and a voltmeter having a resistance value of 40kR is connected across the 40kR resistor.
What is the reading on the voltmeter?

6.

When a 10R resistor is connected across a battery, the


current 1s measured to be 0.18A. If similarly tested with a
2 5 0 resistor, the current is measured to be 0.08A. Find the
c.11l.l'.ol' ~ h c
bnttcry and its intcrni~lresistancc. Neglect thc
resistance of the ammeter used to measure the current.

7,

' I wo groups ol' resistors A and B are conncctcd in scrlcs.


Group A consists of four resistors of values 2R, 4R, 6R and
8R connected in parallel and group B consists of two
resistors of values 10Q and 15R in parallel. If the current i n
the 4R resistor is 1,5A, calculate ( a ) the current in each of
tlic rornaininp ~.t.sistors, ( h i !hc. supply voltage, (c) the
v o l t ; ~ ~( Icr o p i ~ c r o s1 hc
~ g r o t ~ p sA i ~ n dR.

I lic \ o l t ; ~ g cof' ; I d c, generator. wl-icrl \ilppl>.~ng


; I current
01' 7 5 t l ( ( 1 ;I l ~ ~ i I ~> dI ~ ,~ C ; I > L I I C [LI
~ I 1~ IOS SL' . I [ I I I C > \ + I I L ' ~ \ board. At the load. the voltage recorded is 10SV ;lnd ~ , h c n

the Ioxi is switched off' the volt;~gerises 10 I I O V . 1:11id~ l i c


internal 'resistance of the generator, the resistance of thk
supply cables and estimate the fault current if a 'shortcircuit' of negligible resistance occurred at the load
terminals.
9.

The ammeter on a swltchboaid. scaled 0-300A is accidentally damaged. The associated shunt is marked 300A.
150mV A small ammeter, scaled 0-1A with a resistance of
0.12R, is available, and the possibility of using this is considered. Find if such an arrangement is possible, and if so,
how it could be achieved using surplus resistors which are
also available.
t

10.

Five resistors AB. BC. C D , D E and EA are connected to


form a closed ring ABCDEA. A supply of 90V is connected
across A D , A being positive. The following is known about
the resistors: AB is IOR, BC is of unknown value R , ohms,
CD is of unknown value R , ohms, DE is 6R and EA is 9 0 .
A high resistance voltmeter (taking negligible current) when
connected across RE reads 34V with H positive and when
c.orl~icc.~cd
:IL.I.O.;~
( ' 1 : rc;~tI\OV w ~ l l i I ! I I O ~ I I I L . 1~, 1 1 1 ~ 1 1 1 1 ~ '
valucs 01' K , 411d K,, thc current in branch ABCD and the

main supply current.

CHAPTER 2

THE ELECTRIC CIRCUIT (CONTINUED):


ELECTRICAL UNITS
All engineering studies stress theneed for units and an introduction to some of these will have been made when the subjects
of mechanics and heat were being covered. Units allow measurements to be taken and calculations to be made. They are essential
to the derivation of formulae from the basics of theory and
enable presentations of related principles to be evolved. Thus
for electrical engineering, even at the Chapter 1 stage, the
ampere, volt and ohm were considered, and although these units
have yet to be defined, their importance in rel~tionto the basic
electric circuit will have been appreciated. The student will also
recognise these units as being amongst those in common
every-day usage. If, however, the impression has been given that
the study of electrotechnology will involve the knowledge of an
entire range of new units, then it is stressed that this is not the
case. The whole modern concept of engineering technology is
based on the universal adoption of SI units and, since some of
these have been encountered in earlier work, it will not be long
before, in this study of electrical engineering, common ground
is being covered and the relevant relationships with associated
units, already treated from the mechanical engineering aspect,
are being stressed.
Before proceeding with any further study of units of the S1
system, it w o u l d a e useful to introduce a historical note and
consider the situation in engineering as it has developed.
Towards the end of the last century two systems of units began
to be employed in engineering; the British or foot-poundsecond (fps) system and a metric or centirnetre-gramme-second
(cgs) system. The British or Imperial system had no merits since
all units of the same kind, such as those of length, area, volume
ther. indeed-there were also
ric and horsepower which were
ntly defined. The metric system
t to industry and commerce but
tages and it was adopted prior
ical circles. In 1873 the British
nt of Science selected the centiunits of !ength and mass for

h;~sc-unit of rime and the tio option of the second. for this
purpose, gave the c e n t i m e t r e - g r a n i ~ ~ ~ c - s e c(cgb)
o ~ ~ dsystrln.
The metric system, in the cgs form, was adopted to a Iiirpt'
cxtcnt li)s electrical cngiuccsing in thc curly dirys 01' dcvclopment. The system had the advantage that all the same kind of
quantities are multiples of ten and it was also international. The
sizes of the absolute unit of the centimetre and the gramme were
found to give rise to difficulties for the desired electrical units
which became either too large or too small for practical working. The use of these absolute units for essential formulae in
engineering work also proved difficult and thus more workable
or practical units had to be devised. Such practical units were to
include the volt, ampere and ohm. In about 1900 practical
measurement in metric units began to be based on the metre.
kilogramme and the second and the aforementioned electrical
practical units.These constituted the unrationalised MKS system.
The next developmeqt came from a fact, which was repeatedly pointed out over the first half of the present century,
that a system of units could be devised to make the practical
units of the volt, ampere and ohm the absolute units of such a
system. If, in addition, suitable adjustments of certain constants
encountered in electromagnetism and electrostatics are accepted,
then a more workable system of units would result. This system
was known as the rationalised MKS system and its adoption
W A S ~.ccommcndcdby tllc Internation:~l L i l ~ ~ t r ~ t ~ Ci)111cl~~~i~i~
mission of 1950. The change to the MKS caused some little
inconvenience to the older electrical engineer and necessitated
the revision of many of the better known works of reference and
text-books. The student was required to appreciate however.
that the new system did not upszt the course of learning in any
way and that, if anything, the 11nits introduced made matters
easier and formulae more manalr,eable.
Prior to 1970, conditions d;3 exist when both the older
Imperial and the newer MKS sydems of units were in use. The
latest extension of.metric units .nto all branches of commerce
and industry has enabled enginiering to evolve the SI system,
the units of which are used throc ghout this book. Thus from the
electrical viewpoint, it can be faid that the SI system is the
rationalised M K S system with :nits in all the other fields of
measurement being fully metrica..cd.
T H E SI S 'STEM
All measurement consists in cc mparison with some standard
or unit. The three fundamental un ts are those of length, of mas.-

and of time. In the SI system the metre is taken as the fundamental unit of length, the kilogramme as the unit of mass and
the second as the unit of time. From the fundamental units, can
be built u p the derived units, which can be further classified as
mechanical or electrical units. Thus Force I S a derived mechanical unit involving a fundamental unit and a derived unit, ie mass
and acceleration. For the SI system, a unit of force, called the
Nekton, has been introduced. Velocity is similarly a derived
unit involving distance a n d - time. So also is acceleration a
derived unit, involving velocity and time. Both velocity and
acceleration are mechanical units. The ampere is really a derived
unit involving force and length but as stated previously it is used
as a fundamental electrical unit. Other electrical units are the
volt i ~ n dt h o ohm which ore nlso dcrivcd units, but thc Joule and
Wutt, although used principally in the past in connection with
electrical engineering, are really derived mechanical units and
will be defined under this heading.
Once the units are recognised and understood, the reader is
advised to discontinue their classification as mechanical o r
electrical units and to accept them as general engineering units.
This applies particularly to the units of work and power. Both
the mechanical and electrical engineering fields are concerned
with common practical appliances or associated problems and a
ready use of the appropriate units, with a correct appraisal of the
magnitudes of the quantities involved, is essential.to the modern
engineer.

MEC HANIC AL* U N IT S


The fundamental units require little definition since they are
accepted standards. Thus the metre is the absolute standard,
taken as the distance between two marks on a certain metal bar.
Similarly the kilogramme is the mass of an accepted standard
lump of'metal. The time unit is the second, which is defined as
I
of a mean solar day.
86 400
Most of the principal SI derived units will already have been
introduced to the student but a revision is made here to allow
an extension into the fieid of' electrical engineering.
UNIT O F FORCE
T H E N EWTON .

This is theforce required to accelerate a mass of'


one kilogramme at the rate of one metre/second 2 . It has been
fbund that the force of gravity acting on a mass of I kg is 9.81
ncwtons ; ~ n d ,since thc force on ;I body due to tlic ciirth's

THE ELECTRIC CIRCUIT

FLFCTKICAI, ONIT?

-- -- - --

- - - --- .

--

21

attraction is termed weight, i t follows that the weight of 1 k ~ l o gramme is 9.81 newtons.
The symbol for force is F but any value in newtons can be
represented by the letter N after the numerical value.
Thus: lkg = 9.81N.
UNIT OF WORK AND ENERGY
T H E J O ULE . This can be defined

as the irork done or energ1


stored when a force of one newton acts through a d~stanceof
one metre in the direction of the force.
The symbol for work or energy is W but any value In joules
can be represented by the latter J after the numerical value.
From the definition, it follows that a force of F newtons.
acting through a distance of s metres, does F x s newton
metres of work o r Fs joules.
Thus: W (joules) = F (newtons) x s (metres)
*

UNIT OF POWER
THE WA TT . Power

is the rate at which work is done or energj is


converted and the unit is the watt. A watt is the power resulting,
when a joule of energy is expended in a second.
The symbol for power o r rate of doing work is P but any value
in watts can be represented by the letter W after the numerical
value.
The definition can be more generally written as

P
Other tbrms are

(wntts) =

I + ' (Joulc5)

r (seconds)

U' = Pr or

CI '

1 = -

The jaule and watt are the units orignally used in electrical
engineering and they will be encountered constantly in electrical
problems. Example 10 is set out here to serve as an introduction
to electrolmechanical relationships.
Example 10. A pump is required to lift 1200 litres of water
through 10 metres in 6 minutes. Calculate the work done in
joules and the power rating of the pump. Assume 1 litre of'
water to have a mass of' 1 kilogramme.
Work done = force of gravity x distance lifted
= (1200 x 9.81) : . 10
= 12 x 9.81 x l C 3 newton metres
= 1 17.72 x 10.' = 11 7 720N1n or 1 17 7103
1 7 720 - 1 I 772 - 327W
Power = work done - -time
6x60
. 36
Power ratlng of' pump would t- 327 wntts

Note. In the above problem no account has been taken of


machine efficiency. This will be introduced in due course, but
for the example, the practical rating figure of the electric motor,
assuming this to be the means of driving the pump, would be
much larger.
ELECTRICAL UNITS
The same fundamental units are used as for the mechanical
.
primary
units namely: the metre, kilogramme and s e ~ o n d The
derived unit is the ampere, which has been adopted as the basic
electrical unit of current and ;IS a I'ourth fundamental unit.
Before considering the definition for the ampere, it is necessary
at this stage, to describe two associated effects, which would be
observed when a current flowed in a circuit.
(1) If the resistance of the circu~twas concentrated in a short
length of conductor, then a temperature rise of the wire in this
region would t
x noted, showing a conversion of energy into heat.
(2) If the circuit was supplied through two wires laid together, then especially if the current is large and the wires
flexible, a mechanical effect would be noted. When the current is
switched on, the wires would be observed to move and this
electromagnetic effict, as it is called, has been used to define the
ampere for the SI system. The factors governing the magnitude
and direction of the force on the wires will be described in the
chapter on Electromagnetism.
UNIT OF CURRENT
THE AMPERE. This

is that current which, when maintained in


each of two infinitely long, straight, parallel conductors situated
in a vacuum and separated by a distance of one metre between
centres, produces on each conductor a force of 2 x lo-' newton
per metre length of conductor.
As stated in Chapter 1, the symbol for current is I and any
value in amperes is represented by the letter A after the nwnerical value. The reader is reminded that practical circuit currents
may rilngc from thousands of amperes to minute values of'
milli-amperes and attention is drawn to the Table of Prefixes of'
Magnitudes as given at the front of this book. Full consideration
must be given to the correct use of the abbreviation which
follows the numerical value.
When a current flows for n given tir~le,a quantity of electricity
is said to be conveyed round the circuit. The quantity which
passes ciin bc sliown to be related to thc work done in the circuit,

THE ELECTRIC CIRCUIT : ELECTRICAL UNITS


- -

- --

--

23

but before this relationship is considered further, it is necessary


to dcfinc qu:~ntityofclcctricity in terms of current : ~ n dtimr
UNIT 0 1 . QUANTIT)'
T HE COULOMB. The

usual unit-sometimes called the ampere


second. For practical purposes a larger unit, for everyday
electrical engineering is used. This is the Ampere hour as used in
connection with the capacity of batteries and for accumulator
charging.
The symbol for quantity of electricity is Q and any value in
coulombs can be represented by the letter C after the numerical
value. Any value in ampere hours is represented by the letters
A h. after the numerical value. Since the quantity ~f electricity
which is conveyed round a circuit would vary with the strength
of the flow of electricity and with time, a simple definition for the
coulomb can be deduced thus:
, A coulomb is the quantity of electricity cdnveyed by a steady
current of one ampere flowing for a time of one second.
Thus Q (coulombs) = I (amperes) x 1 (seconds)
or Q (ampere hours) = I (amperes) x t (hours)
From the above, the following can be deduced:
1 ampere hour = 1 ampere x 1 hour = 1 ampere x 3600
seconds = 3600 ampere seconds = 3600 coulombs. Thus
1A h = 3600C.
Ex:~mplc I I . C'onsidct. I~?c:~mplr
5, w l ~ c ~:I ch:~ttcr.vof' c 1 n . f
42V and ~nternalresistance 752 is used to supply a clrcult of'
three resistors 2, 4 and 8 0 in series. If the current is switched on
for 30 minutes, find the quantity of electricity which would have
been conveyed.
Total resistance of circuit = 7 + 2 + 4 + 8 = 2152
42
Circuit Current = -- = 2A
;1
Quantity of Electricity = current x time in seconds
= 2 x 30 2 60 = 3600C
or Quantity of Electricity = current x time in hours
- 2 x + = 1Ah.
The passage of an electric current results in energy being
expended. This energy may appear as the work done by the
rotation of an electric motor, as the action of heating up a
furnace element o r as the agency responsible for the electrolytic
dissociation of a salt solution. The relation between conveying a
quantity of electricity round a circuit by an applied voltage and
the resulting work done can be used to derive the units of voltage

and resistance in terms of the coulomb and the joule which have
already been defined.
UNIT OF VOLTAGE
THE VOLT. This is

the unit of electromotive force and potential


difference and can be defined as the potential difference required
between two points in a circuit, if one joule of work is to be
done when passing one coulomb of electricity between the points.
As stated in Chapter 1 , the symbol for voltage or e.m.f. is V
and any value in volts is represented by the letter V after the
numerical value. In accord with the remarks made concerning
the representation of current, the reader's attention is drawn to
the Table of Prefixes of Magnitudes, and to the correct use of
the Abbreviations.
From tho dofinition set out tibovc it is stated that the work
done by part of an electrical circuit = quantity of electricity
conveyed x voltage applied across that part of the circuit.
Thus : fl Goules) = Q (coulombs) x V (volts)
or W (joules7 = Z (amperes) x t (seconds) x V (volts).
Other forms are W =

- $

v2t

or W = It(IR) = 1 2 ~ t
Example 12. Consider Example 1 1 . A battery of e.m.f. 42V
and internal resistance 7 0 is used to supply a circuit of three
resistors, 2, 4 and 8 0 in series. If the current is switched on for 30
minutes, find the energy converted (as heat) by each resistor and
inside the battery &self.
Circuit current was found to be 2 amperes
Using form W = 12Rt then energy converted in
2 ohm resistor = 22 x 2 x ' 30 x 60 = 14400 joules
4 ohm resistor = 22 x 4 x 30 x 60 = 28 800joules
8 ohm resistor = 2* x 8 x 30 x 60 = 57 600joules
7 ohm battery = 2' x 7 x 30 x 60 = 50400 joules
Total energy converted by the circuit =
14 400 + 78 800 + 57 600 + 50 400 = 151 200 joules
( ' l / t ~ X The
.
total energy converted by the entire circuit may be
Ii)utltl 1'1 orll Cj' =- I.'// joulcs
= 42 x 2 x 30 x 60 = 151 200 joules.
The definitions of' Power and Energy have already been con\~dered,but i t would be as well to summarise the points of'
Importance, n:rnlely t h a t power is the rate at which work is done

1,'ro111 c i ~ * ( I i ~ ~ - t iqct
o r ~ 0111
~
: I \ N T \ C 11' - 1'11 : I I I ~ I 1 1 l o l l o \ ~ ~
! I \I . I I
P = VI or P (watts) = V (volts) x I (amperes).

The iibove is a most importi~nt reli~tinnship.I t c i ~ n;11so k


expressed in the following f'orms:
P

vZ

12R or P = R
The attention of the reader is drawn to the following which
must also be known.
W
Since P = - or W = Pt itfollows that
t
joules = watts x seconds and that a joule is one watt
second.
Now a joule is a small unit of energy and for practical purposes a
much larger electrical unit of energy is used. This is the kilort,irtr
holrr, abbreviated to k W h and is also know^ as the commerci:il
unit of electricity or more commonly as 'a unit'.
Since one kilowatt hour = one kilowatt x one hour
= lO!N watts x 3600 seconds
So one kilowatt hour = 3 600 000 joules.
Example 13. A 220V electric fire is rated at 2kW. Find the
current taken when the fire is switched on and also how much it
would cost to use the fire for 5 hours with electricity being
~ unit.
charged at 0 . 6 per
=

I OOU

Current taken = L
7
= Y4YA
220
Electricity used = 2 x 5 = 10 kW h = 10 units
Cost = 10 x 0.6 = 6p.
UNIT OF RESISTANCE
THE O H M . This was

defined in Chapter 1 as the unit of' sesislance and in terms of the volt and ampere thus:-a resistor has a
value of one ohm resistance, if one ampere passes through i t
when a potential difference of one volt is applied across ~ t ends
s
Now that the relations between the ampere, volt, joule and watt
have been defined. i t is possible to give a l'ust1ie1-definition t'or
the ohm which is associated with power or energy dissipation
Thus the ohm can be defined as:--that resistance which when
one ampere passes thrclugh it produces power at the rate. of one
watt. Alternatively, the ohm is that resistance in which a current
of one ampere flowing 'or one second generates a joule of energy.
For a resistor the ensrgy produced by current flow appears a
heat. and the followin: is of' importance.
Since P = F I and I/ = I R then P = ( 1 R ) I
or P = I'R as , leveloped earlier.

Power dissipated in a resistor is thus proportional to the current squared. If the current was doubled by raising the voltage,
the power dissipation would be four times as large. The temperature would rise in proportion and assuming the resistor was
capable of carryng its normal current only and had very little
capacity for working at a higher temperature, then a 'burnout'
would occur. The same limitations apply to cables, electrical
machines and switchgear. Electrical equipment is assigned a
rating which, on full load, enables it to operate with a safe
temperature rise. An increase of the normal rated current.
brought about by overloading or by an overvoltage, results in a
temperature rise proportional to the new current squared. The
total tcmpcruturc will rise very rapidly as the overcurrent occurs
and if this is maintained then damage will result. Damage to
electrical insulating materials can occur because of sustained
overloads and overheating should be regarded as the main cause
of failures of electrical machines.
Example 14. A hot-plate of a ship's electric galley is fitted with
a control marked Low, Mediurn and High. The heating element
consists of two equal sections, lvhich are connected in parallel
for High and in series for Low. Only one section is used for
Medium. If the plate when set at high is rated at 2kW on 220V,
find the wattage rating when the control is set at Low and at
Medium.
Current taken at High. Two sections in parallel

Current taken by 1 section

= 4.545A
220
220 = 48.41i2
Resistance of 1 section = 4.545
Resistance of 2 sections in series = 96.82i2
Current taken at Low. Two sections in series
=

Power dissipated = 220 x 2.27 = 499-4W


or wattage rating = 500W approximately.
Current taken at Medium. One section only across 220V
or I = 4.545A
W i ~ t t i ~ grating
c
I S onc h ~ ~ lthat
l ' of the High setting = IkW
or wattage rating = 220 x 4.545 = 1OOO Watts = 1kW.

T H E E L E C TRI C C IR C U IT : ELECTRICAL UNITS

27

EXAMPLES RELATING MECHANICAL A N D


EI,ECTRICAI> ENERGY
The best understanding of the various units as discussed
earlier, is achieved by considering examples where mechanical
work is performed by electrical means o r vice versa. The
relations between the units will need to be introduced a t all
times, but it is necessary before proceeding to such examples, to
stress that no m a c h n e is perfect and that its overall performance
is measured by its efficiency.
EFFICIENCY

The symbol usually used is q-the Greek letter eta. In all


apparatus and machines, losses of energy occur due to bearing
and brush friction, air turbulence, unwanted electrical currents.
escape of heat, etc. These losses result in the output of such
apparatus o r machines, when measured as work, being always
less than the input when this is measured in th'e same work units.
The ratio of the output to the input is termed the efficiency. Thus :

output
input - losses
or =
input
input
output
0r
output + losses
Efliciency is usually expressed as a percentage.
Example 15. A dicscl cnglnc I ~ ; I S ;I mc;~surctl~ntlicatcdpowcr
01' 7.5kW and a mechanical cllicicncy of' ti5 per cent. I t drlvcs a
generator which supplies a lamp load at 110V. How many 60W
lamps can be supplied, if the efficiency of the generator is
measured to be 88 per cent? Find the total load current.
The output of the engine = input x eficiency
Efficiency

3 3

At the coupling betwees engine snd generator, i t can be


assumed that there is no loss of energy, so the power input to the
generator must be the powei output of'the engine and it followx
that:
Output of engine = i n p ~ , tto generator = 6.375kW
Thus generator outpu! = input x efficiency

5610W

Number of lamp.; = 5610


- = 93-5 say 93
60

28

REED'S BASIC ELECTROTECHNOLOG'I'

Load current

93 x 60 - 558
110 - 11
= 50.73A say 51A

or alternatively ;
5610
Load current = - = 51A.
110
Example 16. A pump is required to lift 12 tonnes of' water
through lorn in 2 minutes. Calculate the power required to drive
the pump, the current taken if driven by a 220V motor and the
cost of pumping at I p per unit. Assume the efficiency of the
pump to be 60 per cent and the efficiency of the driving motor is
85 per cent.
Work to be done = Force opposing gravity x distance lifted
Note 1 tonne = 103kg
Thus work to bc done = (12 x lo3 x 9.81) x lonewton
metres
= 117.72 x 10 4 Nm
Also 1 177 200Nm = 1177.2kJ
This is the oiltput of the pump. The input would be greater, ie
100
1 177.2 x - = 1962kJ
60
Since the pumping is to be accomplished in 2 minutes or 120
seconds, the power input during thls time = 1 962 000
120
= 16 350W
Thus power required to drive the pump is 16.35kW
The output power rating of the motor must be 16.35kW and
100
the input power would'be 16.35 x ---- = 19.24kW
85
19 240 - 962
Current taken by motor = ---- - -= 87.45A
220
11
2
19.24 = 0.641kw h
Energy used = 19.24 x - = 60
30
Cost = 0.641 x 1 = 0 . 6 4 ~ .
Example 17. The electric motor used to drive a ship's winch
has an efficiency of 86 per cent. The winch can lift a mass of'0.5
tonnec through ;I distance of 22m in 22 seconds. The winding
gear ol' ~ l winch
~ e has an efliciency o!' 60 per cent. Calculate the
power rating of the motor and also the current taken from the
220V ship's mains.
Work done by the winch = 500 x 9.81 x 22 newton metres
= 107 910Nm or 107 910J
'This 1s tllc output 01' t11c winch or the output of the winding
gcnr. T l i c input to the winding gear woi~ldbe
.L

The input to the winding gear would also be the motor output
= 179 8505

Since the lifting is done In 2 2 seconds, the motor would give


out power during this time.
- 179 850 = 8 1 7 5 ~
22
For a motor output of 8175W. the input power would be

Thus power rating ot motol

Input or motor current

I \q

51 1\U

eo6
43 ? I A
220
=

Example 18. .4 storage batter! 15 providedJ'or emergency ubtr


aboard a ship. The batter!, I S ;irr-,ingt.cIto suppi) certain essent~,ii
services during the period of lime taken to start-up the 'jtand-b!'
generator. The principal load to be supplied by the battery is the
'emergency' motor for an electric-I~ydr:~ulic
steering gear. Thi,
motor is rated a t 220V. 15kLV. and has an etlic~encyof' 88 per
cent. The battery I S to be ol' a capacit) jufticient to oper:tte ti115
motor and an additional lighting load of'twenty 60W lamps f o r ;I
period of 30 mlnutes. Fst~rnatethe size of' the battery and ;11\o
l t h ~llscll~llgc
ctlsrcl~~,
Outpllr of' lnotol = 15hM'
100 187.5
= 17.04SkM'
Input to motor. .= 15 r
YK
11
Input current to motor = -17- 045
= 852
- - 3 = 7 7 5tl
130
11
L ~ g h t ~ nload
g = 60 * 20 = 11WW
- 60 = j 4 j A
Lighting current = ''0
----220
11
Total current = '7 5 + 5 45 = 82 '75A
or D~scli,~rge
curlent = 83 95.A
S ~ z eot b,~trerj= 113 95 x = 41 (17514 h
G R O U P I N G O F CELLS
Ohm's law stales that the current in n circuit can be ~ncreased
by raising the potential difference a ~ p l i e d3c1.c)~~'
tlie C ~ I - C ' U oI ~r
by decreasing the circuit resistance. I!' the supply source is a
generator. the applied p.d. can be vari :d by controlling tiie e.m.f.
being generated in the machine. but i.' a battery is the source ot'
energ! then tiie applied voltage cann 11 be varied eabilk. Since a

battery consists of a group of cells and since the e.m.f. of any cell
is fixed, being decided by its chemical composition, then a larger
e.m.f. or a greater current can only be obtained by appropriate
arrangement of the cells. The cells can be connected in series,
parallel or series-parallel arrangements.
SERIES CONNECTION. For this arrangement the -ve terminal of
a cell is connected to the +ve terminal of the adjacent cell as
shown in the diagram (Fig 13a). The arrangement is more
simply depicted by Fig 13b. .4 battery of 3 cells in series is shown.

-44-F
Fig 13(b)

Fig 13(a)
From Kirchhoff s voltage law, the e.m.f.-of the source is equal
to the sum of the e.m.f.s taken round the circuit and thus for a
battery of n e l l s in series, the e.m.f. = e.m.f. of 1 cell x n. Also
since this is a series circuit, then the current in any 1 cell is the
circuit current. The internal resistances of the cells are also in
series and should be treated in accordance with the deductions
already made for the resistance of a series circuit. These points
are illustrated by the example.
Example 19. A battery consists of 4 cells in series, each of e.m.f.
1.5V and internal resistance 0.6R. Find the current flowing, if
bVoLTs

2.4

onns

JPltlrt---t.

2 OHMS

0.6 OHM

Fig 14

THE ELECTRIC CIKC U I S


.-

31

FLLC'TRICAL UNITS
.-

-- - - -

-- -

thc br~ttcryis conncctcci to two rcsislors o f 2C;1 :~ncl0 , O C l wh~cll


are connected in series. The arrangement is shown by the
diagram (Fig 14).
Total e.m.f. = 4 x 1.5 = 6V
Total battery internal resistance = 4 x 0.6 = 2,452
Total circuit resistance = 2.4 + 2 + 0.6 = 5R
6 = 1.2.A
So circuit current = 5
Other values of' interest would be
Battery terminal voltage = 1.2 x 2.6 = 3.12V
o r Battery ,,
,,
= 6 - (1.2 x 2.4) = 6 - 2.88
= 3.12V
Voltage drop across each resistance = 1.2 x 2 = 2.4V
and 1.2 x 0.6 = 0.72V
Current in 1 cell = circuit current = 1.24.
PARAL LEL CONNECTION. For this arrangement, the t v e
terminals of' all the cells are connected together as are all the - ve
terminals. The arrangement is as shown in the diagram (Fig 15).

Fig 15

,.---'

From Kirchhoff's current law, the i-3tal current 1s the sum of


the currents in each branch. Thus tti\: total current from the
battery is equal to the sum of the curr.:nts available from each
cell. For correct working, the e.m.f. o each cell should be the
same. So also should the internal resist.ince although this is not
essential. If n cells are in parallel, the toial current is n times that
given by one cell, but the battery e.m.1'. is that of any one cell.
This latter point can be reasoned from the fact that li' the r v e
terminal of A is 3V abo1.e its -- ve ter-mi 131and t h c + \ e ter-rninal

of B is 2V above its -ve terminal, then the +ve connection


between A and B is 2V above the -ve connection. If this is
carried on for cell C and any further number of cells then it is
seen that the whole +ve connection is 2V above the -ve connection, ie the battery voltage is 2V.
The battery internal resistance is obtained from the parallel1
resistance formula, ie it is - th of a cell resistance. The battery
n
resistance once determined, is added to the external resistance to
give the total circuit resistance as in the following example.
Example 20. A battery consist of 4 cells in parallel, each of
e.m.f. 1 -5Vand internal resistance 0.6R. Find the current flowing
if connected to a resistance of 2,@2. The arrangement is shown
in the diagram (Fig 16).
1.5 VOLTS
I

OlOHH

Fig 16

,-.
E.m.f, of'battery = e.m.f. of 1 cell = 1.5V
t
0.6 = 0.1 5 0
b ~battery = Internal r e s i ~ t of
4
Total resisf'ahce of circuit = 2.6 + 0.15 = 2.750
1:. 57
Current =
= 0,545A. Other informatien would be
2.75
,-,Terminal voltage = 0.545 x 2.6 = 1.418V
' ~ ~ i ,
,, = 1.5 - (0.545 x 0.15) = 1.5 - 0.082
_ _ i;,, '.i
= 1.418V
0.545
C u ~ . ~ . c n01't 1 cell = --= 0,136A
6

I
-1

I
-1

-I
-I
--

f
-1
-1

I.

S E K IE S - P A R A LLEL CONNECTION . TO build up this arrangement a


number o f cells which are connected in series are then conncctcd
In parallel w ~ t ha similar number of cells in series. The arrange~ l l r ' t l f 1s . I \ ~110'1\\'11
ill tllc' di:~gram(Fig 17) :lnd is used to providc
1>01li I I ~ C I ~ C ; I ~\.011:1gc
C(~
; ~ n dc t ~ r r c n1.01.
~ :I circ\~it.Tlic cells in

7'tIIi IiLEC'I'HIC' (.'IKC'IJI'I' : til l:(."I KI( ' A L C ' W I I S


-.. ---.
-

~~

--

.;cric\ provide the incre;i\c.cl c.1111'


supl'ly file c.\[I.,Ic.urrcnt

33

-~

:1ri(1

thr p;ir.:~llclh : ~ n h \o f ( T I ~ \

Fig 17
The procedure for solving problems fhllows the reasoning
already covered for the series and paralIe1 arrangements. but
should be taken step by step.
Example 21. Ten cells each of internal resistance 3Q and e.m.f.
2V are connected in two banks of 5 cells per bank. They are then
connected to an external load resistance of 2851. Find the load
current and the p.d. across the battery terminals. The arrangement is shown in the diagram (Fig 18a).
TWO

BANKS

5 CELLS
1 VOLTS
3 OHMS
PtR CELL

-L

Fig I 8 ( a )

E . m . f . of a bank = 5 x 2 = 10V = battery e.m.f


Resistance of 1 bank = 5 x 3 = 1552
15
Resistance of battery =
= 7,551

-,

Total c l r c u ~ tresistance = 7 5 + 20 = 27.551


10
Clrcult o r load current = --- = 0,364A
27 5
P.d. o r termlnal voltage = 0,364 x 20 = 7.3YV
0.364
Current per cell = current of 1 bank = ---- = 0.182A
2
Example 21 (continued). I f the battery 1s rearranged with 5
banks of'? cells in each. find the new current and voltage. The
arrangement is shown in the diagram (Fig 18b).
E . m . f .of a bank = 2 x 3 = 4V = battery e.m.f.
Internal resistance o f a 1 bank = 3 x 3 = 6 R

3 CfLLS
3

20

OHMS

OHMS

PLO CfU

Fig 18(b)
6
Internal resistance of battery = -5 = 1.m
Total circuit resistance = 20 + 1.2= 2 1 . m
4
Circuit or load current = -= 0.188A
21.2
Terminal voltrtgc 20 x 0.188 = 3.77V
or 4 - 1.2 x 0.188 = 4 - 0.23 = 3.77V
0.188 = 0.0376A.
Currentlcell = current of 1 bank = ----

THE ELECTRIC CIRCUIT. ELECTRICAL UNITS


--

35

C'lIAPTER 2

PRACTICE EXAMPLES
1.

An electric hoist is required to lif't a load of'2 tonnes to a


height of 30m. The cage has a mass of 0.25 tonnes and the
lifting operation is timed to be completed in 1; minutes. If
the 220V motor is metered to take a current of 50A, find the
efficiency of the installation.

2.

Thirty cells each having an e.m.f. of 2.2V and an internal


resistance of 0.3R are so, connected to give a supply e . n ~ . fol.
22V. If the arrangement is then connected to three 20V, 10W
lamps in parallel, calculate (a) the terminal voltage of the
battery, (b) the current taken by each lamp, (c) the power
wasted in each cell.
C

3.

A pump delivers 12 700 litres of water per hour into a


boiler working at 15 bars. The pump whlch is 82 per cent
efficient is driven by a 220V motor, having an efficiency of
89 per cent. Calculate the current taken by the motor.
Assume 1 litre of water to have a mass of I kg and 1 bar =
1O5Nlm2.

4.

A resistor o f ' 5 f I I \ ~ ~ 1 1 1 r 1 r c 1l0r ( :I


I h : ~ l t c r yIII:I(I(.1 1 1 7 o f ' f o ~
. .
slrn~larccllx In seriex. IXacll cell has a n c.111.f'.ol' ? . ? V irncl llic

current which flows is 1.4A. If the cells were connected In


parallel, find the current which would flow through the 5R
resistor.
5.

A five-tonne cargo winch 1s required to lift a load of 5


tonnes at 36,5m/rnin. Calculate thc. power rating of the 220V
driving motor if the efficiency o f t le winch gearing is 75 per
cent and that of the motor can be taken as 85 per cent.
Calculate also the current take1 from the ship's 220V
mains.

6.

A 220V diesel-driven generator 1s required to supply the


following on full load. (a) Light] ~g load comprising one
hundred lOOW and two hundred 6 ) W lamps. (b) A heating
load of 25kW. (c) Miscellaneous sn all loads taking a current
of 30A. Calculate the required po ver output of the diesel
e n g n e when the generator is supp ying all the loads at the
same time. Assume a generator elf ciency of 85 per cent.

~ ~

7.

A battery is made up from three similar correctly connected dry cells in series. The open-circuit e.m.f. is
measured to be 4.3V. When the battery is connected to an
unknown resistor the current i s metered. to be 0.4A and the
battery terminal voltage as 4.23V. If one of the cells of the
battery is reversed and the circuit made up as before, estimate
the new current value.

8.

A 150W, lOOV lamp is to be connected in series with a


40W, llOV lamp across a 230V supply. The lamps are
required to operate at their rated power values. Determine
the values of suitable resistors to be used wit11 the lamps and
make a sketch showing h o b they would be connected.

9.

A resistor of 0.5250 is connected to the terminals of a


battery consisting of 4 cells, each of e.m.f. 1.46V joined in
parallel. The circuit current is found to be 0.8A. Find the
internal resistance of each cell.

10.

Twelve cells, each of e.m.f. I.5V and internal resistance


0.225R, are arranged four in series per row or bank, u.itli
three banks in parallel. The battery so formed is connected
to a load consisting of a series-parallel resistor arrangerncnt.
made up of a 2R resistor connected in parallel with a 712
resistor, these in turn being connected in series with a 2 . 2 ' 1
resistor. Find the battery terminal voltage, the power
ratings of the resistors and the energy converted into he~itin
the complete circuit if the arrangement is switched o n for
1 hour.

I I A I r I I I< 1

CONDUCTORS A N D INSI'I :ITORS


The reasons as to why certain materials arc good conductors
of' electricity whlle others are not. will be considered in det~iil
later when the electron theory is studied; here i t can be stated
that a substance which freely allows the passage ot' electricity is
classed as a conductor. Examples :ire metals, carbon and certain
liquids4hiefly solution of salts, acids or alkalis. An insulator
can be defined here as a substance which will not allow the free
passage of electricity. Examples are rubber, porcelain, slate,
mica, some organic materials and certain liquids-notably oils.
RESISTANCE O F A CONDUCTOR
\.ARIATION O F C O N D U C T O R R E S I S T A N C E LVITH * D I ~ I ~ N S I O S SA N L )
MATERIAL
The resistance or 'ohmic value of n conductor. \uch a i a c o ~ l

of'wire, can be altered in different ways. Thus if'co~lsof'different


lengths of' the same wire. lr jame material and same crosssectional area. are measured for resistrince. their ohmic ~ a l u e s
would be fbund to varq in d ~ i e c tproportion to their lengths.
Again if coils of wire of the same materia! and length, but o f
tlifycrcnt cro\\-ccc~ior; ;Ire r n c ; ~ \ ~ ~ r ~hcir.
r d . rc\isl,lnc.c L ; I ~ I I C \
would be I'ound to vary In inverse propcrrtlnn to tlie areas of' the
wires with which they are wound.
A similar series of comparative measurements witti coils 01'
wire of the same length and cross-sectional area but different
material, would show that the resistance value varied with the
conductor material.
The elementary tests described above, indicate that the resistance of a conductor or resistor can be altered by varying its
dimensions or the nature of material used, and the relation of'
these factors to the actual conductor resistance will now be
examined in detail.
l(a) DIME N S I O N S . Resistance of' a conductor is proportion,~lto
its length or, more simply and using an example, i t can be said
that the conductor resistance of a lOOm length of cable will be
double that of a 50m length of the same cable. This can be
readily shown as follows:
Let Rq ohms = the resistance of a 50m length. Then two 5Om
sections in series would have a resistance of R ohms
Hence R = RA RA = 2RA

But the length has been doubled


So 2 x Length = 2 x Resistance of original length.
Summarising :
Resistance is proportional to Length or R cc I.
l(b) DIMENSIONS. Resistance of a conductor is inversely proportional to its area or, more simply and using an example, it
can be said that the conductor resistance of a lmm 2 cross-sectional area cable will be twice that of the same length of cable,
of the same conductor material but of 2mm2 o r twice the
cross-sectional area. Thls can also be shown thus:
I.ct the resistance of the l m m 2 cable be RA and suppose an
identical cable to be connected in parallel with it. The resistance
of the combination would be R ohms.
R
0,. j( = A
l lcllcc I - I i I R

KA

RA

Thus the resistance of the combination is the original cable


resistance, but the area of the combination is twice that of the
original cable
or 2 x Area = x Resistance of original length.
If the reasoning was repeated for four lrnrn 2 cables in parallel,
the area would be four times the original and the new resi'stance
would be R =

h.
The resistance of a 4
4

1.0 o r 4mm2 cable

would be 4of the original cable resistance and the area would be
4 times that of the original cable.
From the above, it follows that doubling the area halves the
resistance of' a mnductor of' the same length and material and
quadrupling the area reduces the resistance to a quarter of the
original value. Thus:
1
Resistance is inversely proportional to Area or R a - .
A

2. M A TE RI A L . The resistance of a conductor depends upon the


material from which it is made. Thus the resistance of' a length
of iron wire is approximately 7 times greater than the resistancc of :\ piccc of copper wire of exactly similar dimensions, ir
same length and cross-sectional area. If therefore resistance
v:~ric.;with tlic n;lturc of thc m:ltcrinl, it is necessary to dcfinc
this property in such a way as to allow a comparison ol'resistance
value for accepted standard dimensions of the conductor
material. The term resistivitv or specific resistance (symbol p
the Greek letter rha), is now introduced. This was originally
expressed in ohms or niicrohms per cm 3 . The modem tendency
1s to slxcil'y i t ;IS---ohm-metreor--microhm-millimetre.
' l ' l ~ cI < c h ~ \ t ~ v 01.
~ l ySl>ccilic, I<csisl;~nccof' ;I m;~tcri;~l
is Ihc

CONDUCTORS A N D

INSULA-rOKS

resistance as measured bctwccn tllc o j ~ p o s ~ I';lccs


tc
01' it c u \ 01
~
unit dimensions.
Tllus I'or rllc di;lgr;~nl(Iiiy 19) ;I c t ~ h co l ' c o l ~ ~ w ~ ~ o l I' sIllcrlr.
~ctc~
is taken and the resistance is measured between the f'aces as
shown by the arrows. The resistivity would thus be determined
or 17.25pRmm.
and would be expressed as 1.725 x 10-"m

..

Fig 19
I t should be noted that the temperature of' the material
is measured at the time the test is made and is f'requently
specified with the resistivity figure. Thus p f h r copper is given as
1.725 x l o - ' ohm-metre at 20C. The reasons for specifying the
temperature in connection with the resistivity figure will be
explained in due course.
It has been seen that R c/_ I and R r _ -.1 Therefore R T -I o r
it = k

'

W I I C I ~ Ck

is

Li

consti~nt.I I ' i~b

~ ; I A C I I;ih , 1 1 1 5L

~ ~ I I ~I,, ~ : I I ] ~

then the foregoing can be written as:

The use of p as the constant k is acceptable since resistance


depends upon resistivity and if in the formula, unit length of Im
and unit area of l m 2 is taken, then R = p. ie the definition of'
resistivity is satisfied. It is the resistance of a unit cube measured
across opposite faces. Problems involving resistiv~tyare solved
P 1 but i t is essential to note
by use of the above expression R = -.
A
the units used for the resistivity figure. If 1 and A are not in these
units, they should be converted first as is shown by Example 23.
Example 22. If 5m of manganin resistance wire, O.lmm
diameter, have a resistance of 267.5 ohms, find the resistivity ot'
the material.
Here R = 267.5Q A = T[ x d 2 = 3.14 x ( 1 x 10-4)2
4
4
= 0.785 x I r l o - ' squ:ire metre
I = 5m

R A = 267.5 x 0.785 x 1 x lo-'


Since R = lo-1 then p = --A
I
5
10-8
o r p = 535 x 0.785 x lo-' = 5.35 7.85
10
= 41.998 x
ohm-metre or 42 x 10-8Rm
= 420 x
= 420 x
x
= 420pRmm.
Exdmple 23. Find the length of wire required to make a 10R
resistor, if the diameter is Imm and the resistivity is 450pQmm.
Here R = 1 0 0 .
p = 450 x
x
= 45 x lo-' ohm-metre
d = Irnm = 1 x 10-Ametre

= 17.44m.
P 1 can be
Occasionally a problem involving the formula R = A
worked by a method of' proportion.
This makes for easier working than finding the resistivity
values and resubstituting in the formula to obtain the answer.
This is illustrated by the example.
Example 24. If the resistance of 1.6km of copper wire of'
0.5mm diameter is 1700. calculate the resistance of I km of iron
wire of 1.Ornrn diameter, assuming that the resistivity of iron is
7 times that of copper.
Resistance of 1600m of copper wire 0.5mm diameter is 170
ohms, then resistance of' lOOOm of copper wire 0.5mm diameter

;ind resistance of lOOOm of copper wire 1,Omm diameter is


17'
Irn x lohms

1600

.l'ore. The iron wire is of double the diameter, so copper wire


of' double the diameter has been considered. Since Area =

"'**.
it
4

i;,iinas

I'OIII. tlmcs
by 4 .

;IS

111i11

wire of twice l l i c diameter will ti;\ve :In Rreil

grc;~t; ~ n dthc rcsist;~ncetherefi)re will bc .reduced

CONDUCTORS AN11 INSIJLATOKS


- .

Proceeding with the colution


Resistance nf I M m (>f' c.ol3pt.r w~l.r1

So reslstancc of' IO(X)m 01' iron

\\II.C

0111111 i n

tli:~mctcr i \

I.Omm di;~lnetcr

VARIATION OF CONDUCTOK KESlSTANCF N'ITH E M P E R A T U R E

M o s t conductors are found to show a \ariation of resistance


when their temperature is changed. Usually this variation
follows a straight line relation as is shown by the diagram (Fig
20). If the resistance of' a res~stor is measured each time i t 5
tempe1:atuce. ic a1ter;ednnd \ke \esu(ts ,t,
phtted,
w
a g,$h svch
as ( I ) o r (2) o r (3) would'be obtained. These graphs woultl cover
the main types of conductor a n d would show that
( 1 ) F o r pure metals, resistance Increases regularly with temperature.
(2) F o r certain metal alloys used f'or making resistors, such as
Manganin o r Constantan, the graph is horizontal. The
resistance is substantially unaffected by temperature.
(3) F o r certain partial conductors. such a s carbon, the
resistance is found to decrease with temperature.

20

40

80

M,

TEHPERATURE

100

OC

Fig 20
TEMPERATURE C O EF FIC IENT O F KESISTANCE. Because of' the
straight-line relationship between resistance a n d temperature.
illustrated in Fig 20, a simple law 1s evident iind :in equation

can be deduced, which within normal ranges of temperature,


allows the resistance R of a conductor at any temperature T to
be obtained in terms of the resistance Ro at OC and a coefficient
(symbol a-the Greek letter alpha), known as the temperature
cueflcient of the conductor material. Tne appropriate equation
is :
R = R, ( 1 aT)
For the diagram (Fig 21) the graph for a pure metal (copper)
is illustrated and enlarged to show how the above formula
has been deduced. It cuts the R axis to give a value R,, the
resistance at OC. If now, the resistance has a value R, at 0C
then at I0C it will inciease by a small amount x. The fraction
.Y .
is taken as the temperature coefficient a of' the metal or
R0
\'
- = a and .r = aR,.
R0
cr is thus the fraction of the resistance at 0C by whlch the
resistance increases for 1C rise in temperature.
I f x = increase in resistance for 1" rise in temperature, then
xT=
,,
p ,,
,,
Thus R = Ro + xT = Ro + RoaT
or R = R, ( 1 + aT)

,3

1
For copper, a has a value= 0.004 265.

234.5

The temperature coefficient of resistance a, is usually based on


average conditions obtaining from 0C to 100C and is thus the

CONDUCTORS AND INSIiLATORS

43

ratio of the incrcnsc in rcsist:rncc per "C' rise in tctnpcrtiturc 10


the resistance at .O0C. Alternatively, the temperature coefficient
of resistance can be defined :IS the increase of resistirnce of 1 ohm
at 0C for 1 "C rise of temperature.
If the graph, as shown in Fig 20, is rising as temperature
increases, then the material is said to have a 'positive' temperature coefficient of resistance; whereas if the graph falls, the
material has a 'negative' temperature coefficient. Usually for
practical work and problems, the ohn?ic value as given for a
resistor will be at a temperature other than 0C and the following expression has been deduced to facilittite the obtaining of the
resistance value at any particular temperature, if its resistance
value is given for any other temperature condition.
Let R , be the resistance at a temperature T I T and R , be the
resistance at a higher temperature T,'C.
Then R , = R , ( 1 + aT,) and R , = R , ( 1 +*zT,)
Dividing, we have
R, 7-2)
- RO (1 + ~ o r R , = R , ( 1 + aT2)
R,
Ro (1 + a T , )
(1 + r T l )
E x a m ~ l e25. The cold resistance of a coil of wire is 2 0 0 a t
15C. 1 i . k heated to give a hot resistance of 23R. Find its
temperature rise, if the temperature coefficient of the resistance
material is 0.42 per cent per "C.
Not(,. Tcmpcraturc cocflicien~can be cxprcsscd as given ; ~ b o v r
f'or the example, but is more ilsually written as 04042 from 0C'
or more fully 0.0042 ohm per ohm at OLC per 'C it1 0.004201R
at O0C/"C.
The problem can be solved indirectly as
20 = R , [I + (0.0042 x !55] or 20 = R , (1.063)
whence R , = 2o - 18.fR
1.063
Resubstituting23 = 18.8 [I
(0.0042 x T ) ]
or 1.223 = 1
(0.0042 x 7 )
2230
and T = -= 53. lC
42
The temp. rise = 53.1 - 15 = 38.ir(3
The following is a mote direct solution.

Thus 1 15 x 1063 = 1 + 0.(@42 x T ,


or 1 222 45 - 1 = 0.0042 x T,
and 0 223 = 0 0042 x T, or T, = 53 1C
As before, the temperature rlse is 38 1 C

Example 26. The filament of a 230V lamp takes a current of


0.261A when working at its normal temperature of 2000C. The
temperature coefficient of the tungsten filament material can be
taken as O.OO5RIR at OCC/"C.Find the approximate current
which flows at the instant of switching on the supply to the cold
lamp, which can be considered to be at a room temperature of'
2OCC.
230
Resistance of lamp (hot) = - = 882 ohms at 2000C
0.26 1
Here R , = R, ( 1
aT,)
8 87
882 = 80.2 ohms
So
R0 = [l-+ (0.005 x 2000)l =
Agi~in
R z o = 80.2 [l + (0.005 x 20)]
or R z o = 80.2 ( 1 0.1)= 80.2 x 1.1 = 88.22l2
230
and current taken when cold = -= 2.6A.
88.22
The example shows how change of resistance with temperature
can affect practical working conditions and allowances to this
end should be made. The 'tripping' of a circuit-breaker or
'blowing' of a main-fuse, could mean the loss of supply to a
large lighting or resistance load. Before any attempt is made to
restore the supply, sections of the load should be isolated, so
that when the main switching-on takes place, only part load is
applied to the supply and this load is then gradually built up to
its f'ull value, b y closing the individual circuit switches. The
reason for 'shedding load' in this manner is related to the h c t
that thc lamps will have cooled when the supply was off and
when supply is restored, a current of seven to ten times the full
load value will be taken as a surge. This current will fall as the
lamps heat up and may only last for a few milli-seconds, but it
could be sulticient to re-trip the circuit-breaker or blow the
main-fuse. Thus a fault condition would be suspected, but in fact
the cause o f the current surge can be explained and appropriate
action taken.

RESISTANCE OF AN INSULATOR
VARIATION O F INSULATION RESISTANCE W I T H DIMENSIONS A N D
MATERIAL
All insul;\tnl. h;is Iwcn dofincd as ii substance which will not

allow the I'ree flow of' electricity. In electrical apparatus,


~n:l(.liil~r\
:111(1 C : I ~ ~ C Sinsi11:itiori
,
i s used to confine the flow o f
t.lec11.1c.ltyIo tile ~.cquilcclclrcuil iilld to plevclll llle c u ~I Z I 11.0111
~ ~

CONDUCTORS A N D INSULATORS
--

--

45
.-

taking 'lea k ; ~ g e path$'. I,r;~hnec current\ arc r n ~ n ~ r n ~ \ hr t\ l


I I I . I ~ I I I tile
~
I C ~ I S I U I I C C0 1 t11r11patti5 a5 large a 5 po5hiblc. I I i u ~
material with a high resistivity ( p ) is used for the insulation, tlic'
lengtli of' the leakage path is kept as Iitrge as possible and tllc
area as sm:lll as possible. I t should be realised at this stage that
insulation does allow current to pass and that by measuring this
currer,t, the soundness of' the insulation can be judged. Instrument such as high-resistance otinimeters or insulation testers-one trade name being 'The Megger', operate on this pnnciple.
The simple diagram of Fig 22 shows the path of' the leakage
currents in the cables forming part of the circuit fieding a lo;~d
and it will be noted that the leakage currents flow radially f'rom
or towards the conductors of the cable. The cebles can be
looked upon as steam pipes, radating heat. The longer the pipe
the more the heat radiated, ie the greater its radiating area.
Similarly for the cables, the thickness of the insulation, shown
PI
as t is actually the length 1 in the formula R = -.while
the area
A
A is given by the curved surface of the insulation. The larger this
surface the easier it is for leakage current to flow.

Fig 22
T o summarise the foregoing, it is panted out that insulat~on
resistance of cable would be measured between core and sheath.
or 'earth' and would be glven by the formula F:

i'1
= -.

Here p
A
would have an extremely high value; for vulcanised rubber it 1s
iO'SRm or 1O9MRm. 1 would be the insulation thickness t and
surface area A would be proportional to the length of the cable.

Thus if the insulation resistance of lOOm of cable was measured


as being 180 megohms, then 200m of the same cable would have
a resistance value of 90 megohms. The basic point is that cableconductor resistance is doubled for double the length, but the
insulation resistance is halved. Doubling the length has doubled
1
the area of the leakage paths and since R a -, if A is doubled R
A
is halved.
It should now be understood why a large electrical cable
installation or machine when tested for insulation resistance may
give a low figure, whereas the value obtained for a smal! installation or machine may be considerably larger. Insulation resistance
is :dso rlffectcd by other frlctors, bcsides the size of the instnllation or machine. Site condition such as temperature, humidity,
cleanliness together with age must be taken into account and the
resistance value means little unless compared with that obtained
for a comparable new installation o r machine. Acceptable
insulation-resistance values for installations and machines are
set out in the appropriate Regulations and the points made
above have been stressed to show that test results should be
treated with due consideration. Conductor-resistance measurements are more straightforward, although here again, special
testing techniques should be employed depending on the type of
resistor or apparatus being measured.
VARIATION OF IQULATION RESISTANCE WITH TEMPERATURE

For electrical apparatus, machines and cables, the allowable


working temperature and hence the currentcarrying capacity of
the equipment is limited almost wholly by the restrictions
imposed by the insulation. The insulation is usually made up
from cotton, silk, rubber, o r plastics, and as a general rule, if
they are subjected to excessive temperatures their electrical and
mechanical properties are impaired. Even if insulation such as
mica or porcelain is not damaged by excessive temperatures, it
is seen from the attached graph (Fig 23) that, like the partial
conductor carbon, the insulation resistance falls with temperature rlse, but here the relationship is not straight line. The
graph can be shown to follow a logarithmic law and thus insulation resistance falls rapidly as temperature rises. An increasing
leakage current flows through the insulation as its temperature
rises and such a current generates more internal heat which may
eventually cause 'breakdown' of the insulation. The allowable
tcrnpcrctturc rise for any electrical equipment which gives a safe
insulation-resistance value, has been determined by experience

CONDUCTORS AND INSULATORS

----

20

- .--

40
TEMPERATURE

80
r

Fig 23
and the power rating of an appliance is in accordance with
accepted specification. For example. BS Specifications or
Lloyd's Regulations may specify a working temperature rise of
50C for a particular motor when performing a certain duty.
This would be when it was developing its rated output in an
ambient or room temperature of'30"C. T h u ~;I total t e m p e r ; ~ t ~ l r c
01' 80C' would Ix allowccl. I ' l i l s lig~rrcvat.lcs 101. t l ~ c~ y p co l
insulation with which the machine is constructed, but for the
example, if the same motor is to work in an ambient of NC,
then the allowable temperature rise will be reduced t o 30C. The
motor would now only be capable of giving a reduced output
and would have to be derated. Alternatively a larger machine
must be used, if the full original power output was still required.
Derating of equipment is necessary to ensure a maximum
safe working temperature for the insulation and for this condition, the insulation resistance will reach an acceptable
minimum value.
Since the insulation-resistance value alters as the temperature
of the equipment alters, and it is also affected by other factors
already considered earlier such as, size of installation, humidity,
cleanliness, age and site conditions, then a true indication as to
the state of the installation o r machine can only be gained by
reference to a record o r log of readings, built up over a period of'
time. Reference to iuch practice has already been made earlier
and it should be accepted that the keeping of such a log is

essential for large electrical installations. Many ships are now


fitted, with insulation-resistance indicators which record leakage
current and thus the state of the insulation resistance. Such
indicators assist the keeping of a log which will show comparative readings for the same temperature rise, taken when the
installation o r machine was new, dry and clean. The difference
between the readings can be used to assess the state of the
equipment at the time of checking, and if an improvement in
readings is deemed essential for safe working, then appropriate
arrangements can be made for cleaning down, drying out or for
;I ~norc.rllo~.oupl~
inspcct'ion and overh;~ul.
RESISTANCE OF A SEMICONDUCTOR
Llt.ctronic dcviccs utilisiny semiconductor materials arc introduced latcr- but studied in depth in Volume 7. It is necessary, at
this stage, to make a reference to the important relationship
between the resistance of a semiconductor and its temperature.
A semiconductor can be described as a material which, for given
dimensions, has a resistance value midway between that of' a
conductor and an insulator of the same dimensions. The main
usage of semiconductor materials is in connection with -s olidstate devices such as re6tifier diodes and transistors, but here we
consider the resistance-temperature property in relation to
l l ~ ~ ~ r l-~ l l ~ .
P

\'ARIATION OF SEMICONDUCTOR RESISTANCE WITH TEMPERATURE

Semiconductor materials have resistance values which alter


appreciably when heated. Germanium and silicon, a s typical
examples, have a negative temperature coefficient which is r:ot
constant, but increases as the material is heated. The relationship of resistance with temperature is in accordance with an
inverse variation and gives a graph similar to that shown for
Fig 23. It will be seen from the graph that as a semiconductor
material is heated, its resistance falls and if a piece of this
material is used as a resistor then the current passed will increase
as the piece heats up. The semiconductor, when used as described here. is known as a thermistor. It can be adapted for use
as a measuring or regulating device. As an example of the
Sormcr, it hiis been dcvcloped for marine work as the detecting
element of an electrical temperature-indicating instrument. The
original therrnometer head consisted of a coil of platinum wire
wliicl~,when heated, altered the resistance of an indicator circuii
so that the latter could be calibrated to indicate temperature. 4
tlicsrnistor- clernent is no\\ being used instead of the platinum

\ + . I I c h(.~n)r.
.
marc. roht15l. (71' im;~lic.rd ~ m e n < ~ o ncirn
\ he locateti

llc~lscl[ I l C *\lo!\ p o [ , , ; 1 1 1 c l g r \ r \ * I glc;l~cll c \ l \ l < l l l ~~ .~l ~l :~l l l l~o r1 .I


givcn tcmpcr;iturc chi~nge.Thc instrurncnt i b !bus morc scnslti\,c
, ~ n d;~ccuratc.
The device can be used as a regulator since i t can alter the
operating current to a controlling circuit when its temperature is
varied. Thus if a thermistor is buried in the windings of an electric
motor, any overheating adjacent to its situation will result in the
therm~stor-circu~t
current increasing until the connected motor
protective device 1s operated. Such thermistor operated units
;ire now being offered for marine usage in'conjunction with
motor starters but care must be taken to ensure that the
thermistors are correctly located and connected.
The use of thermistors is now becoming so common for
electronic circuitry that mention must be made of the fact that
research and development have resulted in unit2 being produced
which have a positive temperature coefficient in contrast to the
more usual negative coefficient characteristic. Such thermistor
elements are obviously special and suited to particular applications.
HEAT A N D ELECTRICAL ENERGY
Energy can exist in several forms. The mechanical, electrical,
thermal and chemical forms are those most used for modem
Industry and the work done when energy is expended can be put
LO use in various ways. Althougl~rllc tcl.ni 'energy I x i ~ l gespended' is commonly used, it should be remembered that energy
cannot be destroyed o r lost. It can only be changed from one
form to another and the obvious convertibility between mechanical and electrical energy is seen in a machlne like the electrical
generator or electric motor. For the former, mechanical energy
is passed in at the shaft and electrical energy is obtained and
utilised in a circuit connected across the machine terminals. The
utilisation may be effected by converting the electrical energy
into heat, light or mechanical energy. For the electric motor
electrical energy is passed in at the terminals and mechanical
energy is passed out at the shaft. The relations between the
mechanical and the electrical units of energy and power have
already been deduced and our studies now continue with the
consideration of the relation between mechanical, electrical and
heat energy.
RELATION BETWEEN MECHANICAL AND HEAT ENERGY

The fact that heat is a form of energy is probably the most


obvious to the practical engineer, who is only too well aware of

the dangers associated with a 'hot bearing', 'slipping belt' or


'clutch'. In these instances mechanical energy is made available
by the prime mover and is being converted into unwanted heat
through the medium of friction. If this conversion into heat is
allowed to continue, the temperature of the associated machine
parts may rise to a dangerous level, when a 'seize-up', 'burn-out'
or fire may result. The examples have been quoted to show that
an elementary deduction can be made showing that the heat
energy produced is proportional to the mechanical energy being
expended.
sPEcrFlc H E A T CAPACITY . This can be found by a comparatively
simple mechanical test. The laboratory apparatus would consist
of a hollow brass cylinder, which can be rotated by a belt drive.
The cylinder can bc filled with a known quantity of water and
made to rotate against a friction surface applied with a known
tension. By simple calculation, the work put in at the driving
pulley can be related to the heat produced at the cylinder. James
Joule, an English scientist, by careful experimental work showed
that 4.187 joules of work are required to produce sufficient heat
to raise the temperature of 1 gramme of water by 1 degree
Celsius or 1 kelvin. Since we are now concerned with SI units,
then if the mass of water is taken as 1 kilogramme it follows that
41 87 joules (42005 approximately) would be required. The joule
is now also an SI unit of heat and thus we have the condition
where this constant of 4200 must be taken into account by introducing the term 'specific hear capacity'. This is defined as the
quantity of heat required to raise unit mass of a material
through a temperature interval of 1 degree Celsius or 1 kelvin.
Different materials would require differing amounts of heat to
produce the same temperature rise on the same mass. The units
of specific heat capacity (symbol c), are heat units per unit mass
per unit temperature. Since, for SI units, the most convenient
unit of mass is the kilogramme then the kilojoule would be the
appropriate size of heat unit to give specific heat capacity in
kilojoules per kilogramme per kelvin or kJ/kgK. In terms of the
Celsius temperature scale, this would be kJ/kg C. Because the
rclation bctwccn thc energy and heat is most readily determined
fbr water and has been taken as 4200 joules, it follows that the
specific heat capacity value for water would be 4.2kJ/kgC. The
values for other materials are also determined by experiment and
can be compiled into the usual tables of physical constants. The
fnllowlng example illustrates a conversion from mechanical to
heat umts which involves the use of differing specific heat
ciipucity v LI* 1ucs.

51

CONDUCTORS AND INSULATORS

Example 27. A motor brake-testing rig consists of a watercoolcd, ~ ~ 1 s t - i r opiillcy


n
; ~ n d:I fixcd f'r:~mcw h i c i ~is rl~:ldcto c:lrry
two s $ m g balances to whch are fastened the ends of a rope
which passes round the pulley. Both spring bnli~nccsIli~npt'rnm
threaded rods which are arranged to be adjustable to alter the
tension on the rope. Tests made on a small motor running at a
full-load speed of 750 revlmin gave the following readings.
Spring balances 16.89kg and 0.55kg. The pulley is hollow 102
rnrn wide, 380mm in diameter (these are outside dimensions).
It has an average wall thickness of 6.4mm. It has a mass of
2.72kg and is designed to be half-filled with water. Estimate the
output power of the motor being tested and the time for which
the motor can be tested before the water commences to boil.
The temperature of the pulley and water is 15C at the start of
the test and the rope diameter is 25mrn. Take the specific heat
capacities of water and cast iron as 4.2 and 0.42kJ/kgC
P
respectively.
2nNT
Power output of the motor is given by the expression 60
Effective load on brake = (16.89 - 0.55) = 16.34kg
Hence restraining force F = 16.34 x 9.81 newtons
380 25 405
- - = 202.5mm
Effective radius =
2
2
Restraining torque = 16.34 x 9.81 x 202.5 x
newton

met rcs

750

x 3.14 x --- x (16.34x 9.81 x 202.5 x


60
joules per second
75
= 3.14 x - x (16.34 x 9.81 x 202.5 x
3
watts
= 3 . 1 4 25
~ x 4 x 4 . 0 8 5 x9.81 ~ 2 0 2 . 5 ~
= 3 . 1 4 ~ 4 . 0 8 5~ 9 . 8 ~
120.25
= 12.827 x 9.81 x 20.25 x
hlowatts
= 1.283 x 9.81 x 0.203 = 12.59 x 0.203
= 2.56kW
Heat energy available per minute = 2.56 x 60 = 153.6kJ
xd2 x width x
Volume of water = 4

Output power

= 2

= c ( 3 8 0 - 12.8)' x (102- 12.8)x:

cubic
millimetres

= 0.785 x 367.2'

89.2
x 2

= 0.785 x 36i.22 x 44.6 x

cubic
metre

0.785 x 3.672 x 3,672 x 4.46 x

= 0.785 x 13.48 x 4.46 x


= 0.785 x 6.01 x
= 4.72 x
cubic metre

Since 103kg is the mass of 1 cubic metre of water


x lo3 = 4.72kg
Then mass of water = 4.72 x
Mass of pulley = 2.72 kg
153.6
Temp riselmin =
(2.72 x 0.42)
(4.72 x 4.2)

Heating time

85

=-=

7.3

11.7 = 12 min (approximately).

RELATION BETWEEN ELECTRICAL AND HEAT ENERGY

In Chapter 2 mention was made of an associated effect which


would be noted when current flowed in a circuit. This effect
would be a temperature rise in any part of the circuit, where
resistance was concentrated and one definition of the ohm
relates the unit of resistance to a joule of energy beinggenerated,
when,a.current of 1 ampere is flowing. Since this energy cannot
be destroyed, this is obviously another instance of energy
conversion from one form to another, and a simple test can be
ma'de to deduce the relation between heat and electrical energy.
Such a test w h l d determine the specific heat capacity of a
material by an electrical method and, since water is the most
convenient substance, an appropriate rig-up is described.
The experimental apparatus consists of a glass flow-tube
which is surrounded by a glass water-jacket-spaced some little
distance from it. The whole is sealed so that jhe space between
flow-tube and jacket can consist of a vacuum. A heating wire
runs centrally along the tube and thermometers are placed at
either end of the tube. The water, the specific heat capacity of
which is to be found, is arranged to flow steadily through the
ruhc i~nd is Ilci~tctl clccr:ici~lly by il known current passing
through the wire. Af'ter a period the inlet and outlet temperatures of the water become constant and this difference in
temperature is noted. The constant rate of flow of the water is
mctcrcd and t h u s the mass of liquid being hcatcd per sccond is
known. The volt;tpe drop :\cross thc hc;~tcris ;rlso mc;\surctl ; \ n d
wc Ilavc tlic fin:~l~.cl:~tionsllip:

CONDUCTORS AND INSULATORS

53

It1 I5

tI1c

IllilS5 0 1 '

V / = rncT
I I ~ U ~ L I(., tI1c 5 [ ~ ~ ~ 1 11c:11
l i C C:lpilclly :111d '1' t11c

temperature rise.
Tlic vi~lucol' 1. would Ix II'ou~ldto tx. 4.2k.l/kp"C'. I r will hc
noted that the specific heat capacity of' water, when determined
either by mechanical or electrical means, is found to be the same.
Different forms of apparatus have been developed to find the c
values of various materials, the electrical method being usuall~.
favoured because of the accuracy with which control of the test
can be effected and measurements taken.
The following examples show how the specific heat capacity
value is used in electrical problems.
Example 28. A brass calorimeter was found to have a mass of'
67 grammes. It was filled with water when the new mass was
131.7 grammes. The temperature of water and container waa
18C. A heater coil was next immersed in ths calorimeter, which
was suitably lagged to minimise heat loss. Find the time taken
to heat the water and calorimeter to a temperature of 33'C, if
the heating was done by passing a current of 2A through the
coil, the voltage drop across which was 7.5V. Take the specific
heat capacity of brass as 0.39kJlkgC and that of water as
4.2kJ/kg0C.
Mass of water = 131.7 - 67 = 64.7g or 0.0647kg
Mass of calorimeter = 67g = 0.067kg
'l'c111~~11.:1lurc
I ~ S Co I ' w : ~ t c:III(I
~ c : ~ l o l . i l n r ~-c ~7 7
I U -- 17C
Heat required by water ( ! ? I ( , ? ] = 0.0617 x 4 . 7 x 15

., calorimeter

kilojoules
0,067 x 0.39 x 15
kilojoules

Total amount of heat required


= 15 [(0,0647 x 4.2) + (0.067 x 0.39)]
= 15 (0.2717 + 0,0261) = 15 x 0.2978 kilojoules
= 15 x 0.2978 x l o 3 joules
Input power = 2 x 7.5 = 15%:
:. Time taken to procruce the,temperature rlse

297'8

- 4-96 -= 5 min (approximately).


60
Example 29. A 220V clectr~ckettle has an effic~encyof 90 per
cent. Calculate the, resi.;tance of the heater-coil and the current
necessary to raise the temperature of 1 litre of water from 15C
to boiling point in 9 ml Utes. Take a l ~ t r eof water to have a mass
of 1 kilogramme and he specific heat capacity as 4.2kJlkg C
-

Since na information is given about the kettle, the effect of


heating it must be neglected.
Heat received by water = 1 x (100 - 15) x 4.2
= 85 x 4.2 = 357kJ
Heat energy put out by the heater (kettle is only 90% efficient)
357
= - kilojoules
0.9
Time of heating = 9 x 60 seconds
357 x lo3 - 3570
So wattage of heater coiU =
0.9 x 9 x 60-0.81 x 6
735 36.75-3.34A
Heater current = - = -220
11
220
Heater resistana = - = 65.9R.
3.34
Example 30. A 120W electric soldering-iron is plugged into the
120V ship's mains for five minutes, the ambient temperature
being 15C. The mass of the copper bit is 133g and 50 per cent
of the heat generated can be assumed to be lost in radiation and
heating the other parts of the iron. Find whether the iron has
reached working temperature in the time specified. .Take the
specific heat capacity of copper as 0.39kJ/kgC and the temperature of melting solder as 310C.
Heat required by the iron = 0.133 x 0.39 x (310 - 15)
= 0.133 x 0.39 x 295
= 15.34kJ
*I

15'34 - 30.68kJ
Heat produced by the element = 0.5
= 30 680 J
Power rating of the element = 120W
Time for 30 680 joules to be expended
- 30 680 minutes
120 x 60
_--306'8 - 4.27 rnin.
72
Since only 4.27 min are required to achieve working temperalure, then the time of five minutes as specified would be sufficient.
Example 3 1. A resistance unit consists of 500 turns of nickelchrome wire, 0.5mm diameter. It is wound on a former 30 x 100
mm and its resistivity can be taken as 1 0 6 0 ~ m mat 15C. At
10O0C,its resistance is 2 per cent greater !han at OC. Determine
the current taken at a temmperature of 3 W C , when the resist:inn: is conncctcd ncross n 250V supply.

55

CONDUCTORS AND INSULATORS


.
.
.
.
.
.
.
.
.
p
.
.

~-

Length of a turn = 2 x 30

+2x

100 = 260mm

N o ol' t i i r ~ ~ s 5 0 0
;

Total length of wire = 260 x 500 = 130 000mm = I?Om


-

106 x 13 x lo-' x 4 - 10.6 x 13 x 4


o r R,, =
rr x 0.25 x
n x 0.25

Also since
R = R, ( 1

+ aT) or

1.02 = l ( 1

+ a100)

a n d a = 1.02 - 1 = 0.0002
100
Similarly R, = R, (1 + aT,)
a n d R, = R, (1

+ aT,) o r

R, = R , 11 f aT2)
(1 + aT,)

where R , = 702R a t 15C.


1 + (300 x 0.0002) 702(1 0.06)
Giving R, = 702
1 + ( 1 5 x 0 ~ 0 0 0 2 ) (1+0.003)
- 702 x 1.06 = 742G
1,003
250
Current taken = --- = 0.337'A
742

CHAPTER 3
PRACTICE EXAMPLES
1.

Ten thousand cubic millimetres of copper are (a) drawn


into a wlre 100 metres long, (b) rolled into a square sheet of'
lOOmm side. Find the resistance of the wire and the resistance between opposite faces cf the plate, if the resistance of
the copper is 17pRmm or 1.7 x
ohm-metres.

A coil of coppcr wire h a s n resistance of 9OR at 20C and


1s connected to a 230V supply. By how much must the
voltage be Increased to keep the current constant, if the
temperature of the coil rises to 60C?Take the temperature
coefficient of resistance of copper is 0.004 2 8 0 / R a t O"C/'C

3.

An electric kettle is fitted with a heater unit of 1 2 0 0


resistance. The efficiency is 84 per cent and the voltage is
220V. How long will it take to heat 0.75 litre of water from
6C to 10O0C? Take the specific heat capacity of water as
4.2kJ/kgC.

4.

Find the length of manganin wire required to make a


1 5 . 7 0 resistor, if the diameter is 0.315mm and the resistivity
is 407p0mm.

5.

The cold*resistance at 15'C of the field coil of a motor is


2 0 0 0 and the hot resistance is 240R. Determine the temperature rise assuming the temperature coefficient of resistance
to be 0 ~ 0 0 4 2 0 / 0a1 OGC/"C.

6.

A 2-core cable, each core of which is 300m long and of


uniform cross-sectional area of 150mm2 is fed from one end
at 240V. A load of 200A is taken off from the centre of tlie
cablc and a load of lOOA from the far end. Calculate the
voltage at each load. A single-core cable of similar material
RROm in length ; ~ n dof uniform cross-scctioni~l i1re;I of'
5 0 m m 2 has a resistance of'0.2190.

7.

The resistance of a I00m length of copper conductor Imm


diameter, is 2.470. Calculate the resistance of a cable 800m
in length composed of 19 similar strands of conductor, but
each 1.5mm diameter. Allow 5 per cent increase in length
for thc 'lily' (twist) of each strand of thc completed cable.

9.

A 200V, 200kW electr~cfurnace is required to ralse 5Wkg


of brass from an inltial temperature of 15C to the melting
point of 910C. If the overall efficiency is 0.8, calculate the
time of the operation. Specific heat capacity of brass is
0.39kJ/kgC.

10.

A 230V electric water heater takes water at a mean


temperature of 16C and the mean temperature of the outlet
per unit of the energy i t
water is 82C. The cost a t 0 . 2 ~
consumes in a g v e n period is 7 4 . 4 ~Determine
.
the quantity
of water used if the efficiency of the heater is 80 per cent.
Take the specific heat capacity of water s 4.2kJ/kgcC.

CHAPTER 4

ELECTROCHEMISTRY
Mention has already been made of the three main effects of
current flow, namely those producing heat, magnetism, and
chemical action. In Chapter 3 the heating effect was discussed in
some length, whilst the magnetic effects will be covered in
Chapter 5. This chapter will be devoted to electrochemistry,
which was the subject of many basic investigations concerned
with the science of electricity. Such studies soon revealed the
chemical action associated with current flow and the reversibility of this action and the electric cell was the principal source
of electrical energy before the principles of electrodynamic
induction were discovered and the electric generator was
developed.
The existence of static electricity, namely electricity at a high
potential and exhibiting itself as a stationary charge, had been
known for centuries, but this was associated with the lightning
flash and friction effects such as the attraction of paper by a
piece of amber when the latter was rubbed. The early and if
somewhat accidental experiments of men like Galvani and Volta
showed that electricity could be produced and controlled by
chemical meajs and led to the first 'voltaic piles' or batteries
being constructed. These consisted of a number of plates of zinc
and copper alternated by absorbent material such as felt, soaked
with an acid or salt solution.
At the commencement of the nineteenth century the only
practical means of producing electricity was by chemical means
and, it is therefore not surprising that, this somewhat separate,
self-contained and special branch of electrical engineering
science was the first to be thoroughly investigated and developed.
'l'l\c luws (/'cloc*trolysis,first propounded by Faraday in 1834,
summarise the basics of theory sufficiently well, to allow modern
ideas on the relation between electrical and chemical action to be
accepted without upsetting earlier, well known and hard won
knowledge.
The conversion of' electrical energy into chemical energy and
the reverse action is an important example of the principle of the
conservation of energy. The theory involved is best studied from
thc first action, namely, the relation between electrical and
chemical energy. This process is called electrolysis.

I:l E('THOI.YSIS

'l'lie passage ot'curren[ [l~rougliii ~ I U [ I O I Iul ,111 Lic~cl,


i ~ l l i ~il )i ~
salt produces a chemical change, explained by the theory of dissociation. The solution is called the c~lrc~rrol~,cc~
i\nd the process
is termed electrolysis. The general theory involved is as follows.
When an eFc-irolyte is first made up, as would result from dissolving copper sulphate crystals in water, some molecules split
Into two, independent of any external assistance, to hrrn rot^.\,
carrying +ve and - ve charges. Up to now, no work has been
done on the nature of electricity and the introduction of'the term
'charge' may cause some concern to the reader. He can be reassured as to this, since the study of the nature of electricity and
the modern conception of the atom will be fully dealt with in
Chapter 13. Most up-to-date textbooks commence their study of
electricity with this very important part of the subject, but the
author feels that the practical engineer is b e g guided along the
lines developed in this book and that supplementary knowledge
can best be acquired at relevant points in the treatment of the
subject. It is sufficient for him to know, at this stage, that from
earliest times the presence of electricity was known because of its
simple attraction and repulsion effects. Thus a piece of amber
when rubbed, will attract light bodies such as pieces of cork or
paper. The amber is said to be electrified o r charged wit11
electricity. Further experiments would show a glass rod. when
I.uI>I>cLI ~ ' i l 1 1h i l l , , to l>cL ~ I L ~ L ~ I I ~ :IINI
I I ~ ~ c1 0I ; I ~ I I ; I Cpicc,cs
~
01' ~ ; I ~ , C
but if 3 similarly treated glass rod was buspendcd b a thread
and brought near the original charged glass rod. a repulsion
effect would be noted. Similarly an ebonite rod rubbed with f'ur.
would be found to be charged and if'brought near the suspended
charged glass rod, attraction would be noted.
Summarising we can say that the electric charges acquired can
be of two types, termed positive ( + v e ) and negative ( - v e )
charges and that like charges repel whereas unlike charges
attract. The allocation of the designation +ve charge to the glass
rod and - ve charge to the ebonite is purely arbitrary, but the
general theory is that all uncharged bodies consist of +ve and
- ve charges which neutralise each other. I!' these charges art.
separated by some applied effort then their presence becomes
detectable, and if they are caused to move from one body to
another or through a circuit then t h e ~ rmovement is explained b\
the passage of' current.
Returning to the subject of' this chapter. we see that for an
electrolyte the molecules split up into charged ions which are
extremely mobile. If two plates, termed the c.lec,trodrs, are

60

RFED'S BASIC ELECTROTECHNOLOGY

immersed in the electrolyte and a current is passed by means of


them through the solution, then a potential difference exists
across the electrolyte between the electrodes due to the applied
potential. The +ve ions migrate, under the influence of the
electric field of the potential difference, to the cathode, namely
the electrode by which the current leaves the electrolyte. Such
ions are called cations. The -ve ions, called anions, migrate to
the anode, namely the electrode by means of which the current
enters the solution. Metal or hydrogen ions always carry +ve
charges and travel with the current to appear at the cathode,
whcrcc~snon-mctnllic ions travel in the opposite direction to the
current and may appear at the anode or may engage in secondary
reactions, some of which will be described shortly.
ELECTROLYTIC CELLS
The whole arrangement consisting of electrodes and electrolyte as described above, is frequently called an electrolytic cell to
distinguish it from a voltaic cell which will be described later in
this chapter. Electrolysis does not occur with solids o r gases and
is only possible for certain liquids Some, like oils, are nonconductors, whereas others, like mercury, conduct without
decomposition. The remaining liquids are electrolytes, which
can therefore be defined as liquids which decompose when
current is passed through them. The electrolytic cell can be
constructed to enable experiments and measurements to be made
with great accuracy. In this form it is frequently referred to as a
Voltalneter
(Sulphuric acid solution)
The diagram (Fig 24) shows the construction of the apparatus
which is made of glass, with platinum electrode plates placed at
A and C. The lead-in wires, passed through rubber corks, are
not exposed to the solution to prevent corrosion, The voltameter
is filled with acidulated water and the platinum electrodes are
connected to a battery. Current passes from the anode to the
cathode and bubbles of gas are given off which rise into the
graduated tubes
If care had been taken before passing current, to fill both
tubes with the acidulated water by opening the taps and then
closing them after all air had been expelled, then certain deductions can be made from the experiment. After a period of time,
the gas collected would be found to be Hydrogen at the cathode
and Oxvpcn : ~ tthe anode. The ratio of the volumes of H to 0
would be 2 : l and the amount 01' gas collected would be pro-

THE W A TER V O L T A MET E R

ELECTROCHEMISTRY

61

Fig 24
portional to the strength of the current and the time for which it
flowed or more generally to the quantity of electricity passed.
In the acidulated water there are considered to be sulphunc
acid molecules which divide into three ions, two of hydrogen
carrying ve charges H + , H + and one with - ve charge represented by SO, - . Note the total + ve hydrogen charge equals
the - ve sulphate or sulphion charge, but the ions migrate under
the influence of the electric field. Thus the H + ions give up their
charges at the cathode and are liberated as hydrogen gas. The
sulphions proceed to the anode, but as they cannot exist in a
free state they combine with two hydrogen H + H + ions or atoms
from the water thus liberating oxygen as in the equation. Thus:
2S04 + 2 H 2 0 = 2 H , S 0 4 + 0,. The oxygen rises from the
anode and collects in the tube above it. The H , S 0 4 goes into
solution and thus the electrolyte is decomposed in that the water
appears to be used up, but the acid content remains the same and
the solution gets stronger; that is, its specific gravity rises.

(Copper sulphate solution)


The diagram (Fig 25) shows the usual arrangement. A copper
sulphate solution, made from crystals dissolved in pure water, is
contained in a glass or glazed earthenware tank. The electrodes
are made from pure copper sheet. The CuSO, molecule is considered to split into two ions, C u + and SO4--. When a p.d. is
applied to the electrodes and current is passed, the copper ions
migrate to the copper cathode to combine with it and give u p

THE C O P PER VO LT AMET ER

'

Fig 25
their charges. The sulphions give up their charges at the anode
and combine with the copper from this electrode to reform
copper sulphate. Thus copper appears to be taken from one
electrode and deposited on the other.
The chemical equations for the electrodes are:
(1) Cathode. CuSO, = Cu + SO,.
(2) Anode. Cu + SO, = CuSO,.
During electrolysis a certain amount of gassing may be noted
at the plates. This would be due to decomposition of water in
the solution as described for the water voltameter. Furthermore
some complex action may occur in the electrolyte due to sulphions combining with hydrogen in the water to form H2S04.
Oxygen from the water is then released to combine with anode
copper to give copper oxide. This oxide will then dissolve in the
H2S04 to give CuSO,. Irrespective of the action the final result
is a simple'one, in that the loss in,mass of the anode equals the
gain in mass of the cathode.
Various forms of voltameter can be constructed to allow
research into electrolysis. Thus a silver voltameter may be used
consisting of silver (Ag) plates and a silver nitrate (AgNO,)
solution. The cxumples described could be connected in series
and the same quantity of electricity passed through all voltameters. If the electrodes were washed and carefully weighed
bel'ore electrolysis LInd then washed and weighed again at'tcr
electrolysis, certain conclusions would be reached which were
first enunciated by Faraday in 1834 by his laws-of electrolysis.
Q U A N I,ITATIVE L AW S O F EL E C TR O LYS IS (Farada y's Laws)
1 , The mHss of an element liberated from or deposited on an
clcctrodc is proportional to the quantity of electricity which has
pil sbccf.

ELECTROCHEMISTRY

63

2. The masses of elements liberated from or deposited on


tn
clcctrodcs by a given quantity o f electricjty are
Atomic*
thcir Chcmicul Equivalent it1
---

-- ~ u b n c y

Consider the first law. It is found by experiment that the mass


of any material deposited or liberated always depends,on the
quantity of electricity which has passed. Thus in K Q coulombs
or rn cc It. This proportion can be modified to:
m = :It
where z is a constant depending on the substance deposited. I is
termed the el~ctrocheiniculequivulenr of the element.
(E.C.E.)
The mass in grammes or kllogrammes liberated by one
coulomb of electricity is called the E.C.E. of asubstance. Thus,
10 amperes flowing through a copper voltameter for 1000 seconds
would result in 10 000 coulombs having passed and 3.3g
- -of
ELECTROCHEMICAL EQUIVALENT

3.3

copper would be deposited. Thus the E.C.E. of cTpper = -10 000


= 0.00033g/C. Similarly that for h y dro g en would be
0.000 010 4, for oxygen 0.000 082 9 and for silver 0.001 118glC.
In line with metrication and the use of SI units, it is more
appropriate to think in terms of the kilogramme and the E.C.E.
can be defined as the mass (in kilogrammes) 01' a substatlcc
liberated by the passage of one coulomb. Thus the E.C.E. of
copper would be 330 x 10-9kg/C. Since the milligrr~mmeis also
a n accepted SI unit, the E.C.E. can be given as mg/C. Thus for
copper it would be 0.33mgIC.
The first law of electrolysis leads to a method of stating the
unit of current, which was considered accurate enough to allow
an original definition for the International Ampere. This then,
was defined as, that unvarying current which deposits silver at
the rate of 1 1 18 x ~ O - ~ kper
g coulomb when passed through a
solution of silver nitrate in water.
The formula already deduced above allows the solution of
problems associated with electrolysis and practical electroplating. The unit in which the E.C.E. is given should be noted.
Example 32. Find the time taken to deposit 11.4g of copper
when a current of 12A is passed through the copper sulphate
solution contained in a copper voltameter.
The E.C.E. of copper can be taken as 330 x
kilogrammes/coulomb.

64

REED'S BASIC ELECTROTECHNOLOGY

m
Since m = zit then t = -

Iz

= 2.88 x lo3
o r t = 2880 seconds = 48 minutes.

EQUIVALENT, VALENCY, ATOMIC WEIGHT


The second law of electrolysis can also be deduced by examinIng the results of tests made with a number of different voltameters in series, having been subjected to the passage of the
same quantity of electricity. The results of the experiment would
show that the mass of' the substances deposited o r liberated at
the electrodes would be proportional to the chemical equivalent
of tlic S L I ~ ~ I ; I I I C C I\ t. w ~ t l l d;IISO hc concluded that if thc i~tomic
weight of any substance is known, its E.C.E. can be found provided the valency is known and the E.C.E. of hydrogen is
assumed. Thus if the chemical equivalents of hydrogen, oxygen,
copper and silver were 1, 8, 3 1.8 and 107 respectively, the masses
of H, 0, Cu a n d A g liberated by the same quantity of electricity
would be in the same proportion and therefore the E.C.E. of a
substance is the E.C.E. of hydrogen multiplied by the chemical
equivalent of the substance. Thus taking the E.C.E. of hydrogen
as 0.0104mg/C, that of silver would be 0.0104 x 107 = 1.1 18
mg/C.
To conclude our deductions from the second law, it would be
well to define the following terms.
VALENCY. This can be described as the combining ratio of'a substance. As an example that of oxygen is 2, whereas that of
hydrogen is 1, so water is represented by the symbol H,O. The
valency of an elemental substance can also be defined as the
number of atoms of' hydrogcn with which one atom of' the
element can combine. Thus the valency of a sulphate is 2 since,
for example, in sulphuric acid H,S04, 2 atoms of hydrogen are
required to combine with the sulphate.
CHEMICAL

ATOMIC' wuc;t{.r. Atomb arc extremely small and determination


of their absolute masses present considerable difficulties. The
Inass of' a n atorn of' hydrogcn is bclicvcd to bc 1.67 x 10- 14g
and i t is still therefore customary, even in SI units, to refer to the
relative weights of the atoms of various substances in terms of
the atom of hydrogen. Thus the term atomic weight is still used
and is the weight of an atom of the substance in relation to the
rnnss 01' a n atom of hydrogen. Thus the value'for oxygen is 16, re
i t has 16 times the weight of an atom of hydrogen.

ELEC TRO C HEMIS T RY

65

This is the ratio of atomic weight


- to
Atomic
Wcighr.
is thus
valency or Chemical Equivalent =
Valency
weight of a substdnce which would combine wit11 one part by
weight of hydrogen or eight parts by weight of oxygen. Thus the
C.E. or combining weight of hydrogen is 1, that of oxygen is 8,
silver 31.8 and so on. as would be fourid in chemical tables.
From the second law we have
Mass of Material X liberated Mass of Hydrogen liberated
Chemical Equivalent of Material X
Chemical Equivalent of Hydrogen
0,- % = C ' E of
also from the expression rn = zlr
InH
C.E. of H
zx-It - C.E. of X or zX = zH x C.E. of X
z, It C.E. of H
C.E. ofcH
But the chemical equivalent of hydrogen = 1
so z,. - = z,- - (Chemical equivalent of substance X)
o r zx = ZH x Atomic weight of substance X
Valency of substance X .

CHEMICAL E Q U I VA L E N T .

Example 33. How many amperes would deposi' 2g of copper


in 15 minutes, if the current is kept constant. Give11the E.C.E. of
hydrogen as 0.0104mg/C, the Atomic Weight of Cu as 63.56 and
rhc Vi~lcncv01'

Then z,,

< ' i:IS


~ 2.

63.57
zH x ---= 0.0104 x 3 1.8

= 0.33mg/C

rn

whence I = - =
21

0.33 x

15x60

Example 34. A voltameter consists of a solution of iinc


sulphate and electrodes of' zinc and carbon If current is passed
in at the carbon electrode, zinc is found to be deposited on the
zinc electrode and oxygen is given 05at the carbon plate. If a
current 3.5 amperes is passed for 1 hour, find the mass of zinc
deposited and oxygen liberated from the solution. The E.C.E. of
zinc can be taken as 338 x 10-gkg/C, the Atomic Weight as
65.38 and the Valency as 2. Take the Atomic Weight of oxygen
as 16 and the Valency as 2.

From relationships already deduced:


rnzN = zz, It or m,, = 338 x
x 3.4 x 3600
= 338 x l o w 9 x 126 x lo2
or Zinc deposited = 425.88 x lo-'
= 4.26 x 1 0 - ~ k go r 4.26g

-)

At wt of Zn
and zo = z,
Valency of Zn
At Lt of 0 / A t 'wt of Zn
-0 =
zZN Valency o f 1 Valency of'Zn
ol'Zn
= At wr ol' 0 x Vatency
-...-Valency 01'0 At wt of'. Z;

Also zzN = z ,
50

Thus z , =

'ZN

A t w t o l ' o = 338
~t wt of ~n

10-9

(Valency
At
ofO0)
Wt

16
65.38

338 x
4.086
or 2, = 82.5 x 10-9kg/C o r 82.5 x 10-6g/C
x 3.5 x 3600
and ,no = 82.5 x
= 8.25 x
x 126 x 10' = 8.25 x 1.26 x l o - '
= 1.05 grammes.
-

BACK E.M.F. OF ELECTROLYSIS

The circuit laws enunciated in Chapter 1 govern the conditions


fhr the majority of practical circuits. The loads of' such circuits
are mainly resistive such as, the coils of' resistance wire in appliances like electric heaters and filament lamps. Such loads,
termed pussive loads, are recognised by the fact that they conform to Ohm's Law. For other types of loads such as the electric
motor and accumulator or storage battery when being charged,
Ohm's law is not directly applicable and they represent uctive
loads, of' which the electrolytic cell is also an example. The
difference between pure ohmic resistance and that offered to the
pass;igc ol'currcnt by a n clcctrolytic cell will now be considered.
The diagram (Fig 26) shows a simple circuit for which the
sourcc 01' supply i h ;I hattcry m ; d c u p of' thrce similar voltaic
cells in series. Assume that the current through an electrolytic
cell made up as shown, is adjusted and maintained at 3 amperes
by the variable resistor provided for this purpose. IS the circuit is
set and the supply potential is then reduced by removing one of'
the voltaic cslls, i t mny well bc iissumed that, as the e.m.f. has
hccn rcduccd to of' the original v;\lue, the current wil,l fall to
3 x 3 = 2 ;lmpc.I.cs. 111 acrual I'ucr the ncw current strength will

ELE C TR O C HE MI S T RY

67

Fig 26

be well below this value, and if the experiment is repeated by


removing another voltaic cell so as to make the supply e.m.f. of
the original, then the final current value will be well below that of
1 ampere, the value expected by an application of Ohm's law.
The experiment shows that an extra current controlling factor
is present in a circuit involving an electrolytic cell and the
results can be explained by considering that a back e.mlf. is
produced by the cell, so that the following equation represents
the conditions :

v = 5' , +

I/<,

Here V represents the voltage applied to the cell, E, is the back


e.m.f, of the cell, I the current causing electrolysis and R, the
internal resistance of the cell.
The magnitude of the back e.m.f., for any electrolytic cell, can
be found by further experiment and is important, in that, it is

Fig 27

responsible for polarisation, which will be discussed later in this


chapter in connection with primary cells. Here the basic action
is described, as i t would be noted for the simple water voltameter
shown in the diagram (Fig 27).
The electrodes are immersed in an electrolyte of sulphuric
acid. When the switch is closed current passes from anode to
cathode and both electrodes become coated with bubbles of gas,
oxygen and hydrogen respectively. Once this occurs the switch
is moved to a second position so as to connect a sensitive voltmeter across the cell only. The main circuit current will have
stopped, but thc voltmeter will register a voltage across tlie
plates, which gradually falls as the bubbles disperse, due to the
flow of the small current through the voltmeter. The voltage o r
c.rn.l'. is due lo the prcscncc 01' the gas bubbles, and i t would be
noted that the cell polarity, remains the same, namely that the
current flows through the meter f'rom anode to cathode in the
external circuit so as to be in the opposite direction in the cell to
the current flow which caused,electrolysis. The value of the back
e.m.f. of polarisation is obviously important in that, if the
applied voltage is less than the polarising e.m.f., electrolysis cannot take place. At start a current would flow but once polarisation begins, the back e.m.f. would rise to equal the applied
voltage and the current would cease.
In the case of water the value of the back e.m.f. E, can also be
calculated thus. I t is known that I gritmme of water when
formed by the ;ombustion of' hydrogen in oxygen produces
approximately 15.96kJ of heat. If now we assume that the
energy required to separate H and 0 in I gramme of' water is the
same, then the electrical energy required would also be 1596kJ.
Since both H and 0 are released by electrolysis it follows that
the total mass of gas o r water released by 1 coulomb would be
= the E.C.E. of hydrogen .+ E.C.E. of oxygen = 0.000 010 4 +
0.000 082 9 o r zH0 = 0.000 093 3 gramme ie 93.3 x 10-9kg are

relaistxl.
Thus 1 coulomb releascs 0.000 093 3g of gas o r liberates
this mass o f water and tlie clectric;~lenergy required to decomIWSC lllis I I I L I S S 01' W I I I U I .
15 900
O~OW09.3 3
I ,40 J O U I ~ ~ .
If this electrical energy is produced by the work done against
the back e.m.f. E,, then the applied voltage would have a value V
where V = E, and the energy produced by the passage of 1
coulomb = V x 1 joules. Thus V x 1 = 1.49or V = 1.49 and
E, = 1.39 volts.
During electrolysis, thc decomposition of' the liqirid results in
t l ~ cions, which Ilirve I~eerldissoci;~tcd,trying to ~.ccombineand
-~

mnkinp this fcndctiky nppiircnt ; I \ ; I hc1c.k r In f [:or water the


br the value
back e.m.f. is 1.5 volts approsimitreiy itnd ~rsoi~ld
which would be substiti~tcdfor I:', in the fnrmul;~: I ' :-I;,, -t- I R , .
Thus for current flow the applied voltage must be greater than
the e.m.f. of polarisation by the voltage drop due to resistance
of the electrolyte.
A back e.m.f. of appreciable value exists for electrolytic cells
made up with electrodes of dissimilar materials, but if both
electrodes are of the same matcriitl, as f'or thc copper voltamctcr.
then the back e.m.f. va'lue is so smitll that i t can be neglected.
This is explained by the fact that tliere is no marked tendency for
the dissociated ions to recombine. They can readily c o ~ ~ i b i nwith
c
the water of the solution as was stated in thc description of the
copper voltameter, and the all important result is that the
passage of current does not produce an overall chemical change
All that happens is that copper is transferred bodily f'rorn anode
to cathode. It should be noted that for the water voltameter.
although the electrodes are of the same material, namely
platinum, yet a back e.m.f. appears when these are coated with H
and 0 gas bubbles. The bubbles have the effect of producing
electrodes of different materials and thus an appreciable back
e.m.f. of 1.5V approximately.
Genera'l observations show that when the products of electrolysis possess chcmic;~l cncrgy. then thc ccluivi~lcnt clcctric;~l
energy I T I U S ~
hccrl supl)liccl t11t.ough L ' I C C ~ I . O I Y S I ~ ;~tldi l IXICL
e.m.f. of appreciable value must exist. An example of this reasoning would be the electrolysis ot' water. The hydrogen and
oxygen formed would recombine in an explosive manner to
form water with evidence of heat and iight. The energy latent f'or
this recombination was derived from the electrical energy put in
during electrolysis and a back e.m.f. of polarisation must have
been present. For an electrolytic cell, such as the copper voltameter, since the product of the process possesses no chemical
energy it can be assumed that a cell using electrodes of the same
material has negligible e.m.f.
RESISTANCE OF ELECTROLYTES. The resistance R , of' a liquid
conductor is proportional to the length and inversely proportional t o the cross-sectional area. It also varies with the nature
of the electrolyte and the concentration but it should be noted
that the temperature coefficient is a negative one. Because of the
back e.m.f. effect already discusst d earlier, the resistance is
difficult to measure. The value obtz ned by dividing the voltage
drop across a cell, by the current f owing will give ,t resistance

value for the electrolyte much greater than the true figure and
would be erroneous because of neglecting the existence of Eb.
P OWER EX PE N DE D D U RI N G ELECTROLYSIS. If the voltage equation
for an electrolytic cell is V = Eb + IRi then for a current flow o f
I amperes the power equation would become VI = EbI 12Ri.
Here the VI represents the power applied to the cell, EbI
represents the power required to produce chemical dissociation
and 1 2 ~represents
,
the heat energy produced in the cell which
results in a temperature rise.
Example 35. Find the voltage required to pass a current of'
4 anlprrcs tlirougli a copper voltameter which has an internal
resistance of 0.014 ohms and a back e.m.f. of 0.25 volts. Find the
power utilised to produce the electrolysis and that wilslcd i n
I ~ c i ~ ~ illlc
n g cleclrolyte. Find also the overall efficiency of' the
voltameter as a plating vat.
Since V = E, + I R , = 0.25 + (4 x 0.014) = 0.25 + 0.056
= 0.306 V
Power utilised for electrolysis = 0.25 x 4 = 1W
Power wasted = 12Ri = 26 x 0.014 = 0.224W

Efficiency of the cell for electrolysis = -= 0.82 o r 82%


1.224
PRIMARY A N D SECONDARY CELLS
Many of the f'undamentals of' voltaic or galvanic action, as i t
used to be called.originally, have already been mentioned in the
earlier pagesof this chapter, but we now consider in detail, the
theory concerned with the conversion of chemical energy into
electrical energy or, in other words, the production of an electromotive force by chemical action. The generation of such an
e.m.f. is best studied by describing the action of a simple cell.
THE SIMPLH V O L T A I C CELL

If a piece of commercial zinc is dropped into a glass jar containing ciilurc sulpliuric acid, tlic zinc is seen to be corroded
away and hydrogen gas bubbles are given off. The jar is also
found to get warm and it cnn bc deduced that Ileat is given oil'
by the chemical action. If' next, a piece of pure zinc is similarly
experimented with, none'of the effects mentioned earlier are seen
to occur nor do they happen if a dissimilar piece of pure metal
such as copper is substituted for the pure zinc.
The chemical action noticed for the commercial zinc is
explained by the presence of impurities in the zinc, the chief of
whicli ;trc iron and Icad A local closed circuit is made, say

bctwccn ;I p:~rticlcol' iron :~ntlt l ~ crinc, since hoth are in contact


and the acid is common to both dissimilar metals. A small cell
is considered to be formed and since current flow is possible, then
the generation of an e.m.f. is thought to accompany the chemical
action. The truth ofthe supposition is borne out by the following
experiments on a simple cell.
Consider two elect~.odesto be immersed in a solut~onof
dilute sulphuric acid as shown by the diagram (Fig 28). The
electrodes should be plates of pure dissimilar metals, such as
zinc and copper and should be placed so as not to touch each
other. No action will be seen to take place for the arrangement
described but if an ammeter and a resistance load are connected
as illustrated, then a current will be seen to flow when the switch
is closed. The current through the external circuit will be from
the copper +ve pole or anode to the zinc -ve pole o r cathode,
and it will be seen that hydrogen bubbles arsgiven off from the
copper plate while the zinc plate is slowly eaten away.

Hycmcm

BUBBLES

w:

.- - - -

Has04

Fig 28
A further point of the experiment is to note that the ammeter
shows the current to fall slowly and that it finally ceases after a
while. The copper plate will be seen at this time to be covered
completely with hydrogen bubbles and if these are wiped off with
a glass rod, the current would'be found to restart and the cycle
of action will repeat itself. The hydrogen bubbles appear to
control the chemical action of the cell which is said to "olarjse'
when the current ceases.
Further simple experiments' with different combinations of
electrodes and electrolytes will be seen to produce different

e.m.l's., as recorded by the voltmeter, which should be a sensitive


-. -.
.. supplying
current, the negative pole of each
Instrument. When
ill-rangement
_-will corrode-'away - . a . j $ - 1 " 0 e < i i i i $ - ~ l l
polar~sationwill be found to occur. A more complete explanation of the observations made can now be developed.
-,

T H E (.ELL E . M . ~ .The voltmeter used in Fig 28 will show that an


e.m.f. or voltage exists between the electrodes of a cell even when
this is on open circuit. When the circuit is completed, current
will flow because of this e.m.f. and chemical action will be
ot>se~.\~ed
\VIIICII ~ O I I ~ I I I I I Lto' S gener;~te;III c.m.f. and mi~intainthc
current. Tllc cIiemic:~laction ol'a voltaic cell results in a definite
c.1~1.l'.
wliic11 is the result of the action of both electrodes o r plates
w ~ t l~~ l clcctr~oly~c.
~ c
11' lie voltl~lc~cr
wits connected on ope11
circuit between the zinc and the solution, the former would be
found to be negative to the latter by some 0.63 volts. When
connected between the electrolyte and the copper plate, the
voltmeter would record 0.47 volts, with copper positive to the
sulphuric acid. With the instrument connected across both
plates a reading of 1 . I V would result as could be expected. The
production of a cell e.m.f. is explained by the electrolytic theory
;~ll-c:td! discussed under electrolysis. I t has been said earlier that
when ;In electrolyte is made up, tlie molecules split into ions
which arc clcctrici~llycharged and very mobile. An electrolyte is
thus a n ionised solution and when a metal is immersed into it,
some of'the metal appears to enter into solution, in that there is
;In irnmcdiate merging 01' surf'acc ions oftlie metal with the ions
of' the electrolyte. Thus the barrier between the metal and
electrolyte is not the surface of tlie former but along some layer
of' electrical potential equilibrium, which has caused ion interchange to cease. The action on immersing iarious metals into an
electrolyte differs for the different metals. Thus for zinc in dilute
sulphuric acid there is a greater tendency for +ve ions to pass to
the solution than ['or f v e ions of'the electrolyte to pass to the
~ i n cThc zinc plntc thus becomes deficient in +ve ions and
becomes negative to the solution by a voltage of 0.63V. F o r
copper in sulphuric acid, a different action takes place. This
metal becomes positive to the solution because there is a greater
tendency for the ve charged hydrogen ions of the electrolyte
to move to the copper than for the +ions of the metal plate to
pass to the solution and the copper rises to a potential of 0.46
\~oils\\,it11 respcct to thc clcctrolyte.
~l'lieinterchange of ions as described, results in a potential
I)c~tigsc!L I bclwcen
~
tlic clcctrodcs and solution which gradually

opposes the interchange irntil this fini~llyce;lsch ;rnd cqirilihriurn


is established. Thus the arrangenlent ot'zinc-copper clcctroclcs in
sulphuric acid as described, results in ;I potentiiil ciiffcrencc of
I. I V between the electrodes. Continuing our investigation of
this type of'cell on open circu~t,we see that the initial Zn ions
migrating to the solution combine with the sulphions to form
zinc-sulphate, libera-ting + ve hydrogen ions which move to and
accumulate on the copper plate. Thus the chemical action is
explained by the formula Zn .+ H,SO, = ZnSO, + 2H. Thc
potentials build up within the cell, quickly bring the ion thjgrations to an end and thus chemical action ceases. If now the
open-circuit condition is changed to that of a closedcircuit, by
joining the copper electrode to the zinc through an external
circuit, the chemical action is immediately noted to recommence.
Current is seen to flow and the formation of zinc-sulphate
continues with the liberation of hydrogen at theccopper plate.
The action of the cell on closed circuit conforms to the following reasoning. The open-circuit e.m.f. of l . l V , which is the result
of the initial ion migration, can now cause a current, the
magnitude of which is determined by the circuit resistance. A
flow of current, as will be seen in Chapter 13, means a movement of negative charges has resulted; passing from the zinc
cathode to the copper anode. The - ve charges move round the
external circuit from the zinc cathode to neutralise the +ve
iinode ch;irgcs, thus making way Ihr f'uriher migrcitions ol' t- vc
hydrogen ion charges in the cell. The initial cell action, as
described for the open-circuit condition, can now continue and
the cell functions by maintained chemical action; provided
polarisation is avoided. Polarisation has already been mentioned,
and will be described again for the simple cell. We can now
conclude our study of cell action by saying that all metal
electrodes produce an e.m.f. as a result of the ion interchange
action with the electrolyte, and that they can be arranged in a
table, in order of the value of their e.m.fs.

ELE~TKOMOTIVESERIES . If any two elements shown in the table


are used for a cell, the element lowest in the series is the +ve
terminal, when considered w ~ t hrespect to the external circuit.
The list comprises the more usual elements which are mainly
metals, but hydrogen and carbon are found to behave like metals
and are included. T o illustrate the use of the table, the ordinary
dry cell as used for
----a hand-tor&
-- can be considered. This cell
uses carbon for the +ve -electrode- and
-- zinc for the -ve electrode.
w-

74

REED ' S BASIC ELECTROTECHNOLOGY

Materials well spaced apart in the table are usually used for
practical cells. For the torch battery the e.m.f, is about 1.5 volts
per cell.
Aluminium
Zinc
Iron
Nickel
Lead
Tin
Hydrogen

Hydrogen
Copper
Carbon
Mercury
Silver
Platinum
Gold

POLARISATION. When the simple cell supplies current, polarisation occurs as described earlier. The circuit current gradually
falls, even though the c h e m i c d action of the cell appears to
proceed. Close examination will reveal that as the hydrogen
bubbles make their way to the copper plate, not all are liberated
here and rise to the surface. Some bubbles stick to the plate and
this tendency increases until the whole plate is covered with
bubbles to result in the cell becoming ineffective as a source of
e.m.f. The layer of gas surrounding the +ve plate causes a
polarising effect because (1) gas has a high resistance, so that
any area of the plate covered with bubbles is almost insulated
and cannot allow the passage of current. Thus the internal
resistance of the cell rises as the gas layer increases and the
circuit current mlls as a direct result (2) As hydrogen covers the
copper plate, it begins to make its presence felt in that it effectively replaces the +ve copper electrode by a hydrogen electrode
and thus reduces the e.m.f. of the cell: It will be seen from the
table of the electromotive series that the spacing between zinc
and hydrogen is smaller if compared with that for zinc and
copper. The cell e.m.f. is thus much reduced giving the final
result as described.
Oncc thc cilusc ol'pol;~risationbccamc known i t Wiis appnrcnt
that, in order to make the simple cell an effective source of'
elccrricr~lcncrgy, ii mclliod of' preventing t l ~ ecollection of the
hydrogen bubbles was necessary.
The simplest forms of'cicpolrrrisrrs which have been developed,
operate chemically, in that they combine with the liberated
hydrogen to convert i t into water; thus preventing the gas from
rcuching the + ve electrode and blitnketinp it. The methods by
which this i s nccnmplished will he seen when the examples of
prit,ttrry cells ;ire stucliccl.

ELECTROCHEMISTRY

75

THE PRIMARY CELL


Utidct t l i ~ sIicnding nrc cur~stdcrcd ~)r.uct~cul
cclls, wl~iclrutr

I
I
I

II
I

sultable for providing a constant e.m.f. when operating under


everyday conditions. They are however, cells which obtain their
electrical energy from chem~calenergy, the active ater rial being
used up in the.process. They differ from secondary cells in that
the latter utilise materials which are not consumed when the cells
provide electrical energy. The secondary cell can be electrically
'charged' so that its electrodes are chemically converted into
materials which enable the cell to provide an active e.m.f. for
supplying electrical energy. In this condition the cell discharges
and the electrode materials again change chemically, reverting
back to those of the uncharged state. The whole cycle of charge
and discharge can then be repeated.
Primary cells suffer from the two main disadvantages of the
simple cell, (1) polarisation and (2) local action.,:
Polarisation is overcome by the use of a su~tablechemical
depolariser which is therefore an essential component of cell
construction. Local action is minimised by using pure metal,
such as zinc free from impurities like iron and lead. In its basic
form the primary cell is a wet cell, which is not used to any
extent nowadays. In the dry form the ~eclanchecell is the most
common and particular attention should be paid to its construction and action.
THE D A N IELL CELL. This cell is now not used, not even in the

laboratory. The only reason for describing it, is to illustrate the


action of a depolariser, which is here a solution of copper
sulphate (CuSO,). The cell consists o f . a porous (unglazed)
earthenware pot which is placed in a copper vessel and contains
the -ve electrode which is a zinc rod. The +ve electrode is the
copper container, and the electrolyte is dilute sulphuric acid. The
diagram (Fig 29) shows a cross-section of the arrangement and
the action is as follows. The porous pot keeps the H2S04
separated from the CuSO, but allows the passage of ions from
one liquid to the other. As for the simple cell, the zinc and
sulphuric acid react to form zinc sulphate and hydrogen when
the external circuit is made. The hydrogen ions make their way
through the porous pot and enter the copper sulphate where
they displace the copper ions to combine with the sulphions to
form sulphuric acid. The copper ions arc displaced towards the
+ve o r copper electrode and are deposikd on the copper vessel.
There is thus no blanketing of the anodc with hydrogen and the
cell gives a steady current without polari sing. The e.m.f. is about

POROUS

-.

.-.
...

----- -- -- - .

COPPf R
CONTAINER
COPPER
SULPHATL

.
.

-.

.-

.. .

:1

..
-..

~-~

--

P
~
ACID

Fig 29
1.1 volts, and the example set out below shows how the energy
available and e.m.f. of a cell can be deduced. Of interest also are
the chemical formulae which explain the action in the cell.
Action at negative plate Zn + H,SO, = ZnSO, + H,
Action at positive plate H, + CuSO, = H,SO,
Cu

ENERGY AN D E . M . F . O F A D A N I E LL CELL. Any chemical action results in an e.m.f. For example that resulting in the formation of
ZnSO, (zinc sulphate) by dissolving zinc in sulphuric acid gives
rise to an e.m.f.which is exactly the same in value as the back
e.m.f. produced when the compound is electrolysed. The same is
true for any other metal such as copper dissolved in sulphuric
acid. Now when I coulomb of electricity passes through the
electrolyte 0400 338 gramme of zinc is dissolved or deposited.
Also whcn 1 grarnme of Zn is dissolvcd in sulphuric acid 6846
joules 01' heat are liberated. This inl'ormation can bc obtiiincd
from the appropriate chemical tables. Similarly 1 coulomb passing through Lllc clcclrolyle dcpositb or dissolves 0400 329 5
gramme of copper, and when 1 gramme of Cu is dissolved in
sulplluric i~cid3700 j o u l ~of~ hci~t; ~ r cliheri~lcd.
Thus the energy released during the passage ol' I coulomb
which dissolves the zinc in a Daniell cell = 0.000 338 x 6846 =
2.32 joules. Similarly the energy utilised during the passage of I
wulornb which deposits the copper in a Daniell cell =
0,000 329 5 x 3700 = 1.12 Joules.
Therct'ore the energy avniliiblc 1.01. driving 1 coulomb through
rtlc cell
2.32 - 1.22 :~1 . I ioulcs.
1

*fhe cflectivc ccll voltage will thercl'orc be I . I volts In order to


C X I ) C I I ~1 ~' 1 , ~ ( l k l l ~1 h' 0 ~1 1 1 ~ ' l > ~ l h h ~ 01'
l ~ C 1 ~ ~ ) l l ~ O111
l lCl~ ~C L~' I IL'II)'.
I

Thus the efl'ective e.m.f. of a Daniel1 cell is 1 . 1 volts.


T ~ I ILECLANCHI:.
:
CI:LL (Wet type). The diagram (Fig 30) shows a
cell used for supplying small amounts of electricity in remote
locations where cells of the dry type have no particular advantage. Up to quite recently such cells were used for railway
signalling in places where no electricity mains were available.

The +ve electrode consists of a carbon rod which is placed i n


a porous pot and surrounded by small pieces of carbon mixed
with powdered manganese dioxide (MnO,) which serves as the
depolariser. The pot is sealed with a layer of bitumen compound
o r pitch. The negative electrode is a zinc rod and the electrolyte
is a solution of ammonium chloride (NH,CI).
The action of the cell may be summarised as follows. Zinc ions
migrate into the electrolyte and when the external circu~tis
made, the further chemical action results In zinc being dissolved
by the electrolyte and zinc-chloride (ZnCl,), ammonia (NH,)
and hydrogen are produced. It is the conversion of zinc into zinc
chloride which provides the energy of the cell. The chloride and
ammonia dissolve in the water of the electrolyte and the hydrogen as ions migrates through the porous pot and reaches the
carbon. The hydrogen ions, while passing their charges to the
carbon electrode, combine with the manganese dioxide, taking
from i t some of its oxygen to form water. The manganese

dioxide is'thus reduced to a simpler oxide (Mn,O,), frequently


known as sesquioxide of manganese.
The solid depolariser is comparatively slow and polarisation
takes place if the cell, is used continuously. When the circuit
current is switched off, the depolarising action continues and the
cell can be used again after a little while. It is best suited for
intermittent duty such as bellringing. The e.m.f. is 1.55V, and the
following chemical formula defines the chemical action.
Action at negative plate :
Z, + 2NH,Cl = ZnC1,
2NH,
H,
Action at positive plate:
H, + 2Mn0, = Mn,O,
H,O.

n i l : L.I:(.I.ANCIIII
CEI.L. (Dry type). One form of construction is
illustrated by the diagram (Fig 31) which is a cross-sectional
view of a typical practical cell.
BRASS CAP

prcw

Fig 31

The depol:\~.iscrof mongancsc dioxide is mixed with powdered


carbon and packed round a central carbon rod. This whole
;~ssemblyis then placed in n linen bag which serves as the porous
pot of the wet cell. The negative electrode is a piessed zinc
cannister which contains the linen bag assembly and the electrolyte, which is made up as a paste of ammonium chloride (salammoniac), zinc chloride, flour and plaster of paris. One method
of closing the cannister is by sealing it with pitch. Because the
cell uses the same materials as the wet type its action is identical
and the chemical formulae as already given also apply.

The form of cell as described, is in most general service: but


otllcr li)r~nsl ~ i ~ vbrcrl
c dcvclopccl I'or incorpuruling inlo lllz lityes
type of battery, as is used for portable wireless sets. The reader
should complement the information given here by referring to
books specialising in the practical treatment of' electrical equipment.

THE SECONDARY CELL ( o r Accumulator)


Because of the importance of this cell as a means of storing
electricity ( i t is sometimes called a storage cell), the reader is
again advised to consult a book giving more details of modern
constructional methods, applications and maintenance requirements. The diagram (Fig 32) shows only the basic construction
n out only the elementary principles. The
and the d e ~ c n " ~ t i osets
modem accumulator uses 'pasted' plates to allow the maximum
use of the available material and the process involved in 'forming' the cell is somewhat involved for setting out in a book of
basic theory. It is hoped however, that the information given
below will provide sufficient knowledge to enable the action of
.
accumulator-to..be understood. Q e- -nickel-iron,
-....
the lead-acid
nickel-cadmium or alkaline battery also funcGons_.on s h i l a r
p r i n c i p l ~ ~ a l t h o u gthe
h plate materials and electrolyte 'differ.
ThZ-Se of cell is also important and should be thoroughly
investigated.
,-

1. CHASCE

2, DISCHAhCE

ot-

CHARGING
SUPPLY

I-

-- -- -- Fig 32
The simple accumulator consists of' two lead (Pb) plates Immersed in dilute sulphuric a d d , the whole assembly being
contained in a glass or moulded ebonite container. The cell has

to be worked into a suitable condition before it can be used for


storing electricity and the process is carried out by alternatively
'cliar_eing' and then 'discharging' the cell. If a d.c. supply is connected to the plates as in the diagram (Fig 32, switch posltion 1 )
and the cell is subjected to electrolysis by passing current
through it, oxygen and hydrogen gas is given off at the electrodes.
As for the water voltameter, the first stage of the reaction would
be decomposition of the acid (H2S04). A molecule of acid
dissociates to produce hydrogen ions and sulphions (SO,). The
sulphions move to the +ve plate, reacting with the water to
li)rin sulphuric c~cid: ~ n doxygen Tllc Ik~ttcrntti\cks (lie -t- ve plate
only to form lead-peroxide (PbO,), which causes the original
Ic;id clcctrqdc to :Issume :r dark brown colour. The hydrogen
3s io115 I S discharged at the cathode and liberated in the gaseous
state. The first chemical action is thus at the +ve plate only but
if the supply is switched off (switch-intermediate position), the
cell is now found to have the properties of a voltaic cell and will
provide an electromotive force.
If the cell is next short-circuited (switch-position 2), it will
behave !ike a primary cell, passing a current for a short time
during which period it discharges. The solution is electrolysed in
the reversed direction and the original negative plate now acts
as the anhde with its lead ions reacting with the sulphions of the
electrolyte to form lead sulphate. Thus at - ve plate Pb
SO,
= PbSO, (lead sulphate). The hydrogen ions from the electrolysis during discharge, move to the original +ve plate, now the
cathode. ~ h c h y d r o g e nnow reduces the lead peroxide to lead
oxide which in turn reacts with the acid to form lead sulphate.
Hz +
Thus at the +ve plate the chemical action is PbO,
H 2 S 0 , = PbSO,
2 H 2 0 . Both plates are converted into lead
sulphate and assume a whitish colour.
If the charging cycle is repeated (switch-position I ) , the
direction of' current flow in the electrolyte is again reversed and
tht. PhSO, on the - t v e plate becomes lead peroxide (PbO,).
'I'his i s a coti~plcxresult of' the electrolysis of thc acid. Sulpliions
move to the +ve plate, react with the water to form H 2 S 0 4 and
o a y g c ~'l'llc
~ . I;~ttc~.
; ~ t t ; ~ cthc
k s +vc plate to lbrln I'bO, and morc
sulphur~c acid. The chemical action at the f v e plate is
SO,
H 2 0 = HzS04
0 and 0 + H 2 0
PbSO, =
I'bO, -t H,SO,. At the negative platc, lead is produced by the
hydrogen ions liberated by the acid decomposition, moving to
this clrctrorlr :lnd reducing tllc Ic:d sulphntc to spongy lead. Tlic
t.licrnic.:~l:~ctinn; I I thc vc pl:~tci s :
I'hSO, + H 2 =. H Z S 0 4 + Ph.

81

EL EC TRO C HEMIS TRY

After a number of cycles of charging and discharging the


platcs bccomc porous und the ci~p:\cityo f the cell is increased.
When a cell is fully charged, chemical conversions are colnpleted and hydrogen is freely given off, resulting in 'gassing'the accepted term indicating a full charge.
The lead-acid accumulator in its practical form is provided
with 'pasted' plates Here the active material is app!ied to plates
in the form of a paste, the back-bone of the plate being a leadantimony grid. One 'forming' charge converts the paste into lead
peroxide on the +ve plate and spongy lead on the -ve plate.
Irrespective of the method of production the charge and discharge action can be summarised by the following chemical
equation.
Discharge ---------+
t
Charge
+ ve Pole
- ve Pole
+ ve Pole
- ve Pole
Lead
Sulphuric
Lead
Lead
Water
Lead
Peroxide
Acid
Sulphate *
Sulphate
PbO, + 2 ~ ~ 3 0 , + Pb = PbSO,
2H20
PbSO,
It will be seen that during discharge water is formed, thus
diluting and reducing the specific gravity of the electrolyte.
During charge, acid is formed and the tests to check a fully
charged cell include :
(1) S.G. of cell charged (1.20 to 1.27); discharged (1.17 to
I

1.18).

(2) ~ o l t i & eon open-circuit, cl?urged 252V txr cell or higher.


(3) 'Gassing' on charge.
(4) Positive Plate-rich dark brown colour. Negative Plateslate grey.
CAPA C ITY OF A CELL. This is the ampere hour figure it can yield on
a single discharge, until the e.m.f. falls to about 1.8V per cell (for
the lead-acid cell). Generally the capacity is based on a 10 hour
rate of discharge, since it decreases as the rate of discharge
increases. Research has shown that the performance of a cell can
also be improved by working the +ve plate at a higher current
density than the negative. This is achieved by keeping the plates
of equal area for convenience, and providing an extra -ve
plate, ie always making the outside plites negative.

be ex ~ressedin terms of ( I ) the


Ampere hour input and output, (2) the Watt hour input and
o u t ~ u tThus:
.
Ampc re hours of discharge
Ampere hour Efficiency =
Am >erehours of charge

EFFICIENCY OF A CELL. T ~ Imay


S

The ampere hour efficiency neglects the varying voltages


during charge and discharge. Since this is important, we thus
can have an energy efficiency compared to a quantity efficiency,
and
Watt hours of discharge
Watt hour Efficiency =
Watt hours of charge
Average
Discharge
Volts
x Amperes x
Average Charge Volts x Amperes x Hours

ours

Example 36. A battery is charged with a constant current of


16 amperes for I I hours after which time it is considered to be
fully charged, its voltage per, cell being recorded as 2.2V. Find
~ t amperc
s
hour efficiency if it is (1) discharged at a rate of 16
amperes for I0 hours, and (2) 28 amperes for 4 hours. In either
C:I';C d~sclit~
rgc w:is d i s c o l l l i ~ l ~ r cwllcn
d
thc vol tagc per cell fell
to 1.8V.

(1) Ampere hour Input = 16 x 1 1 = 176


Ampere hour output = 160
160
.: Efficiency = - = 0.91 o r 91 per cent.
176
(2) Ampere hour input = 16 x 11 = 176
Ampere hour output = 28 x 4 = 112
:. Efficiency = 112
-= 0.63 o r 63 per cent.
176
Example 37. A 12V accumulator is charged by means of a
constant current of 16A passed for 11 hours. The p.d. during
charging varies as shown. The battery is then discharged a t a
constant current-of 16A for 10 hours, the p.d. again varying as
shown. Calculate the Watt hour Efficiency of the battery.
Start

Reading No
1
2
3

4
5
6
7
H

9
10
II

Time

Charge
10.8V
1I .ov
1 l.5V
1 1.8V

12-ov
12.2v
12.4V
12.6V
12.8V
13.0V
13.1V

Discharge
12.6V
12.4V
12.2v
12.0v
1 1.8V
1 1.6V
1 1.4v
1 1.2v
11.ov
10.9V
10.8V

83

ELECTROCHEMISTRY

Efficiency =

1162

l 6 lo =
12.20 x 16 x 11 1342

0.867 or 86.7%.

CHARGING PROCEDURE. British practice uses the 'constant


current' method. This is also American practice but on the
Continent the 'constant voltage' method is favoured. For this
latter method the charging supply voltage is kept constant and is
substantially higher than the battery e.m.f. for the discharged
condition. The charging current is high initially but falls as the
back e.m.f. of the battery rises. Thls method gives a lower charging time than the 'constant current' method, but due to the
violent chemical action and heat generated in the battery there is
danger of 'buckling' the plates, unless the battery is specially
constructed.
For the 'constant current' method arrangements must be
provided for increasing the 'voltage applied to the battery as
charging proceeds and the back e.m.f. rises. If a generator is used
and Z is the charging current, R , the internal resistance of the
battery and E, the battery e.m.f. at start of charge, then the
applied voltage must be V = E b+ IR,. . . start of charge (1).
If E,, is the battery e.m.f. at endof charge, the applied voltage
would have to be V, = Eb,+ IR,. . . end of charge (2).
Thus subtracting (1) from (2) vuriittion of voltage would bc
V , - V = Ebl- Ebor thc applied voltage must be increased by
an amount equal to the rise of the battery e.m.f.
If a constant supply voltage is used for charging, then a
variable resistor is required to obtain the neqssary current
control, and its value will be reduced as charging proceeds.
Let V be the supply voltage, Z the charging current, Ri the
internal resistance of the battery, E, the battery e.m.f. at start of
charge and R the control resistor. Then:
V = E b + ZR,+ ZR . . . start of charge (1)
If Eb,is the battery e.m.f, at end of charge and R , the new
value of the control resistor. Then :
V = Ebl ZRi+- JR, . . . end of charge (2)
Subtracting (f) from (2) 0 = E,, - Eb + IR, - IR
or (E,,- E,) = I(R - R,).Thus the control resistance
must be reduced from R to R, as the battery voltage rises from
Eb to Ebl.
Example 38. A 24V enlergency battery IS to be charged from
the 110V ship's mains v hen the e.m.f. per cell has fallen to a
C

minimum value of 1.8V. The battery consists of 12 cells in series,


has a capacity of lOOA h at a 1Qh rate and the internal resistance
is 0.03R/celI. If charging continues until the voltage/cell rises to
2,2V, find the value of the variable resistor needed to control the
charging. The charging current can be assumed to be equal to
the maximum allowable discharge current.
Discharge current =

10

= 10 amperes = charging current

At start of charge, battery voltage = 12 x 1.8 = 21.6V


Battery internal resistance = 12 x 0.03 = 0.36Q
Then 110 = 21.6

+ (10 x 0.36) -t (10 x

R)

At end of charge, battery voltage = 12 x 2.2 = 26.4V


Then 110 = 26.4 + (10 x 0.36) + (10 x R,)
or R1,= 110- 30 - 80 = 8 ohms.
10
10
Thus the variable resistor should have a value of 8.48 ohms
and be capable of being reduced to 8 ohms. In practice a unit of
9 ohms would be used which would be reduced by adjusting the
sliding contact un ti1 the charging ammeter recorded the correct
current. Further adjustments would be made periodically as
charging proceeds. It is important to note that besides the
ohmic value of the resistor, the wattage rating must be specified.
For the unit in the example, a rating of 1 2 =~ lo2 x 9 = 900
watts is required. The control resistor must be capable of dissipating up to this power as heat during the charging, although
this waste of power will decrease slightly as charging proceeds.
For example at the end of the charge the power wasted in the
resistor would be lo2 x 8 = 800 watts.
The most important point to stress is the correct connecting
up of thc butrcry for clltrrging, io vc tcrminal of battery to vc
of mains; -ve terminal of battery to -ve of mains. It is
.;urpristng how tnc~ny1i111cslllis olclllcntctry rcquirctnenl is overlooked through carelessness. For incorrect cgnnection, no
control of the current would be possible with the equipment
provided and damage of the ammeter, control resistor, o r
battery could result.

ELECTROCHEMISTRY

85

CHAPTER 4
PRACTICE EXAMPLES

1.

An accumulator is charged at the rate of 6 amperes for


18 hours and then discharged at the rate of 3.5 amperes for
28 hours. Find the ampere hour efficiency.

2.

The mass of the cathode of a copper voltameter before


deposit was 14*52g,and after a steady current was passed
through the circuit for 50rnin, its mass was 19.34g. The
reading of the ammeter was 5.1A. Find the error of the
ammeter, taking the E.C.E. of copper as 330 x 10-?kg/C.

3.

, A 90V d.c. generator is used to charge a battery of 40


cells in series, each cell having an average e.m.f. of 1.9V and
an internal resistance of 0.0025R. If the fotal resistance o f
the connecting leads is l R , calculate the value of the
charging current.

4.

Nicke1.S~to be deposited on the curved surface of a shaft


lOOmm in diameter and of length 150rnm. The thickness of
deposit is to be 0.5mm. If the process takes 8h, calculate the current that must flow. The E.C.E. of nickel is
302 x 10-9kg/C. The density of nickel is 8600kg/m3.

5.

A nickel-alkaline battery is discharged at a constant


current of 6A for 12h at an average terminal voltage of
1.2V. A charging current of 4A for 22h, at an average
terminal voltage of 1.5V is required to re-charge the battery
completely. Calculate the ampere hour and watt hour efficiences.

6.

A battery of 80 lead-acid cells in series is to be charged at a


constant rate of 5A from a 230V, d.c. supply. If the voltage
per cell varies from 1.8 to 2.4V during the charge, calculate
the maximum and minimum values of the required control
resistor. If the ampere hour capacity of the cells is 60, state
the probable charging time required, assuming that the cells
were in a completely discharged condition at the commencement of the charge.

7.

A metal plate measlrsring:5@nrn ,by 150rnm is to be copperplated in 30min. Calculate the current required to deposit a

thickness of 0.05mm on each side (ignore the edges). The


E.C.E. of copper is 330 x 10-gkg/C and its density is 8800
kg/m3.

8.

A battery of 40 cells in series delivers a constant discharging current of 4A for 40h, the average p.d. per cell
being 1.93V during the process. The battery is then completely recharged by a current of 8A flowing for 24h, the
average p.d, per cell being 2.2V. Calculate the ampere hour
and the watt, hour efficiencies for the battery.

Thirty lead-acid accumulators are to be charged at a constant current of 10A,.from a 200V d.c. supply, the e.m.f. per
cell at the beginning and end of charge being 1.85Vand 2.2V
respectively. Calculate the values of the necessary external
resistor required at the beginning and end of charge,
assuming the resistance of the leads, connections, etc to be
1R and that thz internal resistance is 0.01R per cell.

10.

When a current of 3.5A was passed through a solution of


copper sulphate, 4.2g of copper were deposited. If the E.C.E.
of copper is 330 x 1 0 - ~ k g / Cand the chemical equivalent
of copper is 31-8, find the time for which the current was
passed through the solution and also the mass of hydrogen
liberated.

CHAPTER 5

MAGNETISM ELECTROMAGNETISM
NATURAL MAGNETS

From very early times it was known to ancient civilisations,


such as those of the Greeks and Chinese, that pieces of certain
types of iron ore have magnetic properties. Pieces of the ore were
known, not only to be capable of attracting and repelling other
such pieces but could also pass on this property of magnetism.
One further known fundamental property of a piece of the ore,
called Magnetite or Lodestone, was that if it is freely suspended,
as shown in the diagram (Fig 33), then it would come to rest in
an approximate geographic North-South direction. The end
pointing north is called a north-seeking or simply a North Pole,
whilst the other end is a South Pole. The piece of ore constitutes
a natural magnet and if brought into contact with a quantity of
iron filings, these would be found to adhere mainly to its ends
or poles.

Fig 33
Further simple investigations made with pieces of the magnetic
ore would show that, if two such magnets are each suspended as
described above and their polarities are determined and marked,
then when the N pole of one suspended magnet is brought near
the N pole of the other suspended magnet, repulsion of the poles
will result. Two S poles brought near each other would behave

11

in a similar manner whereas, a S pole brought near the N pole of


the other magnet will produce an attractive effect. Thus every
magnet is seen to have two poles of unlike polarity and that like
poles repel whereas unlike poles attract.

ARTIFICIAL MAGNETS

A piece of iron can be converted into a magnet and made to


exhibit properties similar to that of the iron ore described above.
Such a piece of iron is an artificial magnet and is said to be
rnagnetised. A simple method of magnetising a specimen is by
stroking it in one direction from end to end with one pole of an
existing magnet, but the most effective method is by electromagnetism, which will be considered later in the chapter.
Ccrltlin mi~tcrii~ls
such us copper, aluminium, Icad, brass,
wood, glass, rubber, etc cannot be magnetised. Thus all known
materials can be classified under the heading of magnetic o r
nonmagnetic substances. Some metals such as nickel, cobalt and
magnesium exhibit very slight magnetic properties, but it is of'
interest to record that, when alloyed with iron very strong
magnetic properties result.
An artificial magnet is usually made in bar o r horse-shoe form.
When tested, the tips are found to constitute poles of opposite
pojarity andjif suspended, a bar magnet will lie on an approxima,re N-S line. The magnetic compass makes use ofthis principle
and ,consists of a short highly magnetised bar magnet which is
pivoted at its centre. A card, calibrated in degrees and/or
geogrdphic points, is mounted below and is used with the magnet
to obtain a 'bearing'. It is necessary to mention here that the
N-S direction as indicated by such a compass is not exactly
geographic N and S. The angle between the lines of magnetic
and geographic N- S, is called the 'variation' and varies for
different parts of the world. If' the magnetic compass is being
used, due allowance must be made for the variation, bef'ore a
map can be truly oriented and used correctly.
Ikl;rrr.
o~*r.ccli~lg
will\ I'UI.IIICI' s[udy 01' magnetism, i t would
be as well to explain why a compass needle lies in the N-S
dircction. Thc c;lrth itscll' hcl~;~vcs
;IS tl~oi~gli
i l conl;lins ;I rnilgncl
having 11sS pole in the region ol'the geographic north and its N
pole near the geographic south. A compass needle placed on the
earth's surface will lie so that its N pole will be attracted to the
magnetic south (geographic north) pole of' the earth and its S
pole will be attracted lo the magnetic north (geographic south).
I:u~.ll~cs
~ ~ i e ~ i t01'i oI I~I C l ~ ; ~ l . t l l ' 111;1gnclisrn
~v
will bc m;~dclater o n
111 t l i i x c~Ii;~ptcr

I
I

89

M A G N E T I S M , ELEC T RO MAG N ETIS M

Summarising the facts deduced so far about natural or


artiticial magnets, we know that every lnugncl has two poles 01'
unlike polarity and that like poles repel whereas unlike poles
attract.
THE MAGNETIC FIELD

This is the space around a magnet where its magnetic effects


can be felt. If a bar magnet is covered by a sheet of paper and
iron filings are sprinkled on the paper, then on tapping the
latter, the filings would be seen to align themselves as shown in
the diagram (Fig 34). The filings would form a pattern which, if
examined closely, would show that lines could be traced from
the N pole of the magnet to the S pole through the space outside
and from the S to N poles inside the mamet

-2
PATHS Of LlNtS Of FLUX
7RACkD
OUT 81 IRON F I L I N G S

Fig 34
The field can also be plotted by using a small compass needle
as shown in the diagram (Fig 35).

Fig 3 i

Field plotting with the aid of a compass needle is undertaken


as follows. Place the magnet on a sheet of' paper and draw its
outline. Set the compass needle against the N pole of the magnet
and, with a pencil, mark a dot at the point in line with and
adjacent to the N pole of the compass needle. Move the compass
until the S pole of the needle is coincident with the original dot.
Mark the new point in line with and adjacent to the N pole of'
the needle. Repeat this procedure until the S pole of the magnet
is reached. Join the dots together to give a 'line of force' or a
'line of J r t x ' which can be described as the line which, when
drawn tllrougli any point in a magnetic field, shows the direction
of' the ~ilagneticforce at that point. Using the compass needle
the field can be mapped out for a considerable distance around
:I IllilgllCt i1tlC1 t l ~ cI ' O I I U W I deductions
II~
can bc made.
1 . Lines of' flux never cross.
2. The lines are always continuous.
If various magnetic field arrangements are plotted as shown in
the diagram (Fig 36) then other conclusions can be deduced.
3. Lines of flux are like stretched elastic threads and tend to
shorten themselves. This explains the attractive effect between
two unlike magnetic poles, which if fiee to d o so will move into
contact, thereby reducing the length of the lines of flux.

MAGNETISM, ELECTKOMAGNETISM

91

4. Lines of' flux which are parallel ; ~ n din the same directton\
repel each other. 'This deduction is clearly seen tor the condition
where two magnets are brought together, with like poles adjacent
to each other. There would be a force of repulsion between the
magnets and if the field is plotted between two like poles a
neutral point would be found where the effects of the two
repulsion forces balance each other and the total effect is as
shown by the absence of control on a compass needle placed at
this neutral point.
The strength of the magnetic field around a magnet will vary
from point to point,. but before this can' be measured and
methods devised for making such measurements, a system of
magnetic units and terms must be introduced. Faraday conceived the idea of the line of flux, as already introduced, and
further suggested the use of these lines to depict the strength of
the magnetic field.
If a unit area at right angles to the lines of flux is considered
then further definitions and terms can be introduced.
A number of lines of flux collectively are said to constitute the
magnetic Flux (symbol @-Greek letter phi) which is passing
through the area being considered.
Another unit of importance is Flux Density-and the value,
at any point, is obtained from the expression:
Flux
Flux Density = -Area
The diagram (Fig 37) illustrates the SI unit of flux or the
Weber. Thus if' 50 lines of flux are shown passing through the
p

area of 1 square metre, then for the plane being considered, the
magnetic flux is 50 Webers. The symbol for flux density is B and
the unit is the Tesla. Thus for any point P in the plane being
considered, the flux density is 50 teslas.
Note. The tesla is a new name introduced for the SI system.
The original unit was the weber per square metre ie Wb/m 2 .
We now have Flux = Flux Density x Area
or @ (Webers) = B (teslas) x A. (square metres).
The above relationship will be used continually when the
study of electromagnetism and magnetic circuits is made and
should be considered a basic and important formula. It is well to
stress here the obvious, namely that lines of flux d o not exist but
the properties of magnets and magnetic fields can best be
assessed by assuming their existence and their having definite
physical properties. It should also be remembered that the field
of a magnet exists in all directions and is not confined to one
plane. A fuller understanding of magnetic theory is rewarding
to the engineer, but there is little space here for a more complete
treatment of the subject. The basics however are summarjsed a s
follows.
MOLECULAR THEORY OF MAGNETISM

A molecule is defined as the smallest particle of a substance


that can exist separately and in any magnetic material every
molecule is thought to be a complete magnet. In a piece of
un~llagnet~sed
pagnetic material the molecules are considered to
arrange themselves in closed magnetic chains o r circuits as
shown in the diagram (Fig 38). Under this assumption i t js
considered that each mo.lecular magnet is neutralised by adjacent

molecul;ir m:ignets so that no magnetism is apparent in the


material. The process of magnetising the material is considered
to be ;~cli~cvcd
by simply arranging the molecular magnets so
that their axes point in the direction of' the magnetising f'orcc.
1 ' 1 1 ~~ ~ ~ ~ o ~ ~I \l\ ' ~o ~l ' ~~ l ~li silYsc~ IsI ~~ I ~N ~ Iby~ ( t~l ~C e~lY)llo\+~i~~g
know11
ohscrv;~lion\ ( I ) Tllcre i.; :I limit to the mount 01' magnetism
Illat C ; I I I I>c I I I I ~ ~ I i tI1 . any
~ ~ ~orlc s;~mplcol' matcl.ial. l'I11s I S

MAGNETISM, ELECTROMAGNETISM

93

explained by the supposition that, once all the molecules had


been 'lined up', no amount of extru mt~ynctising force ctln
increase the strength of the magnet. (2) When a magnet is broken,
the ends of' the moleculur mugnets are exposed und the broken
pieces are found to be magnets themselves. (3) If heated to about
100C and allowed to cool a magnet is weakened. If the.magnet
is heated to red heat, the magnetic properties are lost altogether.
Similarly if a magnetic material like hard steel, is cooled in a
strong magnetic field then it will set as $ permanent magnet.
It is considered that during heating, energy is transferred to the
magnet which causes oscillations of the molecular magnets which
tend to break the 'lining up' and results in these magnets taking
up random directions. Similarly for the cooling process, as
energy is passed from the hot material, the oscillations decrease
in magnitude and violence and the molecular magnets are
allowed to settle in the direction of the magnetising field.
A more modern.theory of magnetism is b&ed on the electron
theory and the conception of the atom. A chapter considering
the electron theory in detail is introduced at a later stage but
here it can be stated that an electron, the smallest known 've
charge, when rotating in an elliptical path, constitutes a circular
current which sets up a magnetic force along the axis of gyration.
'In a molecule the magnetic effects of the electrons of the atoms
may neutralise each other giving little resultant effect. Again a
spinning electron also sets u p a magnetic field along its axis of'
spin. If the fields due to the etyects of spin balance out, due to
electrons spillning in opposite directions, then the material is
non-magnetic. A magnetic material is the result of' the fields not
balancing out, but to explain the overall apparent effect, i t is
thought that rather than single atoms or molecules being concerned, it -is a group of molecules which act together. Such a
group is called a 'domain' and is considered to function like the
more elementary molecular magnet already described.
ELECTROMAGNETISM
Earlier theory has referred to an association between magnetism and electricity and this was more specifically mentioned in
Chapter 2 when the electrical units were defined. The discovery
of a relation between an electric current and magnetism was made
in 1820 by the scientist Oersted, when he accidentally noted that
a wire arranged above and parallel to a compass needle, caused
deflection of the latter when a current was passed through the
wire. Reversal of the current caused a reversal of the deflection.
Further experiments on the shape, direction and strength of the

MAGNETISM, ELECTROMAGNETISM

95

FIELD

Fig 40
mum on the circumftrence. Outside the wire the flux density
varies inversely as the distance from it.
The diagrams (Figs 40a and 40b). make use of: the conventional method of' indicating current direction. Consider an
arrow ie current entering tlic surfiice of' the paper and receding
I'rom tlic vicwcr, thcn tllc I'c;~tllct~ctl
crltl w o t ~ l ~hc
l 4ccn '1'I11\
would be shown with a cross. Similarly current flow low;il.db thc
viewer would be shown with the tip ot' the iirrow it. a point 01.
dot. The relation between the direction of' the lines of' f u x a n d
the current is best summarised by Maxwell's 'Right-Hand
Screw' Rule. This depicts the association that, if current flows
in the direction in which a right-handed screw moves forward
when turned clockwise, then the resulting field yill be in the
direction of' turning the screw. I f the current is reversed, the
screw should be unscrewed and the field would be reversed, o r
would be in the direction of'turning the screw ie anti-clockwise.

(2)

FIELD D U E TO A C U R R E N T - C A R R ) ' I N G C O N D U C T O R BENT T O


FORM A SINGLE L O O P

The diagrams (Figs 41a and 31b) show the loop,, tl3e current
and the lines of flux which tend to encircle the conductor a s
deduced from condition, ( I ) above. The resulting field can be
plotted by locating the loop in a sheet of' cardboard as shown.
The result can be considered as the field taken thrbugli the
section XY of the loop and the similnl-ity wit11 the field of a s1iol.t

94

REED'S BASIC ELECTROTECHNOLOGY

magnetic fields associated with current-carrying conductors


arranged in the form of loops and solenoids were the subject of
much work by famous scientists such as Faraday, Maxwell and
Gilbert. The lesults of the discoveries made then led to the
deduction.of certain fundamental relationships which are now
part of accepted basic theory. The shape of the magnetic fields
due to simple arrangements of current-carrying conductors will
now be considered.

( 1 ) FIELD DUE TO LONG STRAIGHT CURRENT-CARRYING CONDUCTOR


The field associated with such a conductor may be determined
by the use of' iron filings o r a compass needle as was described
earlier in the section on magnetism-Figs 34 and 35. Assuming
tliat thc currcnt is kept constant during such a test, a field consisting of concentric lines of' flux would be confirmed. The
diagram (Fig 39) shows a vertical wire passing through a sheet of
cardboard. The directions of' the current and lines should be
particularly noted since this is fundamental knowledge.

CURRENT

Fig 39
Fk~rtlicrtests would s h o w that if' thc current is reversed, thc
liclcl wuuld s c v c i x and I ( ~ l r chtrc~lg[ll01' ~ h cliolcl was ~ucasu~.ed

b) an appropriate sensitive instrument, then consideration of'


the results would give a graph as illustrated by the diagram (Fig
40a), which shows 1-lux Density ( B ) plottcd to n base ofdistance
( A ) I'rnm thc centre 01' thc conductor.
I t will be seen that inside the circular conductor, the strength
01' licld 01. Ilus dcnsi[> vuric.; I'~.omzc1.o at thc centre to ;I maxi-

,
bar magnet will'be recognised. Thus the loop can be considered
to set up a magnetic polarity which can be determined from first
principles.

AXIS

U U
t

4 f

CURRENT
AWAY FROM
OBSERVER

(b)
Fig 41
.L

(3) FIELD

DUE TO A CURRENT-CARRYING CONDUCTOR WOUND AS A

SOLENOID

The next logical step in electromagnetic field investigations is


!'or a coil of wire, which is basically a collection of' several loops.
A solenoid is a form of a multi-turn coil where the axial length
is much greater than its diameter. The turns of wire can be
wound in an open spiral o r placed close together so that they
totlcll, pl.oviJcJ i n s i ~ I ; ~ ~wire
c d is used. The insu1;ition most
commonly used is either one of the modern synthetic enamels o r
;I f i h r n ~ ~
tn:ltcri:ll
\
s\~clias cotton o r silk in the f'orm o f thrc;ld.
tape or braid. 'fhe turns ol'a solenoid may be arranged in several
layers provided the current travels through the turns in the same
direction. When the field is investigated by plotting with a
compass, i t is found to be as shown in the diagram (Fig 42). I r
will be seen th;~t; \ I 1 thc turns tend to produce ;\ mngnctic field i n
tlie same direction. so that this can be deduced by considering
tllc liclcl 01' ;I .sir~glclu1.11or lool>. TI1c t 1 l r . n ~ unitc lo send ;I

MAGNETISM, ELECTROMAGNETISM

97

straight field up the centre which comes out at the ends, opens
and spreads out to return to the othcr cnd, giving the surnc
distribution of lines of flux as would be obtained from a bar
magnet.

L_

-3

9 =,l,,oh~A'?
-

attributed to the solenoid


when carrying current. The polarity can be detemined by finding
the direction of the lines for any one turn by applying the RightHand Screw Rule but additional aids are useful, the easiest of
which being the Right-Hand Rule. This is explained as follows,
and is shown in the diagram (Fig 43). Place the right hand on the
coil with the fingers pointing in the direction in which current
flows. Then the thumb will point in the direction of the N pole.

RIGHT HAND

Fig 43
INTRODUCTION OF AN IKON COKE

By iron is meant, at this stage, a magnetic substance. The iron,


if made the core of a solenoid, strengthens the field by concentrating the flux and more clearly defining the poles. A magnetic core appears to allow the passage of flux more readily
than does air. The reason for this will be introduced in later
studies, but experiment shows that the most perfect type of flux
path is where the whole of the magnetic circuit is formed from
magnetic material. Where this is not practicable the air gaps o r

air paths are kept as short as possible and good examples are
found in the electromagnetic paths for the flux in the electric
bell and the electric motor or generator, as illustrated by the
diagram (Figs 44a and 44b).

.Electromagnets are preferred to permanent magnets in


industry f'or two main reasons. (1) They can be made more
powerf'ul $anp_e_rmanent magnet% by providing the desired
m m s i n g force, ir solenoid coils with sufficient turns and
energis~ngcurrent. (2) The m x n e t i s m can be~controlled,ie i t
C1n77c s\\;itil~cdon and off' or varied gradually by controlling
the current. Altllough mucll more will be said about the material
u w d for the core of' electromagnets. general practice can be
~ ~ ~ l l l l l l ll\L!~l
.ll

~ l l l l ,\ ~

Permanent magnets are made of llurd s t e e l because tI11s


n<ifcri:14
retains its rnac*.
The material is said to have a high
'1.etentivit7
Elcctrornagncts Iiacc n core of' .roJi rr.011 a s i t is more
m;?~nt.tiscdhut Ioscs i t 5 ~n:~gnctic~npertiesjn_or_e
quickly. TIie
-------.---y.natc~.i;~l
is s l i r l to Ii:~\c;I I I I ~ I I 'si~sccptibility'.Tlii~ssort iron is
I I ~ ~ \~ ~l l~ \Cc c ~ ~ ttl ll, lll l~ \lCL!l
l~

MAGNETISM, ELECTROMAGNETISM

99

CONDllcrrll~IN A MAGNliTlC 1,ltil.D


Oersted's experiment with the compass needle and currentcarrying wire showed that, a force must be produced when the
current is switcllcd on, to bring about deflection of' the needle.
Converse action can be assumed, ie if the needle was fixed and
the wire was sufficiently flexible, then movement of the latter
would be noted when the current was switched on. Further
investigations, to discover the nature, action and magnitude of
the effect detected, lead to an accepted fundamental-that a
force acts upon a conductor wheo it is carrying current and
situated in a magnetic field provided it is lying at right angles to
the lines of flux. The importance of this fundamental truth
requires fuller consideration and it would be useful to reconsider the electromagnetic effects which allow the ampere to be
defined as a fundamental unit of the SI system.
In Chapter 2 the phenomena leading to the aefinition oPthe
ampere were mentioned and the points made previously are
revised here in the light of electromagnetic theory. If a circuit is
supplied through two wires laid together, then especially if the
current is large and the wire flexible, a mechanical effect would
be noted. This would be particularly noticeable when the
current is switched on and off, since the wires would be seen to
move. The action noted.can be explained with our knowledge of
the field associated with a long straight conductor.
Consider the diagram (Fig 45), which shows two conductors
carrying current as shown. When the current in both conductors
is in the same direction then the resultant magnetic field is such
FORCE ON A CURRENT-CAKHYING

NEUTRAL
AREA

Fig 45

as to embrace both conductors. If the current in each conductor


IS of the same magnitude then, by Ma.xwel19sright-hand screw
rule, the fields between the wires will cancel and the outside lines
of flux will unite to make a field which encircles both conductors.
If the lines of flux are likened to elastic threads, tending to

MAGNETISM, ELECTROMAGNETISM

101

side. Thc lines of' flux ;ippe;lr to bc strctchetl ; ~ n din tending to


return to their shortest length, :I I'orce is exerted on the conductor
pushing i t out of'the wily i\s shown. This i~ctionf o r ~ n sthe h;~sis
of' operation of the electric motor and the reader should pay
careful attention to the points being madi here.
It can be shown that the force acting on the conductor varies
directly with ( I ) the strength of'the magnetic field (2) the strength
of the current in the conductor (3) the length of the conductor in
the magnetic field.
Summarising: Force cc Strength of field x Current in
conductor x length of conductor in the magnetic field
or F a BII where F is the force on the conductor in newtons,
B is the flux density in teslas, I is the current in amperes, I is the
length of the conductor in the field in metres.
The above relationship can be converted to the expression
F = BII if the correct unit of flux density is chosen to allow this
equality. This unit is the tesla and is defined below. Thus we
have the important formula :

F (newtons) = B (teslas) x I (amperes) x 1 (metres).


It will be noted from now on that the ex~ressionflux
will be used to an increasing extent in preference to strength of
magnetic held. This is because, if the idea of using lines of flux
to depict a magnetic field is accepted then. the strength of' t h e
magnetic field ciln bc rcprcsentcd by tlic clcnsity of' tlic linc4
Lines well spaced apart would suggest a weak field, whereas a
strong field could be represented by lines closely packed together. Thus field strength can be measured by the density ol'
these lines o r by the flux density which in turn can be defined in
terms of other accepted SI units.
UNIT OF FLUX DENSITY

This unit is readily defined in accordance with the relationship


F = BIl because the units fbr F, I and 1 are already known.
Thus B is defined in terms of the other three factors and the u n ~ t
of flux density or the tesla is the density of magnetic field such
that a conductor carrying 1 ampere at right angles to the field
experiences a force of 1 newtonlmetre length acting on it.
UNIT OF FLUX

The terms fiux and flux density were introduced earlier in


this chapter when it was mentioned that the expression flux
density was determined by dividing the total flux by the area
through which it passed. Thus:

I
!

1 04

REED 'S BAS IC ELEC TROT EC HNO LOG Y

so as to produce a continuous solenoid effect. From the theory


point of view the magnetic circuit would be unaffected if both
coils were placed on one pole and connected in series, but the
practical arrangement gives the more symmetrical'layout. Consider each coil to have 2000 turns of thin wire and the coil
current to be 1.5 amperes. Then the totul magnetising force
producing the flux for this machine would be (2000 x 2) turns
x 1.5 amperes o r 6000 ampere turns. By symmetry the flux
through the poles and armature splits (Fig 47a) and returns
through both halves of the yoke of' the machine. This distributi011 01' f u x can be seen to be identical wit11 the arrangement in
the diagram (Fig 47b), where all the flux returns through one
yokc --a lay-out much used in the earlier types of electrical
machines employing a horse-shoe shaped electromagnet system.
From the work done so far it will be noted that we consider a
certain flux density to exist at a point by virtue of a magnetising
force producing it. An analogy can now be made with the
electrical circuit and such reasoning will allow a clearer understanding of the magnetic circuit and associated problems.
MAGNETOMOTIVE FORCE O R M.M.F.

A complete path is followed by a group of lines of magnetic


flux and this path is called the magnetic circuit. In an electric
circuit current'is due to an e.m.f. a a d i n a magnetic circuit, flux is
considered to be due to a . magnetomotive force (Symbol F)
caused by current flowing through a coil,of wire. Thus the !n.tnf.
is the total ~nagfitisingf'orce produced by a solenoid coil and is
measured in ampere-turns (IN).It is stressed at this stage that,
from now on, the terms magnetising force, magnetic field
strength o r magnetic field intensity a r e used for the force o r
m.m.f. acting over one metre length of the circuit and that the
total force for the circuit is called the magnetomotive force.
Thus magnetising force H is the m.m.f./metre length

--

.I'IIc I ) ; I \ \ ; I G C 01' ~ I I L . I1t1x I I ~ I ~ O L I -~ ,-(lie


I I -.I I I ; I ~.-I T c.irc\l$
~- I ~
I S con.
\~deredto be rebtrlctcd by the rc,llrc,furrc,c,of the circuit. lieluctancc (symbol S) cirri be likened to the resistance in an electrical
circuit and is found to bc proportional to the length of the
magnetic
circuit and inversely-proportiongl & t h e ar+ and the
.I ~ ~ ~ ~ . ~--t ~-w t ; i 3pl.Eii
~ j 1
tlc;\l
1 ~h Xi yet
~ ~to be said about
Ik~.~K7Cfi~l~ty.
but 1'01. tlic 111omc1ltO U I . analogy can be sct out in
~ < I I ) L I I ~ I1 I0 1 I I I , I \ I ) C I O L V ,
--

--

-y--

105

MAGNETISM, ELECTROMAGNETISM

Electric Circuit

--I

Quantity

Unit

e.m.f. ( E )
Current (I)
Resistance (R)

Volt
Ampere
Ohm

Magnetic Circuit
Quantity

Unit

m.m.f. ( F )
Ampere-turn
Flux (@)
Weber
Reluctance(S) Amp-turn/
Weber o r
AjWb
Also

~lso

Other Comparables are


Electric
Force ( 8 )
E
-1
Current
Density (J)
I
- A
-

./- -

Volts/metre

Ampere]
metre 2

_ ._I_.

Magnetising
Force ( H )
F
- 1
Flux
Denslty (B)

. ,

Amp-turns/
metre
Tesla

- @
-

The above conception of the magnetic circuit also allows


formulae to be deduced, for the magnetising force in the fields of
various current-carrying conductor arrangements, such as the
long straight conductor, single loop and multi-turn coils like
solenoids and toroids.
FORCE, MAGNETIC FIELD STRENGTH OR lNTENSlTY OF
LONG, STRAIGHT, CURRENT-CARRYING CONDUCTOR
Consider the arrangement shown in the diagram (Fig 48a).
The conductor is assumed to be of infinite length, to be situated
in a vacuum and to carry a current of I amperes. The conductor
forming the return path to the supply source, is considered to be
a t such an infinite distance away that its current will unaffect the
magnetic field near the conductor being considered.
The conductor arrangement constitutes a single turn and the
m.m.f. F i s then I turn x Iamperes o r F = lampere-turns. Consider any point on a line of' flux distant r metres from the centre
of the conductor. Then the magnetising torce H a t this point will
MAGNETISING
A

F
F
be the m.m.f./metre length of flux path or H = - = I
2xr
I
whence H = -- ampere-turnstmetre o r amperes/metre.
2xr
The above result can now be used to find the flux density
resulting from a certain magnetising force and the permeability
( p ) of the medium in which the field is established can . b e
deduced. Briefly at this stage permeability can be defined as the
ratio of the flux density in a m e d ~ u m
the~ mxnetising force
fhducing i t . The value o f t h e permeability for freespace @,)is
iicxt deduced but ~t is found that there is no appreciable difference for this value whether it be for a vacuum: air o r any other
non-magnetic material.

--

- /

Fig 48
(b)
P ERM E A BI L IT Y OF F RE E S P A C E (or the Magnetic Space Constant)
The diagram (Fig 48b) shows Fig 48a modified to suit further
reasoning. The conductor, in a vacuum, is represented by A and
now carries a current of' 1 ampere which flows away from the
observer. The magnetising force, at any point P distant 1 metre
(a)

from A is given by H

1
= -

2x

ampere-turnslmetre since both I and

; ~ r cunity in the formula derived earlier.


Ncst ;~ssumctllc tlux density at point 1' to be R tcsla. Then
( 2 ) the force on ;I metre length of conductor placed at P, parallel
I0 A illrd ~ill'lyillg11 ~ L l l ' l ' d l l l 01' I illlll>Cl'C ~ ( > l l l d
2 X i0-'
newtons. T h ~ is
s known I'rom the definition of'the ampere. Also
( b ) the force on a metre length of this conductor is given by
R I I newtons. o r is
B (teslas) x l (ampere) x l (metre) = B (newtons).
INIS IS equating ckpl.cssions ( ; I ) i ~ n d(h) f'or the t'orce on the
condi~ctorwe sec t1ie:t the value o f ' B fbr the condition considered
woulcl be 2 x I0 ' I I C W I O I I S i111d

r.

107

MAGNETISM, ELECTROMAGNETISM

Flux density at point P-

-B
4

MagnetisingforceatpointP

-.

~ -X I O - '

1 /2n= 4x x 10-'

SI units

of free space-symbol I(,


Ampere-turnsirnetre for air = Flux Density
4x X lo-'
Example 41. It is required to produce a flux of 0.018Wbacross
an airgap 2.54mm long and having an effective area of 24 x 10square metres. Find the ampere-turns required.
Area of Gap = 24 x

square metres -I a = 0-75T


Required flux density B = 0.018
24 x
24,

The length of the air gap = 2.54mm = 0.254 x 1 0 - ~ m


So total ampere-turns needed = 59.7 x lo4 x 0.254 x
= 59.7 x 0.254 x lo2
= 15.15 x lo2 = 1515At.
Example 42. A wooden ring having a mean diameter of 200
mm nnd u cross-scctionul urea o i 400mmJ is wound uniformly
with a coil of 300 turns. If the current passed through the coil is
5A calculate the value of flux produced in the coil.
The m.m.f. of the coil = 5 x 300 = 1500At
The mean circumference = ~c x 200 = 628mm = 0-628m
1500 - 15 x lo2
The magnetising force H = Atlm = -0.628
0.628
The flux density B = p o H
= 4 x x x lo-' x 2380
= 1.256 x 2.38 x l o p 3 = 0.003T
Total flux @ = BA
= 1.2 x 1 0 - W b
= 0.003 x 400 x
= 1.2pWb.
Example 43. The magnet system of a moving-coil instrument
provides a flux density in the air gap of 0.25T. The moving coil,
of 120 turns, is carried on a former of (active side) length 25mm
and width 18mm (between air-gap centres). If the coil carries a
current of 2mA, calculate the turning moment on it.

F = BIl newtons
= 0.25 x 2 x
x 120 x 2 x 25 x
= 0.25 x 2 x 2 x
x 120 x 25 x
= 30.0 x
= 3 x 10-3N
Torque = F x radius of' coil
=3
10-3
9
10-3
= 27 x l o v 6 newton metres = 27pN m.
The subject matter covered in this chapter has dealt with
sufficient basic theory, terms and relationships to allow the
general study of Electrotechnology to proceed. Regarding the
magnetic circuit, u p to riow, only nir o r non-magnetic material
has been considered for the medium of the magnetic field
i ~ s s o c i i ~ l ~witti
c l nn C ~ ~ C I ~ O I I I The
~ I ~ nccd
~ C ~ .for introducing il
magnetic material into thc circuit to providc a well defined flux
path, will require extensions of' our investigations into the
magnetic circuit, which can be made more conveniently at a
later stage. Chapter 12 will be found to cover the more chmposite
magnetic circuit and the further aspects of ferromagnetism.

M A G N E T I S M . E L E C T R O M A G N E TIS M

109

PRACTICE EXAMPLES
1.

A conductor carrying a current of 100A is situated in and


lying a t right angles to a magnetic field having a flux density
of 0.25T. Calculate the force in newtonslmetre length
exerted on the conductor.

2.

A coil of 250 turns is wound uniformly over a wooden


ring of mean circumference 500mm and uniform crosssectional area of 400mm2. If the current passed through the
coil is 4A find (a) the magnetising force (b) the total flux.

3.

A current of 1A is passed through a solenoid coil, wound


with 3200 turns of wire. If the dimensions o f t h e air core are
length 800mm, diameter 20mm, find the value of the flux
produced inside the coil.

4.

Two long parallel busbars, each carry 2000A and are


spaced 0.8m apart between centres. Calculate the force1
metre acting on the conductors.

A moving-coil pcrmi~ncnl rn;ignct inslrl~rncnt I l ; ~ s ;I


resistance 01' IOR and the flux density in [lie gap is O ~ I ' I ' .' l ' h c
coil has 100 turns of wire, is of mean width 30mm and the
axial length of'the magnetic field is 25mm. Il'a p.d. of5OmV
is required for full-scale deflection, calculate the controlling
torque exerted by the spring.

6.

An air gap of length 3mm is cut in the iron magnetic


circuit of a measuring device. If a flux of 0.05Wb is required
In the air gap, which has an area of 650mm 2, find the
ampere-turns required for the air gap to produce the necessary flux.

7.

A straight horizontal wire carries a steady current of 150A


and is situated in a uniform magnetic field of 0.6T acting
vertically downwards. Determine the magnitude of' the
force per metre length and the direction in which it acts.

8.

An armature conductor has an effective length of400mm


and carries a current of 25A. Assuming that the average flux

density in the air-gap under the poles is 0.5T, calculate the


force in newtons exerted on the conductor.

9.

10.

In an electric motor the armature has 800 conductors


each carrying a current of 8A. The average flux density of
the magnetic field is 0.6T. The armature core has an effective
length of 250mm and all conductors may be taken as lying
on an effective diameter of 200mm. Determine the torque
and mechanical power developed when the armature is
rccolving nt 1 0 0 revlrn~n.
Two long straight parallel busbars have their centres
25mm apart. IS each cnrrics a current of 250A. calculate the
mutual fbrce/metre run.

1
I

CHAf''1'EK 6

ELECTROMAGNETIC INDUCTION

I
I

U p to now, the only method considered' for producing an


electromotive force has been that due to chemical action. In the
progression of electrical knowledge, history shows that this also
was the pattern followed and that electrochemistry was the first
branch of the science which was developed and was to play its
fhll part in subsequent electrical investigations of' the early
nineteenth century. At that time electricity was a subject of
interest to the scientist only and had not as yet been accepted as a
medium which could be put to use for engineering processes.
The -chemical cells as were then known, could not produce
sufficient energy for practical purposes, nor kad any electromagnetic devices been evolved which could be put to engineering
applications. As was mentioned in Chapter 5, it was only after
the relation between current and magnetism was discove'red that
attention was turned to various associated findings. These
inevitably disclosed the related phenomena of electromagnetic
induction and led to the development of machines which were
subsequently to interest the engineer as means of producing
either electrical o r mechanical energy.
The initial electrotnagnctic ' i t r c l r ~ c - ~ i o t rinvcstiyntions i ~ r c
attributed to Michael Faraday, who in 1821 showed that when
the magnetic flux linked or associated with ;In electricnl circuit
is changing, an e.m.f. is induced in the circuit. This e.m.f. is
shown to last only whilst the change of flux is taking place and
the faster the change the greater the e.m.f.
The flux linked with a circuit, which invariably consists of a
coil of insulated wire, may be changed in different yays. Thus:
(a) A magnet could be moved in the vicinity of a coil of wire.
This principle is used for the Alternator-a particular for12
of electrical generator.
(b) A coil of wire could be moved in the vicinity of a magnet.
This principle is used for the d.c. Dynamo o r Generatorthe more accepted modem term.
(c) The flux could be changed by varying the cuirent in the
energising coil of wire. The ampere-turns are thus varied and
the flux produced varies accordingly. This principle is made
use of in the operation o f t h e Transformer o r the Spark-coil
of a petrol-engine ignition system.

For the above three ways of e.m.f. generation, it will be seen


that cases (a) and' (b) involve relative physical movement between the magnet and the coil. Case (c) however, involves no
such movement and the generated e.m.f. is achieved in a stationary coil with which the associated flux only is changing. Thus
there are two distinct forms of e.m.f, generation o r inductionto use the alternative term. These forms are referred to under two
basic headings: ( 1 ) Dynamic Induction, (2) Static Induction.
Before these two methods are considered in detail it would be
well to give some attention to >he meaning of an expression
wllich will bc uscd frequently, namely Iflux-IitlX-trgc.~'.
Earlier stcdies on magnetism -showed that the
ficld of' ;I magnet can be represented by lines of' flux emanating
f'rom the poles. The strength of the flux can be represented by
the number of lines and is measured in webeis, while the flux
density is measured in teslas. The flux lines make complete loops
and the associated conductor or coil of wire in-which the e.m.f. is
induced can also be considered to consist of' a number of turns.
Since the rfumber of lines of' flux associated with the turns are
ref'erred to as flux-linkages, then a magnet with poles of flux
strength 3.4j1Wb associated or linking with a coil of 500 turns is
said to result in a condition of 3.4 x
x 500 = 04)017
weber-turns. The diagram (Fig 49) shows the basic idea of' the
explanation.
F L U X - LI N K A G E S .

CURRENT

F'AKADA) 'S L A b ' O F ELECTROMAGNETIC INDUCTION

This summarises the known relationship deduced for the


generation of e.m.f. by electromagnetic induction and can be
stated as: the magnitude of the e.m.f. produced, whenever there
is; ; I cli;~ngcof' flus linkctl with a circuit, is proportional to the
I.:klc ol' cllar~gc01' I l u x ~ - l ~ t ~ h a g c ~ .

EL E C T R O M A G N ETI C I N D U C T I O N

113

LENZ'S L AW

This identifies a phenomenon always noted for the e.m.f, produced by induction. The law can be stated as:-the direction of'
the current due to the induced e.m.f. will always set up an effect
tending to oppose the change which is causing it.
This can be further explained in terms of the magnetic field
which wo'uld be set up by the current caused by the induced
e.m.f. Thus if the flux-linkages tend to increase, then the field
produced by the induced current. resulting from the induced
C.m.f., will tend to oppose this effect, ie it would tend to oppose
the flux-linkage build up. Similarly, if the flux-linkages tended to
be reduced, as when the current in a coil is switched off, then the
induced e.m.f, will induce a .current which, if allowed to flow,
would tend to keep up the,flux-linkages to their original value.
It is stressed that the action of the induced current does not
succeed in preventing the change, but would try to do so during
*
the period when the change is occurring.
Faraday's law is capable of being expressed in mathematical
form and thus formulae can be deduced for both static and
dynamic electromagnetic induction. These will be considered
under each appropriate heading.
STATIC INDUCTION
The generation of an e.m.f. by static induction is considered
first, because it can be briefly dealt with here and then left to a
more appropriate stage in our studies. It is of purticular jntcrcst
in connection with ~nductanceand the theory of the a.c. circuit.
The study of dynamic induction is however, of immediate
importance, to allow electrical machines to be introduced as
soon as possible and will be given most of the attention in this
and subsequent chapters.
b.M.F. DUE TO STATIC INDUCTION

Consider the diagram (Fig 50) which shows two coils A and B
of insulated wire. Coil A can be connected to a battery through a
switch, whereas B is wound over or placed adjacent to coil A and
is connected to a sensitive centre-zero type voltmeter. This type
of instrument is used because, as the pointer is positioned at the
centre of the scale, a deflection to the left or right depending on
the polarity of the supply can be registered.
At the instant of switching on the current in coil A, the flux
can be imagined to grow outwards and to cut the turns of coil B.
The initial growing is shown by the dotted flux lines becoming
fuller until the final.condition (full lines) is reached. The cutting

/-I - -

Fig 50
of coil B by the flux of A, results in an e.m.f. being induced, its
magnitude and direction being governed by Faraday's and
Lenz's laws. The flux-linkages, ie flux linking with the turns
( N , ) of coil B increase and if the linking flux grows to a value of
@ webers from its original zero value, then the rate of change of
flux-linkages will be the flux-linkages divided by the time ( t , )
taken for them to grow, ie the time taken for the current to
reachrits final value I amperes in coil A. Thus if the resistance of
coil A is RA ohms and V,, is the applied voltage to coil A then

I=

.,
Y

amperes.
RA
flux-linkages - NB@volts.
and the e.m.f. induced in coil B =
time
tl
For this equation NB = turns of coil B, @ is the flux in webers
linking with it and t , is thc time taken for the energising current
to reach its final value I. It could be assumed that value I is
reached immediately the switch is closed;'because the flow of
electricity is considered to be instantaneous, but here we have a
condition wherc thc current takes an nppreciable time to reach
its full value--due to the inductance of the arrangement. This
action will hc considcrcd in detail later, but to revert to the
lnit~uleflkcts being observed; it will be seen that when the switch
for the primary coil A is closed, the voltmeter pointer gives a
'kick', say to the left, showing an e.m.f. to be induced in coil B--A

y.

the secondary circuit. The value of e.m.f. EB =

!+!&and the volt11

meter will show the polarity of coil B to be such that the current,
which flowcd through the instrument, was in such a direction

ELECTROMAGNETIC INDUCTION

115

through the coil as to set up a secondary flux, which would be


opposite to tlic originul flux cP und wotlld try to stop i t growins,
A further point of importance to note would be that, although a
kick of the voltmeter pointer would be seen, yet it would return
to the zero position even though current in coil A was allowed to
flow indefinitely. Thus an e.m.f. is induced only during the time
when the flux-linkages were changing. Further experiments with
coil B would show that if the number of turns of wire were
doubled, then the induced e.m.f. would be twice as large, even
though the flux 0 of coil A was the same. The flux-linkages have
been increased and the induced e.m.f. rises in proportion.
Consider next the instant of switching off the current in coil A.
The voltmeter will again be seen to kick-to the right this time,
showing an induced e.m.f. of reversed polarity. The flow of
current in coil B is such as to try and maintain the flux to its
where

original value @ and again EB =

t 2 is

the time taken

17

for 'switching off. It can be noted here that t , need not necessarily equal t,. If the switch is opened quickly, the current of A
will be interrupted very quickly and EBcan be larger at switching
off than at switching on; when the rate of growth of the flux is
controlled by the inductance and resistance of the circuit.
Up to now we have only considered the induced e.m.f. in coil B
and this is said to be due to Mutuul Induction, ie the mutual action
of cail A on 0. Wc now turn our rlttcntion to Sc~lflttrlriction, icv
the conditions appertaining to coil A itself. At the instant of'
switching on, the flux grows outwards and in so doing, cuts the
turns of coil A-the primary circuit. An e.m.f. is thus induced
N 0 Here N A is the turns ofcoil A, 0 is the linked
given by EA = -A.
t,

flux and 1 , the t&e taken for the current to reach its full value.
As before, the direction of the Self-induced-e.m.f. EA will be such
as to cause a current to flow in the opposirgdirection through the
battery and will produce a flux which will try to oppose the
build-up of flux @. We can now see the reason for the opposition
to the growth of current in coil A at the instant of switching on
and why the current I takes some little time to reach its full value.
As before when the switch is opened, the flux collapses and in
doing so, again cuts the turns of coil A, inducing a voltage of
reversed polarity, which tries to keep the current flowing.
Appreciable arcing will be observed at the switch contacts, but
if the latter is operated quickly, the circuit will be interrupted
quickly in spite of this and EA will be ineffective. It is stressed
however, that this self-induced e.m.f. at 'switching off, can be

extremely large in some instances where a large number of turns


of an energising winding are associated with a strong magnetic
flux. An example would be the opening of the field circuit of a
large alternator o r d.c. generator and special arrangements are
necessary in order to limit the e.m.f, to a safe value and prevent
'break-down' of the insulation by this large induced voltage.
Sufficient has now been said about static induction to allow
consideration of simple practical problems. It will be apparent
that the induced e.m.f, is proportional to the rate of change of
flux-linkages and since this rate alters from instant to instant
then, if instnntancous values arc not being considered, an average
value must be taken and generally we can take the formula as set
out to give the average e.m.f. and to serve all conditions. Thus:
Induced e.m.f. E,,

("I

- @') volts,

where N is the number of turns of the coil with which the flux is
linked, O, is the original value of flux in webers, @, is the final
value of fhe flux in webers and t is the time in seconds during
which the change is taking place. This general expression serves
conditions of switching on and switching off and also intermediate conditions, when the flux is changed from one value t~
another,.
Example 44. (Self-induction). A coil of 800 turns is wound o n
a wooden former and a current of 5A is passed through it t o
produce a magnetic flux of 200 micro-webers. Calculate the
average value of e.m.f, induced in the coil when the current is (a)
switched off in 0.08 seconds (b) reversed in 0.2 seconds.
N@ - 800 x 200 x lob6 - 16 x lo4 x
(a) E,, = t
0.08
8 x lo-'
= 2 volts
(b) E.,

("I

- *,)

here O, = @, but is in t h e reverse

direction. I'hus flux change is from @, to zero and then up again


+ a,) = 2@, webers.
to 0,or a total change of (0,
Thus E,, =

800[200 x l o L 6 - ( - 200 x
0.2

117

ELECTROMAGNETIC INDUCnON

Example 45. (Mutual-induction), I f the coil of the above


example has a secondary coil of 2000 turnh wound onto it, find
the e.m.f, induced in this second coil when the current of 5A is
switched off in 0.08 seconds. ( I t can be assumed that all the flux
of 200pWb created by the 5A current in the primary links with
the secondary coil.)
2000(200 x
0.08
2
x
20
- -= 5 volts.

E." =

- 0)

=2

lo3
8 x

200

lo-2

Note how the e.m.f. of the secondary is - = 2.5 times the in2
duced e.m.f. value in the primary. It is in direct proportion to the
2000 = 2.5. This is the basic principle of the transturns ratio ---800
r
former and ignition system spark-coil. It shows how a large
voltage can be induced in a secondary coil by the flux associated
with a low voltage primary coil. For a petrolengine ignition
system, the e.m.f. in the secondary may be in the region of 8000
volts compared with the 12V applied to the primary. This is
achieved by using the appropriate turns ratio for the primary
and secondary coils, by providing an iron magnetic circuit to
concentrate the flux fbr maximum linkage and by interrupting
the primury circuit quickly by un cnginc-drjvcn cu~li-opcratcd
switch.
DYNAMIC INDUCTION
As was mentioned earlier, this condition covers the cases
where there is relative movement between a magnetic field and a
conductor. Obviously this concerns either a stationary conductor and a moving field or a stationary field and a moving
conductor. To avoid repetition of basic theory, the immediate
explanations and diagrams will refer to a fixed field and moving
conductor.
The diagrams (Fig 51) show a field as produced by two
permanent magnets and a conductor which is moved so as to cut
the field, thus altering the flux-linkages. The reasoning can be
seen as in interpretation of Faraday's and Lenz's laws and three
cases are shown.
For case (a) there is seen to be no change of flux-linkages, ie
no cutting of the field. The conductor is merely moved at a
velocity of v metreslsecond in the same direction of the lines of
flux and no e.m.f. is recorded on the voltmeter. For case (b) the

Fig 51

conductor is moved at right angles to the field of flux-density B


teslas and the voltmeter shows a constant deflection. The fluxlinkages can be copsidered to be changing since the flux lines arc
cut or can be imagined to stretch and snap as the conductor
passes through, to reform again behind the conductor. If' the
c.onduclor i s movcd from left to right, a polarity i s notcd,

119

ELECTROMAGNETIC INDUC~ON

which reverses if the conductor is moved from right to left.


Alternatively, if the field is reversed so that the flux lines are
considered to pass from a bottom N pole to a S pole a t the top
of the diagram, and the conductor is moved from left to right,
then a reversed polarity will again be indicated. The investigation
will show further deductions. Thus:
The magnitude of the induced e.m.f. varies with the speed of
cutting the field or rate of change of flux-linkages. Hence
E a V . Again, if the field being cut is varied by altering the
density of the flux, then the e.m.f. will vary as B or E a B.
Obviously also, the longer the conductor cutting a field, the
greater will be the magnitude of the e.m.f. and E a I. Summarising these three conditions we see that E a Blv. Here 1 is
the length of the conductor in metres.
Case (c) of the diagram shows the conductor cutting the field
at an angle 8. It is an intermediate condition between cases (a)
and (b) and is best treated by resolving v iflo two component
velocities at right angles to each other. Consider v cos 8 to be
the component velocity in the direction of the flux lines, then
v sin 8 will be the other component velocity a t right angles to the
field. In accordance with the reasoning for cases (a) and (b) we
see that velocity v cos 8 will be responsible for no induced e.m.f.
whereas velocity v sin 0 will be responsible for such an e.m.f. and
E K v sin 8.
E w Rlv qin 0 will be A more general expression than those
already deduced since i t will cover all conditions.
For instance for the condition of case (a) 8 = 0" and
since sin 0" = 0 :. Blv sin 0" = 0 or E = 0 as already stated
Again for case (b). If 8 = 90" then sin 90" = 1 and
Blv sin 90" = Blv giving E a Blv.

1
I

E.M.F. DUE TO DYNAMIC INDUCTION

As explained above, the induced e.m.f. is found to be proportional to B, I, v and the sine of the angle made by the direction
of' cutting, and the direction of the field. The actual magnitude
of such an e.m.f. can however be deduced in more definite terms
thus :
Consider case (b) of Fig 51. In 1 second, the area cut by a
conductor of length 1 metres and moving at a velocity of v
metres/second is lv square metres. If the flux density in this area
is B teslas, then the flux cut per second by the conductor = Blv
webers. Using Faraday's law, we see that Blv can be used as a
measure of the magnitude of the induced e.m.f. in volts or
Induced e.m.f. E = Blv volts

If case (c) is considered the flux cut is proportional to the


component of the velocity perpendicular to the field
or induced e.m.f. E = Blv sin 8 volts.
The above formula can also be deduced as follows; this
approach may appeai- to the reader to be more satisfactory as a
proof. The diagram (Fig 52) shows a conductor Q, carrying a
current of I amperes in the direction shown. As before the flux
density of the field is taken as B teslas and the length of the conductor as I metres. Using fundamentals already set out in the
chapter on electromagnetism, it is known that a force is exerted
on a current-carrying conductor in a magnetic field. Thus the
conductor in the diagram experiences a force BII newtons
urging it to the left. Accordingly a force of BIl newtons must be
applied in the opposite direction to oppose movement of the
conductor.

Fig 52
8
.

Consider the.conductor to move, from position Q to position


P spaced x metres away. Then work done by the conductor in
moving from Q to P = Force x distance = BIlx newton
metres or joules.
Let E volts be the e.m.f. induced in the conductor as a result of
cutting the field, and t seconds the time taken to complete the
operation.

Then Mechanical Power expended = 'I1' watts and if this


t

appeared as electrical power, it would be El watts


BIIx
Blx
or EI = and E = volts
I
t
Obviously 5 = velocity of cutting v
t
or as before, E = Blv volts.
From the above, the following deduction can also b e made.

ELeCIROXAGNETIC INDUCTION

12 1

Blx is (the fluxdensity x area) or the Bux Q, cut by the conductor


in moving from position Q to P in time t seconds and since

0
E = -Blv then E = B x a r e a = t

Thus E (volts) = @(Webers) and we have an alternative


t (seconds)
formula for the e.m.f. generated in a conductor cutting a
magnetic
field. In this form, it is sunilar to that deduced for the
NO
statically induced e.m.f. namely E = -where N is the number
t
of turns of the coil, and O is the change of flux.
flux-linkages or E = 0 since
The formula can thus be E =
t
time
the flux-linkages are numerically equal to @, there being only one
conductor.
'
cut and thus we have an alternative
Important Note. - is
t
time
way of stating Faraday's law, which can now be expressed as:
The e.m.f. generated in a conductor is proportional to the rate of
cutting lines of flux or is proportional to the flux cutlsecond.'
The abo,ve form of Faraday's law is more applicable to
dynamic induction and will be uscd scpeutcdly in connection
with the Generator, Motor and Alternator.
@

Example 46. A conductor is moved to cut a magnetic field at


right angles. Find the e.m.f. induced in it, if the average density
of the field is 0.45 teslas, the length of conductor is 80mm and
the speed of cutting is 8.88 metresisecond.
In the Formula E = Blv we have
E = 0.45 x 0.08 x 8.88 = 0.32 volts.
An alternate solution could be:
Area swept by the conductor/second = 0.08 x 8-88 square
metres. The flux in this area would be 0 = 0.45 x 0.08 x 8.88
webers
and e.m.f. = flux cut per second
0.45 x 0.08 x 8.88 = 0.32 volts.
or E =
1
Example 47. A four-pole generator has a flux of 12mWb/pole.
Calculate the value of e.m.f. generated in one of the armature
conductors, if the armature is driven at 900 rev/min.
= 0.048Wb
In 1 revolution a conductor cuts 4 x 12 x

1
Time of 1 revolution of the armature = -minutes

900

:. Rate of cutting flux =

Thus E =

0.048

--i- =
l-5

Flux cut per revolution


time taken to complete a revolution

0.048 x 15 = 0.72 volts/conductor.

Although this unit has been introduced, it has not a s


yet been satisf'actorily defined, since the accepted .definition is
based on the principle of electromagnetic induction. Thus we
have :
'The weber is that magnetic flux which, when cut by a conductor
in one second, generates in the conductor an e.m.f. of value
equal to one volt.'
Alternative ways of defining thc weber o r SI unit of flux are :
'An e.m.f. of one volt is generated when a conductor cuts flux a t
the rate of one weberisecond', c+r'an e.rr,.f. of one volt is generated when the flux linked with cne turn changes at the rate of
one weberlsecond.'
THE WEBER .

DIRECTION OF INDUCED E.M.F.( H A N D RULES)

The direction of the induced e.m.f. can be deduced from first


principles, using Lenz's law o r by the application of a rule which
was first enuniinted by Professor T. A. Fleming and is now
commonly known as F L E M IN G ' S R IG H T - H A N D RULE . It should be
noted that there is also a Left-Hand Rule and to avoid later
confusion in the students' mind, the point is made a t this stage
and the following aid is suggested for memorising the appropriate
rule to suit the circumstances. Thus the Generator is studied
hqji~rethe Motor and for the average person, the use of the
Right-hand I S prcf'crred hcfirc~ that o f t h e left. Therefore use the
Ripht-11;lnd rule f'or the Generator and the Left-hand rule for
tlic Motor. The (;enerator is n machine concerned with thc
generated o r induced e.m.f. in armature conductors and thus for
condittotis 01' elccl~.oclyli;~~llic
i~l(lucliorl,wc ~ s ctllc t,ight-lli~ntl
rule which can now be explained.
R U L E (Fieming's). Consider a conductor in a
magnetic field as shown in the diagram (Fig 53).
Imagine the tnagnetic ficld to bc in tht: direction from Ief't to
~.it..Iil; ~ n dthc conductor to be moved at right Angles and upwards
with a velocity of v rnetres/second. The e.m.f. across the ends ol'

R I G HT - H A N D

ELE C TR O M A G N ETI C I N D U C TI O N

123

Fig 53
the conductor is assumed as shown, ie the polarity is such that,
if the ends of' the conductor are joined externally through an
ammeter, current will flow as indicated. Its direction in the
conductor is 5ccn :~ntlif' :tttcntion i s ylvcn to Fig 53b, it will be
deduced that the field due to the conductor current is clockwise,
to strengthen the field at the top and weaken the field at the
bottom. Thus according to Lcnl's law, opposition is o f i r c d t o
the motion of the conductor as one imagines the field lines
concentrating or massing bef'ore the conductor, stretching and
then snapping. A force of opposition to the direction of movement is apparent and the assumed polarity must be correct to
confirm the action which takes place in practice. If a reversed
polarity was assumed, the current would be in the opposite
direction and field weakening would occur above the conductor
and strengthening below. This would result in a driving force
behind the conductor which would be a motoring rather than a
generating condition. There is no opposition to moving the
conductor and since such a condition is not possible, this
alternative e.m.f. polarity assumption must be incorrect.
Since the' original assumption actually is confirmed by practical conditions, the right hand can be drawn and used to find the
direction of the induced current and therefore the induced
polarity. This is shown in the diagram (Fig 54).

Fig 54
To use the rule, place the thumb, index finger and second finger
of the right hand at right angles to each other. Point the index
linger in the direction of' the flux lines and the thumb in the
direction of moving the conductor. The current in the conductor,
due to the induced e.m.f. would be in the direction indicated by
the second finger. For the example being considered (Fig 53),
current would be into the paper as deduced from first principles.
T H E SIMPLE MAGNETO-DYNAMO
Once the principles of electromagnetic induction were discovered, it soon became evident that the way was open to
constructing a machine, in the true sense of the word, which
could convert mechanical energy into electrical energy and thus
generate electricity as a result of being driven by a prime-mover,
such as a steam engine or water turbine. The idea of making
insulated condu?tors move through a stationary magnetic field
presented no difficulties for a small machine and so the basic
construction of such a magneto-dynamo followed fundamental
requirements. A typical machine is therefore, illustrated in the
diagram (Fig 5 9 , and consists of permanent magnets to provide
the field and a simple coil which is mounted on but insulated
from a shaft which can be rotated. In order to allow contact to
be made with the moving conductors, they are connected to
slip-rings which are mountcd on but insulated from the shaft.
Fixed 'brushes' in turn, contact the slip-rings to make sliding
connections and allow an external circuit to be enexgised.
It will be seen that the coil consists of two 'active' conductors
which have been designated AB and CD. These are connected
in series by the connection BC wbich, together with the front
connections to the slip-rings, plays no part in the generation of
e.m.f. but merely serves as a means of carqhng current to the
external circuit. The load resistance of the external circuit has

Fig 55
been shown as concentrated in R and is connected to the
terminals X, Y of the machine.
Consider the operation of the machine as follows:
As one conductor AB moves down through the field, the other
CD moves up and the induced e.m.fs. will be such that A is +ve
relative to B and C is + vc rclutivc to D. Thc induced currcnt, if
allowed to flow, would be as shown by the arrows and, since it is
from terminal Y to terminal X through the external circuit, Y is
+ve with respect to X. It would be well for the student at this
point to try the right-hand rule for himself and satisfy himself as
to the polarity of the terminals for the half-revolution being considered. It should be noted that the right-hand rule as described,
can be applied here to conductor AB, the condition being that
AB is moving from the top vertical position round past the
centre of the magnet pole and then onto the bottom vertical
position. The position where it moves past the pole at right
angles is of particular importance, being a condition of maximum e.m.f.
After the coil has rotated a half revolution, conductor DC
begins to move downwards and AB upwards. The polarity
induced is now in reverse to that for the 1st half revolution, D
being +ve relative to C and B is +ve relative to A. Terminal X
is now the +ve terminal and Y is the negative. An alternating
e.m.f. is generated, as shown in the diagram (Fig 56), which also

illustrates four positions of the coil viewed from the slip-ring end.
For position 1, A and D are moving horizontally along the field
and no e.m.f. is being generated. A sirnilar.condition exists for

Fig 56

'

position 3 , but for positions 2 and 4 maximum e.m.f. is being


generated, since field-cutting at right-angles is taking place. For
intermediate positions, the general condition of e.m.f. generation, ;IS rcprcscntcd by E a Blv sin 0, is followed, since ttlc
conductors are cutting a uniform field at an angle but are moving
a t a uniform velocity. Thus the e,m.f. generated a t a n y instant is
not constant but varies and it is customary to use a small letter
for what is termed the instantaneous value.
The expression e = Blv sin 0 volts gives the magnitude of the
voltage being generated, provided the correct units are used
when substl.uting. If the voltage is plotted to a base of' revolutions, degrees or radians, a waveform such as:that illustrated will
be obtained.

ELECTROMAGNETIC IN D U C T IO N

127

THE SIMPLE D . C . GFNRRATOR

The simple magneto-cly~rirrwn ~ u c l ~ ~u!,~ l dc\cr~hccl


c,
sbovc
(Fig 55), or electrical generator, as it is now called-to use the
modern term; is seen to have n uniform field arranged to be cut
by conductors as shown. It will provide an e.m.f. whose magnitude varies sinusoidally, that is, the e.m.f. polarity and value
follows a sine waveform. A sinusoidal waveform is desirable for
a.c. working but for the d.c. generator, modifications are necessary to achieve a substantially constant unidirectional voltage
magnitude and polarity. It is now apparent that a distinction is
being made between the generation of direct current and alternating current and from here on the division between the two
methods of generating, transmitting and utilising electrical
energy will become marked. In this book, it is hoped that the
study of both d.c. and a.c. theory will be followed simultaneously
and the reader is asked to give the approprizte chapters equal
attention. D.C. theory has up to now received more attention by
the marine engineer, mainly because of the type of installation
with which he is familiar. A.C. installations are however, for
quite a considerable time now, being used to an increasing extent
for ship work and are almost universal ashore. It is not proposed
to enter, at this stage, into a discussion as to the relative advantages of alternating current over direct current or vice-versa; but
it is stressed that the major portion of electrical theory is conccrned with u.c, circu~tsand milchines and that if Inter study
difficulties are to be minimised, then full attention must be given
to a.c. theory right from the beginning.
The first of the modifications referred to above for the d.c.
machine, involves the introduction of an iron or magnetic
material into the armature or moving-coil part of the assembly.
The coil made up from insulated wire or strip, is wound onto an
iron armature which is carried on the shaft. The magnet system
is also provided with iron pole-shoes or extensions as shown in
the diagram (Fig 58). Since the length of the flux path through
air is now reduced to two small air-gaps, the remainder being
through the iron of the armature and field system; the flux
density or B value in the air-gaps is increased and. the conductors will therefore cut a stronger field. Again as the air-gaps
are now small and of constant width, the flux lines will cross
them as shown and the field will be uniform over the pole-faces.
The moving conductors thus pass from a small arc with substantially no magnetic flux into a large arc ~f constant flux
density. The flux lines are seen to be radial in the gaps and are
cut at right angles for most of the distance under the pole. The

Fig 57
e.m.f. waveform is now as shown in the diagram (Fig 57), ie it is
proportional to the flux density through which it passes.
COMM~JTATION.T O

obtain a constant unidirectional e.m.f, or to


produce the true d.c. generator as is used in practice, the next step
in modification is, to fit a form of automatic reversing switch.or

ELECT RO MAGNETIC IN D U C TI O N

129

oammutator which, even tl~ouyht l ~ dmovingcoil oontinwa to


generate an alternating e.m.f., ensures that a unidirectional or
'rectified' e.m.f. appears at the terminals of the machine. The
diagram (Fig 58) shows how a commutator is fitted. It consists
of a metal slip-ring which is split into two parts, each insulated
from the other and from the shaft. The ends of the coil are
connected to each half or segment of the commutator. The
stationary brushes are so adjusted that they bridge the gap in the
slip-rings at the instant when the e.m.f. induced in the coil has
zero value and is due to reverse.
The diagram (Fig 59) shows the side view of the commutator
and the reversing action of the switching arrangement can be
more clearly seen. The diagrams can be considered t o b e complementary to those of Fig 56 although only conditions for
positions 2 and 4 are shown. It is apparent that the obvious
position for the brushes is on the 'magnetic geutral axis' a d that the brush Y will always be the +ve and brush X the -ve
terminal. The new shape of the waveform is also shown.

Fig 59

A+
Position 2

1+
Position A

130

REED'S BASIC ELECTROTECHNOLOGY

For position 2 it will beTseen that - ve end D of conductor CD


is connected to the +ve brush Y, whereas the +ve end A of AB
is connected to the - ve brush X. For position 4 when the e.m.fs.
have reversed in the conductors of the coil, end D which is now
ve is connected to the -ve brush X and end A of AB is now
- ve and connected to the +ve brush Y. It is well to remind the
reader here, that the brush polarity is decided by the direction of
current flow in the external circuit. Thus current flows from Y
(the +ve brush) to X (the - ve brush) and then to A onto B, etc.
The apparent anomaly of ve end A being connected to a - ve
brush and so on is thus cxplaincd. Tlic resulting effect of thc
commutating action is to produce a pulsating but unidirectional
e.m.f, at the terminals of the generator.

PRACTICAL REQUIREMENTS.TO obtain a more uniform e.m.f., the


two-part commutator and single coil can be repeated to give a n
arrangement employing a greater number of segments and a
larger number of coils. Each coil can consist of a number of
turns to give a larger output voltage.
The example shown in the diagram (Fig 60) is an armature

/-.

Fig 60
with two coils at right clngles. It follows thl~tfor this arrangcmcnt
when coil A develops maximum e.m.f., coil B generates nq e.m.f.
and whc11 tllc Irrmi\turc rot;~tcsIlirot~gli;I rlui~rtcro f ;I revolution,
the conditions would be vice-versa. The accompanying diagram
(Fig 61) shows the waveforms of the generated e.m.fs. The
generator terminal voltage ncver falls to zero but it is obvious
that two distinct disadvantages are still evident. Firstly, all the
~ x ~ ~ l d u c tarc
o r s not used to mnximum ndvantage since only one
coil ; ~ t;I time is being ernployed for supplying the external
circuit. Secondly, but or prime importance is the new condition

131

ELECTROMAGNETIC INDUCTION

EM.WE
0 COILS
AL 0

' \

r
I
\

I
I

I
I

::
n

2 ~i
C

Fig 61
of commutation. Since the brushes must be placed in a position
to contact the coil in which e.m.f. is being generated, it follows
that if the generator is on load, ie supplying current, then at the
instant when the connected segments leave the brush, since an
e.m.f. still exists and current is flowing, arcing will take place at
the brushes. If coil A is bcing considered : ~ n dFigs 60 and 61 :ire
noted, it will be seen that at the instant when the gap between
segments is being bridged by the brushes, coil A.is still cutting
the field and coil B has only just entered the field. Thus coil A
tends to be short-circuited by a coil in which the e.m.f. may not
have risen to the required value and current will flow in the coil
B. This current is diverted from the !oad current and also
adversely affecting the commutation. If the number of coils is
increased, the tendency would be to give a smoother output
voltage but continued arcing at the brushes would persist. This
arrangement Is obviously not satisfactory.
In the early period of development of a satisfactory arrangement for the armature conductors many ideas were introduced.
One such arrangement was incorporated in the Gramme-Ring
Armature, which involved a special construction, in that the
armature iron q c u i t was built up as a ring and the conductors
were connected in series, with tappings being brought out to the
commutator segments. The conductors were thus part of a
continuous winding, but it will be noted that only the conductors
on the outside of the ring are active, whereas that part of the

132

REED'SBASIC ELECIROTECHNOUXiY

winding which is on the inside and at the ends of the ring, cuts
no flux, and is thus responsible for no e.m.f. This was a most
uneconomical arrangement since it wasted conductor material.

Although the Gramme-Ring Armature is shown (Fig 62), it is


stressed that this never was a commercial proposition. The only
reason why it is being described here is that it shows the student
how a continuous closed winding can be made to operate as a
suitable source of generated e.m.f. Furthermore satisfactory commutation is possible and all 'active' conductors in the air-gaps
arc used to advantage. It is not intended that the reader should
give the arrangement undue attention nor should he consider
the actual details worth remembering. All his attention should be
given to the 'drum' type armature winding which is described
next and dealt with more fully in books on the practical aspect
of modem maohine construction and operation.
The ring arrangement shows how a continuous winding is
possibk, in which the induced e.m.f. automatically divides the
armature into two parallel paths, the flow of current being as
shown. The coils in which no e.m.f. is being generated are shortcircuited as they pass under the brushes and since there is no
current, no sparking occurs and commutation is correct. All
other active conductors are however used to advantage and are
connected in series to supply the e.m.f. at the terminals. Even
though the armature is rotating, the disposition of the active
conductors in space can be considered to be stationary. As any
one coil passes from one side of a brush to the other, it leaves
the series circuit of one parallel branch of the armature winding
to be replaced by a similar coil, which has just been cornmutated
o r short-circuited by passing under a brush. Thus one coil enters
into the series circuit of one of the parallel paths as one coil
leaves and the total e.m.f. being generated is substantially
constant.

E L E C TR O M A G N E T I C I N D U C T I O N

133

It should be noted that for a 2-pole machine the armature


winding is i~utomiiticallydivided into two pnrnllcl poths. For a
4-pole machine the winding will be divided into four parallel
paths and so on. The armature can now be looked upon as a
battery made up from a number of similar cells. An armature
with one hundred conductors, being wound for a 4-pole
machine, will mean twenty-five conductors are in series for one
parallel path and there are four such parallel paths. If 1V is
generated in one conductor, the e.m.f. of 1 parallel path is 25V.
This is also the e.m.f. of the machine, since as for a battery, the
e.m.f. of the arrangement, is the e.m.f. of one parallel path.
Again if the size of the conductor used for the winding is'suitable
for carrying 10A, then the current carried bq'one parallel path is
10A and the total current that can be expected from the armature
is 4 x 10 = 40A.
THE DRUM W I N DI N G . The diagram (Fig 63) h o w s the basic

arrangement. This armatwg winding arrangement is accepted


as the only modern method of connecting the active conductors
together. The amount of 'copper' on the armature is used to
maximum advantage since, except for the overhang at the back
of a coil and the front connections to the commutator segments,
the winding consists of lengths of copper conductors which are
so placed to cut magnetic flux and thus generate e.m.f. The basic
winding uses a number of coils in series between brushes, these
coils being arranged at constant angles to each other. T h e
resultant e.m.f, is thus more uniform and larger since many coils
in series are employed. The connecting up of such coils presents
difficulties; but any requirement can be realised by .proper
choice of coil numbers, the span of a coil, number of parallel
paths, etc. An example of a simple drum winding can now be
considered.

Fig 63

i
1

I
I

I
i
1

A, B, C, D, etc are insulated conductors fitted into slots cut


into the iron of the armature. There are also four commutator
segments Nos 1 , 2 , 3 and 4. The conductors may be connected to
each otlier and to the segments in a variety of ways and one
possible arrangement is shown in the diagram.
With rotation as shown, the e.m.f. in B, C and D are from
front to back, while for F, G and H the e.m.f. are from back to
front. The full lines show coqnection to segments and the dotted
lines connections, which constitute the overhang of the coils, at
the back.
St:~rtinpat the -ve brush on segment 1 , current enters the
armaturc from the external circuit and divides into two parallel
paths. One path passes to A and flowing down this comes at the
I l i ~ c l IO I,', IICIICC
L I iICI.oSS
~
to scjilncnt 2 and Lhcn onto und
down C to H, I'rom where it rises u p and goes to the brush on
segment 3. This would be the +ve brush. The other current path
is from segment 1 to conductors D, G, B, E and onto segment 3
or the +ve brush. There are thus two circuits in parallel and it
will also be noticed that, as a brush passes from one segment to
the next, one coil is short-circuited and the brush must be
located so as to short the coil at the instant when its e.m.f. and
resultant current is zero. Such an instant is shown in Fig 63b
giving correct commutation conditions for the short-circuited
coils between segments 1 and 2 and between segments 3 and 4.
Example 48. A slow-speed d.c. generator has an armature of
diameter 3.0m and active conductors of length 510mm. The
average stseng4li of the field in the air-gap is 0.8T and the
armature speed is 200 revimin. If the armature has 144 conductors arranged in 8 parallel paths, find the e.m.f. generated
at the machine terminals.

Using formula E = Blv volts


then R = 0.08 teslas. 1 = 510 x 10-3m and v is obtained
t US :--

In I second the :ll-m;lhlrc rcvolvcs

2s
or -10. t i ~ r e s .Also i n
60
3

1 revolution one conductor travels

xd = 3.14 x 3 = 9.42m

So in 1 second the conductor travels 9.42 x

10
= 3.14 x
3
=

10

31.4m

E . M . F . generated perconductor = 0.8 x 510 x 1 0 - 3 x 31.4


= 0.8 x 5 . 1 x 3.14
12.8 volts
-2

Now

~ l i c r c;II.L.144

c o n t l ~ ~ c ( o111
r \ X 1~:11;1llcl
I):IIII\
144
= IS
So the conductors in serles in each pnrnllel p;rtil =
13
'Thus the e.m.S. generated in 1 parallel path = 18 i( I3 8
= 230.4V
-

A.C. A N D D.C. THEORY


The reader will be required from now onwards to give equal
attention to both the a.c. and d.c. theory appertaining to electrical technology. T o assist in this, the following chapters in Volume
6 will treat both aspects of theory alternatively, thereby ensuring
correct progression along both channels of study. It should be
noted that the amount of a.c. theory to be covered is considerable and will occupy the major part of Volume 7. Here in Volume
6, under the broad coverage of the d.c. theory, is included further
basics on d.c. machines, the magnetic circuit, circuit conditions,
methods of solution and thermionics, whereas the a.c. theory
proceeds with basic fundamentals of circuit conditions and
systems. Inductance and capacitance have also been introduced
because of their importance to a.c. circuits and in connection
with the capacitor, the treatment has been extended to include
the electron theory, basic electronics and electrostatics. The following diagram will help to explain the course b a n g followed.
C ' I 1Al''I'Iil~0
(Electromagnetic Induction)

CHAPTER 8
(The D.C. Generator)

CHAPTER 7
(Basic A.C. Theory)
I

CHAPTER 10
(The D.C. Motor)

CHAPTER 9
(The A.C. Circuit)

CHAPTER 12
(Electromagnetism)

CHAPTER 1 1
(.4.C Circuits and Systems)

CHAPTER 14
(Miscellaneous Clrcult
Cond1tlon5 dnd Method\
of Solut~on,Spec~,il
,4pplicCitlon\)

CHAPTER I?
(The Electron The01 !,
B'isic electronic^ ,rnd
Electroctat~cs)
CHAPTER 15
(Electron~c\)

CHAPTER 6
PRACTICE EXAMPLES
1.

Calculate the e.m.f. generated in the axles of a railway


train when travelling at 100km/h. The axles are 1.4m in
length and the component of the earth's magnetic field
density is 40pT.

2.

Find the generated e.m.f./conductor of a 6-pole d.c.


generator having a magnetic fluxlpole of 64mWb and a
spccd of I000 rcvlmin. If there arc 4611 conductors, connected
in six parallel circuits, calculate the total generated e.m.f. of
the machine. Find also the total power developed by the
armature when the current in each conductor is 50A.

3.

An iron-cored coil of 2000 turns produces a magnetic flux


of 30mWb when a current of IOA is flowing from the d.c.
supply. Find the average value of induced e.m.f. if the time of
opening the supply switch iS 0.12 second. The residual flux
of the iron is 2mWb.

4.

A one-turn armature coil has an axial length of 0.4m and


a diameter of 0.2m. I t is rotated at a speed of 500 revlmin
a field of uniform flux density of 1.2T. Calculate the
magnitude of the e.m.f. induced in the coil.

5.

When driven at 1000 rev:min with a flux/pole of 20mWb,


a d.c. generator has an e.m.f. of 200V. If the speed is increased to 1 100 rev/min and at the same time the flux/pole is
reduced to I9rnWb/polc, what is thcn the induced e.m.f.?

0,

A coil 01' 2oU turns 1s rotated a t 1200 ~.sv/m;nbetween thc


poles of an electromagnet. The flux density of the field is
0.02T i~ndthe xis of roti~tionis at right angles to the
direction of the field. The eff'ective length of the coil is 0.3m
and the mean width 0.2m. Assuming that the e.m,f. produced is sinusoidal, calculate (a) the maximum value of
e.m.f. (b) the frequency.

7.

A coil of 19120 turns la wound on an iron core and wltn a


ti1111 V ~ I I ~
01'I CC L. I II . C I I ( [lowitip, 111 tl~cC I I . L . I ;II ~flttx
I, (TS
.lrnW\> i~ ~~rodiicrtl
Wl1c.11rllr c ~ l ~ . c . uI.; ~ opcnctl.
l
lhc ll!~x I'ic!!.:

(TI

to ~ t sres~dual value of 1 5rnWb In 4 0 m s . Calculate the


:lvcr:lgc V:IIIIC 01. t i i c I I H I I I < T ( I r 111 l'

8.

The armature ol' a four-pole generator rotates at 600 rev/


min. The area of each pole-face is 0.09m2and the flux
density in the air-gap is 0.92T. Find the average e.m.f.
induced in each conductor. If the armature winding is made
up of 210 single-turn coils connected so as to provide four
parallel paths between the brushes, find the generator
terminal voltage.

9.

A solenoid 1.5m long is wound uniformly with 400 turns


and a small 50 turns coil of lOmm diameter is plaaxj inside
and at the centre of the solenoid. The axes of the solenoid
and the coil are coincident. Calculate (a) the flux linked
with the small coil when the solenoid carries a current of 6A
and (b) the average e.m.f. induced in the sm51I coil when the
current in the solenoid is reduced from 6A to zero in 50111s.

10.

Two coils A and B having 1000 and 500 turns respectively


are magnetically coupled. When a current of 2A is flowing in
coil A it produces a flux of 18mWb, of which 80 per cent is
linked with coil B. If the current of 2A is reversed uniformly
in O.ls, what will be the average e.m.f. in each coil?

CHAPTER 7

BASIC A.C. THEORY


Introduction is made by quickly revising the relevant fundamentals of Chapter 6. The diagrams (Fig 64a and b) show an
elementary form of a.c. generator in which a coil is rotated in a
uniform magnetic field. The sides of the mil, ie the conductors,
cut the magnetic flux and thus, an e.m.f. is induced which, from
first principles is e = Blv volts. The letter e, for the value of induced e.m.f., has been introduced here because this value is not
constunt but, as will be seen, vi~siesfrom instilnt to instent. Thus
even though the w i l is rotated at a uniform velocity v, the rate
of cutting i s not constant, but depends upon the angle at which
the conductors cut flux. The velocity can be resolved into a cutting component ( v sin 8) and a noncutting component ( v cos 8).
The cutting velocity component only is responsible for e.m.f. and
can be used to give a general expression which gives the e.m.f.
at any instant, as e = Blv sin 8 volts.

)8\j\
\

-.
- - -(h)
Fig 64

/
/

BASIC A.C: THEO RY

139

THE A.C. W A V E F O R M

In tlie expression t1 = Shl sin 0 , us for any alternator, 61. 1 and


can be assumed to be const;lnt and made equal to K. The
expression now becomes 61 = K sin U and a further value for K
can be obtained if we cons~derthe instant when the coil sides
are cutting the field at r ~ g h tangles. Velocity component v sin 0
generates a maximum e.m.f. which can be designated as Em.
At this instance e = Em and we can write Em = K sin 8.
But sin 8 = sin 90 = 1 . :. Em = K
Substituting back in the expression, we have:
e = Em sin 0
The above is an Important equation which shows that the
generated e.m.f. varies sinusoidally. e is termed the instantaneous
value and Em the maximum value.
If attention is now turned to a waveforr~plotted to a time o r
angle base, it will be remembered, from work dready done elsewhere on vectors, that a sine wave can be deduced from the vertical component of a rotating vector-for electrical work, such a
rotating vector is called a phasor. If the length of the phasor is
made to represent Em,then for any angle 8, the instantaneous
value is the vertical projection and this also can be used as an
ordinate for the waveform, when plotted to an angle or time
base. The diagram (Fig 65) illustrates the procedure for deducing
a waveform and the method is summariscd thus:
I)r.;tw ; I c11.clcof' t a d 1 ~ 1m5 : ~ d cequ;tl

I O 1 1 1 In;tullrlillri
~

V;IIIIC oI'tlic

wave. Startlng from the h o r ~ ~ o n t amove


l,
the phasor through a
known angle and project the vertical value onto an angle or time
scale. Choose suitable scales so as not to distort the sinusoidal
shape of the wave. The following remlnder shows the connection
between the construction and the representation of a sinusoidally
induced e.m.f. Since, from the triangle illustrated,
e
- = sin 8
so e = Em sln 0.
Em

Fig 65

This expression is in the form deduced previously but can be


further modified to suit the representation by a phasor. In
accordance with accepted procedure, assume that the phasor Em
rotates from the zero or horizontal position in an anticlockwise
direction with an angular velocity of o radians/second (ois the
Greek letter-small omega). Then 6 = wt where t is the time in
seconds and the equation can now be written as e = Emsin at.
The diagram (Fig 66) following below, shows some of the
terms used in connection with a,c. theory. Periodic time = the
time for 1 cycle. The frcqrienq- f of the wave = the number of
complete cycles in the interval of' 1 second. In accordance with
S1 recommendations, the name hertz (Hz) is now being adopted
lor frequency Ineuuurctncnr. ,This rcpluccs tl~coldcr tcrm of
cycles per second (CIS).Present marine practice uses either 50Hz
or 60Hza.c. systems. The maximum value reached by the wave is
also called its peak value, or its amplitude and, as mentioned
earlier, the value at any instant is termed its instantaneous value
and is denoted by a small letter such as e . In passing, we can
observe that sinusoidal current conditions can also occur and
that the expression for a current following a sine-wave law can
similarly be written as i = I, sin cot.

LI CYC L E

---------C(

Fig 66
Following the introduction of the sine wave, as derived from a
phasor, and the generation of a sinusoidal e.m.f. by a rotating
coil, the treatment can be combined still further by the following
deduction. The phasor is assumed to be rotating at a constant
angular velocity of o radianslsecond and the waveform, if
considered to have a frequency ol'j'hertz, will stretch out to cover
in 1 second, an angle of 360fdegrees or 2nf radians. The phasor
meanwhile will have passed through ro radians in 1 second and it
l'ollows l l ~ i i t ( I J

C;III

1)c C~LI;IICLI
to 2 ~ , / '01.

3 0 0 f ' . 1 . 1 ~c;~rIicr

14 1

B A S I C A . C . THEORY

dcclt~ccdcxprcs.;ions c:in r ~ o whc writtcn


form, namely:
( > = E,,, sin 7nfr.

111

~ l i c i rmc;sr

\I\CTIII

The above is the first fundamental formula of a.c. theory and


should be given full attention. It is important to note that if
or 3.14 is substituted for rc, then the angle will be in radians
and can be converted into degrees by multiplying by 57.3. The
simpler method is to substitute 180- for rr, thus converting into
degrees directly.
Example 49. Find the instantaneous value of a 50Hz sinusoidai e.m.f. wave of maximum value IOOV, at an instant of time
0.003 seconds after the zero value.
Substituting in e = Em sin 2rcfr we have
e = 100 sin (2 x 180 x 50 x 0.003)
o r e = 100 sin (18 x 3) = 100sin 54" = 190 x 0.809
and e = 80.9V.
hnportant Note. A catch problem can occur when the instantaneous value is given and the time is required. Attention should
be given to the following example, which illustrates the point
being made.
Example 50. Find the first time after zero, when the instantaneous value of a sinusoidal current wave is 6.8A. The
maximum value is 12A and the frequency I S 50Hz. Find al.;o thc
,cconcl tirnc :il'lcr / c r o

Here i = I , sin 2x11


o r 6.8 = 12 sin (2 x 180 x 50 x r)
6.8
Thus- = sin (180 x 100 x r )
12
and 0.566 = sin (18 x 103 x 1). Further :elution of this equation can only be made by reference to sine tables, from which an
angle can be found whose sine equals 0.566.
Thus let 0 = this angle, then sin 0 = 0.566.
0 is seen to be 34" 30' o r 34.5"
*.
s i n e = s i n ( l 8 x lo3 x ( ) o r 18 x lo3 x t = 34.5

34'5 = 1.9 x lo-' = 0.0019sor 1.9rns


18 x 10
The second time value required, is obtained by finding the
time for a cycle and then subtracting the inverval, from a zero
value, necessary for the instantaneous value to attain a height of
6.8A.
Thus time for
cycle = & seconds = 0.01s.
So second time required = 0.01 0.0019
= 0.0081s or 8.1ms.
and t =

REPRESENTATION O F
SINUSOIDAL ALTERNATING QUANTITIES
Earlicr it hiis been shown that an alternating voltage o r current
can be represented by an expression such as e = Em sin 2xft o r
i = I, sin 2xft and that this method of notation conveys all that
is required to be known about the quantity, ie the fact that it
follows a waveform whose amplitude, frequency and instantaneous value, at any particular time, can be found. This method
of notation is called trigonometrical representation.
TKIC;ONOMETRICAL REPRESENTATION .

This is useful for two quantitics which are alternating, but not necessarily in the same
simultaneous manner. Thus an alternating voltage of 50Hz can
C ~ I I I ~i111
C ill1~1.11ilting
CuI.rcnt ill i\ circi~itwhich will altcmate a t
5 0 1 4 ~ .'l'he current need not however, be in phase with the
'voltage, which latter may reach its maximum value a little time
before the current reaches its maximum value. The voltage is said
to letrd the current or the current to lag the voltage. There is a
plzusr diflrrence between the two quantities or between their
waveforms and such a phase difference is shown by the inclusion
of the phase angle (in radians). Thus if two current waveforms
are represented by
i, = I,, sin 2xft and i, = I,, sin (2xft + F), it means that the
3
180
second waveform leads the first by an angle of 2 radians o r 3
3

I,, sin (2rrfr - h) is


6
180
seen to lag the first or reference waveform by --- = 30".
6
The trigonometrical form of representation, being a mathematical expression, can be used for the usual trigonometrical
o l > c . ~ ; ~ t ~\ oI I C~I ~i \ ;.I S multiplication.-division, expansion, etc and
such ~ ~ p p l ~ c a t ~will
o n sbe well illustrated in the course of a.c.
tllcoc ;IS 11115IS ~ i c ~ l o ~ x d .
= 60 . A third%aveform written as i3 =

This, commonly used for a.c. quantities


bucll as current, volr;~gc,flux, ctc, Iins already hccn introduced
in terms of vectors. In Volume I1 dealing with Mechanics a
vector was introduced and defined. Since voltages or currents
whose magnitudes and directions are known, they
arc qu~~ntitics
can be described by rotating bectors but since 'phase' is usually
i~\\~ol\~txI,
i~ is IIOW customary to represent these by phasors.
T11\1\;;I volt;~gc~ I i ; ~ s c;ln
o r be drawn to scale, its length reprcb c l l ~ i ~111c
~ gI I ~ ; I ~ , LI II ~~ 01'~C tlic voIt;~gc; I I I ~tl1c dircclion in wllich i t
P I ~ A S O RREPRFSENTATION.

143

BASIC A . C . THEORY

acts can be shown by an arrow. This technique has already been


I I I ~ I ~ O L I L I C ~~L;I I I . I I ~ I111 it115 C ' I I . I I ) ~ L; I I.I LI! , \vc L , , I I I I I O \ \ ,
OCL.L.C!
tt)
consider the accepted methods ol' phasor oper:\tlcln
The rclation bctwccn and tllc gr;~pllicaldcduc~ionol', :I waveform from a phasor has already bccn covercd. S111cct'or I I I O S I
common and practical a.c. work, waveforms and instantaneous
values are of comparatively little im?ortance compared with
magnitude and phase, the use of a phasor alone, as shown in t h e
diagram (Fig 67), makes representation much simpler. Even
further simplification and understanding can result I'rom the
correct use of phasor diagrams. Such diagrams are used to
illustrate a.c. circuit relationships and are particularly useful if
more than one current and/or voltage 1s being considered at the
same time.
Note. For most practical work r.m.s. (root mean square) values
are used in electrical engineering. The full mewing of the term
will be dealt with later in this chapter, but it is mentioned here,
because it is more convenient to make'a phasor equal to this
value rather than the maximum value-since the representation
for a maximum value holds good for an r.m.s. value. Thls
modification will however only be introduced a t a later stage.

PHASOR

DIAGRAM

Fig 67
Phase difference can also be shown by phasors. Conslder two
50 hertz sinusoidal voltages represented by El, and E,,. The

phase angle 4 is known, the voltages being of the same frequency


but out of phase. The voltages can be written as r , = El,, sln tot
and e, = E,, sin (ot- 4 ) where the angle 4 can be assumed to
be say 60" o r

X
-

radians and e, = E,, sin (wt - - ) The wave-

3 '

forms can be drawn graphically as described earl~erand slnce


the first leads the second then, if the instant when the first 14
going through its zero value is considered as the start of the
angle or time scale, the first wave can be cons~deredas the

reference and the phasor diagram can be drawn as shown in the


diagram (Fig 68).
-*

- - -

-1

.
t*

--

I-A

Fig 68
11 will be YCCII t l l i ~ t1'01.llle phnuor diuyram, wc n~crelydepict
the two phasors and their relation to each other. The first phasor
has been taken as the reference and the second is seen to lag it by
an angle 4. Direction of rotation is anticlockwise so E2, is
behind E,, by the angle $. If an instant 8 degrees later in time
has to be considered then the diagram can be drawn as shown
(Fig 69), the horizontal being taken as the zero time or reference
axis.

Fig 69
ADDITION AND SUBTRACTION O F
ALTERNATING QUANTITIES
When two or more sinusoidal voltages or currents act in a
circuit the resultant can be obtained in either of the following
ways ( 1 ) By Trigonometrical Methods (2) By Phasor Methods.
(1) ~KIGONOMETKICALMETHODS. These methods require a
good knowledge of trigonometrical identities and follow recognised procedures. Examples of their uses will occur 6n later
studies.
(2) PHASOR ME T H O D S . The resultant of two or more phasors
may be obtained (a) Graphically or ( b ) Malhernalically.
(:I) Thc Gtrrld~icrrlMrthod is performed by sctting 0111 thc

BASIC A . C . THEORY

145

pl1;isors to scale a t the given phase angles, completing the


par;illelogram or polygon and ~ncasuring tlrc ~.esult:~~rt.
'l'lrc
diagram (Fig 70) shows the method employed. Phasor addition
is shown. T o subtract a phasor, reverse its direction and proceed
as before,
1.1 the diagram, phasor addition is shown: ( I ) by completing
the parallelogram to obtain the resultant of two phasors and
then using this resultant with a third phasor to obtain the final
resultant, (11) by completing the polygon as shown. Both
methods are cumbersome and have the disadvantage that errors
are cumulative.

PHASOR DIAGRAM

PHRSOR ADDITION

--I I PHASOR

ADDITION

Fig 70

If the resultant of two individual wz qeforms is required, then


either of two procedures can be follov ed.
The first procedure uses the known fact that the sum of any
two sine waves of the same frequency 1; itself a sine wave. Thus
any instantaneous value on the resulta , t wave is the sum of the
individual instantaneous values taker from the other waves.
Each waveform is drawn graphically in accordance with the
method already outlined, care being ta.<en to displace one from
the other by the given phase angle. -ly adding instantaneous
values, as shown in the diagram (Fi : 71b), the resultant instantaneous value is obtained to give a point on the resultant
wave. In the example e = r , t e,. Ot; er points are obtained in
a similar manner and a smooth cur e drawn by joining the
points.

146

REED'S BASIC ELECTROTECHNOLOGY

Note. One waveform should always be used as the reference,


the other and the resultant being drawn to its base and zero
value.
The alternative procedure for obtaining the resultant waveform is as follows. Since the individual voltages or currents can
be considered as phasors, the resultant of any two or more
values can be obtained in the same manner, as already described
above. If the parallelogram is completed, as in Fig 71a, the
resultant Em will give the maximum value of the wave of the
resultant. Using Em as the radius of the largest circle, the
resultnnt waveform cnn be deduced as before. As an example, if
Elm= 10 volts, E,, = 6 volts and the phase difference is 60,

(a)
Fig 71
(b)
then the resultant Em would be found to be 14 volts,'~, woulg
also be found to lag 21" 45' behind El,. The same procedure as
to magnitude and phase angle would give the resultant r.m.s.
value, if r.m.s. values were used for the component values
instead of maximum values.
The second method, of obtaining the resultant waveform as
described above, is obviously the quicker method if the componcnt wavcfoniis arc not required. The resultant phasor may
be obtained graphically or by one of the mathematical methods
: I S detailed next.
(b) The Marhematicul Method can be performed( in one of'
two ways: (i) by using the Cosine Rule, (ii) by resolving into
horizontal and vertical 'components.
(i) The Cosinc Rule or a modification of the sarne, can be used
to advantage if the resultant of only two phasors is required.
Consider the diagram FI 72 . T h ~ nthe resultant E can be
oht;tinc<lfmln E - l
'
i
~ wl~creEr, and

BASIC A.C. THEORY

147

E , are the given phasors and b, the angle between them. The
ph:~sr :tnplc 0 of Ihr r c s ~ ~ l l : ~c:ln
n t hc oht:~inrtlfrom thr Sinr
Rule.
ti
1:
= -_.L
Thus
s i n ( 1 8 0 - 4) s i n 0

Fig 72
For more than two phasors, the resultant is used with a t h r d
phasor and so on. The method, next to be described, is advocated for the summation of more than two phasors, since it is
quicker and is to be encouraged, being the method on which the
treatment of series and parallel a.c. circuits is based.
(ii) Horizontal and Vertical Components. Any phasor can be
split into two components, which are at right angles to each
other and together produce the same effect as the original
phasor. Thus in the diagram (Fig 73) the e.m.f. phasor E can be
split into a horizontal and a vertical component. If E lies at an
angle 4 to the horizontal, then {he horizontal component will be
E cos (I, a n d the vcrtic;tl cotnponcnt will hc f : ' c o s (00 (1)) o r
E sin b,

Fig 73
Fig 74
If all the phasors as shown in the diagram (Fig 74) are to be
added and the resultant obtained then:
The sum of the horizontal components would be
EH = El cos
EZ cos 42 + E3 cos 4,.
Similarly the sum of the vertical components would be
E, = E , sin dl
E, sin 4, - E, sin 4,.

+
+

Note. Due allowance must be made for the signs. Thus if the
vertical components are considered to be +ve when acting
upwards, then E3 sin 4 , must be subtracted from the sum since
it acts in the downwards or - ve direction.
and r$,
The resultant E is obtained from E = d m Z
the angle at which it acts, can be found from the sine, cosine o r
tangent values. Thus cos

EH
-.

E
The method is illustrated by the following example.
Example 51. Find the resultant of the following currents
I , = 5 sln wt
jl =
i,

4 sin ot
3 sin

9
-

g)

kul

Express the resultant ii a trigonometrical form ie in the same


form as the individual quantities.

Fig 75
The diagram of Fig 75 should be considered with this solution.
Then I, = 5 cos 0 + 4 cos 60 + 3 cos 30
= ( 5 x 1)
(4 x $) (3 x 0.866)
= 5 + 2 + 2,598

--

0-59XA

5 sin 0 + 4 sin 60 - 3 sin 30


(5 x 0) + (4 x 0.866) - (3 x +)
0 + 3.464 - 1.5
' = 1.964A
7--1
t I , = \/9.59s2 t 1 ,9642
From whicll I = \I,,
-- \ '9)-TT-,-g= v'95.87 = 9.78.4
And I,

=
=
=

~~

149

BASIC A.C. THBORY

= 12" (approx) = 15 radians

+.
(
;5)
Note. The following points are of interest in this example.
So i = 9.78 sin wt

(a) In line with mathematical practice, phasors drawn to the


right and those drawn upwards are given +ve signs, whereas
those drawn to the left and those drawn downwards are assumed
-ve. Thus in the example all the I, components are +ve. The
phasor diagram should be considered. For the I,, it will be seen
that 3 sin 30" is considered to act downwards and is therefore
subtracted from 4 sin 60" which acts upwards.. (b) The resulting
sign of I, also indicates whether the. resultant I is in the 1st or
4th quadrant, ie whether it lags or leads the reference which in
this case is the horizontal. In the solution 4 i ~ f o u n dto approxi7T

equal to - radians. The


15
resultant can thus be written as shown.
It will be noted that in the treatment introducing the Mathematical Method and in the above example (Figs 72 to 75), the
suffix m has been omitted from the e.m.f. symbol E. This is intended to illustrate that, as stated earlier, the method is equally
applicable to maximum values and to r.m.s. values. The meaning
of r.m.s. values will next be considered but as these are the most
commonly used in a x . work, it is important to appreciate that
phasor representations, applications and solutions will be used
without further introduction.

mate to 12' which is

ROOT ME A N SQUARE AND AVERAGE VALUES


R.M.S. OR EFFECTIVE VALUE
The magnitude of an alternating current vanes from instant
to instant and the power dissipated in a resistance varies
accordingly. The energy given out over a period of time manifests itself as heat. A resulting temperature rise is attained, which
is steady and can be considered to be due to a constant power
dissipation, ie due to the passage of a constant current which
gives the same heating effect in the same time. Thus from a hea,ting aspect, any value of alternating current can be assumed to
have an equivalent value of direct current. WFen it is remembered
that the heating effect is proportional to 'current squared since P = 12R, then the magnitude of this equivalent value can
be deduced as follows.
7

150

REED'S BASIC ELECTROTECHNOUX~Y

Let I amperes be the equivalent direct current which has the


same average heating effect as the alternating current of varying
instantaneous value i amperes.
For the d.c. condition
Energy expended = (current2 x resistance) x time
= PRt
hor the a.c. condition
Energy expended = (mean or average of i2R) x time
= (mean or average of i2) x R x t
By assumption since both energy conditions are considered to
be equal then I ~ R
= (mean
~
or average of i 2)Rt
or I = Jmean or average of' i
Thus the effective o r r.m.s. value of an alternating current is
the square root of the mean or average of the squares of the
instantaneous values. This is true for the shape of any half cycle.
Note. The above deduction shows where the term 'root mean
square' comes from, as the alternative to 'effective'. In practical
electrical engineering, the term r.m.s. is the most commonly
used, but a further older and little used alternative is the virtual
value. Thus r.m.s. value, effective value and virtual value all
mean the same.
The following definition will be found useful.
'The r.m.s, or effective value of an alternating current or voltage
1s that value of direct current or voltage which, when passed
through or applied to a given resistor for a time of 1 cycle,
produces the same amount of heat as the alternating current or
voltage.'
&
It will be noted that in the definition, voltage has been mentioned although the r.m.s. value of a voltage wave was not
specifically mentioned earlier. It will be seen that the heating
effect on a resistor of value R ohms, has been taken as the basis

vZ
R

of discussion, but an alternative to P = P R is P = , so a


r.m.s. value of an alternating voltage wave of instantaneous value
v , could have been deduced in a manner similar to that set out
earlier.
The r.m.s. value of a waveform can be obtained graphically as
shown in the diagram (Fig 75). To do this, the instantaneous
values of current or voltage should be plotted to a time or angle
base, suitable scales having been chosen. Subdivide the base o f
one half cycle into equal divisions and erect the mid-ordinates
I , , i : , i,, etc up to in. Measure these to scale and substitute in the
cxprcsslon

151

BASIC A.C. THEORY

I=

tll

+ i Z 1 + 1,' +
n

, , , in

It will be seen that only a half cycle has been 'considered,


because the next half cycle is similar to the first, even though it
is considered -ve. As the square of the current ordinates is

Fig 76

required, +ve values will result and the r.m.s. average, if taken
over a complete cycle will be the same as for a half cycle. It
should be noted that the Mid-ordinate Rule has been applied to
the ordinates squared and not to the ordinates directly. If the
latter had to be done, the average value would be obtained. This
will be described later in the chapter.
Example 52. The following are the. results of measurements
.taken at intervals over a half cycle of alternating voltage:
Time (t milliseconds)
0
0.45 0.95 1.5 2.1 2.5 3.1 3.9 4.5 5.0
Voltage (V volts)
0
20 36 40 37.5 33 32 31 20
0
Calculate the r.m.s. value and frequency of the wave.
The solutiofi is shown in the diagram (Fig 77), from which i t
can be seen that, if the watieform is plotted to suitable scales,
then dividing the base into ten equal parts, mid-ordinates can be
drawn and measured as indicated.

I
I

Fig 77

Then v , = 12
v 2 = 32
v3 = 39,s
V , = 39.5
v , = 35
V , = 32.5
\!, = 31 5
L,, = 3 1
rJ, =

and v I 2 = 144
v z 2 = 1024
v3' = 1560
= 1560
v S 2 = 1225
v,' = 1056
v T 2 = 992
v 8 2 = 961
v,' = 784
v L O 2 = 121
Total 9427

28

= 1I

- 9427
---- = 942.7
Mean or average of v 2 10

1t.M.S. valut:

d942.7 = 10d9.427

= 10 x 3.07 = 30.7 volts

Tlme for a half cycle


i \ ~ l t -.T

[:or

1 x 5

I ocXj

;I \IIIC'

5
lo00

= ----th

seconds, so time for a whole

I second :ind frequency


loo

L L . ; I V C , I ~ I Cr.111

L,

100 hcrrr.

V ; I I I I C can hc sllown rnallicrnal~c;~lly

153

BASIC A.C. THEORY

--

to be 0 707 times the maximum value. The most direct approach


involve\ 11 knowlcclgr of' Cnlculus but tlir ftlllnwit~ggrnphicnl
method illustrates the relationship.
Consider an tllternat~ngcurrent of sinusoidal wi~veformhaving
maxlmum or peak value of 4 amperes. The current-squared
curve can be plotted to represent the instantaneous values of the
heating effect; the mean height of this curve can be determined
and will be found to be half of the maximum height, ie half of
16 = 8. This is shown on the diagram (Fig 78). The number of
amperes of continuous ccrrent that would give the same heating
effect will be 4 8 = 2.828 amperes and thus is the square-root
of the mean of the squares of the current, ie a true r.m.s, value.
2.828
The ratio of r.m.s. to maximum value = 4

R.M.S. value
or
- 2.828 - 0.707 and this ratio is true for
Maximum value
4
1
any alternating current or voltage which is sinusoidal.
As mentioned earlier, the r.m.s. value is always used and
assumed in practical electrical engineering. Sine-wave working
is also assumed and any departure from this will always be

I5

)r
C

z1
0

"W

p:
d

u
3
3
I-

'25
a

50

60

90

1
2
0

DECREES

Fig 78

I
5
0

1
8
0

154

REED S BASIC ELECTROTECHNOLOGY

clearly stated. Thus if a supply voltage is given as 220V, this


would be the r.m.s. value and actually the voltage varies over a

220

cycle between zero and - = 31 1.2 volts.


0.707
AVERAGE VALUE

This is the true average value,-as is understood mathernatlcally, of a half cycle since that of a full cycle is zero. The average
value of a waveform is of particular interest for devices which d o
riot depend on the effect of current squared. An example, of such
a device, would be a rectifier. Rectifiers have been developed to
t u n v c r l i~itcrilntingcurrent into direct current, withaut the need
of rotating pachinery, and are available in various forms. In the
domestic type of 'mains' radio set, gas or vapour-filled valves are
provided which convert the a.c, mains voltage to a direct voltage
for operating the other valves in the set. Alternatively a 'metal'
or semiconductor rectifier may be used, since these are being
developed to suit all requirements. For the marine industry gas
or vapour type rectifiers, the most common of which is the
'mercury-arc' rectifier, have not been used to any extent for
shipboard work, although they may be seen in shipbuilding a ~ l d
dockyards for providing the 'shore supply' to d.c. ships. Metal
and semiconductor rectifiers are however, being used in various
forms for a x . ships, in sizes both large enough to supply dirpct
current for the 'de-gaussing' gear and small enough to build into
moving-coil indicating instruments, to enable these to be used
for a.c. circuits.
The d.c. current or voltage has a value equal to the average
value of the a.c. waveform being rectified and for a non-sinusoidal wave it can be obtained graphically, as follows:
Referring to the diagram (Fig 76) of the waveform cons~dered
earlier, let I,;, equal the average value, then
Example 53 Cons~der :he same waveform and ord~n:ttec
the \ ~ i b j ~ c01 t L X , I I I ~ ~52I C 1-lnd tllc ,~vcr:~gc
vLiluc
1
2
+
3
2
+
3
9
54-39
5
+
3
5
+
3
2
5+31
5-1 31 +- 2 8 + 1 1
lIc1c I., --

\ + l l ~ c l ,lI I C

lb--

10

292A or Average value

292A.

For a sine wave the average val~leis 0,6365 times the maximum
value. This can be proved mathematically or graphicallv. As f o ~
the r.111.s.value. the most direct method of calculation involves a
knowlcdpc of C;~lculush r ~ :tlic grnphic;iI mcthod can he checked

BASIC A.C. THEORY

155

hy plotting s ~ i n ewnve, to n time or angle base, Sub-divide the

base into equal divisions, erect the mid-ordinates and obtain the
average value by using the mid-ordinate rule or substituting in
the expression
As stated earlier, the ratio of average
- to maximum value is
2- or Average value - 0.6365. This ratio is true for
rr
Maximum value
3:14
any alternating current or voltage which is sinusoidal.
FORM FACTOR

This factor, when given a nunlerical value, states how near a


waveform approaches the theoretical ideal sine wave. For any
waveform, it can be defined as the ratio of the r.m.s. to the
t
average value.
R.M.S. value
Thus Form Factor =
Average value
For a sine wave, the Form Factor is 1.11. This can be obtained
from :
0.707 Maximum value 0.707 = 1.11
-Form Factor =
0.6365 Maximum value
0.6365
Example 54. For the problem already considered, in obtaining
the r.m.s. and average values. the form factor would be

PEAK FACTOR

The term 'peak factor' may occasionally be encountered when


dealing with a.c. waveforms. This can be defined as the ratio of
the maximum value to the r.m.s. value. Thus:
Peak Factor = Maximum value
R.M.S. value
For a sine wave the Peak Factor would be

CHAPTER 7

EXAMPLES

PRACTICE
1.

2.

Three circuits carrying currents of I,, I, and I, are joined


in parallel. I, = 4A, I, = 6A, Lagging 30" on I, and
I, = 2A, leading 90" on I,. Find, by a phasor construction
drawn to scale, the resultant current and its phase angle with
reference to current I,.
A sinusoidal, 25Hz a x . voltage has a maximum value of
28208V. Find tho time interval, after the zcro valuc, when

the voltage wave reaches (a) its first, (b) its second instantaneous value of 200V.

3.

A sinusoidal e.m.f. of lOOV maximum value is connected


in series with an e.m.f. of 80V maximum value, lagging 60"
behind the IOOV e.m.f. Determine the maximum value of
the resultant voltage and its phase angle with respect to the
lOOV e.m.f.

The following figures give the instantaneous values of an


current, which varies smoothly over one half
cycle.
6

Time (milli-seconds)
0

Current (amperes)
0
0.4
Time (milli-seconds)
8

0.75 1.1

1.4

1.7

1.9

2.0

10

Current (amperes)
1.8 I 3 0
Plot the curve ot'current and find its r.m.s, value. Calculate
when the above current flows through a
the power diss~p;~tecj
reslslance ol' 811.
5.

Three currents of peak values 10A, 17.32A and 20A


respectively meet in 21 common conductor. Thc 17.32A
current lags the 10A current by 90 electrical degrees and
leads the 2OA current by 60 electrical degrees. Draw a
phi~sordi;~gr;lmi~ndfind the v;~lucof the, result;~ntcurrent,
giving its pli~isereli~tionwith respect to the IOA current.

157

BASIC A.C. THEORY

I
I

6.

An alternating valtnge, in the form of n aine wave, havinu


a peak value of 340V, is applied to the ends of a 24n
resistor. Calculate the r.m.s, value of the c u m n t in the
resistor.

1
I
I

7.

Represent by phasors, the following e.m.fs., e l = 100 sinot.

e, = 50cos o f , e, = 75 sin
I

Determine by calculation
el
e,
e,
e, = E sin (ot 6).

+ + +

8.

Two alternators are arranged to be coupled to the same


prime-mover in a manner which allows the phase-angle,
between their generated e.m.fs., to be varied. If the machines
are connected in series and generate lOOV and 200V
respectively, find the total output voltage when the phase
difference is; (a) zero, (b) 60,(c) 90, (d) 120, and (e) 180 .

!C

9. . A stepped alternating current wave has the following


values over equal intervals of time.
Value (amperes)
4
6
6
4
2
0
0
- 2 - 4
Time Interval (seconds)
0-1 1-2 2-3 3-4 4-5 5-6 6-7 7-8 8-9
ClC.

P16t the waveform and find what value of direct current


would give the same heating effect.
10.

The 50Hz e.m.fs., induced in four separate coils of a n a c


generator, are each of maximum value 4V and are successively 10 degrees out of phase. If these coils are connected
in series, find by calculation and phasor construction, the
resultant maximum value, expressing this in the form
e = Em sin (wr O), where 8 is the angle of phase difference
with respect to the first coil.

CHAPTER 8

THE D.C. GENERATOR


It is not intended in this book to cover the finer points of d.c.
machine construction, operation and maintenance since these
matters are better appreciated if dealt with in a practical
manner. It is necessary however, to set out here the basic features
of the machine as it has been developed to date and also to follow
up on the theory already commenced in Chapter 6. Once the
general construction of the machine is outlined, full attention
crln tllcrl bc given to furthcr tllcory und it crln bc connidcrcd
from its functional point of view, which classifies it under the
heading of (1) The Generator (2) The Motor.

D.C. MACHINE CONSTRUCTION


The principal features of the machine will be described under
(a) the field system o r stator and (b) the armature or rotor.
I N TERPOLE

l-3SYMBOLS USED
FOR THE D C
MACHINE

FIELD
SERIES
F l E LD

P
Fig 79
( a ) FIELD Sl'S'TEM
This includes the magnet arrangement comprising the poles
;111(i Y ( I ~ C .tlic field <oils : ~ n dititcrpolcs (whcn fitted). Notc t h ; ~ t
the interpoles are essentially part of the armature electrical
circuit and will be mentioned later under this latter heading.
I > O I . I S A N D \ . O I < I : . Tllc Sot.mcr arc thc cores of the machine
electromagnets and are usually fitted with pole-shoes which

iotlct'ntrate t l i ~ticlcl across the air-gaps in which the conductors


riio\.c Thc yokc is ;In extcnstorl of the milpnct system, forms the
111;1111 I'~.;~rnc.
ol' [he machtnc a n d sc.r.ves 10 carry flux from and to

--

THE D . C . GENERATOR

159

the poles. The diagram (Fig 80) shows typical ways in which the
flcld system cnn hc constructed,
The poles and yoke can be constructed from cast steel or
fabricated from mild-steel sheet which has been cut and rolled

Fig 81

160

REED 'S BASIC ELECTROTECHNOLOGY

into shape. The poles may be part of the yoke, but for modern
machine construction, they are more usually- built u p from thin
laminations, riveted together and shaped to include the pole
shoes.
field coils are basically of two types (i) Shunt
Coils which consist of a large number of turns of fine wire a n d
(ii) Series Coils which are made from a few turns of thick cable
or conductor. The shunt coils are built u p on a 'bobbin' o r
'former' but the series coils may be self-supporting. The diagram
(Fig 81). shows a typical cross-section of construction; the
~nsuliitionused is decided by the class of machine and its duty.

HELD COILS. The

wnoLE C O I L
VARNISH
IMPhtCNAllD

COTTON
TAPE
FORMER OF
LEATHERDID 0

D O U B L E C O T T O N C O V E R ED
OR ENAMELLED WIRE

Fig 81
(b) THE ARMATURE
This consists of the armature core, the windings, the shaft
and commutator'The brushes, although not part of the armature, are considered here since they work in conjunction with
the commutator.
This is built up from iron laminations which
are clamped between two end plates. The laminations are
insulated from each other to minimise induced circulating
currents, called 'eddy currents', and if clamping bolts are used
to pass through the core as shown in the diagram (Fig 82), then
thcy must be insulated. Modern techniques use stamped lamina[Ions w h i c h ;Ire pressed onto and 'keyed' to the sllnft, the end
plates being screwed onto the shal't for smaller machines. For
larger designs a 'spider' is employed which allows ventilation and
keeps the iron required to a minimum.
A R M A T U R E CORE .

M.INDINGS. Tile number of conductors, their size, shape, etc, are


decided by thc design requirements for the machine. The
d i ; ~ g r . ; ~(~Fni g 8 3 ) \bows :I ~ y p i c a l method of locating and

161

THE D . C . GENERAT O R
LAMINATIONS 0 1 W

6 1 0 ~YlPt
~ 0 1 CONSTRUCTION

.LAHINATIONS

MORE MODfPNCONSTRUCTION

Fig 82
holding the coil sides in position. In the sketch, a wedge made
of' bitkelised pitpel or b;tkclised I';tbric is sllown, but open slots
with a closing piece of fibre and 'binders', made from hightensile steel wire, are frequently used. For small machines, mainly
motors: as for vacuum-cleaners, cabin-fans, etc, the armature
windings consist of enamelled or cotton-covered wire placed in
position by hand winding. Semi-enclosed slots are used with a
fibre insert closing the slots.
The method of arranging the conductors to form a closed
winding will be considered in greater detail after the machine
construction has been dealt with.

Fig 83

SHAFT. This is made from the best forged mild steel and is
designed so that it will not deflect unduly when running up to its
maximum speed.

COMMUTATOR. This consists of copper segments, insulated from


each other by mica. The thickness of the top of a segment may
be up ta $' and the segments may be mounted on but insulated
from a sleeve, which is secured to the shaft, and are clamped
by an end-ring which can be bolted or screwed as shown in the
diagram (Fig 84). Insulated conc-sh;~ped rings, made from
micanite are used to insulate the segments from the stee! clampi ~ i g;~ssc.ml~ly.
Tllc ;1r-nl;rturc \\,inditlgs ; ~ r chrottpllt ollt and
soldered ro the segments. The mica must be undercut between
:~di:~ccnc
segments.

Fig 84
BRUSHFS. Any one brush is pressed onto the commutator by
means of the pressure arm and is connected to the holder by
means of a pig-tail of braided copper wire which is moulded into
the brush. One or more brush-holders may be carried on an
insulated spindll! which is mounted Gn the brush rocker-ring.
The brush r~cker-ringis arranged to be clamped firmly once
the brush position has been set. Brushes of modern d.c. machines
are always of moulded carbon and graphite, the grade of hardness being chosen to suit the running condition;. The diagram
(Fig 85) shows a typical arrangement.

Fig 8 5

1 63

THE D . C . G EN ERAT OR

BEARINGS.
For .most industr~illd.c, m:~chincsthc bearings are of
the ball or roller type. Advantages are (i) its axial length is
shorter than that of the journal type (ii) after initial packing with
grease, service for a long period can be obtained. Journal bearings, ie sleeve types, give quieter running and are frequently
preferred for marine work,. since they resist 'transmitted'
vibration troubles better. They are usually of the 'ring-oiler'
pattern. The steel shaft runs in a brass or cast-iron sleeve lined
with white metal. For small and medium size machines, the
bearings are. carried in the end shields, but for large machines,
the bearings are carried in separate pedestals.

D.C. ARMATURE WINDING ARRANGEMENTS


The simplest winding possible would be built up from singleturn coils of span equal to one 'pole pitch', ie 180Yor a two-pole
machine. For a four-pole machine the coil sfan would still be
one pole pitch but now 90 mechanical degrees. This is illustrated
by the diagram (Fig 86).
I POL PITCH
r90*MKHANCAL

I WLE P I T H
~ 1 8 0 .MECHANICAL

QQr
Fig 86
In practlce it is not usual to make the span equal to one pole
pitch exactly and many small machines have an odd number of
slots. Each slot carries two coil sides, ie it contains more thafi
one conductor. D.C. windings are usually of the 2-layer type, a
a l l side lying at the bottom of the slot and another at the top.
Sometimes more than 2, such as 4 , 6 or even 8 coil sides may be
contained in 1 slot since it may not be practicable to have too
many slots. There are two basic methods of connecting up the
conductors on an armature after they have been formed into
either single or multi-turn coils, and the complete winding falls
into one of two distinct types namely (a) a wave winding or (b) a
lap winding.
(a) The W A V E or two-circuit Winding. This winding results in
there always being two paths in parallel irrespective of the

number of poles of the machine. Two sets of brushes only are


necessary but it is usual to fit as many sets of brushes as the
machine has poles. The diagram (Fig 87a) shows the essential
layout.

(a>

SEGMENT I

(b)

Fig 87
(b) The L A P o r multi-circuit Winding. This winding results in
as many paths in parallel as the machine has poles. There are a s
many sets of brushes as the machine has poles. The diagram
(Fig 87b) also shows the essential layout.
In building up a winding it is essential to connect coil elements
in such a manner that the induced c.1n.f. in the conductors add,
in much the same way as cells are connected in series so that their
e.m.fs. add to give the required battery voltage. Thus conductor
X i s in series with conductor Y which occupies relatively the same
position as X but is under a pole of reversed polarity. The coil
element so formed by conductors X Y should then be connected
in series with a similarly placed coil element under a pair of
poles so that the required voltage for a parallel path of the
armature can be attained in this manner. For a wave winding
the connection can be readily seen from the diagram already
introduced and for a lap winding the same rule is followed.
cxccpt [lint all thc coil clenicnts under n pais of poles are connected in series before the winding progresses to connect up the
condl~ctorsunder tllc nrut p;lir of poles.
The example, which follows, .will illustrate both simple lap and
wave windirigs. A small armature is to be designed to have oneturn coils-one turn comprising 2 conductors. There are to be 8
coils. There will be I co:nmutator segment to a coil, ie 8 commut;\tnr segments. If only 2 coil-sides arc to be accommodated
in n slot then there must !>e ti armature slots and if a four-pole
hystcm is to Ilc ~~sccl
[ I I C I I tI1t1.cwill bc 2 sIots,I~>oI~.
This will give
it fnle pnlc pitch 01' 3
thc pole pitch being thc n i ~ ~ n b eofr

165

THE D.C.GENERATOR

armatura slots divided by the number of polcs. Sincc the sides of'
a coil should be under the influence of the correct field poles, the

winding pitch must be as nearly as possible cqunl to the pols


pitch. Thus the winding pitch would also be equal to 2 o r a coil
should embrace 2 teeth.
The L A P winding is considered first, being suitable for this
armature. Now for such a winding, the connecting up of the
conductors is such that the winding progresses round the
armature by being pitched alternatively forwards and backwards.
For our example, if (Fig 88) is considered, it will be seen that
conductor No 1 is connected to No 6 which is spaced 2 teeth
away. No 6 is then connected to No 3 and so on. The winding
thus progresses by 1 slot until it is closed by all the slots having
been occupied and conductor No 15 being connected to No 1
through N o 4.
#.

Fig 88
If now, for our example, a W AV E winding Is required then a
preliminary examination would show that this could not be
achieved. If the winding started at No 1 proceeded to No 6 and
then on through Nos 9 and 14 it would close back onto conductor No 1. It is obvious that an armature with 8 slots would
not be suitable for such a wave winding and one of 7 or 9 slots
should be considered. A nine-slot armature winding would give
a winding pitch of length slightly less than the true pole pitch
length and is considered as suitable. Consider now the diagram
(Fig 89). Here conductor No 1 is connected to No 6 as before
which in turn is connected to Nos 9, 14, 17 and then to No 4, ie

%C~*CT~CAL CWCCTWU

Cplr

Fig 89
the winding passes into the slot beyond that at which the start
was made. The winding, thus does not close immediately and if
the connecting-up proceeds as described, it will be seen that the
winding will progress four times round the armature before the
close is made at the starting slot by conductor No 1 1 being
joined to No 1 through No 16. This then would be a suitable
winding but 9 coils would be used with 9 armature slots and 9
commutator segpents.
More details on armature windings will be found in a book
dealing more fully with the practical subject since machine
design and armature winding is specialists' work. However, it is
of interest to find where the brushes are to be placed on the
co.mmutntor and one accepted way is to draw out the 'equivalent
ring' winding,
EQU I V A L E N T K I N G U ' I N D I N C ; ~ . The diagrams (Figs 88 and 89)
?;l\c>\t Iiow tllc c o ~ i ~ ~ c c t iol'
o ntllc conductors c ; ~ nhc sct out. On
the armature winding diagram, current flow is assumed in the
c o ~ l d i ~ c t o r~rndcr
s
tlic polcs. this current flow hcing duc to t l ~ c
induced e.m.1'~.'l'hus ~ t ' I'or
, the conductors under a N pole, the
direction of the current is assumed from the bottom of the page
to the top, then for those under a S pole, it would be from the
top of the page to the bottom. If next, the winding is drawn out
its a ring winding. as shown at the bottom of each figure and the
correctly then:
conductors are ~nk~rked
1;o1.fhc I ;rp wintling N o I sliot~l(iIc*;~d
to No 6 and thcn onto
3 ; ~ n t lso on.

------

THE D.C. GENERATOR


- -- - -- -

167
--

For the Wave winding No 1 should lead to No 6 and then onto


9 and so on.
The assumed current direttions can be transferred frorn tlie
annature winding and druwn in to s t ~ o w111i11 :
For the Lap winding the currents meet at the junctions of Nos
8 and 5 , I6 and 13. These would be i-ve points or the brushes
should be placed on segments Nos 3 and 7-these being + v e
brushes. The -.ve points would be a1 the junction of Nos 4, 1
and 12, 9 or the -ve brushes should be placed on segment Nos
1 and 5. The ring winding also shows that there are 4 paths in
parallel between the +ve and - ve terminals of the machine.
For the Wave winding the currents meet at the Junction between conductor Nos 6 and 17. It will be noted that nc current
(indilced e.m.f.) has been attributed to conductors which are in
the position between the poles. Thus a +ve point is at the junctiorl
6-9 o r 14-1 7, there in effect being no potential d ~ in ploop 9 and
14. A brush could be placed at either commutator segment No 4
or 8 and to maintain uniformity, arushes may be placed at botl:
these points and connected as sllown to form the +ve termina!
of the machine. Similarly - ve points occur at the junctions of 2
and 13 or the actual joints 2-5 and 10-13. Brushes may be placed
at these points, there being no current (induced e.m.f.1 in loop 5.
10. As for the +ve terminal, brushes are placed on segment Nos
2 and 6 and ,joined togcther to form the - ve terminal. The ring
winding, ; ~ l s oshows t l i : ~tI1er.c
~
:Ire 2 p:1111\In p:~r;~llel
for :I w:~vr
winding.
T H E D.C. GENERATOR
Theory of the d.c. armature and commutator has shown that
commercially, direct current is best obtained by using an
armature wound with a number of coils so connected, that all
the coils, except those being short-circuited by the brushes, are
in the circuit. The armature being a continuous closed winding
splits itself electrically into a number of parallel paths. It has
also been shown that there are two fundamental ways of winding
an armature (a) with a lap winding or (b) with a wave winding.
An important rule can now be stressed which is that:
For a lap winding the number of parallel paths in the armature
is always equal to the number of poles.
For a wave winding the nu'mber of parallel paths is always
two, irrespective of the number of poles provided for the
machine.
Interpoles or commutating-poles, sometinies termed compoles, d o not perfarm any function' of the main poles and are to
be disregarded for the rule just enunciated:

THE E.M.F. EQUATION

Consider the diagram (Fig 90) and the factors for a machine
as glven below. A simple expression for the composite grmature
is now deduced and it is stressed that this is of the utmost
importance. I t must be memorised and the student should be
capable of proving it from first principles.
Let N = the speed of the machine in revlmin. P = the number of poles. @ = the flux/pole (webers). Z = the number of
armature conductors. A = the number of parallel paths of the
armature winding.

Fig 90
N .
In one second the armature revolves - times
60

and in 1 revolution, one conductor cuts a flux of P x @


webers
N
:. In 1 second one conductor cuts P@ - webers
60
From F a r a d a y " ~law the magnitude of the e.m.1'. generated in
volts is given by the flux cutjsecond
P@'V volts.
so the e.m.f. generated in 1 conductor = -

60

If the armature winding is divided into A parallel paills then


the e.m.f. of one parallel path is also the e,m.f. of the machine.
7
Now in a par:illel path there are conductors in series,
A

P@N 7
so e.1n.l'. ol' I parallel pat11 = e.rn.1'. Iiol'the machine = -- x 2
60
A
P
Thus E = Z@N x - volts. where E = the generated voltage.
60
A
Example 55. The armature of a four-pole, shunt generator is
lap wound and generates 216 volts when running at 600 rev!min.
The armature has 143 slots with six conductorslslot. I f this
armature is rewound and wave connected. determine the e.m.f.
generated at the same speed and fluxipole.

TH E D . C . GENERATOR

From t t ~ ce.m.f.cqrlntion 21 6

169

--

216 webers
60 x 144
Note. This is a Lap-wound armature so A = P = 4.
For a Wave-wound .armature A = 2

or @ =

CHARACTEKISTlCS

By characteristics are meant graphs which show the behaviour


of any type of machine when under.investigation. As an example,
consider the e.m.f. equation. It is noted that, far any given
machine, all the factors except @ and N are constant. The
ZP
equation can be written E = k @ N where k = 60A
Thus E a 0,if N is kept constant and E cc N , if @ is kept
constant.
If @ and N are both varied then E will vary accordingly. Thus
we see that the voltage generated can be controlled by varying
the speed or the flux of the machine. This can be shown by
deducing the 'no-load' characteristics.
ASSOCIATED MAGNE'T'IC' C'I IIC'U17' EFFECTS
Since the magnetic circuit forms an essential part of the d.c.
machine, i t is necessary here to consider two effects which
influence generator characteristics. U p to now the electromag~et
with an iron core has not been covered in our studies and it is
therefore necessary at this stage, to mention briefly two of the
factors relating to the magnetising of an iron or steel specimen.
The first noticeable effect is that of residual magnetism.
Experiments with a piece of iron will show thai when such a
specimen is made the core of an electromagnet and it. is magnetised by passing a current through the energising coil then, when
the current is switched off and the magnetising m.m.f. has been
renioved, the magnetism o r magnetic flux will not completely
disappear. In other words, some magnetism remains when, in
theory, it should be non-existent. Such residual magnetism will
be discussed fully in Chapter 12 and for the moment it will be
sufficient for the reader to appreciate that the effect does occur.
The second factor which is of importance, is the saturation
efSect of an iron sample when it is being subjected to a magnetomotive force. If a magnetic circuit uses iron as the medium for

conveying the flux then, as the magnetomotive force is increased,


the flux increases in proportion up to a stage, when the straightline relation between the m.m.f. and the flux @ or the consequent
flux density B, is no longer followed. Thus if, for an iron sample,
the B value is plotted against H-the magnetising forcelmetre
length, a graph called 'the B-H curve' is obtained. The method
of testing a sample and a detailed explanation of the results
obtained will also be discussed in full in Chapter 12, but the
point being stressed here is that the resulting graph will be a
straight line for only a short section of its length. It will be of the
form shown in the diagram (Fig 91) which indicates that the iron
sample appears to saturate, ie no matter how much the m.m.f. is
incrcascd, once the curve bends over and flattens out then i t
would be apparent that no increased flux 0 or consequent B value
would be produced, irrespective of the strength of the magnetising force.

Fig 91
The flattening out or saturation effect is considered to be due
to r ~ l lthe molecular magnets having oriented themselves in the
direction of the magnetic field and thus, it can be expected that,
the saturating effect will make itself apparent when investigating
the relation of rhc gcncratcd voltagc E lo thc flux 9 in thc
magnetic circuit of the d.c. machine.
THE NO-LOAD CHARACTERISTIC

Theory of the machine has so far shown that the generated


voltage is dependent on the machine flux and speed and the nolo;~dcIinr;~ctcristics,n1rc;tdy mentioned, can now be considered
as detailed under the following (:I) and ( b ) headings.

171

THE D . C . GENERATOR

(a) V A R I A n o N OF E WITH N (Flux Qr constant). A pennanentmagnet generator is seldom used for practical applications, but
the investigation to be considered, can be made by controlling
the current of separately-energised field electromagnets. This
current, referred to as the field current I, when flowing through
the field coils, gives effect to a m.m.f. which results in the flux in
the air-gaps. If this current is kept at a constant value I,,, then
the flux will be constant, and tests can be made by varying the
speed at which the machine is driven and by noting values
together with the corresponding voltage being generated.
Since flux @is constant.and as E a N, a straight-line graph as
shown by (1) of the diagram (Fig 92), will result. If the field
current is next adjusted to a smaller value I,, and is kept constant, then when the test is repeated, a straight-line graph such as
(2) will result and the deduction assumed, namely that E varies
directly with N, will be proved.
C

-----cN

Fig 92
(b) VARIATION OF E WITH @ (Speed N constant). As explained
above, variation of flux can most readily be effected by controlling the energising current I, in the field coils or the 'exciting
,current' as it is often called. If no residual magnetism is presznt in
the field system then, if I, is increased, the m.m.f. is increased and
the flux in the air-gaps increases. The generated e.m.f. increases
accordingly and a B-H type of curve (1) as shown in the diagram
(Fig 93), is obtained if E is plotted to a base of I,. Note. @ cannot
be readily measured but its effects can be gauged by knowing
the appropriate values of the exciting current.
Curve ( I ) at first increases as a straight line, flattening out to a
horizontal as the magnet system saturates. When saturation has

occurred, if the field current is reduced, Curve (2) will result. This
curve will be found to lie slightly above the original curve ( I ) and
it is seen that for decreasing values of I,, the values of E are
above those obtained for the ascending curve (1). The cause of
the difference between curves ( 1 ) and (2) is Magnetic Hysteresis,
which will be fully discussed in Chapter 12. When the field
current is eventually reduced to zero, some generated e.m.f. is
found to be present while the machine is run at the constant
speed N. This e.m.f. is due to the residual magnetism; which will
be shown later to be essential, if a generator is required to be
self-exciting. The e.m.f, due to residual magnetism can only be
removed by dernagnetising the field system. IL' the value of I, is
increased again, Curve (3) will be I'ollowed which closes up on
curvc ( I ) . Thc dingram has bccn drawn to accentuate the difference between curves ( 1 ) and (2). In the modern machine this
difference is not appreciable and if a mean curve is drawn, shown
dotted, this is known as the 'm~gnetisation'or 'open-circuit
characteristic' (O.C.C.) curve.
The Magnetisation Curve or O.C.C. is of sufficient importance
to require the direction of students to this effect. It will be
mentioned repeatedly during theory relating to generators
(alternating current and direct current) and motors and many
problems will require reference to it before they can be solved.
It is-plotted in a standard fashion as Generated Voltage or
E . M . F . to a base of Field Current. This point should also be
noted.

Fig 93

Il

TYPES O F D.C. GENERATOR


llndcs I ~ I gcncral
S
heading, the tnachinc can be classified In
different ways. S ~ n c cthe typcs of generator arc altered by

THE D.C.GENERATOR

---

173
--

variations of the magnet system, in that either the magnetic


material or the connection of the field energising coils c i ~ nrliffcr ;
in this book the machines are described in the following order.
(a) The Permanent-magnet type of Generutor. (b) Tile Sepi~riitelyexcited type of Generator. (c) The Self-exciting type ot' Generator, which may be furth'er subdivided under the practical
headings of (ci) Shunt-connected, (cii) Series-connected, (ciii)
Compound-connected.
(a) THE PERM A N E N T - M A G N E T TYP E O F G E N E R A T O R
This type of generator is not used to any great extent because
of the difficulty of making large permanent magnets and of
varying the magnetic field so as to control the generator output.
The most common use is for specialiskd applications such as for
electrical tachometers (speed indicators), hand-operated insulation testers (the Megger is an example) and primary exciters for
p
large alternators.
THE LOAD CHARACTFERIS~C.Since 0 is cnnstant, the load characteristic will be almost identical to the no-load characteristic
which has already been considered. A tachometer arrangement
Is considered in the diagram (Fig 94).

-N

Fig 94
If the voltmeter used is sensitive, le it requlres very little
current, the generator output current will be small- so that
armature voltage drop (I,R,) will be negligible. Here R, is the
ohmic resistance value of the armature and I, the armature
current. Thus the load terminal voltage V is approximately
equal to the generated e.m.f. E and the voltmeter can .becalibrated in revolutions per minute.

(b) THE SEPARATELY-EXCITED TYPE OF GENERATOR


Knowing that E a N if @ is constant, then the no-load
characteristic will be a straight line as has been considered
earlier. If however N is constant and @ is varied the Characteristic
will vary as the B-H curve and an open-circuit characteristic
(O.C.C.)as described earlier will result. The two variations of the
characteristic are shown in Figs 92 and 93.
TH E L O A D CHARACTERISTIC . This characteristic is obtained by
setting the field current at a value which gives the normal rated
voltage at the correct speed, and by applying load in stages so as
to inkc currents of value between zero and some 25 per cent
overload.
F o r a srn;~llgenerator such loading can best be applied by
swilching in banks of sinlilar wattage lamps which tire thus connected in parallel. If the terminal voltage V is plotted against the
load current IL, the External Load Characteristic would be
obtained as is shown in the diagram (Fig 95).

INTERNAL LOAD
C HARACTtR!STIC
VOLT DROP Wt
TO ARMATURE
REACTION

'
I

EXTERNAL LOAD
CHARKTERlSTlC

IL

LOAD

VOLT DROP W t
TO IaRa

CURRENT

Fig 95
If the machine is stopped and the armature resistance R, is
rnc;isurcd by the ~mmcter/voltmetermethod and a separate
low-voltage supply, then the I,R, voltagedrop line can bc
plotted as shown. If various I,R, voltagedrop values are added
to the external characteristic the Internal Load Characteristic
can be obtained by construction. The difference between this
line and the horizontal line of the theoretical generated e.m.f. E
will illustrate the voltage drop due to armature reaction effects.
Armiiturc rcactiorl will bc explained very much Inter in detail

175

T H E D.C. GENEKATOK

(Book 7), but can be described here briefly, by saying that the
passage of current through the armature scts up a magnetic
field which interacts with the mais field, tending to weaken and
distort the latter. Thus the magnitude of the generated e.m.f. is
reduced and commutation is effected adversely.
The load characteristics have been introduced to illustrate the
effects which are responsible for a voltage drop inside the
generator itself, when the machine is on load. In problems the
armature reaction effect is seldom mentioned but the armature
is usually credited with a resistance value greater than its ohmic
value in order to allow for a total internal voltage drop. The
voltage equation would be :
E= V
I,R,
The separately-excited generator in its d.c. form is only used
for specialised applications such as, machines used for supplying
current to electroplating vats. Under these conditions some 6000
to 10 000 amperes may be required at voltages of 6 to 12 volts
and the output can best be controlled by varying a 'separatelyexcited field.

(ci)THE SHUNT-CONNECTED

GENERATOR.

The diagram (Fig 96) shows the typical diagram of connections for this machine. Here the armature current is fed to both
the load circuit and the parallel field circuit which, although i t
lakcs a vcry smull current in co~npurisonwit11 thc loud, niay
require to be considered-especially for problems.
Thus: I , = I, + I,.

LOAD
A VARIABLE

R E S I S T A N C E .)

Fig 96
The shunt field of the machine is connected 'across the
terminals of the generator. The field coils form a high-resistance
circuit, they are wound with many turns of fine wire, ie the
ampere-turns are produced by a small current value and a large
number of turns.
As before: E = V + I,R,

Example 56. A four-pole, wave-wound generator delivers


40kW at 200V. Its armature has 181 turns and a resistance of
04IlR. The air-gap fluxipole is 0.02Wb. Calculate the speed of
tlie machine, neglecting any voltage drop at the brushes and
taking- the shunt-field resistance as 500.
kilowatts or I - 40000
Load current =
- -=200A
voltage
200

200
50
So armature current I, = 260 4 = 204A
Voltage drop in armature = 204 x 0.01 = 2 4 4 V
Shunt-field current If = - = -= 4A

R,

Generated voltage = terminal voltage

voltage drop in

i ~ l ' l l i i l Urt!
t

Thus N = 840 revlmin.


Z'ote Here the armature has been assumed to be wound with

s~ngle-turncoils, ie 2 conductors/turn, since no extra data is


gi\en to assume otherwise.
Since the shunt-connected generator
utilises the principle of self-excitation, it is necessary at this stage
to explain the theory involved. If the field system has residual
magnetism, then rotation of the armature will generate some
small e.m.f. TJlis e.m.f. will cause a field current which will produce more flux, which in turn causes more e.m.f., hence more
field current. more flux and e.m.f.. to give a continual building-up
condition. The voltage continues to rise and only steadies when
the voltage drop across the field equals the terminal voltage.
.Yore,. The field current must be in the correct direction through
the field coils. to assist the build-up of the original residual flux.
Summarising. the conditions necessary for self-excitation

T HE O RY O F S E LF - E X C IT A TI O N .

* I 1.L'

1 Iicrc must

~.cs~du:il
magnet~snl-sufficient to generate a
the armature is rotated at the correct speed.
1 1 . l I I C xllunt-liclcl c11.ci11t
111ust be C O I ~ ~ ~ I ~ L I~OI I II CSSO
~
connected that current flow will cause a flux build up, to assist the
original residual flux.
iii. The shunt-field circuit resistance must be less than the
c , r i t i c ~~.csi.vtrlttc,r
~l
;IS determined from the opkn-circuit characteristic (O.C.C.) when the machine is running a t a particular speed.
('l.it~c;~l
~.chiht;~~lcc
will t7c csplaincd ; ~ n dtlcfincd hclow to
as4ist tlic i~ncicrct;iiidingol' the conditions for satisl'uctosy selfI.

!>t.

s n l i ~ l lc.rn.l'.when

excitation to occur. It must be emphasised that the subject


ml~tter.:llrcndy dealt with under the heading of self-excitation
and that to follow, is of the utmost importance and tbrms the
subject of many examination questions and problems.
THE MAGNETISATION CURVE OR O.C.C. APPLIED TO SELF-EXCITATION
CRITICAL RESISTANCE

The diagram (Fig 97) illustrates, as a means of revision, the


circuit and characteristic which is obtained by making tests with
a generator being separately excited. The initial part of the
O.C.C. graph is somewhat complex, in that the effect of resiilal
magnetism is apparent, but if it is assumed that the graph
started from zero then it is usually straight, since the magnetic

SEPLRATE
SUPPLY

M A t N E T l SM

Fig 97

Ir

circuit of a machine involves air-gaps and saturation conditions


are reached only gradually. Full saturation conditions are
seldom attained. It is again stressed that the O.C.C. is dependent
on speed.

When the field is shunt-connected, provided the conditions


set out above ale fulfilled, the generator will self-excite and an
open-circuit voltage value is attained, where the voltage drop in
the shunt field is equal to the generated terminal voltage. This
condition is illustrated in the diagram (Fig 98) and is best
understood by considering the associated O.C.C. and field
voltage-drop line.

REGULATOR

Fig 98
I'hc mngnctisat~oncurvc !'or a n y particular speed N is drawn
from test results obtained by separate excitation. Imagine the
sI1~1ntfield and regulator to Ilavc a ~.csistancc01' R, ohrns. 'T'llcn
assuming a current of value If amps to flow, the field voltage
drop will be I,R, volts. Plot this value ( e ~ a m p l epoint R ) .and
extend the straight liae through R from zero to cut the O.C.C. at
point P. For this point of intersection (P), the voltage drop
across the field equals the applied terminal voltage and conditions a r c h;ll;~nccd.Considcr illso thc I, condition shown, wherc

THE D . C . GENERATOR

179

the generated voltage SQ is greater than the field voltage drop


K Q by SR volts. More current will flow in the flrld circuit
because of this voltage difference and both graphs will rise until
a point of intersection is reached. The reasoning and procedure
for estimating the O.C. voltage is further illustrated by the
following example.
Example 57. A d.c. generator when separately excited and
driven at 1000 rev/min gave the following test values on open
circuit.
0.16 0.48 0.66 0.8
1.0 1.29
Field current (A) 0
O.C. e.m.f. (V) 6.25 50
150 200 225 250 275
The field windings are then shunt connected. Find (i) the
voltage to which the machine wil,lself-excite on open circuit when
driven a t 1000 revlmin and the resistance of the shunt-field
circuit is 24m. (ii) the value of the regulator resistance to be
added or subtracted from the field circuit to allow the generator
to self-excite to 237.5V. (iii) the value of the critical resistance at
this speed.

Fig 99

(i) Plot the O.C.C. a s shown in the diagram (Fig 99) and using
the graph, take any value of field current, viz 1 ampere. The
voltage across the field circuit with 1A flowing would be
1 x 240 = 240V. Plot this point (R) and draw the field voltagedrop line through the origin as shown. Then the O.C. voltage to
which the machine self-excites is 257V.
(ii) For the machine to excite to 237,5V, note this value on the
O.C.C. and join it to zero to obtain the new field resistance
voltage-drop line. Note the field current for 237.W; this is 0.88A,
Then from Ohm's law, the fieldcircuit resistance is

:. Resistance to be added = 270 - 240 = 30R.


(iii) Neglecting the start of the graph (due to residual magnetism), draw the tangent through the origin. Read any voltage
value on this tangent and the corresponding field current.
Example :-155V and 0.5A.Then the critical resistance would be
155 ohms = 31012.
0.5
The effect of altering the shunt-field
resistance or regulator can be seen by referring to the example
and the graph (Fig 99). Reduce R, and the slope of the field
resistance voltage-drop line becomes less and the point of intersection with the O.C.C. moves higher up, ie the O.C. voltage of
the generator is raised. The converse occurs with R, increased. If
R, is increased until the field voltage-drop line lies outside the
magnetisation curve, there will be no point of intersection and
the generator will not self-excite. Thus for any point on such a
voltage-drop line, the voltage required is not available, as can be
seen from the O.C.C. If R, is reduced, the slope of the field
voltage-drop line decreases until the line lies along or becomes
tangential to the O.C.C. The resistance value deduced from the
field voltage-drop line will thus fall until it attains the value given
I)! tllc I ~ n cranpcnt~nlto tlic O.C.C. Tllc rcsistancc valuc obt:~i~lcd
for this condition is called the Critical Resistance. The term
c \ l ? l ; ~ ~ r~~t h\ c l f ; ~ l i t I is clcpcn(1crit 0 1 1 s p c c t l . T l ~ u sfor ;Iny o n c
speed. IS the field reslslance is made less than the critical resistance, the machine will self-excite if the other conditions are
satisfied.
C RI TI C A L RESISTANCE.

THE L O A D CHAKACTEKISTIC. The

test circuit and the resulting


graph is shown by thc diagram (Fig 100). The armature resis111nccR, is nlcnsurcd by rl S C ~ ) ~ I I . ~ Ik~sCl .

THE D.C.GENERATOR

181

Fig 100
The External Load Charaxeristic is plotted from the test
results obtained and the In1 :rnal Characteristic is drawn by
construction already descr-bed for the separately-excited
machine. Due allowance sho\,ld be made for field current since
armature current and not lir c current is used for the armature
voltage drop.
Features of the Load char ~cteristicsare ( 1 ) the rapid fall off of
terminal voltage (ii) the bent,-back of the characteristic on itself.
(i) This condition is expl lined as follows. When the external
circuit is connected to a I )ad, there is a voltage drop in the
armature. The terminal vo,tage falls, resulting in a decrease of
field exciting current. T h ' s in turn causes the external characteristic to droop more t Ian it does for the separately-excited
machine. The armature rt action effect is as for the separatelyexcited machine, ie it is r~sponsible for a decrease of generated
voltage--equivalent to an ncreased armature voltage drop. Thus

182
-

REED'S BASIC ELECTROTECHNOLOGY


--

--

this can be taken into account by crediting the armature with a


R, value 'greater than its ohmic resistance.
(ii) As the load resistance is decreased, load current will
increase at first with a resulting fall in terminal voltage. This
effect tends to slow up the increase of load current. At first the
decrease of external load resistance'with consequent rise of load
current predominates and a rising current with falling terminal
voltage is shown on the graph. At a certain value of current the
demagnetising effect of armature reaction, the armature resistance voltage drop and the loss of field current due to reduced
voltage, combine in effect, to produce a terminal voltage which
results in less load current even though the load resistance is
decreased and the curve bends back on itself. The armature may
thus be short-circuited-a self-protecting cffect being produced.
OA is caused by residual magnetism, but a sudden short-circuit
may cause an excess armature reaction effect which tends to
cancel residual magnetism, thus demagnetising the machine
which may then fail to self-excite when the short-circuit is
removed. The machine will then require to be remagnetised
before i t can be put back into operation.
The shunt-connected machine can be used for most purposes
where a simple generator is required. Examples would be for
battery chargers and small lighting-sets, such as motor-car
dynanio and electrical systems.
(Cii) THE SERIES-CONNECTED GENERATOR
The diagram (Fig 101) shows the connections for this
machine, which is only used for specialised work. It is however of
sufficient importance for the student to spend some little time in
studying its behaviour and characteristics, which are used to
advantage in the compound generator.

Fig 101

183

T H E D . C . GENERATOR

The series field of this generator is designed to be connected in


the main armature circuit to the load. The field coils are wound
with a few turns of thick cable, ie the field ampere-turns are
produced by a large current and a small number of turns. Thus
I, = If = I,. The terminal voltage on load is V and the generated e.m.f. E is greater than V by the internal voltage drops in
the armature and series field. Thus:
E = V + If& -t I,R, = V + I,(R, + R f )
The theory involved is as for the shunt-connected machine and the same conditions apply. It should be
noted that the load resistance constitutes the field regulating
resistance and thus for any particular speed there is a critical
resistance value. If the load. resistance, ie the field-circuit
resistance, is less than the critical value, the machine will selfexcite. U:the circuit resistance is above the critical value for that
particular machine speed, self-excitation and voltage build-up
will not occur.

SELF-EXCITATION.

THE LOAD CHARACTERISTIC. Consider the circuit shown in the


diagram (Fig 101). The circuit switch is closed with the load
resistance set at maximum value. Load resistance is then
gradually reduced until, at some point in the test, the machine
is observed to self-excite. Load current and terminal voltage
settle at some definite value but if the load is next altered, new
vpltage and current values will be obtained, which should be
noted. This should be done for both decreasing and increasing
load resistance values, until the full external load characteristic
is obtained. The diagram (Fig 102) shows the characteristic.

,~ T H A R A C T ~ R I S T I C
INTERNAL

Fig 102

IL

The machine is next slut down and R, and R, are measured


separately. The armature and series-field resistance voltage-drop
lines are next drawn and the internal load characteristic can be
deduced. The effects of armature reaction could also be investigated if an O.C.C. (obtained by separate excitation at the correct
speed), was supe~imposedon the characteristics.
The machine has the following disadvantages. (i) It cannot
self-excite until the load circuit is completed and its resistance
value is made less than the critical resistance. (ii) The voltage to
which it self-excites is dependent on the load current and very
little control of this voltage is possible. (iii) The load characteristic is a rising one and is unsuitable, in fact dangerous, since it
could result in the 'bum-out' of the load.
The series generator is never used for normnl generating
purposes, but only for special applications such as a series
booster. The machine in this form or variations, has been used
for marine electrical systems but only for specialised applications. Examples would be for particular types of electric propulsion and winch control arrangements.
(ciii) THE COMPOUND-CONNECTED GENERATOR
As this generator utilises both series and shunt fields, its
characteristic can be considered as being made up of shunt and
series-machine characteristics. The ultimate shape of the
characteristic will depend on the relative strengths of the
individual fields, but it must be noted that the shunt field is the
basic requiremelt and thus it is the performance of a shunt
generatpr, which is being improved upon.
The diagram (Fig 103) shows
how a machine can be connected ir, either 'short' or 'long' shunt.

TYPES O F ELEC TRIC AL CONNECTION.

5HORT

SHUNT

Fig 103

LONG S H U N T

185

THE D . C . GENERATOR
.-

There is no i~ppreciabledifference in the resulting generated


voltage as will be seen from the example.
Example ,58. A l IOV, compound genenitor has nrmutun.,
shunt and series-field resistaxices of 0.06!2, 25R and 0 4 4 R
respectively. The load consists of 200 lamps each rated at 55W,
110V. Find the generated e.m.f. and the armature current. if the
generator is connected (a1 long shunt, (b) short shunt.
200x 5 5 ---11000
(a), LONG SHUNT. Load current I, =
110
110
= lOOA
110
Shunt-field current I,, = - = 4.4A
25
Series-field current I,, and armature current I, = 100 + 4.4
= 104.4A
The generated voltage E = V + I,(0.06 + 0.r04)
= 110
104.4(0.1)
or E = 110 + 10.44 = 120.44V.
It will be noted that since there are two fields with different
values of current, symbol I, has not been used for field current,
but instead symbols I,, and I,, have been used in both Fig 103
and this example.
(b) S H O RT SHUNT . Load eurrent I, (as before) = lOOA = I,,
The voltage drop in the series field = I,,R,,

100 x 0.04

= 4v

The voltage applied lo the shunt field = terminal voltage


t voltage drop in series field
= 110 + 4 = 114V
114
Shunt-field current I,, = - = 4.56A
25
Armature current I, = 100 + 4.56 = 104.56A
E = V + voltage drop in series field + voltage drop In
armature
= 1 10 + 4 + (104.56 x 0.06) = 1 14 + 6.274 = 120.27V.

rl PES OF FIELD A RR AN G E ME N T . The series field is usually so connected that the flux ~t produces adds to the shunt-field flux. For
such an arrangement, which is the most common, the machine
is said to be cumulatively connected. All generators, used for
supplying lighting and power for electrically driven auxiliary
machinery aboard ship, have this connection. If the series field I S
connected so as to weaken the shunt field, the generator is
diferentially connected. This arrangement is used for specialised
work only and is not common. An example of its application
would be for certain types of welding generator.

THE LOAD CHARACTERISTIC . The diagram (Fig 104a) shows the


graphs which would be obtained by directly loading the machine
in the manner already described. Curve (a) shows the characteristic ihicti would be obtained if the shunt field only was
used. Curve (b) would be obtained with the series field only and

Z
T5-

Fig 104(a)

IL

I .OVER COMPOUNDtD
2 . L ~ V E L O R F L A T COMPOUNDED

I
D

I UNDER COMPOUNDfD

Fig 104(b)

IL

curve (c) would result from'the use of both fields. Any point on
this load characteristic could also be obtained by adding the
voltages obtained from graphs (a) and (b), for any one value of
load current.

THE D.C.G E N ER A T O R

187

Fig (104b) shows how the load characteristic of a compound


gcncrritor is vnricd by nltcriny tllc rclrttivc strenyth of tlie rericy

field. Flat-compounding is required by most Regulations. The


curve is not quite flat and the rise in voltage between no load and
full is called 'the hump'. It may be 6 to 7 per cent for small
generators, but for normal cases it is about 2 to 3 per cent. Overcompounding is used to compensate for the voltage drop in a
supply line, as was done in the past for land systems when direct
current was used extensively. This is shown by the example.
Example 59. A factory is sited some distance from the generating-station and takes lOOA at 200V. The resistance of the supply
cable is 0.02nlcore. Find the percentage compounding required
for the generator.
The voltage drop in the line on full load = 100 x 2 x 0.02
= 4v
To supply 200V to the factory, the generated veltage should be
204V.. An overcompounded characteristic as shown in the
diagram (Fig 105) would be suitable and the compounding is

Fig 105

usually expressed as the rise in voltage on full load to the O.C.


voltage.
DC DC 204 - 200
Thus percentage compounding = - = - =
CB
A0
200
o r = 0.02 x 100 = 2 per cent
Thus the generator requires to be 2 per cent overcompounded,
Further work o n the d.c. generator will be necessary for more
advanced studies, but for the present sufficient knowledge has
been ~ ~ c q u i r etod allow some study of the motor to be made. The
I'c~liowingc x n ~ ~ ~ pIhr111
l c s h ussl'ul conclusion lo this cl~aptcr.
Example 60. A shunt generator is to be converted into a com~ ' o u n t lfiulicr.;ltor.b y tllc ;~cltlrtioriof' ;I scrics-licld winding. F;ronl
a test on the machlne with shunt excitation only, i t is found that
a field current of 3A gives 440V on no load and that 4A gives
440V at the full-load current of 200A. The shunt winding has
1600 turns/pole. Find the number of series turns required/pole.
Ampere-turnslpole required to give 440V on O.C.
= 3 x 1600 = 4800At
Ampere-turnslpole required to g v e 200A a t 440V on load
= 4 x 1600= 6400At
Full-load ampere-turns must be increased by 6400 - 4800
= 1600At
But these 1600At/pole are to be obtained from the series field
which passes 200A
1600
Thus the req$red number of series turnsipole = - - 8.
200
Example 61. A four-pole, compound generator has a 'lapwound armatyre and is connected in short shunt. The resistances
of' the armature and fields are 0 , I R and SOR(shunt), 0.08R
(series). The machine supplies a load consisting of sixty IOOV,
40W lamps in parallel. Calculate the total armature current, the
cwrrcnt ~itrmitturcpath ; ~ n dthe generated c . n ~ . f .
Slnce this is a lap-connected armature A = P
For onc Ii~rnp,slncc I'

1'1

40
I = 100

= 0.4A

T h e load current I,, = 60 x 0.4 = 24A


Voltage drop in senes field = 24 x 0.08 = 1.92V
Voltage across shunt field = 101.92V
101.92
Shunt-field current = = 244A
50
Armr~turccurrcnt = 24 .t 2.04 = 26.04A
~

189

THE D.C.GENERATOR

*"04 - fj.51A
Current per armature path = 4

Generated Voltage = terminal voltage


voltage drop in
voltage drop in armature
series field
= 100 1.92 (26.04 x 0.1)
= 100
1.92 2.604
= 104.524V.

+
+
+

CHAPTER 8
PRACTICE EXAMPLES
.1.

The armature of a four-pole, shunt generator is lap


wound and generates 216V when running at 600 rev/min.
The armature has 144 slots with 6 conductors~slot.If the
armature is rewound to be wave connected, find the e.m.f
generated at the same speed and fluxlpole.

:I compound-wound, long-shunt d.c. generator has an


output 01' 250A :it 220V. The equivalent resistances of the
armature. series and shunt windings are 0.025, 0.015 and
17CQ rcspcctivcly. 11' tlicrc is ;I 2V volti~gcdrop iicross the
brushes, find the induced voltage.

3.

The curve of induced e.m.f. against excitation current for


a separately-excited generator when run on n o load a t 1200
revlmin is given by :
E.m.f. (V)
15 88 146 196 226 244 254
Excitation Current (A)
0 0.4 0.8 1.2 1.6 2.0 2.4
Deduce the voltage to which the machine would selfexcite if the shunt-field resistance was set at 90R and the
machine was run at 900 revlmin

4.

A 220V. four-pole, wave-wound, shunt generator has an


armature reshtance of 0.1R and a field resistance of 50R.
Calculate the fluxlpole, if the machine has 700 armature
conductors, runs at 800 revimin and is supplying a 38kW
load.

"

In a 250kW. 440/480V, overcompounded generator, the


flur/pole required to generate 440V on no load is 0.055Wb
at 620 rev/min. The resistances of the annature, interpoles
;ind scrics ficld arc 0.01.0~005i~nd0.005Q respectively. Find
the fluxipole required at full load, the speed now being
fKKl revlmin. Ncglcct the current ti~kcnby thc shunt ficld.
6.

Estimate the series-turnsipole required for a 50kW, compound generator required to develop 500V on no load and
550V on full load. Assume a long-shunt connection and that
the ampere-turns required per pole on no load are 7900
\\'licrc;~stlic ampere-turns required per pole on full load are
1 1 200.

THE D.C. GENERATOR

191

I
I
1

7.

A four-pole milchine has :I lap-wound rirmature with 90


slots each contalnlng 6 conductors. 11' the m'lch~ne runs ,it

1500 rev/min and the fluxipole is 0.03Wb. calculate from


first principles the e.m.f. generated.
If the machine is run as a shunt generator with the same
field flux, the armature and field resistances being 1.052 and
200R respecti\fely, calculate the output current when the
armature current is 25A.
If due to a fall in speed the e.m.f, becomes 380V, calculate
the load-Current
ina 4052
load.
- ---8. A d.c. generator gave the following O.C.C. when driven a t
1000 revlmin.
Field Current (A)
0.2 0.4 0.6 0.8 1.0 1.2 1.4 1.6
Armature Voltage (V)
e
32 58 78 93 104 113 120 125
If the machine is run as a shunt generator at 1000 rev/min,
the shunt-field resistance being 100R, find (a) the O.C.
voltage (b) the critical value of the shunt-field resistance, (c)
the O.C. voltage if the speed was raised to 1100 revlmin, the
field resistance being kept constant at 10052.
9.

Calculate the input power required to drive a shunt


generator when giving an output of 50kW at 230V. if undcr
thcse conditions the bei~ring,friction, windage and core loss

is 1.6kW and the total voltage drop at the brushes is 2V. The
resistance of the armature is 0 - 0 3 W and that of the field
circuit 55R.
10.

A d.c. generator when separately excited and run at 200


rev/min gave the following test results:
Field Current (A)
0 1 2 3 4 5
6
7
8
9
O.C. Voltage (V)
10 38 61 78 93 106 115 123 130 135
The field is then shunt-connected and the machine is run
at 400 rev/min. Determine, (a) the e.m.f. to which the
machine will excite when the field-circuit resistance is 3 6 0 ,
(b) the critical value of the field-circuit ,resistance, (c) the
additional resistance required in the shunt-field circuit to
reduce the e.m.f. to 220V, (d) the critical speed when the
field-circuit resistance is 36Q.

CHAPTER 9

THE A.C. CIRCUIT (CONTINUED)


The method of introduction to the a.c. circuit chosen here. is
again not the recognised approach as used in most text books.
Experience in teaching the subject has convinced the author that
the essential fundamentals should be introduced as soon as
possible and the details should be considered as experience is
acquired. Conscqucntly at this stngc, a number of new terms are
mentioned and relationships are stated. These should be
memorised as soon as possible.
IMPEDANCE

For the a x . circuit, conditions are followed which are somewhat comparable with those for Ohm's law, as it has been
propounded for the d.c. circuit. Thus we can consider a modification of Ohm's law as applicable to the a.c. circuit; where the
current is found to be directly proportional to the applied
voltage and also to be inversely proportional to the opposition
of the circuit to the flow of current. This opposition is called the
circuit Impedance (Symbol-2. Unit-the ohm) and is due to
more than the mere ohmic resistance R of the circuit. The difference between Z and R is now considered further.
For the a.c. circuit, the current flowing is given by
Applied voltage
Impedance
~ h u Current
s
= --------voltage
o r I (amperes) = V (volts)
Impedance
Z (ohms)
Note the variations of the relationship. Thus:
.b

/=- or C'=I%orZ=-

For 111cd . c clrcult. it i b known I I ~ : I II = fwllere R i s tile ohmlc


R
rcsist:lncc of the circuit. l f ; ~wire. of rcsist;~nceR ohms, is laid out
as a straight conduc~orand connected to a d.c. supply of V volts,
17

the current I when measured, would be given by

amperes. If

the same wire is next connected to an a.c. supply of V volts, tlie

cirrrent I, when measured, would be given by - amperes and


woultl Ilc

I ' o ~ l r l t l to

Ilc o f

tllc si1111cI I I ~ I ~ I I ~ I 11s


L I f~oCr thc d . c . t c s ~ .

THE

A . C . CIHCIJIT ( C O N T I N U ~ ~ D )

1 93

For this case Z and R would be equal or the impedance would he


made up of' res~stanceonly. 'I'he circuit is then said to be 'resistive'
o r 'non-inductive'.
If the wire is next wound into a coil or solenoid and the same
voltage V is applied, t h e current wilJ be smaller, ie the new
impedance would be greater than the ohmic resistance. Again, if
an Iron core is inserted into the solenoid, the impedance would
increase funher are i t would be apparent that the impedance is
made UD of ohmic resistance tozether with Sonit! extra C t T z i d l limiting' quantity. This extra iuantity is termed .Keactance9
(Svmbol-X. Unit-the ohm) and it should be noted here that
t h i impedance is not given by a straight-forward arithmetical
summition of resistahce R and reactance X but by a rightangled relationship where :
Impedance = l/~esistance2 Reactance2
r
or Z (ohms) = d~~(ohms) X' (ohms)
The relationship between R, X and Z can be represented by
the 'Impedance Triangle' as shown by the diagram (Fig 106).

+
+

Fig 106
The angle 4 is called the 'Phase-angle' and cos 4 is a measure
of the 'Power Factor' of the circuit. It is apparent that much
explanation is now necessary to assist with the understanding of
the terms introduced and the relationships stated. The reactance
as mentioned, is furthermore of only one particular form, namely
inductive reactance. If a coil with its associated magnetic field is
concerned with the a.c. circuit, then its inductive reactance must
be known, which in turn requires a knowledge of the Inductance
and the supply frequency. The term 'inductive reactance' is thus
associated with an inductor coil-usually iron-cored. An alternative to the term inductor is reactor-usually used for a large coil
built to pass heavy currents and strengthened to withstand the
great associated electromagnetic forces.

194

R EED ' S BASIC ELECTROTECHNOLOGY

In contrast to inductive reactance there is also capacitive


reactonce, a term associated with a condenser o r capacitor-to
use the modern and advocated name. The capacitor. and
capacitive reactance will be considered in due course, but
immediate attention is now given to inductance and inductive
reactance.
Example 62. An inductor (sometimes called a ch,oke-coil) has
an ohmic resistance of 3R and a reactance of 4R. If it is connected to a 20V a.c. supply, find the current which flows and the
power factor at which the coil o p r a t e s . Note the diagram
(Fig 107).
Here R = 3R and .Y = 4R. Also since Z = J-R~+XZ
V
20
The current I = z = -5 = 4A
R
The circuit power factor is given by cos 4 = - (from the
z
3
impedance triangle). Thus cos 6 = - = 0.6 (lagging).
5
The term 'lagging' is associated with an a.c. circuit which contains inductive reactance. A more complete explanation will
follow as the theory is developed.

Fig 107
The diagram shows how the circuit is represented. It will be

seen that all the ohmic resistance is considered to be concentrated in a resistor R and the reactance in an inductor X, even
though they together constitute the impedance Z of the chokecoil. The dolled rcctunglc rcprcscnts the cliokc-coil and will
generally be omitted in future diagrams.
INDUCTANCE
Basic electromagnetic induction theory, as covered in Chapter
6. has alrc;ldy shown t h a t whenever thc magnetic flux linked with
II circuit is c l i ; ~ n ~ i nagn, c.ti1.f. is induced in the circuit. F;~r;~day's
Iiiw shows that the value ol' the induced e.m.f. IS proportioniil to

THE

A.C.

195

CIRCUIT ( C O N TI N U E D )

the rate of change of flux-linkages and this e.m.f. only exists


wllilst ;I cliangc 01' flux-linkitycs is t;lkirlg plilcc. Sincc, for itn u.c.
circuit, the current is changing continually, then the associated
flux of a coil carrying such a current , ...,o changing continually.
This means t h a t t h e flux-linkages are changing and an induced
e.m.f. is thus being continually generated. By Lenz's law this
would be a 'back e.m.f.', tending to oppose the change which is
causing it.
SELF - INDUCTANCE .

Any circuit, in which a change of its fluxlinkages produces an induced e.m.f., is said to be self-inductive
or to possess self-inductance (Symbol-L). As the statically
induced e.m.f. of a coil is produced by a changing flux, which in
turn is caused by a changing current, it is more convenient to
consider inductance' in terms of current change.
N@
Written mathematically, we already know thaLE,, = t

If only a single turn is considered or IV = 1 , the expression IS


@
0
Eav = o r Ea, cc -.
Assuming that the flux produced is
t

proportional to the current producing it, then we can write


I

Eav cc -. If a constant is introduced into this proportionality


1

expression, it becomes E,,

Li
-.
I

Notce. 11'o vcry short inst;~ntof time is considcrcd, thc cxprcs-

di
sion can be written as e = L-, since a rate of change is

dt

involved. This form of expression is introduced here for the benefit of students who prefer mathematical proofs and developments
to be made by the use of calculus. Similarly the earlier expression
for induced e.m.f., as already stated, can be written in the form
d@
e = N-. A minus sign is frequently placed in the expression to
dt

show that the induced voltage is a 'negative or back e.m.f.' as


stated 'by Lenz's law.
From the expression Eav =

g,
a value for the constant L can
t

be deduced in terms of the knpwn units of E, I and t . The unit


of inductance is called the Henry (Symbol-H), which can be
defined thus :
A circuit has a n inductance of 1 henry, if an e.m.f. of 1 volt is
induced in it, when the current changes at the rate of 1 ampere
per second.

From expressions already deduced, we know that:


N@
LI
E.,=- I
also E = 1

The above is an important expression for L in terms of fluxlinkages and current. I t can be further developed into' the
statement that:
Inductance (in henrys) = Flux-linkuges/ampere.
When the current in an inductive c~rcuitis made to change, it
is apparent that, due to the property of inductance being present,
the current value will, at the instant of change, be controlled by
mom than mcrely the voltage and resistance. During the changIng o r rrunsipnr conditions, a back e.m.f, is being generated and
new conditions of voltage balance occur. Such transient conditions involve detailed consideration, and since inductance is
only of consequence for a d.c. circuit, a t the instant of switching
on o r switching off and at the time when the current is being
altered, its effect will need to be considered separately. A full
treatment is given in Volume 7. For the a.c. circuit, since current
is varying sinusoidally and is thus changing constantly, inductance will have an important and continuous effect. This is
considered next.
r N m c n v E R E A C T A N C E . Imagine a choke-coil having no resistance and only inductance of value L henrys. An alternating
voltage of V rolts is applied giving a current of I amperes. The
diagram (Fig 108) represents the current and voltage conditions.
Assume the current of Iamperes (r.m.s. value) to be sinusoidal.
LI o r = L (rate of change of
Since the induced e.m.f. = -

current), then at point 'a' the current value is zero, but it is


increasing at its maximum rate, since the slope of the waveform
is steepest at this point. Maximum induced e.m.f. will therefore
occur at this instant 'a' and, since this is a 'back e.m.f.', by Lenz's
law it must oppose the supply voltage. The voltage waveform
contlitions will hc ;IS illustr:~tcd; ~ n dthe corresponding ph;~sol.
diagram is also shown with I used as the reference phasor. At
point 'b' on the current wave, no e.m.f. is induced since current is
maximum and not changing at this instant. Thus there is a 90"
phase difference between the current and the induced voltage
( E x ) and a further 90" difference between the current and the
applied voltage (V,). Notr. This condition applies to a circuit
with Incluctancc only.

197

T H E A . C . C I R C U I T ( C O N T I N U ED )

Fig 108
Again referring to Fig 108, it is seen that, as the current rises to
its maximum value I, in the first quarter cycle, flux-linkages
LI, are set up, since L =

N@
o r N@ = L I
I

---

to zero in the second quarter cycis, the llnkages are destroyed.


For the next half cycle the same number of l~nkagesare set up
and destroyed.
The ,change of flux-linkages in 1 cycle = 4LIm and the change
of flux-linkages in 1 second = 4fLIm (f is the frequency). Also,
since the average value of induced e.m.f. = rate of change of fluxFlux-linkages - 4 f L h
linkages Then :rver;ige c m f = - -time

Thus back e.m.f. Ex,, = 4fLIm volts.


The supply voltage is equal and opposite, its value being V,
(r.m.s.) o r VX,, (average). Since r.m.s. values are preferred the
following conversion is necessary.
2
Since VxaV= - Vx, and V,, =4fLI,
K
2
Vx,= 2rcfLIm
then - Vx, = 4fLIm or
K

giving 0.707Vxm = 2nfL x 0.7071, or Vx = 2rcfLI.


Thus the voltage drop in an inductor o r reactor is seen to be
proportional to the current and a constant which involves the
circuit inductance and the supply frequency. This constant is
given the name 'reactance' and since it is for an inductive
circuit, we represent it by the symbol .Y with the suffix L.
Thus X, = 2nfL ohms and for a purely inductive circuit
Vx = IX, where X,,= 2 ~ f L .
Inductance reactance is measured in ohms and is seen to be
proportional to both frequency and inductance. Since resistance

1
1
I
1

has been neglected, the phase relationship between the reactlve


voltage drop I.Y, and current is seen to be fixed at 90' or these
quantities are in quadrature with respect to each other. This
condition will be deduced again as the various conditions of the
a.c. circuit are considered.
1 . C I R C U I T WITH PUKE RESISTANCE
The c~rcuitconditions are illustrated by the diagram (Fig 109).

Assume a sinusoidal voltage of value v = Vmsin of to be


applied to a purely non-inductive resistor of value R ohms. The
applied voltage has to overcome the ohmic voltage drop at every
I'

instant or we can write i = - and maximum current will occur


R
Again since
when the voltage is a maximum since I, =

R'

.
I

L!

- -

%
sin wr
R

. i

= I, sin'wr.

Thus we see that the circuit current is also sinusoidal and is in


phase with the applied voltage. The phasor diagram can also be
drawn as shown.
0.707 V ,
o r I = -Y
R.M.S. values can be used since 0.7071, =
R
R
The power cond~tioncan be deduced thus:
Powcs at any lnstant IS given by p = \\i
or p = V , sin wt I, sln wr = V, I, sin 2 wt
( 1 - co5 2 (I)!)
*
= ,'L I ,
--.
. . . .
2
Average power P
= Average value of
-

' l m

0 Since the average value of a cosine.wave is


/cI'o.

1 90

1 H E A.C. CIHCUIT (CON'IINI:LL))

. , . l , \ , ~ , ) --

"111

1111

1.1

t.2
t 2
or P = VI (watts).
From the expression marked thus*, it will be seen that tllc
power wave is a periodic quantity, always +ve and of twice
supply frequency. These facts would also be confirmed if the
power wave was plotted by obtaining values of v and i for corresponding instants in time and multiplying these together to
give p, the power value at that instant. The resulting power
wave would be seen to be fully displaced above the horizontal
and its maximum value would be equal to VmIm.Being syrn- '
metrical, its average value would be obtained from the distance
its axis is displaced from the horizontal. This would be

u.
This value would then be a measure of the average power
2

p = U= V" x --Im or P = V I (watts).


2
t.2
.\/2
For the resistive circuit being discussed, power is found to
equal the product of voltage and current, but further considerations will show that this is only rue for non-inductive circuits.
In general, if an attempt is made to co-relate the power with the
voltage and current or volt amperes of the circuit, it will be
found that the product of V and I must be multiplied by a
factor, which for convenicncc I:;called the 'power factor'.
For the condition ol' a purely resistive circuit, il' we write
P = VI x power factor, then it is obvious that the power factor
must equal unity. It will again be seen later that the power factor
is related to the ratio of the resistance and impedance of a
R
circuit and that it can be obtained from -. From the impedance
Z
R.
triangle of the general circuit, - is the cosine of the phase-angle
z
4 between circuit voltage and current. Thus cos 4 = R = the
t

Thus:

&

power factor.
The assumption already made-that P = VI x power factor
can now be written as P = VI cos d. Furthermore, since here
R
R
the circuit is resistive Z = R or cos 4 = - and - = 1, giving

the condition of unity power factor already mentioned,


Note. The following deduction is also of value and is seen to
be identical to that for the d.c. circuit. Since P = V I cos d we
R =V IR = I x I x R or P = I ~ (watts).
can write P = VIR

z z

Esample 63. An electric fire rated at 2kW, is connected to a


??OV s u p p l ~ Find
.
tlic currcnt which will flow and the resistance
value of the fire element.
Since an electric fire consists of a heating element which is
purely resistive. it can be assumed that the circuit operates a t
unity power factor. Thus cos 4 = 1 o r the general expression
P = 1/I cos d becomes P = C'I
Therefore /

220

Here Z = R

2000=9.1A
= 220

:. R = 24.252

whence R - 2000 = 24.2.Q.


82.8 1
2.

CIRCUIT WITH PURE INDUCTANCE

The basics of this condition have already been introduced in


this Chapter under Inductance and Reactance, but since further
deductions are necessary, the circuit is again illustrated by the
diagram (Fig 110) which shows a coil with no resistance, but
having inductance 01' vnluc L henrys.

Fig 110
Assume a sinusoidal current of value given by i = I?, sin or to
tllc' coil. Since 1 is wrying sinusoldally, the
~ l ~ ; ~ p nficlci
c ~ i isc ; ~ l s ov;lrving ; ~ n d;I sinusoid~~l
sclf-induced c.ni.f.
15 bet
ol)po511ig[lie :il)pltcd volt:igc ~ I every
I
i~lb[:~t~
'T'rc;~tn~en[
t.

tw tlo\vlng thsnupli

THE A.C. CIRCUIT

20 1

( CONTINUED )

of the a.c. circuit with inductance only, showed that the e.m.f. of
self-induction or 'buck c,m.f.' can bc rcyr~rdcdan boing aquivalent to a voltage drop, which is caused by the current and by a
property, with which the circuit is credited. This property is
termed lnductivc Rcuctancc (Symbol-XL. Unit-the ohm).
Thus we have Ex = V , = I x X,. X , has already been shown
to equal 2xfL.The associated phasor diagram can now be considered with the waveform. Ex is the e.m.f. of self-inductance,
displaced 90" behind the current I and Vx is the supply voltage,
being 90" ahead of the current and. 180" out of phase with Ex.
V, is thus always equal and opposite to Ex.This is illustrated by
the-waveforms and can also be shown thus:
It has been seen that the e.m.f. of self-induction can be written
di
mathematically as e = L -. By Lenz's law, since it a t all times
dt
dl
opposes the supply voltage, we can write v = L t.Also v = - e
dr
di then e = L d ( I , sin a t ) =
and since e = L1, cos
or
dt dt
e = o L 1.
or Similarly v can be deduced a s

(sin

v = o L I,.,, (sin or

;}

i).

Note. v is i80" ahead'or anti-phase with>e.


Since wl, is the reactance X , . i t bcinp remembered that
o =

2ltf wt'

I~~IVC

and v = V,,, sin

giving the 90"

phase displacehent' between the current and voltage waves a s


shown on the diagram. Note. V, = X, I,. The relationship
X , = 2xfL is a fundamental and has been deduced earlier.
The power condition can be deduced thus:
Power at any instant is given by p = vi

or cos of
sin 2 o t

= V , I, sin
=

v, I,

7
L

Thus the instantaneous power p

I
\

I
!

of-

x
sin 2
4 2
4 2
= VI sin 2 or . . . *
Average power P = Average of value of V I sin 2 or
= 0. Since the average value of a sine wave
is zero.
=

From the expression marked thus*, it will be seen that the


power wave is a sine wave of double frequency. It will be
symmetrically disposed about the horizontal and the average
value will be zero. This would also be confirmed, if the power
wave was plotted from values obtained from the voltage and
current waves. Since the axis of the power wave is not displaced
above but lies along the horizontal, the value of the 'average
power utilised must be zero because the + ve halves of the power
wave are exactly equal to the -ve halves. This indicates that
when power is taken from the supply, to establish the magnetic
field associated with the coil. it is returned to the supply when
the magnetic field collapses.
If the general expression P = V I x power factor is adopted
I'ol. this circuit, thcn l l i c powcr fi~ctormust c q u i ~ lzcro; sincc
P = 0.If used in the form P = VI cos 4, then cos 4 = 0. T h i s
R
can be further deduced from cos 4 = as stated earlier. Here
0
R = 0 therefore cos 4 = - = 0.

,
L

Summarising we can say. that a circuit with inductance only


and no resistance would be purely imaginary but would give rise
to a zero power-factor working condition.
Example 64. A 220V, 50Hz supply is applied to a choke-coil of
negligible resistance and the circuit current is measured to be
2.5A. Find the inductance of the coil and the power dissipated.
220 880
Since Z = then Z = - = - = 88R
4
2.5
10
Now R = 0 :. X, = Z or X, = 88Cl
also X, = 2xfL
88
- 0.88
XL -SOL=2
x
3.14
x
50
- 3.14
27rf
o r L = 0.28H
0
Also as R = 0 cos 4 = - = 0
88
:. 1' = 220 x 2.5 x 0 = 0
Alternat~velysince P = 12R
t l i c l l 1'
2.S2 x O = 0
3.

CIRCUIT WITH RESISTANCE AND INDUCTANCE IN SERIES

Consider a pure resistance and a pure inductance in series as


shown in the diagram (Fig I I I ) . I t should be noted that for the
practical choke-coil or reactor, resistance and inductance are
pliysic;~llyinseparable, but for ding~.an~mntic
purposes they can
l)c

~ , I ~ o \ \ ;~I \I I t w o i ~ i ( l i t , i ( l ~~~~; O
~ lI I I ~ O I I C R
I I ~; ISI I ( ~1.

THE

A . C . CIRCUIT ( C O N TIN U ED )

203

Since the circuit conditions for both rmistnnce and inductnnce


liuvc uI~.cudybccn clcult with tl~cn,l'rom the deductions already
made, it can be considered that if a current I is assumed to flow,
two voltage drops VR = IR and V x , = I X , will exist. These
must together form the applied circu~tvoltage and for greater

Fig 1 1 1
convenience we can assume that the applied voltage V consists
of two components: One such component VR would be the
voltage necessary to overcome the resistance voltage drop of the
circuit and the other component V , would be the voltage
necessary to overcome the reactance voltage drop or would
oppose the induced back e.m.f. of self-induction. Since these two
components are at right-angles to each other, as shown by
consideration of Circuit Conditions 1 and 2, it follows that the
applied voltage is the resultant of the two components. The
relationships being discussed can be illustrated by the diagram
(Fig 112) which shows the relevant waveforms and also the
appropriate phr~sors.

Fig 112
F o r the phasor diagram, thecurrent is common to both components, it being a series circuit, and is used as the reference
phasor. The resistance voltage drop V R = IR is in phase with

current and is-drawn horizontally. The reactance voltage drop


V,, = I X , is at right-angles to the current andehas been drawn
vertically. The e.m.f. of self-inductance has also been shown but
will be omitted from now on since it serves no useful purpose on
the phasor diagram.
The resultant of VR and V X Lis obviously V, the applied voltage and the current is thus seen to lag V by an angle 4 which is
the phase-angle of the circuit.
Simplification of the phasor diagram can be useful in that it
allows the appropriate 'voltage triangle' to be extracted, which
in turn can be modified to give the 'impedance triangle'. These
are shown in the diagram (Fig 113) and are used to prove the
assumptions already made at the start of the Chapter.

Fig 113
Thus from th2 voltage triangle we have V =

and

m b = -VvR
also since VR -.I R and VXL = I X L the above can be written as
v = , C I +~( I X , ) ~= 14If Z is taken to be the equivalent impedance of the circuit then
V
Thus IZ = I
whence we have
v = IZ o r Z = -.
I
the impedance triangle relationship of:
R
Z = r R2 - + X L 2 and cos 4 = -

The power condition for the R. L series arrangement can now


be deduced as follows:
The diagram (F:ig 114) sllows tllc lx~sicwaveforms of 1, and i,
redrawn to allow the power wave to be deduced.
Let v = V m sin or be the applied voltage and
i = I , sin (oi - d ) be thc circuit current lagging the
voltage by the angle d,
Thcn the instantaneous po\vcs p = 1,;
= I/,,, sin n)i x
I , sln (wr - @)

Fig 1 14
o r p = Vm I, sin o t sin (ot- 4)
cos 4 - cos ( 2 0 t 2

4)

-v'2

d2
Thus p = VZ cos 4 - VI cos ( 2 0 t - 4).
The Average power P = Average of VI cos 4 - Average of
vz cos ( 2 0 1 - 4 )
Whence P = VIcos 4 - 0. Since the average of a cosine
WCIVCis

0.

Note. Vl cos 4 is a constant quantity, thus its average value is


apparent.
In the expression, as finally deduced, P = VI cos 4, we recognise the term power factor as it has been already introduced.
We now see how it can vary between the limits o f 1 and 0, to
enable the extreme conditions of pure R or pure L to be satisfied.
If we examine the power wave we also see how power factor o r
cos 0 is involved with the actual displacement of the power wave
axis above the horizontal. The expression marked thus * shows
that, here again, the power wave is periodic and of twice supply
frequency. It consists of ve and - ve areas, showing that some
power can be returned to the supply, the amount depending on
the power factor at which the circuit operates. The greater +ve
net result of the power wave area, the greater the power consumption of a circuit and the nearer unity the power-factor
condition.
It should be noted that the only component responsible for
power consumption is the resistance and that the deduction
made previously may well be repeated here to advantage.

206

R EED ' S BASIC ELECTROTECHNOLOGY

v we can write:
,

Thus since P = 12R and I =

Power factor will also be the subject of further treatment later,


but i t may be useful to point out, at this stage, that the product
1.1 is frequently referred to as the 'volt amperes' of the circuit
and may suggest the 'apparent power'. P, we know, is the 'true
power, so we have the relation.
True power = Apparent power x cos 4
; ~ n dhcncc tlic nanlc powcr factor for cos 4.
Example 65. A circuit has a resistance value of 25!2 and an
~tidi~c-tiincc
v;lli~eo f 0 3 H . If i t i c connected to a 230V. 50Hz
supply, find the clrcuir current, the power ['actor and the power
d~sslpation.
X, = 2rfL = 2 x 3.14 x 50 x 0.3 = 314 x 0.3 = 94.2Q
Z = 2 5 2 9 9 4 . 2 2 = m 0 = 97.5Q
230
I = --- = 2.36A
97.5
R
25
Power factor = cos 4 = - = - = 0.256 (lagging)
z 97.5
P = V I cos 4 = 230 x 2.36 x 0.256 = 139W
o r P = I Z R = 2.36' x 25 = 139W.
It will be noted that the word 'lagging' has been introduced
after the power-factor figure. This is usual to indicate whether
the current la@ or leads the voltage. The latter is possible for
certain circuit conditions which will be considered as study
progresses, but it can be assumed that inductive circuits always
operate with a lagging power factor. The term is used for the
circuit current with respect to the applied voltage, ie the current
lags the voltage.
Example 66. A coil of wire dissipates 256W when a direct
current of 8A is flowing. If the coil is connected to an alternating
irpplicd voltage of 120V, the same current flows. Find the
resistance, and impedance of the coil and the power dissipated
011 i l , C .

D.C. condition. Since I = 8A and P = 256W, then the applied


756
voltage must be '
,= 32V. The coil resistance must also be

22
8

4Q

A.C. condition. Since I = 8A and the applied voltage is 120V


120 30
thcn thc coil impedance must be - = - = 15Q
8
2

THE A.C. CIRCUIT ( CO N TI N U ED )


.
- -- - - ---- - ---

207
---

The resistance is, as for the d.c. case = 4R


The power tiissipi~tetl 121< or H 2 x 4 - 256W, I N f n r the
d.c. condition.
The latter part of the problem could have been solved by

P = V I cos 4 = 120 x 8 x cos 4 and cos 4 obtained from z


4
Thus cos 4 = - = 0.266 (lagging)
15
and P = 960 x 0.266 = 255.4W
CAPACITANCE
The property of capacitance is attributed to an electrical
condenser or capacitor, to use the modern term. A more complete treatment of the capacitor will be made in Chapter 13, but
since the component is mainly associated with a.c. circuits, it is
necessary at this stage to describe the arrangement and action
briefly, in order to enable a.c. circuit theory to d n t i n u e .
If two conductors, arranged as plates, are separated by insulation, such as air or mica and are connected to a d.c. voltage, then
at the instant of making connection, a current will flow. This
current is of maximum value a t the instant of switching-on but
gradually dies away to zero value. This is termed a 'charging'
current. and the action is explained by considering the insulation
to be put into a state of electrical stress. The effect of this stress
can be likened to a 'back e.m.f.' which gradually builds u p in the
cupucitor to opposc tllc Supply voltuyc. Oncc tllc cupucilor Ilus
been'charged and this voltage has built up, its presence becbmes
apparent from the fact that, if the supply voltage is lowered, this
back e.m.f. asserts itself, and causes a current to flow in the
reverse direction, ie i t causes a 'discharging' current.
Since an alternating voltage is varying all the time, either
rising o r falling, it follows that, if it is applied to a capacitor or a
circuit with capacitance, then a 'to and fro' or a.c. current will
flow. This is explained by the fact that, as the voltage across the
capacitor plates rises, a charging current results and as the
voltage falls,.a discharging current results, due to the stress e.m.f.
already mentioned. The magnitude of the current depends on
the Capacitive Reactance (Symbol-X,. Unit-the ohm) of the
circuit. This term is introduced to correspond with inductive
reactance and an expression for it will require to be developed.
A unit for capacitance must however, be introduced first. It can
be shown that the quantity of electricity or charge which passes
into a capacitor is dependen1 on the applied voltage. Therefore
Q cc V or Q = CV where C is a constant depending on the

construction of the capacitor, [e upon its dimensions, nature of


the insulation used between the plates--called the dielectric,
upon thc sp:~cing between the plates, etc. A measure of this
constant. termed its capacitance, can now be given as a definition
in units already known to us. Thus:
capacitor is said to have a unit capacitance of 1 Farad if 1
volt charges it with a quantity of electricity of 1 coulomb. It
should be remembered that a coulomb results from an average
current of 1 ampere passing for 1 second. It is also pointed out
that a farad is a very large unit and the micro-farad (pF) is the
I I I O ~ Cl~r;\c~ici\I
1111il. 1pF = I Farad x
I t should bc noted that current only flows if the volt:~gcacross
the plates is changing. In the above expression V is the change'in
j ) . d i~crosstllc plalcs I'rom 0 to V volts, wtlcn thc switch is closed.
If the p.d. increases by v volts in t seconds and i is the average
charging current, we can then write:
For very short instants in time, the above will become:
Cdv
1 = -dl

C A P A C ITI V E KEACTANCE . The action of a capacitor when a n


alternating voltage is applied across its plates can now be
considered in detail and is illustrated by the diagram (Fig 115).

Fig 1 15
As the voltage rises from 0 to I f , in a quarter cycle, the charge
or1 tl,c ~>l:\tc'srises 10 ('J",,,, since Q = CV. During the next

qu;\rter cycle, the cliargc frills to zero. For the next quarter cycle,
r l ~ cclla~.gcrises lo - ,'L(
and again finally falls to zero for the

21 1

THE A . C . C IR C U I T ( C Q N T I N U E ~ '

Example 67. A capacitor of value 200pF is connected across a


220V. SOH7 supply mains, Find the curretlt which would be
recorded and the circuit impedance.

1 o6
x 3.14 x 50 x 200
lo6
- lo' ohms
6.28 x lo4 - 628

Here X, =

1 O6 =
2nfC 2

220 = 13.8A
or I = 15.92
Since there is no circuit resistance then the impedance is made
up of reactance only or Z = X, = 15.9253.

The current is given by

5.

Xc

CIRCUIT WITH RESISTANCE AND CAPACITANCE

The diagram (Fig 117) illustrates the circuit conditions and the
technique employed for its consideration is similar to that used
for the inductive 'circuit of Condition 3.

hT
V

"Kc

Fig 117
The applied voltage v is considered to be resolved into two
components V , and V x c . One component V , overcomes the
resistance voltage drop due to the passage of current I, and
the other component Vxc maintains the charging current of the
capacitor and is at all times equalled and sustained by the
internal stress voltage. As seen from Condition 4, there is a 90"
phase displacement between V,, and I. If current is used as the
reference for the waveform and phasor diagram, since it is
common to R and C (this being a series circuit), then the conditions shown can be deduced. If the voltage triangle (shown
heavy) is extracted, the impedance triangle and relationships can
be found thus:
v=
vxc2= J ( I R+) ~( 1 x 3=~ I
J

c+

circuit, where it was said that a choke-coil without resistance


was not possible, here it is pointed out that a capacitor has
negligible resistance and thus Circuit Condition 4, as set out
below, can exist and is practical.

4.
fi

CIRCUIT WITH PURE CAPACITANCE

This condition has already been considered above in detail.


r'e circuit diagram however is shown (Fig 116) and the wavei m n s and phasor diagram repeated, since the power condition
has yet to be considered.

Fig 116
The power at any instant p = vi
or p

Vmsin

wt

x I,,,sin

= I/, 4, 'sin or cos of


=

sin 2

ot

= V l sin 2

ot .

V
a

. .*.

v2

d2

sin 2 o t

Thus average power P = 0 since the average of a sine wave is


zero. The expression marked thus * shows the power wave to be
periodic and of double frequency. The diagtam also shows
power to be + ve at the times when the voltage is increasing and
energy is being put into the electrostatic field of the capacitor.
When the voltage decreases the power is shown as -ve, ie energy
i \ hcinp, ~.rc.ovcl.ctlfro111 I I I C ficltl ; I \ l l ~ c i ~ p i i c i t o lc. l i s c l ~ i ~ ~ ~ g1 c1sd
thus no power is wasted. The power wave is seen to be symmetrical about the axis and the circuit power factor is zero. Thus
if the expression P = VIcos q5 or P = V I x power factor is
applied to this condition. it follows that-cos 4 = 0, as is borne
R
0
= -- = 0. 111c1.tbeing 110 rcsrstitncc
I
I I
7 l

209

THE A . C . CIRCUIT(CONTINUED)

last quarter of a cycle. The total change of charge for a complete


cycle i s t l i u ~4CIf,,,n ~ i dthiu occtlrtr /times second, Tlie nvernge

Q
current during this time is I

or I,,

41'('C'm
= 4/CVm since
r

For a sine wave I,

is I second.
K

I,,. Thus I, = - x 4fCVm


2
2
= 2TcfCVm o r in r.m.s. values 0.707 I, = 2nfC x 0.707Vm and
I = 2KfCv.
v
]I
Whence - = -= Xc. Hare X, is called the capacitive
I
2nfC
reactance and the expression is in line with that developed for
inductive reactance.
1
lo6 ohms. Here C is in
megohms or 2xfC
Thus Xc = ZnfC
P
microfarads.
X, itself is usually measured in ohms. It should be noted that
the current wave is seen to lead the voltage wave by 90" since
maximum current occurs at the instant of maximum rate of
charge of voltage. If a phasor diagram is drawn, then it follows
that the current phasor I is 90" ahead of the applied voltage
phasor V.
The above conclusions can also, be shown thus:
Lct v = V,, sin ( I J / he the sinusoidal volt;~gei~ppliedacross the
dv
C'rl ( b ' , sin ( 0 1 )
plates of the capacitor. Since i = C- then i =
nt
dt
= - x

. The capacitor current is in


1

quadrature with the voltage. It is also sinusoidal. If -is called


oc
the capacitance reactance and is made2qual to X,

then f = - sin wr + - and i becomes a maximum, when


"
Xc
the wave becomes a maximum or sin ot - = 1

i +3

. .
V
v m or 0.7071, = 0.707Vm giving
Thus I,,, = I =xc
xc
x
Summarising here V = IX, as for the inductive circuit, except
1 = -1that here X - --- 0c
2Kfc'
The current leads ,the voltage by 90' and unlike the inductive

If Z is the circuit impedance then - = Z. =


I
or==

Jw

As before cos

\/R7 +

xC2

4=2
R

The power relation 'follows the form already used several


times.
Thus power at any instant p = vi
where I = I,,, sin or and v = Vmsin (ot - 4)
then p = Vm I, sin ot sin (or - 4)
cos $I- cos ( 2 0 t - 4)
= v m Im
2

'i

x
cos 4 - cos (2ot - 4)
d2
d2
= VI cos (tl - V I cos (2wt - 4).
Average power P = Average of V I cos 4 - Average of V I
cos ( 2 0 1 - 4) or P = VI cos 4 - 0. Since the average of a
cosine wave is zero.
Thus P = VIcos +the
form of expression already encountered several times. If the power wave is plotted as before it
will be as shown in the diagram (Fig 118).

or p =

-A

Fig 1 1 H
The power wave is seen, as before to be periodic, of double
Srrqucncy and consisting of + ve and - ve sections. The average
value can be found from the amount by which the axis is displaced above the horizontal and this displacement varies with
the power factor of the circuit. Thus Conditions 1 and 4 are also
covered. If .Y, = 0 and the circuit is purely resistive then
cos 4 = I ;~nd the wavc will be fully. displaced above the
linrizc~nt:~l.
I f R = 0 and the circuit is piircly cnp;~citivcthcn

213

THE A . C . C IR C U IT (CONTINUED)

4 = 0 and the wave will be symmetrical about the horizontal


g i v i n g i'
0. As I'ol. t l ~ cintluctivc c i r c i ~ i t ,tho only component

cos

responsible for the dissipation of power is resistance.

As before P = 12R or P = I x I x R
R
and P = V I - or P = VI cos

= 1'1-

4 as already deduced.

Example 68. A 500W, l00V bulb is to be connected across


250V, 50Hz mains. Find the value of the capacitor required to be
connected in series.
500
Current taken by bulb is - = 5A

100

100
Resistance of lamp = - = 2 0 0
5

250
On 250V, impedance of the circuit is to be - = 50Q
5

d m 2 io~

JWTS = 10
=
= 1 0 0 = 10 x 4.58 = 45.8Q

T ~ U SX, =

Again X,

and 45.8 =

2 5 - 4

1 o6
2nfC

1 O6
1 o6
or C =
2 x 3.14 x 50 x C
3.14 x 100 x 45.8

I 04 = 69.5pF.
giving C = 144

THE SERIES CIRCUIT


From the work which has now been completed and the
various circuit conditions considered under the Cases 1 to 5, we
can see the. general form of technique employed for a series
circuit. The phasor diagram is easily drawn with current being
used for the reference phasor. From this diagram is deduced the
circuit relationships and expressions. This method is employed
below.
\

INDUCTIVE IMPEDANCS IN SERIES

The diagram (Fig 119) shows the circuit arrangement and the
deduced phasor diagram.
Impedances A and B, consisting of resistances and reactances
of values RA, RB,XA and X, ohms respectively, are connected in
series. From the phasor diagram we can deduce an expression
for the total circuit impedance Z, noting that it is not equal to
Z,
Z,.

214

REED'S BASIC ELECTROTECHNOLOGY

"no

"R*

Fig 119

Usinn the diagram we have:

If Z is the equivalent circuit impedance then:

or summarising, for more than two inductive impedances,


Z = J(R, + R B R c . . .)'
(XA XR X C . ..)2
Aiso the power factor is given by:

The example given below s'hows how simply the above


expression can & adapted for practical use.
Example-69. Two coils A and B are connected in series to
50Hz mains. The current is 1A and the voltage across each coil
is measured to be 45V and 70V respectively. When the coils are
connected to a d.c. supply, the current is also l A , but the voltages
across the coils are now 20V and 40V respectively. Find the
impedance, reactance and resistance of each coil, the total
circuit impedance, the applied a.c, voltage and the power factor
of the complete circuit.
On D.C.
On A . C .

THE A.C. CIRCUIT ( CONTINUED )

21 5

d m 2

= 1oV/7' - 42 = 10\/49 - 16
= 10,'X=: I0 x 5.74 = 57*4n
Total R = 20 40 = 6051

Also XB =

Total X = 40.3 57.4 = 97.7f2


Total impedance Z = J60'
3737l = l0\1'6' t 9.77'
= 1 0 J K C T = 1 0 m = l o x 11.4
= 114R
Applied voltage = 114 x 1 = 114V

Circuit power factor = - - -- 0.53 (lagging).


2'
114
INDUCTIVE AND CAPACITIVE IMPEDANCES IN SERIES

The diagram (Fig 120) shows the arrangement

Fig 120
From the phasor diagram we can deduce the expression for
the total circuit impedance Z. I t will be noted t h a t althougli ,!'I
and VRB are in phase and can be added, VxAand V X Bare ant]phase and the resultant of the vertical phasors must be obtained
by subtraction. Thus from the resulting final diagram (shown
heavy) :

If Z is the equivalent circuit impedance then,


Summarising Z = J RA + R , ) ~+ ( X , - x , ) ~
Also for the circuit, the power factor
V,-IR
R
cos qb = - - - = - or, cos 4 = R* + RB
v IZ z
z
The above two circuit conditions give rise to the general series
circuit, which is set out below.

2 16

REED~SBASIC ELECTROTECHNOLOGY

THE GENERU SERIES CIRCUIT

From work already done on circuit theory, a fundamental


expression can be deduced from the phasor diagram (Fig 121).

I
Fig 121

Since it is a series circuit, current is common and can alsg be


used as the reference phasor. It will be noted that the condition
being considered is similar to that already considered for
Inductive and Capacitive Impedances in series, except that all
resistance of the circuit is taken as being contained in one
resistor R. Then for the phasor diagram.
V , = IR and is in phase with the current
V X L= I X L and is 90" ahead of I
V,, = I X , and is 90" behind I
V x , and Vxc are 180" out of phase or anti-phase and a phasor
difference can be obtained where V x = V,, - V,,. Here V X Lis
assumed greater than, Vxc
Further deduotion from the diagram is possible, thus:

v = JvR2+ vX2= J v R 2+ (v,,,- vXc)'


= J ( I R ) ~+ ( I X , - 1 x 3 ~

If Z is taken as the equivalent impedance of the circuit, then

v
Z=T

Example 70. A series circuit is made u p of a choke-coil of


~.csistancc2 0 0 and inductance 0.08H. connected in series with a
100iiF capircitor. I f thc cir.cuit is connected across 200V, 50Hz
mains. find ( a ) the circuit current a n d ( h ) its powcr fr~ctor.

THE A . C . C I R C U IT ( CO N T I N U ED )

217

Here X , = 2xfL = 2 x 3.14 x 50 x 0.08 = 25.25).


1
1 o6 - - - - --- - -lo'- - 3177m
314
,ye
= 2 x 3813 x 5 0 n lo()
Resultant reactance = X = X, - X , = 25.2 - 31.75

- 2;rc

-- - 6~55fl

The - ve sign denotes that the capacitive reactance predominates


and that the phasor diagram will be as shown (Fig 122).

Fig 122
From the diagram as before V =
or Z =

tG-c
d m 2

The power factor is glven by cos

R
20
4== 0.95
Z= 21

(leading), ie the current leads the voltage since the capacitive


reactance of the circuit predominates.
KESONANCE. An

sion

z=

'R'

examination of the general serles circuit expres-

{ ? n f ~-

can a r m when 2 ~ f =
L

1'
2nfc 1

-I

will show that a condition

in magnitude. ;c the capacitive


2 XJ'C
reactance equals the inductive reactance. Under thls condition
Z = R and the circuit is said to be in a 'state of resonance'. The
current passed will be limited by the value of R only and although
large voltages can be present across components L and C, their
effect on the supply voltage V is not evident. Voltage resonance
as this condition is called, is used to advantage for some practical
purposes, especially radio work.
>

Example 71. A coil of unknown inductance and resistance is


connected in series with a 25R, non-inductive resistor across
250V, 50Hz malns. The p.d. across the resistor is found to be
l5OV and across the coil 180V. Calculate the resistance and
~nductanceof the coil and also find its power factor.
The circuit diagram and phasor diagram (Fig 123) are shown.

Fig 123
This example is important in that it involves basic fundamentals and yet has a simple solution.
The phasor diagram is first explained with the various voltage
drops considered in detail. V , is the voltage drop across resistor
R = IR. V , is the voltage drop across the coil and is the
resultant of two voltage drops, V , across the resistance of the
coil = Ir and V, across the reactance of the coil = IX,. V , is in
phase with current and V L is 90' ahead of the current. From the
phasor diagram i t is apparent that V is the resultant of V , and
V, and that the expression given for simple phasor summation
can bc applicd .hcrc.
.I~-.
Thus V = \/I.', +
2VRV,cos4,,
01.250'
150' + 180' + 2 x 150 x 180 x cos gL
:. 62 500 = 22 500 + 32 400 + 54 000 cos 4,
or 54 000 cos b,.
- = 62 500 - 54 900
7600
COS q L= ---- - - = 0,141 (lagging)
0'76
54 000 - 5.4

THE

A.C.

CIRCUIT ( CONTINUED )

The impcdilncc of thc coil

l80
0

219

?On

Resistance of coil = Z cos 4, = 30 x 0.141 = 4.23Q


Reactance of coil = J302 - 4 ~ 2 =
3 ~29-7i2
29.7
Inductance of coil =
= 0.0945H
2 x 3..14 x 50
Power factor of coil = cos 4, = 0.141 (lagging).
Example 72. A moving-iron voltmeter with a resistance of
1732Q and an inductance of 0.625H registers 1 lOV with maximum deflection on a 50Hz, a.c. circuit. It is required to be placed
in a 230V, 50Hz a.c, circuit in series with a non-inductive resistor.
Find the value of R, the required resistor.
Xofmeter = 2 x 3.14 x 50 x 0,625 = 314 x 0.625
= 196.250
Z of meter = J 1 7 3 2 ~ + 196.25' = 100J11.32' + 1.96'
110
Current for full-scale deflection = -= 0.063A
1 744
On 230V. New circuit impedance must be
and 36502 = (1732
13 322 500 = (1732
( 1 732

+ R)'

+ R)' + 196.252
+ R)2 + 38 433.7

= 13 284 066

(1732 + R)2 = 13.28 x 106


or 1732 + R = 3.64 x lo3 = 3640
R = 3640 - 1732 = 1908R.
Example 73. A coil of resistance 10R and inductance 0.1H is
connected in series with a capacitor of capacitance 150pF,
across a 200V, 50Hz supply. Calculate (a) the inductive reactance, (b) the capacitive reactance (c) the circuit impedance (d)
the circuit current (e) the circuit power factor (f) the voltage drop
across the coil (g) the voltage drog across the capacitor.
(a) Inductive reactance = 2nfL = 2 x 3.14 x 50 x 0.1
= 3 1.4Q
(b)

Capacitive reactance

- = - :

27tfC 2 x 3.14 x 50x 150


lo3
=21.2*
3.14 x 15
(c) Resultant reactance = 31.4 - 21.2 = 10.2R (inductive)
= J102 + 1 0 . 2 ~
Impedance =

200
(d) Circuit current = -= 14A
14.28
lo - 0.7 (lagging)--since
the circuit
(e) Power factor = -14.28
reactance is net inductive
(f)

Impedance ofcoil

= ,/lo2

+ 31.42 =

dl00

a14 x33R33 = 462V


Voltage drop across coil

+ 986

(g) Voltage drop across capacitor = 14 x 21.2 = 296.8


= 297V.
Note. The point already made in connection with resonance.
Although resonance is not occurring here, the condition is
working towards this and large voltages can be built up across
coniponcnls. Thus thc fuct that thc voltt~gcsacross the coil and
capacitor are larger than the supply voltage is in accordance with
theory and all the values are in order.

CHAPTER 9

PRACTICE EXAMPLES
1.

A circuit has a resistance of 3R and an inductance of


0.01 H. The voltage across its ends is 60V and the frequency
is 50Hz. Calculate (a) the impedance ( b ) the power factor
(c) the power absorbed.

2.

A 100W lamp for a lOOV supply, is placed across a 220V


supply. What value of resistance must be placed in series
with it so that it will work under its proper conditions? If a
coil is used instead of the resistor and if the resistance of the
coil is small compared to its reactance, 'what is the inductance of the coil? The frequency is 5OHz.rWhat is the total
power absorbed in each case?

3.

An inductive load takes a current of 15A from a 240V,


50Hz supply and the power absorbed is 2.5kW. Calculate
(a) the power factor of the load (b) the resistance, reactance
and impedance of the load. Draw a phasor diagram showing
the voltage drops and the current components.

4.

Two inductive circuits A and B :\re conncctcd in series


across 230V, 50Hz mains. The resistance values are A 1200 :
B 100R. The inductance values are A 250mH; B 400mH.
Calculate (a) the current (b) the phase difference between the
supply voltage and current (c) the voltages across A and B
(d) the phase difference between these voltages.

5.

Two coils are connected in series. When 2A d.c. is passed


through the circuit, the voltage drop across the coils is 20V
and 30V respectively. When passing 2A a.c. at 40Hz. the
voltage drop across the coils is 140V and lOOV respectively.
If the two coils in series are connected to a 230V, 50Hz
supply, find the current flowing.

,6.'

' A simple transmission line has a resistance of 10 and A


--'reactance at normal frequency of 2.5Q. It supplies a factory
with 750kW, O$pf (lagging) at a voltage of 3.3kV. Determine the voltage at the generator and its power factor. Find
also the output of the generator and draw the phasor
diagram.

cr>TYT

-:--

A non-inductive resistor of 8 R is connected in series with


an inductive load and the combination placed across a lOOV
supply. A voltmeter (taking negligible current) is connected
across the load and then across the resistor and indicates
48V and 64V respectively. Calculate (a) the power absorbed
by the load (b) the powerabsorbed by the resistor (c) the
total power taken from the supply (d) the power factors of
the load and whole circuit.

'netted

A circuit, consisting of a resistor and a capacitor con-.


in series across a 200V 4 0 H i supply, takes a current
of 6.66A. When the frequency is increased to 5OHz and the
\~olt;lpe maintained at 200V. the current becomes RA.
( ' i t l ~ ~ ~ 111r
l i ~vitluc
l ~
ol' r c ~ i s t a n c cand cupacirance and sketch
a phasor diagram (not to scale) for either frequency.

9.

A coil, having an inductance of 0.5H and a resistance of


6012, is connected in series with a 10pF capacitor. The combination so formed is now connected across a sinusoidal
supply and it is found that, at resonance, the p.d. across the
capacitor is 100V. Calculate the current flowing in the circuit
under this condition. Sketch the phasor diagram (not to
scale).

10.

A certaln coll has a resistance of 400R and, when connected to a 60Hz supply, an impedance of 438R. If the coil
is connected in series with a 40pF capacitor and a p.d. of
200V, 50Hz is applied to the circuit, find the current and the
p.d. across the coil and the capacitor.

CHAPTER 10

THE D.C. MOTOR


A d.c, machine will run ns i\ motor if its field i ~ n diirmuture nre
connected to a suitable supply. The 'motoring' actlon is based
on the fundamental law described in Chapter 5, which stated
that a force is set u p on a conductor which lies in a magnetic
field and carries current. The diagram (Fig 124) shows the basic
arrangement for revision purposes.

F IS THE FORCE ON THC


CONDUCTOPI TENDING TO
TURN THL ARMATURE

-F
DlPtCTlON OF
FORCE FROM F I R I T
PRlNClPLfI

Fig 124
DIRECTION 0 1 : 1:OUCE

The four small diagrams (Fig 125) show that, in order to


reverse the direction of the force and thus the direction in which
the armature will rotafe, it is necessary to reverse the current in
the conductor with respect to the magnetic flux.

FOPCE O'WN
(0

~ORCU
EP
( CU R REN T
R E V ER S ED )

(bl

FO RC E UP
(FIELD
R E V E R S ED )
(c)

F O R C E ' D O WN
( C U R R EN T L FI ELD
R E V E R S ED )

(dl

Fig 125
The practical aspect of this rule should be remembered if a
motor is found to.run in the incorrect direction when first connected up. Reversal of rotation can be obtained by interchanging the supply leads to the armature circuit. A hand rule

has been developed to help memorise motor action and is


comparable with that enunciated in Chapter 6 for the generator.
L EF T - H A N D P U L E (Fleming's). The diagram (Fig 126), shows
the practical interpretation. The first and second fingers are
made to represent the flux and current respectively, as for the
right-hand rule. The direction of force o n the conductor will
then be represented by the thumb. Note. As for the right-hand
rule, the thumb, index finger and second finger must be placed at
right angles to each other.

Fig 126
MAGNITUDE OF FORCE

From the first principles set out in Chapter 5, it was shown


that the force acting on a conductor in a magnetic field, is proportional to the flux density, the current and the active length of
the conductor ia the field. The law was summarised by the
fundamental formula F = BIL newtons but, it is pointed out
here that, the magnitude of the force also depends on the
inclination of the conductor to the direction of the field. It is a
maximum when they are at right angles.
Example 74. Calculate the force in newtons, as established on
;I conductor, O.5m long, carrying a current of 500A in and at
right angles to a magnetic field of uniform density 0.8T.
S~ncr.I: = R I I .
'Then F = 0.8 x 500 x 0.5 = 8 x 5 x 5 = 200 newtons.
I t should hc nc~rcd.tllat,i f thc conductor is sit~r;~tcd
o n ;In
a1'matul.c at a 1.irciiu5 I . metres, then the torque produced on the
shaft can be expressed as F x r. newton metres.
BACK E.M.F. OF A MOTOR

If the motor is i~llowedto ro~.;tedue to the torque produced


hq' the armature conductors. then these same conductors will cut
;I mi~gnctic
ficlti i ~ n dfrom Faracii~y'slaw i t is known tllat an e.m.f.

THE D.C. MOTOK

225

will be induced, the magnitude of-which will be given by the


g c ~ ~ c l . c i lCo ~
S I, ~ I . C ~ ~ I iOl kI ~1 1,

111in

Ilcc~i tlcvclol~ctl 1 1 1 <'lir~ptcrH,

namely :
From first principles, it can be reasoned that the direction of
the induced e.m.f. will be such as to oppose the applied voltage
and a condition of balance must result. This is also supported
by the fact that since, the direction of rotation would be opposite to that for a generator which is to be operated under the
same directions of flux and current in the armature conductors:
then the induced e.m.f. opposes the current flow and can be
termed a 'back e.m.f.'. This e.m.f. must always be less than the
terminal voltage V, so as to allow the motoring condition. Thus
the armature is seen to start as a passive load, but as it rotates, it
accelerates until the condition of balance is at;ained when the
supply voltage is equal to the voltage drop in the armature
plus the back e.m.f. being generated. This balance condition is
expressed by the voltage equation set out below and the motor
armature operates as an active load.
VOLTAGE EQUATION

V = Eb + I,R,. This equation explains the voltage conditions


as they occur for the armature circuit. Here V is the voltage
applied to the armature, Eb is the back e.m.f, being generated
and I,R, is the armature voltage drop caused by thc urmilturc
current I, passing through the annature resistance R,. If a
problem is encountered where the brush voltage drop is given,
then due allowance must be made for this.
It should be noted that the equation is comparable with the
generator terminal voltage equation V = E - IaRa and a
thought about the difference in the two equations will surnmarise the basics of generator and motor action.
CURRENT EQUATION

Eb + I,Ra then IaRa = V - E,


v - Eb
and I, = R.
The equation i i this form Shows how the motor current is
dependent on the value of the back e.m.f. being generated.
The starting conditions are also illustrated.
,
v
At start Eb = 0 .: Ias= R.
But R, is usually very small so as to minimise the armatureSince V

resistance voltage drop for working conditions and thus I, will


be very large. For example. a 220V motor having an armature
resistance of 0.4R may take a full-load current of 52A, but if
started without making special arrangements, the starting cur-

220 - 550A. Such a large


rent Ias would be given by Ia5 = 0.4
starting current could give rise to undesirable starting conditions.
It could 'blow' a fuse, or cause too rapid acceleration-resulting
In mechanlcnl or electrical damage through excessive sparking
; ~ tthe comrnut;\tor and, i t is for this reason that, the starting
current 1," is limttcd hy tllc usc o l ' : ~'st:~rtcr'.Thc bi~sicfeature c f
the starter is a variable resistance which is inserted into the
;~rrni~t\lt'c
c i r c ~ ~;i t~ st;~rtinp
t
;\nd is prildui~llyredi~ccdor cut out
as the motor accelerates LIP to speed.
. Here R, is the full
At the 'instant of star'ting' Ia5 =
Ras + Rs
value of the starting resistance.
SPEED EQUATION

The equation is also essential for understanding the action of


a motor. It is most conveniently obtained by rearranging the
terminal voltage equation and using the generater expression
thus :
Since V = E, + IaRa.Then E, = V - IaRa.
Z@N
P
E , being a generated e.m.f., its
But E, = -- x

60

-,

magnitude
c a n be determined from the generator
formula.
Z@N
P
Hence -- x - = 1 ' - I Ra a
60
A
60 A
- 'aR.)
x -- revolutions per minute.
orN =
Z@
P
Example 75. Calculate the full-load speed of a motor operating
from ;I 440V supply, given: R, = 0.750, full-load armature
current is 55A, the flux/pole is 0.02Wb and that it is a four-pole
machina with a simple wave-womd armature with 43 slots and
12 conductors per slot.
Number of armature conductors = 43 x 12 = 516 = Z
For a wave-wound armature A = 2. Also P = 4 and d, =

'"

THE D.C. MOTOR

227

SPEED CONTROLLING FACTOR!,

'1.11~ dcducl~ur~s
scl out I)clow, ilr tlcr rvc~l I t o 1 1 1 OIC %IWCLI
equation, are of the utmost importance and should be considered in detail by the student. He should ensure that he fully
understands the implication of each deduction.
it is obvious that for any
Since

particular machine only certain variables affect the expression.


Thus 60, A , Z and P are all constants and can be written as K.
Then we have N = K( V - IaRa)
@

or N = K 2 since E, = V - f,R,.
@
K( V - IaR,)
is considered, then for the
If the expression N =
@

purposes of approximation, the voltage drop &R,, being small,


KV
V
can be neglected and we now have N = - or N cc - (approx).
@
0
Thus,speed can be controlled by varying V o r @ and
Variation of V gives direct speed control, whereas
Variation of @ gives inverse speed control.
In deducing the above it should be remembered that the true
reliltion is N r Eb hut under working conditions the v;llue of 6,
(I,

not very dill'erent l'rom that 01' V, the I , H , vol~agcdrop bc~llg


small. The practical application of the deduction leads to the
basic systems of motor speed control in that:
Variation of the voltage across the armature terminals produces a direct variation of speed, ie raise.the armature voltage
and speed rises, lower the armature voltage and speed falls.
In contrast;
Variation of the field flux produces an inverse variation of
speed, ie lower or weaken the flux and speed rises, strengthen
flux and speed falls.
V
The- relationship N cc - (approx) will also be used to deter-

is

Q,

mine the shape of the motor speed characteristics, when these


are being considered.
Example 76. The armature resistance of a 200V shunt motor
IS 0.4l2.The no-load (this is the term used when the motor is
running light, ie not loaded) armature current is 2A. When
loaded and taking an armature current of 50A, the motor speed
is 1200 revlmin. Find the approximate no-load speed.

O n No load. Back e.m.f. Ebo = V - IaoRa


= 200 - (2 x 0.4) = 200 - 0.8
= 199.2V
On Load. Back e.m.f. =,E,, = V - I,, R,
= 200 - (50 x 0.4) = 200 - 20
= 18OV

Since this is a shunt motor, the field is unaffected by the load1112 ;~nn;~turc.
and @, = @.,

ing of

'

-=

/a
=

since K, 6, and 0,cancel

) 0,
Eh
5
'
199.2 - 20 x 199.2
= 1200 X -----No = Nl x
Eb,
180
3
0

TYPES O F D.C. MOTOR


As for the generator, the motor-field windings can be con-

nected in shunt, serles or a combination of both to give a


compound arrangement. The main point to remember is that
the motor is a machine which, at all times, is taking current from
the supply and that the fields are a load, additional to the
armature circuit.

(:1)

( a ) T J l l i SHIIN'T MOTOR
' 1 ' 1 1 ~~ I I , I , ~ I I I ~ C I I ~1 5C \II II OLW I I

Fig 127

(b)

111 ~ I I C diiigri1111(1:ig 127~1)


and i t
will be seen that I, = I, + I,,. The supply voltage V is applied
to b o t h the armature and the field circuits but here is an instance
where the 'equivalent resistance' treatment for a parallel circuit
cannot be applied to find I, because, although R,, is a passive
load. tllc urmuturc is a n activc load when the machine is running.
The sli\~ntmotor is suhst;lnti:illy ;i cnnst;~nt-speedmachine, used

tor niosl duties.

229

THE D . C . MOTOR

Here I,, =

- cind

I, = I ,

I,,,

f(.,l

(b) THE SERIES MOTOR


The ilrri~nyementis shown in rht tlii~gr;im(Fig 127h7. Ilcrt I,
= I,, = I,. The voltage equation is modified slightly in .that, if
V is taken as the supply voltage then allowance must be made
for the voltage drop in the series field and the equation should be
written as:
V = I,,R,,
I,R,
E b = Eb IaRa I,R,,
or V = E,
Ia(Ra R,,).
The voltage as applied to the armature is equal to V minus
the voltage drop in the series field and any voltage drop at the
brushes if this is mentioned. As the machine current rises with
increase of load, the voltage across the armature falls and speed
is affected accordingly. By design, the ohmic value of R,, is kept
as small as possible for this type of machine. his is a variable
speed motor, used mainly for traction, hoist, crane and winch
work.

(c) THE COMPOUND MOTOR


As shown by the &agram (Fig 128), this motor, like the
generator, can be connected as a long-shunt or short-shunt
machine. Again the two fields can also be connected to assist or
oppose each other magnetically. If the resultant flux is strengthened by the arrangemcnt, the ficlds arc said ro bc 'cumul:~tivcly'
connected. If the fields are however connected to weaken each
other, then the motor is 'differentially' connected-an arrangement which is rarely used.

LONG SHUNT
CONNCCliD

SHOll SHUNT
CONNlCltD

CUNULATlVClY
CONNlClfD

DIfICLtNTIAUI
CONNlCltD

Fig 128
Most marine motors are cumulatively-compounded machines.
The relative s-trengths of the shunt and series fields are decided
by the type of performance required and this statement will be
considered further when the characteristics are studied in detail.

230

R EED ' S BASIC ELECTROTECHNOLOGY

THE POWER EQUATION

This equation is important because it shows the conversion


from electrical to mechanical power and the cause of electrical
losses. I t .is also used for deducing the Torque Equation. Procedure is simple and will give the student no difficulty provided
he understands and has masterkd the voltage equation.
Since I' = E, + I,R, and the armature is the agent by which
the electrical energy supplied is -converted into mechanical
energy, then the following deduction is possible;
Multiply the expression by I, and study the result.
Tlliis I ' = E, -t I, R becomes

Ebla + 1, 'iR,,
O h v i o ~ ~ s I'In
l > ~is ;I mc;isure of the power input to the armature
=

clrcult. /,'K, indicates a resistance loss and is [he power lost by


being converted into heat in the armature itself. It is known as a
Copper Loss and is due lo the resistance of the armature. EbI,
must be a measure of the power developed by the armature. T h ~ s
can be seen if the expression is arranged thus:
vra
Ia2R,
=
Ebla
Input Power
Copper Loss Output Power
Xole. The Output Power EbIa is in watts and is the mechanical
power developed by the armature conductors and is not a true
measure of the shaft output power until f i e machine mechanical
losses, such as those due to friction and windage, have been
subtracted. For a problem, when data concerning the mechanical losses is not given, then only an estimate of the shaft output
power can be o%tained in terms of the. electrical output.
Example 77. A four-pole motor has a wave-wound armature
with 594 conductors. The armature current is 30A and the flux
per pole is 0.009Wb. Calculate the total power developed when
running at 1400 revjmin. Estimate the shaft output power if the
mechanical losses are assumed to absorb 10 per cent of the
dcvcloped powcr.
For this machine P = 4, A = 2. Z = 594 and 0 = 0.009
%@,h' 1'
594 x 0409 x 1400
4
Also Eb = --- x - =
60
A
60
2
594
x
0.0
x
14
x
7
or Eb
- - = 5.94 x 3 x 14
'b

M1
--

= 249.48V
The power developed = Eb/, = 249.48 x 30 = 7484.4W
= 7.5kW (approx)
Sirlcc 1ncc1lnnic:iI powcr loss = 10 per cent of 7.5kW
= 0.75kW
tl1c.n shal't output power = 7.5 - 0.75 = 6.75kW,

THE D.C. MOTOR

23 1

THE TORQUE EQUATION

This i s u n imporlut~t cxprca*ic~n, cdlcn rccluirstl t c ~ hc tic:


veloped from first principles for examinat~on purposes. The

method used here involves the power and voltage equations and
is considered to be the simplest.
Since the electrical power output of the armature = E,Ia watts
and the mechahical power developed is given by:
2n x speed (revlmin) x torque (newton metres)
60
2x,YT
EbIa = Then we can write:

60

Substituting for E, in terms of machine data, we have;

P
P
x Z@I, - = 0.1 59Z@Ia 2 x 3.14 x 60
A
A
P
or T = 0.159Z@Ia- newton metres.
-

TORQUE CONTROLLING FACTORS

As for the speed equation sc. for the torque equation, the
factors which influence the torque can be determined. Thus for
irny one particular machine 0.159, Z. P and A ;Ire all constants
and when considered togzllizr can be written iis K .
Thus we have the expression T = K@Ia or T cc @Ia. This
means that the torque developed varies directly with either the
flux and/or the armature current and this fact will also be made
use of for problems and when considering machine characteristics. As a preliminary point of importance, it can be stressed
here that, for a shunt motor for differen4 conditions of loading
(9 is substantially constant and so T a I,. For a series motor
however, @ is not constant and is frequently taken as being
proportional to I,. Therefore, if (9 a I, and T cx @Ia, we can
write for a series motor T a I a 2 . This deduction is used in the
example.
Example 78. A series motor when running at a speed of 600
revlmin develops 3kW and takes a current of 40A. If the starting
current is limited by means of the starter to *A, find the starting
torque. Neglect the effects of armature reaction and assume that
the magnetic circuit is unsaturated.
Since the magnetic circuit is unsaturated, it can be assumed
that @ r~ I,, c~ I,. Thus we have T x @Ia or T = ~ 1 , ~ .

There are alsg, for this problem two torque conditions.


Thus: when running TI = K I a 1 2 .
At starting T , = K I a s 2 o r KIa2,.
When running at 600 revimin, the output = 3kW and thus
T , is glven by:

7.2 - K I
Also - -a or T = TI(?)'
= 47.8($)
TI
K/q2
And stilrting torque 7', = 107.6Nm.

47.8 x 9
4

MOTOR CHARACTER ISTICS


'I lie bel~aviuurol' sllurl~,series and compou~idmotors can be
illustrated by means of characteristics, which can be considered
under ( a ) Electrical Load Characteristics (b) Mechanical Characteristic, The Electrical Character.istics show speed and torque
in terms of armature current whereas the Mechanical Characteristic shows speed related to torque, assuming a constant applied
terminal voltage. The Electrical Characteristics are important,
in that they show the performance of the machine when loaded.
The Mechanical Characteristic shows the suitability of the motor
for any particular application.
The characteristics may be checked by making a load test on a
type o f motor, but the theoretical performance may be
reasoned from the two expessions ?!ready deduced, namely
Nx

.1.

a.

(approx) and T

01,.

THE SHUNT MOTOR


(;I) ELECTRICAL CHARACTERISTICS. SPEED.

If flux @ is Constant,
assuming a constant'applied voltage V, then N may be considered as constant over the load range, since IV cc V and V is
constant. Speed is unaffected by I, and the theoretical graph is
shown d n ~ l c din thc di;\gr:~m(Fig 129), ;IS N, ;\gainst I,. This
motor' is deemed to bc a constant speed machine although, in
pr;~cticcthc spccd docs f;tll slightly with ioad. as shown by the
graph N. 'fliis is explained by the I'act that thc back c.m.f. does
reduce slightly (the fall from no load to full load being some 2
per cent for large machines and some 6 per cent for small
m;~chincs),duc to the armature voltage drop I,R, increasing.
Altliougli tlie field current I,, is constant and flux @ is constant,
tlie armature reaction eff'ect causes the overall resulting flux @ to

i-

dl.op sllglitly. S ~ n c ch' cc s ,~t should be constant


@

11'

the I:, and

---

--

233

THE D.C. MOTOR


-. -

--- - - .-.-

@ variations are proportional. Weakening of flux however


mc:ilrs ii rise in r1rm:lture current tlt~cto the corrcspondina drop
i l l L,.' l ' l ~ cI,&, drop lncrcuscs us u rcsull, urld lllus llle speed

lowering effect of a reduced Eb is greater than the speed raising


effect of a falling @. The net result is that the speed fulls slightly
over the load range of I,.
7

la

Fig 129

Fig 130

TORQUE. T varies as 1, giving a straight line through the origin,


since @, Is assumed constant. In practice @ is weakened by
armature reaction and T drops as a result, departing from the
theoretical straight line T, as shown. The torque available at the
shaft is everywhere lower because of the lost torque due to
rotational losses. Thus two torque characteristics are shown in
Fig 129.

( b ) M E C H A N I C A L CHARACTERISTIC . A S illustrated by the diagram


(Fig 130), this is obtained by plotting N against T and is seen to
be slightly drooping.
Shunt motors are considered to be constant-speed machines
and have only about a 4 per cent drop in speed from no load to
full load. As stated earlier they are used for all constant speed
drives such as for machine tools, centrifugal pumps, purifiers,
etc.

---+
Fig 131

10

THE SERIES MOTOR

(a) E LE C TRI C A L CHARACTERISTICS. SPEED. For this machine, the


load current value is also that of the field current and, allowing
for the effect of armature reaction, it is seen from the diagram
(Fig 13 1 ) that the useful flux 0 is only slightly less than that given
by the magnetisation curve of 0,. Since @ increases with load
1 .
: ~ n dN vnrics :IS - 11 follows t l l ; ~ r the slxcd must drop and t l ~ c
@'

curve will conform to that for inverse variation (a rectangular


Irypcrbola), I1;1tlcl,111g
O L I ~ i15 sa1~11.a
tion of 0 occurs. Thc no-loird
flux is small and speed can be excessive. It is for this reason
that a series motor should never be run 'light'. It is liable to
'race'* and be destroyed by centrifugal force. Like the shunt
motor, N is lower than N , for reasons already described.
'TORQUE.

Saturation oftlie tield is not normally achieved over the


; I \ ' ~ ~ I I I I C171.o11o1.1io11;rI
C~
to I,. Then

\r;oll\it~gIoi~tlI : I I I Hi111(1
C (1)i 4

235

THE D.C. MOTOR

since torque is proportional to @ x I, we have, the deduction


m ~ d ee ~ r l i e r that, T , w la2,Tho curve thorefore follows a
parabola. On heavy loads, as @ commences to saturate, T a I,
and the graph tends to follow a straight line passing through the
origin. This is shown by Fig 131. As for the shunt motor, due to
machine losses, the torque available at the shaft is less than the
develo d torque. At start T cc la2and the starting torque is
very hig . This is one of the advantages of this type of motor.

(b) MECHANICAL CHARACTERISTIC. This is shown by the diagram


(Fig 132) and is given by plotting the N and T values, for the
same armature-current value, as obtairied from the electrical
characteristics. The result is a curve similar in shape to the speedcurrent curve as shown by the diagram (Fig 131).
Series motors are variable-speed machines, giving a low speed
on heavy loads. They are ideal for traction, winch, hoist and
fan work. Their excellent starting-torque characteristic can be
used to advantage where heavy masses have to be accelerated
quickly, as for lifting or traction.

Fig 132
THE COMPOUND MOTOR

As has already been stated, the field connections can be such as


to give a cumulative or differential flux result. The former is
usual and the latter is used for only exceptional motor duties.
The shunt and series motor have in themselves such good
characteristics, that compounding is only used as a means to
minimise disadvantages which may occur'in the basically connected machine. Thus for example, the series motor tends to race

on no load. This effect can be limited by providing a stabilising


shunt field, and the compounding for any machine can thus be
arranged to give either a strong shunt, ,t,eak series effect o r a
strong series, weak shunt field combination. The characteristics
will therefore be considered with these two arrangements in mind.
Here the two fields assist
each other to give a resultant strengthening of flux. The machine
characteristics will depend on the relative strengths of the fields.
(1) Strong shunt-weak series. The characteristics of the shunt
motor ilre so good, that in practice i t is quite suitable for most
drive duties. The provision of a weak series field will not
materially alter the load characteristics, but this field does give
1111 i ~ l ~ p r o v c.stitrtitlg
d
toryilc, 11s cxpli~incdbelow.

C U M U L A T I V E CONNECTION O F FIELDS.

(a) ELECTRICAL CHARACTERISTICS. SPEED. The diagram (Fig 133)


shows the characteristics. Since the net flux rises due to the
series field, this will have a speed-lowering effect since N a

v
@

(approx). The speed will tend to sit down slightly more than it
would for the same machine without a series field. If the series
field is weak, its effect is not appreciable on the speed characteristic, which differs little from that of the shunt motor. However,
when the machine is coupled to a flywheel, a stronger series field
can be used, so that sudden application of load causes momen-

T H E D.C. MOTOR

23 7

tary slowing down with a rise of 1,. The motor speed tends to
'sit down' and the required driving pnwcr i s ohttlinctf from thr
flywheel which, due to its momentum, glves up sonir ol 11senergy
and does work. This arrangement enables the motor and the
electrical sjstern to be protectcd'1'1~ortiulldue stlock :111dis U S C ~
in connection with motors driving specialised loads, such as the
rolls in steel-works, presses and hammers, some types of
compressors etc.
During starting, when voltage is applied to the shunt
field, due to its self-inductance-it being a winding of thin wire
and many turns, a back e.m.f. is induced whlch tends to oppose
the shunt field current. Thus the shunt field current builds up
very slowly and the torque ( T oc @I,) is small in spite of the
large armature current. A series field arranged to pass the starting current I,,, will produce a flux to strengthen the shunt flux.
Thus the net flux at starting will be very muck larger and an
improved starting torque would be obtained which may be used
for starting against heavy loads, such as those encountered for
compressors, centrifugal pumps, certain machine tools, etc.
Once the machine accelerates, the characteristic will follow that
of a shunt motor, and the effect of armature reaction will alter
the theoretical characteristic from T I to T as shown (Fig 133).

TORQUE .

(b) MECHANICAL CHARACTERISTIC. This characteristic is generally


similar to that for a shunt motor.
( 2 ) S~rongseries-weuk sliwrt. Hcrc ugain the cliuracteris~icuf'
the series motor makes it so suitable for its appropriate applications that, it is obvious, its basic performance features will be
retained. Its major disadvantage, eg the tendency to race on light
load, requires to be removed and this is the main function of the
shunt field.
(a) ELECTRICAL CHARACTERISTICS. SPEED. It will be seen from the
diagram (Fig 134) that although the net flux varies, ie it follows
the magnetisation curve, yet it never falls to zero as it does in the
case of the series motor. In effect the shunt field predominates
on light loads and the machine will run as a shunt motor at a
predetermined speed. Once load is applied, the series field
asserts itself and the speed characteristic passes from that of the
shunt machine into that of the series machine. The tendency for
racing on no load has thus been removed and this is the typical
characteristic for a ship's d.c. winch. Nore. The effect ofarmature
reaction and voltage drop on E, is seen at the higher current
values and speed N is lower than the theoret~calvalue N,.

Ia

Fig 134

Since the motor behaves like a shunt machlne 9n light


loads, the torque characteristic commences as a straight line
through the 'gin and then. becomes parabolic as the series field
8.ength. The armature reaction effect gives a slight
increases 1.
reduction 01 net flux with the consequent falling off of torque T
from the theoretical graph T , (shown dotted).

TORQUE.

(b)

ME C H AN I C A L CHARACTERISTIC.

The characteristic for this

THE D.C. MOTOR

239
-

motor is shown by the diagram (Fig 135). It is seen to be similar


to the electrical pad chr~rnctcrinticu~rtidci~tihc tlodr~c'cd,! I N witr
described for the series and shunt mach~nea.It is apparent that
the exact shape and position of the graph will depend on the
relative strengths of the shunt and series tields.
This can be used to maintain a constant speed eg as for an alternator drive. Increase of
load results in an increase in the series flux and, as the fields are
in opposition, the resultant flux is decreased. The speed, being
inversely proportional to flux, increases to compensate for the
fall due to the application of load. Thus the machine speed and
alternator frequency tends to remain constant but the armature
current increases appreciably to provide the required torque
with a reduced flux value. A greht disadvantage of the arrangement is the field cancelling effect at starting and the fact that,
due to the series field establishing itself quick&, the machine
may start to run in reverse. Special arrangements have therefore
to be made when starting a motor with a differentially-connected
field system.
Example 79. A 220V shunt motor runs on light load at a
speed of 1250 rev/min and takes a current of 2.8A. On full load
the current taken from the mains is 40A and owing to armature
reaction, the flux per pole is 4 per cent less' than the no-load
value. Calculate the speed on full load if the armature resistance
is 0.29Rand the field rcsistnnce is 165R.
No load. Voltage
- across shunt field = 220V
220 = 1-332A
Current through shunt field = 165
Armature current = 2.8 - 1.33 = 1.47A
Voltage drop across armature = Ia0R, '1
1.47 x 0.29
= 0.426V
and Ebo = 220 - 0.426 = 219.574V
Full load. Current through shunt field as before = 1.332A
Anna.ture current = 40 - 1.332 = 38.67,A
Voltage drop across armature = I,, R, = 38.67 x 0.29
= 11.23V
and Eb, = 220 - 11.23 = 208.77V
Now since Eb a @N
:. E, = K@N
Eb
00
and
But @, = 0.96G0
Eb,
QlNl
E
= No x Eb, = 1250 x 208.77
OoNo or
bo=
,..
Eb 1
0.96@0N1
0.96 x Eb, 0.96 x 219.57
Thus N , = 1238 ie speed on full load = 1238 revlmin.
DIFFERENTIAL CONNECTION OF FIELDS.

Example 80. A 220V series motor is working with an unsaturated field taking a current of l00A and running at 800 revlmin.
Calculate at what speed the motor will run when developing
half the toraue? The total resistance of the motor is 0.1R.
1 -.. Here T = KI,'
But T, = 0.5T,
1a22

7-2

T =loo2
so0.5
1
- or Ia12= 1W2x 0.5 = 0.5 x lo4 = 500C
T,,
1,:
and I, = 1 0 4 50 = 10 x 7.07 = 70.7A
A I S under
~
the first condition E,, = v - I,, (R, + R,)
= 220 - (100 x 0.1)
= 220 - 10 = 210v
1 Indcr the second condition E,, = V - I, (R, + R,,)
= 220 - 170.7 x 0. I )
= 220 - 7.07 = 212.93V
But Eb = KO N o r
E

Eb,

also O rr I,
@21V2

I a I N l or N2 = I., N1Eb2 = 100 x 800 x 212.93


70.7 x 210
Eb~
I,] N 2
l a l Eb,
and N2 = 1150 revlmin
Thus speed at 4 torque = 1 1 50 revlmin.
SO

MOTOR STARTERS
The need for a starter to work in conjunction with a motor
was mentioned earlier in the chapter, when it was seen that, at
the instant ofStarting since the machine is not rotating, there is
no back e.m.f. The current is consequently limited by the armature resistance alone, unless some arrangements are made t o
include additional resistance in the armature circuit. Thus for all
but 'fractional output power' motors, which have w i t e a n
appreciable resistance, a resistor is inserted into the armature
circuit and then removed in steps, as the motor accelerates u p
to its correct running speed. The arrangement used is incorl>or;~tc~J
in :I ~ ~ t l icl; ,I I I c ~ a '11101or starter' or lnorc simply a
'starter' and consists of a tapped resistor and a switching device
W I I I L ~ I I c11~11ilc\
1 1 1 ~ - IC.\I\I:II~L.L.
to 1~ ~ > I . ; I C I I I ; I I I r~ c ~ l t ~ ;IIICI
~ ~ cf li ~ i i ~ l l y
c u t out altogether. 'l'lle starter may also incorporate other special
attachments which may be considered necessary for the safe
operation of the motor. Thus it may include protective arrangements to safeguard the motor against the adverse effects of a
rt~iuccdworkin3 i,oltagc or :In overcurrent.
:lltliougli motor sti1rter.s will be studied later in more detail, i t
I \ ' I I ) ~ Iop1.1;11~
l o I I I C I I I I ~ I 11c1.c
I
l I i i 1 1 llic Sor111of starter ncccssilry

24 1

THE D.C. MOTOR

for any particular machine is mainly decided by the duty for


which the motor i s being used. Thus i t may be o f the manuallyoperated or automatie type. Ir trrlty be clealgt~otl far moroly
starting and stopping the motor and this may require to be done
only once a day. In contrast, the duty moy he such ns to require
the motor to be started and stopped almost continually for long
periods, as is necessary when working a winch or hoist. Such a
starter is mure frequently referred to as a 'controller'. A further
point of importance to note is that the starting resistor is not
cut out in equal sections from the armature circuit, but that the
resistance values of these sections follow a Geometrical Progression.
The foregoing observations indicate that the starter is of
sufficient importance to require detailed attention. It is an item
of equipment which requires both careful and routine maintenance and a thorough knowledge cd its function is necessary both
from a theoretical and practical point of view.*

SPEED CONTROL
As for the starter, so for the full treatment of speed control,
much additional study has yet to be made. It is proposed here.
only to deal with the basic methods whereby the speed of a d.c.
motor can be controlled and, in this connection, the reader is
reminded of the basic deduction N a

4
or
@

N a - (approx).
@

Thus varying the voltage applied to the motor armature and


keeping the flux constunt will vary the syccd i n direct proportion.
This is termed 'Voltage Control'. Varying the flux of the machine
and keeping voltage constant will vary the speed in inverse
proportion and is termed 'Field Control'.
F IE L D C O NTRO L . This is introduced first, since it is the most usual
type of control. When a motor is loaded, its speed will vary
with load. It may be desired to adjust the speed for any load
condition ie keep it constant throughout the working range or to
raise it above the normal running speed. Field control is used
because its adaptation into the field circuit is easily achieved,
control is smooth and effective and little energy is wasted as heat.
It must be remembered that this type of control will give speed
variation in an upward direction only. It is used for raising speed
above normal and as flux is weakened, for the same driving
torque, armature current will rise. Note. T a 01,. Thus the
motor may be of larger dimensions, if speed variation is required
and .interpoles (compoles) must be fitted to ensure good commutation throughout the working range.

VOLTAGE CONTROL. This is achieved in various ways for the


different kinds of d.c. motor but the fundamental requirement is
to reduce the voltage applied to the machine armature. Thus a
large variable rheostat may be connected. in series with the
armature o r the latter may be supplied from a variable voltage
supply. The method is always used to lower speed and control
is in a downward direction only.
A wide range in the adjustment of motor speed can be
obtained by combining field and voltage control and the methods
of applying these are sufficiently important to require further
detailed study. T o meet the requirements of the duty for which
~ h cnlotor is rcquircd, the starter and speed controller may be
incorporated into one unit which, though simplifying the
electrical circuit requirements, apprirs to complicate the theory
01' the control. Since the correct application and use of a motor
is of prime importance to the practical engineer, it is hoped that
the additional treatment, gven to the d.c. machine in the next
book, will be regarded as a necessary continuation of theory and
that too long a break is not introduced into the period of study
before the necessary advancement is attempted. If such a break
does occur in the student's studies of Electrotechnology then,
he would be well advised to revise the work undertaken in this
chapter..before proceeding to the work in Volume VII.
Example 81. The armature of a motor has 660 conductors
whose effective length is 410mm; of these, only 0.7 are sirnultaneously in the magnetic field. The flux density is 0.65T, the
effective diameter of the armature is 300mm, and each conductor
carries a curren't of 80A. If the armature speed is 800 revlmin
calculate the output power developed.
Force on one conductor is given by F = BZL newtons
.: F = 0.65 x 80 x 410 x
= 52.0 x 41 x
= 2132 x lo-' = 21.32N
Number of conductors in the field at any given instant
= 0.7 x 660
:. Total force = 21.32 x 0.7 x 660 = 9.85kN

Tt~rqllc force

r ; ~ d i l or
~ s 7.

91150 x O!: newton metrcs


2

Thus T = 9850 x 0.15 = 1477.5Nm


2 x 3.14 x 800 x 9850 x 0.15
And power developed =
60
- 124kW
Ex;implc 82. A shunt motor takes 180A. The supply voltage is
4OOV. the resistance of the shunt field is 200f2, and that of the

243

THE D.C. MOTOR

armature 0.02R. I f there i.; a voltage drop of 2V at the brushes.


calculute ( 1 1 ) tile hrlck c , m , f ,o f llrr tncltor (h) thc outprlt powcr
developed (c) the ellic~ency, neglecting all losses I'or which
information is not given.
Shunt-held current

4IK)
-

212

200
Armature current = 180 - 2 = 178A
Armature voltage drop = 178 x 0.02 = 3.56V
(a) Back e.m.f. = 400 - 3.56. - 2 (voltage drop at brushes)
= 400 - 5.56 = 39444V
(b) Output power developed = 394.4

loo0

178 = 7 0 . 2 k ~

output - 394.4 x 178 - 70.2


Efficiency =- --4 x 18
input
400 x 180
17.55
or q = ---- = 0.975 or 97.5 per cent.
18
c
Example 83. A four-pole d.c. motor with a lap winding is connected to 200V supply mains. The armature carries 600 conductors and has a resistance of 0.3Q. The resistance of the shuntfield circuit is 100R, the flux per pole is 0.02Wb. On no load, the
armature current to 3A. If the normal full-load current in the
armature is 50A, determine the drop in the speed of the motor
from no load to full load. Neglect the effect of armature reaction.
Back e.m.f. on no load Eb = 200 - IaoR,,
Shunt-field current =
= 2A I,,, = 3 - 2 = I A

?E
100

.'. Eb. = 200 - (1 x 0.3) = 200 - 0.3 = 199.7V


No-load speed is given by No where:
N
P
600 x 0.02 x No 4
E~ = -Z@
- O O x - o r 199.7 =
60
A
60
4
Thus 199.7 = 0.2 x No
Back e.m.f. E,, on full load is given by:
E,, = 200 - I,, R, = 200 - (50 - 2) 0.3
Eb, = 200 - (48 x 0.3) = 200 - 14.4
or E,. = 185.6V
Since Eb,= KO, N , and assuming a constant flux, then 0 ,= @,
E =
or A
N, =N ! !
Ebo
KG0
Ebo
998'5
185'6 - 5 x 185.6
Thus N , =
199.7
Full-load speed = 928 .revjrnin.
"1

Example 84. Calculate the first resistance step of a starter for a


240V shunt motor having an armature resistance of OaSR, if the
maximum current limit is 60A and the lower limit about 45A.
Let R, = the total resistance of the series resistor put into the
armature circuit. Then if 1,- is the armature current at start
I,, = 60A
240 - 240
and also I,, =
R, R, 0.5 R,
-.s

As the motor starts and ;~cceleratesup to speed, the starter


Ilandle is kept in position until the current falls to 45A. Thus the
starting resistance is still in citcuit, but a back e.m.f. is building
10 ii !h;tl V U ~ yivcn
U ~
by kqb,
Here E,, = 240 - 45(3.5 + 0.5) = 240 - 45 x 4
= 240 - 180 = 60V
At this stage the handle is moved and a seztion of the starting
resistor is cut out. Let R, be the new value of the total starter
resistance. The current rises to 60A but the back e.m.f. does not
change until the motor speed changes. Thus at the instant of
moving the handle
240 = Eb, I, (R, R1)
o r 240 = 60 + 60 (0.5 + R , ) whence

or R, = 3 - 0.5
* = 2.5R. Thus the resistance removed during
the first movement of the handle after switching on, is 3.5 - 2.5
= 1R
The first resistance step is thus 1R.

THE D.C. MOTOR

245

CHAPTER 10

PRACTICE EXAMPLES
1.

A 1 10V series motor has a resistance of 0- 1 m . Determine


its back e.m.F. when developing a shaft output of 7.5kW
when the efficiency is 85 per cent.

L.

A 500V d.c. shunt motor has an input of 90kW when


loaded. The armature and field resistances arc 0.ln and
lOOS2 respectively. Calculate the value of the back e.m.f.

3.

A 460V, d.c. motor takes an armature current of 10A at no


load. At full load the armature currmt is 300A. If the
resistance of the armature is 0.025R, what is the value of the
back e.m.f. at no load and full load.

4.

An armature w~ndingof a d.c. motor conslsts of 240 conductors arranged in four parallel paths on an armature
whose effective length and diameter are 400mm and 300mm
respectively. Assuming that the average flux density in the
air gap is 1.2T and that thd input to the armature is &A,
calculate (a) the force in newtons and the torque in newton
metres developed by one conductor (b) the total torquc
developed by the complete winding, assuming that all the
conductors are effective (c) the power output of the armature
in watts, if the speed is 800 revlmin.

5.

A maripe shunt motor is used for driving a3'freshwater'


pump and is found to take an armature current of 25A at
220V, when running on full load. The speed is measured to
be 725 revlmin and the armature resistance is 0.252. If the
field strength is reduced by 10 per cent by means of the
speed regulator and the torque remqins unchanged, determine the steady speed ultimately attained and the armature
current.

6.

A shunt generator delivers 50kW at 250V and 400 rev/mm.


The armature and field resistances are 0.02Q and 50R
respectively. Calculate the speed of the machine when
running as a shunt motor taking 50kW input at 250V. Allow
2V for brush-contact drop.

7.

A 105V,3kW d.c. shunt motor has a full-load efficiency of


82 per cent. The armature and field resistances are 0.25R and
90R respectively. The full-load speed of the motor is 1000
revlmln. Neglecting armature reaction and brush drop,
calculate the speed at which the motor will run at no load if
the line current at no load is 3.5A. Calculate the resistance
to be added to the armature circuit, in order to reduce the
speed to 800 revlmin, the torque remaining constant at fullload value.

8.

A shunt motor runs at 1000 ~ v / m i nwhen cold, taking


50A from a 230V supply. If the armature and field windings
both increase in average temperature from 15C to 60C, as
the motor warms up; determine the speed when the motor
is warm, given that the armature resistance is 0 . 2 0 and the
field resistance 20012 at 15C and that the total current
drawn from the supply remains constant. Neglect brush drop
and armature reaction and assume the magnetic circuit to be
unsaturated. (Resistivity temperature coefficient 0.40 per
cent from and at 15C.)

9.

A four-pole, shunt motor has a wave-wound'armature


having 294 conductors. The flux per pole is 0.025Wb and
the resistance of the armature is 0.35R. Calculate (a) the
speed of the armature (b) the torque developed,'when the
armature is. taking' a current of 200A from a 230V supply.

10.

A shunt motor runs at 600 revlmin from a 230V supply


when taking a line current of %A. Its armature and field
resistances are 0.4R and 104.5R respectively. Neglecting the
effects of armature reaction and allowing a 2V brush drop,
calculate (a) the no-load speed if the no-load line current is
5A. (b) the resistance to be placed in the armatulrecircuit in
order to reduce the speed to 500 revlmin when taking a line
current of 5OA. (c) The percentilge reduction ip the flux per
pole in order that the speed may be 750 revlmin, when taking
an ; % m a t u r ecurrent of 3OA with no added resistance in the
arrnarure clrcuit.

CHAPTER 1 1

A.C. CIKCUL'I'S (contisued) ANL) SYSTEMS


POWER IN THE A . C . ClRCUIT

From the various circuit conditions considered in Chapter 9,


ie resistance in series with inductive reactance, resistance in

series with capacitive reactance and resistance in series with


both inductive and capacitive reactance; it was seen that, the
current flowing was sinusoidal and displaced from the applied
sinusoidal voltage by an angle 4, termed the phase angle of the
circuit. The general expressions were :
For voltage v = Vmsin wt and for current i = I, sin (or - 4)
a lagging phase angle being assumed for convenience.
The instantaneous power p = vi = VmImsin o t sin(wt - 4)
{cos $ - cos (*Of - 4)
o r p = VmIm

= VIcos $ - VIcos ( 2 0 1 and average power P = VI cos 4 - 0

4)

1'

or P = VIcos 4
VI is frequently called the 'apparent power' of the circuit and
P is referred to as the 'active power'.
Then active power = apparent power x power factor.
The reason for calling cos 4 the 'power factor' can now be
readily seen. I t is the factor by which the apparent power must
be multiplied to obtain the active value of power expended in u
circuit.
active power or cos 4 = P =I*R
So power factor =
apparent power
VI I Z I
=
as deduced
earlier.
The following is also of interest.
Since active power = apparent power x power factor
then P (watts) = VI (volt amperes) x cos 4
Thus wattage is given by the volt amperes multiplied by the
power factor. This can be expressed by W = V A cos 4 or
k W = k VA cos 4.
Note. The term kVA is an accepted method of.giving the
rating of an a.c. generator, motor or transformer and it must be
remembered that it does not indicate the power rating. More
information is required before the latter can be deduced, and
the power factor is usually specified at the same time. The volt

amperes o r V A of a circuit is a term in itself but more has yet to


be said about its usage. It has been retained from the early days
of electrical engineering, before standardising of terms and
symbols was recognised as being beneficial and V A o r k V A , as
a rating, is now used internationally for a x . circuits and
machines.
ACTIVE AND REACTIVE COMPONENTS

These terms are usually used in connection with current but


under certain conditions can be applied 'to voltage and power.
Consider the phasor diagram (Fig 136), for a simple a.c. circuit
with current lagging the voltage by an angle 4. If tlie current I
is considered to be split into its two quadrature components I,
and I, as shown then I, = I cos 4 and I; = I sin 4 .

Fig 136
Since I c o s 4 is a current, in phase with the voltage V and we
know VI cos 4 is the measure of the power expended in a circuit,
then it appears that I cos 4 is the component of current which is
responsible for pewer dissipation. Thus I c o s 4 is called the
active power, wattful o r working component of current. Similarly I, = / sin 4, being always at right angles to voltage, is
responsible for no power and is called the reactive, wattless o r
idle component of current. The example further illustrates these
terms.
Example 85. A single-phase a.c. motor of 15kW and 90 per
cent efficiency is run from a single-phase supply of 400V. Find
the current taken from the mains, if the motor operates at 0.8
power factor (lagging). What is the value of the active current,
the reactive current and the motor rating in volt amperes.
Motor power output = 15k W = 15 x 1000 watts
Motor power input = l5 low x 100 watts
90
I'llc \ult ;~tnpercrating

16'67 - 20.84kVA
0.8

Thc linc clirrent is ohtilined by dividing the volt-:~mpercv ; ~ l ~ ~ r


20.84 r 1OOO = 52.1A
by the supply voltage. Thus I =
400Active component ol'cut.ret~lla= 52 1 x 0.8 = J1,7A,
Reactive component of current = I, = 52.1 x 0.6 = 31.3A.
Note. If 16 667 was divided by 400, then I, would have been
obtained directly.
16 667
Thus 1, = -= 41.7A. I could then be, obtained by
400
4 1-7 - 52.1A and I. as before, by I sin 9 = 52.1 x 0.6. It is well
0.8
to point out the simple relation for sin 4 being 0.6 when cos 4 is
0.8. This is obviously referring to a right-angled triangle of sides
10, 8 and 6. Similarly for examples, cos 4 is frequently given as
0.707 or sometimes 0.7. This is referrin8 to a righ--angled
isoscles triangle and sin 9 in this case is also 0.707 or 0.7
(approx).
THE PARALLEL CIRCUIT

The parallel circuit is being treated under a separate heading,


to remind the student that procedure is different to that for the
series circuit. Nevertheless it will be seen that the method
employed follows the familiar technique of phasor summation,
ie that of resolving into the horizontal and vertical components
or, to bc marc in line with the littest terms introduced in this
chapter, into active and reactive components. The branches 01'
the parallel circuit are made up of simple R, X, or .Y, values in
series, and all work done in this connection will be in no ~ 3 4 '
altered. For a parallel circuit it is pointed out that the same
voltage is applied to all branches and it is usual to work with V
as the reference for the phasor diagram. The current condition is
often written as I = 7, + f, + T3 etc. The dash above the I is to
remind one that, this is a phasor summation and not an arithmetical one. Thus all correct operations for a phasor summation
must be performed.
INDUCTIVE IMPEDANCES IN PARALLEL

Assume two inductive impedances to be connected in parallel


as shown in the diagram (Fig 137). Impedance Z1is made up of
a resistance R, and inductive reactance X, whereas Z , is made
up of resistance R, and inductive reactance X,. The phasor
diagram and circuit relationships are also shown. Since V is
common to both branches it is used as the reference phasor.
The .problem is to find I where I = f1 + f2.

Fig 137

Here I, =

v and I, = v
I

2 2

Resolving into active and reactive components, using arbitniry signs, we hnve I, = I, cos 4, + I2 cos
and I, = - I , sin 4, - I , sin 4,. I t should be remembered
that these phasors are vertically downwards.
Then I =

d m 2and cos 4 = I;.Here cos 4 is the power

factor of the whole circuit.


Example 86. In the circuit shown above, let R, = 3R and
X, = 4while R, = 822 and X, = 642. If the applied voltage
is 20V, find the total current supplied and the power factor of
the complete circuit. Find also the total power expended.
2, = J i i 7 - T T 7 = 4 P T i 2 = @ i = 5 5 n
20 = 4A
Then I, = 5
z2=4-=
J m = m = l 0 ~
20
and I, = - = 2A
10
3
4
cos 4, = - = 0.6 (lagging) sin 4, = - = 0.8
5
5
8
6
cos 4, = - = 0.8 (lagging) sin 4, = - = 0.6
10
10
Also I, = (4 x 0.6) + (2 x 0.8) = 2.4 + 1.6 = 4A
I , = - (4 x 0.8) - (2 x 0.6) = - 3.2 - 1.2 = -4.4A
Whence I

v-4T-+4T
= fltT3

5 Y5A
4
C~rcuitpower factor cos 4 = --- = 0.67 (lagging)
5.95
Power expended = 20 x 5.95 x 0.67 = 80W
The above can be checked thus:
Power in b n n c h I = I I 2 R 1= 4, x 3 = 48W
Power in bianch 2 = I Z 2 R 2= Z3 x 8 = 32W
Total 8 0 ~ .
=

A.C.CIRCUITS (continued) AND SYSTEMS

25 1

INDUCTIVE AND CAPAClTlVE IMPEDANCES IN PARALLEL

Thc prowdurc for solviny prablcma, rtmwcir~tcdwitti tliin lypc


of circuit, follows that outlined above, except that due allowance
is milde for the directions and signs when addin8 the rcrrctivc
components. Thus in the diagram (Fig 138). impedance Z, is
made up of resistance R, and capacitive reactance X , in series.
The phasor for the reactive component of current is drawn
vertically upwards and is allocated a +ve sign, whereas the
reactive component of current for branch 1 is allocated a -ve
sign. The total of the reactive components is thus a difference, as
will be noted. The voltage is again used as the reference for the
phasor diagram.

Fig 138
As before I, = I , cos 4 , + I , cos 4,
and I, = - I , sin 4 , + I , sin 4,. I, will carry either a + ve
or - ve sign, decided by the relative values of I, sin 4 , and
I, sin 4,. Thus the resulting reactive component will act either
upwards o r downwards and the resultant circuit current miiy kx
lagging or leading .as shown ,by the example. As before
I
I = dm
and cos 4 = . ; The qualifying term lagging o r
leading is decided by the sign of I,.
Example 87. A circuit consists of two branches in parallel.
Branch A consists of a 20R resistor in series with a 0.07H
inductor, while branch B consists of a 60pF capacitor in series
with a 50R resistor. Calculate the mains current and the circuit
power factor, if the voltage is 200V at 5OHz.
Branch A. XA = 2 x f ~ =2 x 3.14 x 50 x 0.07
= 314 x 0.07 = 2252
RA = 20Q ... Z A = .JFT@
=
= 29.7R
20
=200 - 6.74A and cos 4 'Thus IA= 29 7
A - ZA
29.7
= 0.674 (lagging)
22
sin 4, = ?A = - = 0.74
ZA 29.7

Branch B. X , =

cos

--

2xfC

1 o6
2 x 3.14 x 50 x 60

10 3
3.14 x 6

50
5
= -= 0.686 (leading)
2,
72.8

Then I, = (6.74 x-0.674)

(2.75 x 0.68)
-t 1.885 = 6.43A
I, = -(6.74 x 0.74)+(2.75 x 0.728)
= 4.55

= - 5
2.005 = - 2.995A.
Note. The mains current will lag, since the effect o f the
inductive branch predominates.
I= J
I
m
2
= .\/6.432 + 2.9952 = d 4 1 . 4 + 9
=
= 7.1A

cos

I, =*6

= 0.902 (lagging).
1 7 . 1
The mains current is 7.1 amperes and the circuit operates at a
lagging power factor of 0.9.
=

P A R A LLEL RESONA~JCE

Before passing on to the more practical applications of parallel


working, it would be well to point out that, a condition of
resonance can occur for the parallel circuit. This condition is
often termed 'current resonance' to distinguish it from 'voltage
resonance' as dealt with for the series circuit. It will be seen, from
the example set out above, that a condition can arise when
I , sin 4, = I, sin 4, and as -these are the reactive components
of cul.~.cntsin inductive and capacitive branches, then they will
oppose each other tending to produce a total reactive component
ol'zcto v;tluc Tlic rcnxtining itctivc colnponcnts will tot;~lto givc
the line current, since I =
+~-0= Ia and the. combined
circuit will operate at unity power factor. The supply current
will also be minimum especially if the resistance values in the
two branches are small compared to the reactance values. This
is illustrated by the pli:lsor diagram (Fig 139). from whicll it is
.;ccn t I i : ~ t .sincc the power. filctors of both branches are low, the

A . C . C IR C U IT S

(continued) A N D SYSTEMS

----

253
--

phase angles 4 , and 4 , are large and I, cos 4, and IB cos &
are small compared to the reactive components.

Fig 139

Large currents can flow in the chokecoil and capacitor


branches, which are very much greater than the main supply
current and these, are therefore, not supplied from the line. On
examining the power waves for an inductor and capacitor, it w ~ l l
be seen that they are directly opposite in phase, as are the current
waves. It can be assumed that as the capacitor discharges, the
power given out is absorbed by the choke in building up its field.
When the field collapses, the power released charges the capacitor
and there is a current due to oscillation of power between choke
and capacitor. Apparatus using such a circuit is an oscillator
and has many applications in radio and electrical filter circuits.
If no supply is available the current is not maintained, due to
energy loss in the circuit resistance which, however small.
cannot be neglected. To maintain the oscillatory current, the
resistance loss must be supplied at the correct frequency from
the external supply source.
POWER-FACTOR IMPROVEMENT
The full meaning and advantages of this technique, which IS
much used in practical electrical engineering work, is best
illustrated by the use of an example, as now considered.

Example 88. (a) Two inductive coils of resistance values 5R


and 8R and inductance values of 0.02H and 0.01H respectively
are connected in parallel across a 240V, 50Hz supply. Find the
coil currents, the circuit current and its power factor.
The arrangement is shown by the diagram (Fig 140). A phasor
diagram is also drawn.

F'ig 140
Branch A. X, = 2xfL = 2 x 3.14 x 50 x 0.02
= 3.14 x 2 = 6.280
Z, =
= 1/25
39.4 =
= 8.02R

Jm2
+

240 = 29.8A cos


1, = 8.02

4,

\644

5
80.2
0.622 (lagging)

= =

6.28
-= 0.78
sin 4,
8.02
Branch B. X, = $ that of b r a n c H , since L is halved
= 3.14R
-- z,*=
= \/64 + 9.9 = \rnn3
= 8.60

,/8r+.nl

= 0.93 (lagging)
3.14
sin 4, = --- = 0.366
8.6
Then I, = (29.8 x 0.622) (27.9 x 0.93) = 18.6 + 26
44.64
1, (29.8 x 0.78) + (27.9 x 0.366) = 23.3 + 10.2

33.5.

It will be noted that the arbitrary - ve sign has not been used
here since, both branches are inductive and there is no doubt as
to, the resultant current being laggin
Then I = w6-m'
= IO&G'-ST
= 10\/19.8
11.2 = 1OJ31
or
I = 55.6A cos 6, = --44'6 - 0.801 (lagging).

55.6

A.C. CIRCUITS

(continued) AND SYSTEMS

255

Example (contd) (b) Find the effect on the main circuit


cumnt and power factor, if a capacitor of 400pF,was connected
across the supply in parallel with the coils.
The phasor diagram (Fig 141), shows the new conditions.

Fig 141
1

Branch C . Reactance of capacitor X c = 2xfC


1o6
- lo'
ohms
Thus Xc = 2 x 3.14 x 50 x 400 - 3.14 x 4

capacitive
and there being no resistance in branch C,-nly
reactance, then cos & = 0 and sin $c = 1
Again I , acts at 90" to the voltage and is wholly reactive, there
being no active component. Then I, as before = 446A
and I, = - 23.3 - 10-2+ 30.25
or I, = -33.5 + 30.25 = -3-25A
It will be seen that the arbitrary signs have been introduced
here, because the reactive current of branch C acts in the
opposite direction to that of branches A and B.
The circuit current is now:
I = d m 2= J44.6'
3 . 2 ~ ~
= 10d4.46' + 0,325' = 10d19.8 +0.106
= 10JI!DC% = 446A

44.6

and cos 4 = -= 1.0 ie unity.


44.6
From the above example it is seen that, by connecting a
capacitor in parallel with the inductive loads, the total line

current is reduced from 55.6 to 44.6 amperes and the overall


circuit power factor is improved from 0.8 (lagging) to unity.
The resulting advantages of the arrangement are now considered
in detail.
ADVANTAGES OF P.F. IMPROVEMENT

For the majority of commercial loads, the current lags behind


the voltage, due to the inductance of the apparatus or the
operating characteristics of motors and control gear. Typical
values of power factor are:
System supplying lighting loads only: power factor (lagging)
= 0.95.
System supplying lighting and power loads: power factor
(litgglng) = 0 7 5 to 0.85.
System supplying power loads: power factor (lagging)
= 0.5 to 0.7.
The lower the p w e r factor, the greater the line c u r r e n l t x d
be__fpr-a9 given load k W or output power rating and the
attendant d & a d V a n t a g G E
(a) the transmission losses in the supply cables or power lines
are increased; these being given by PR, where R is the
cable or line resistance.
Thus for a given amount of power transmitted, the

----

current at 0.7 power factor is - x current at unity power


0.7
factor = 1.43 x current at unity power factor. Also the
transmissiorrloss at 0.7 power factor is (1.43)2 x loss at
unity power factor = 2 x loss at unity power factor.
(b) Because of the larger currents resulting from a low
power factor, there will be a greater voltage drop in thc
supply lines resulting in a lower voltage a t the load.
Conversely the size of the conductors must be increased to
keep the voltage drop figure to an acceptable value.
(c) Again because of the larger current resulting from a low
power factor, the size of the current-carrying conductors
in transformers, control-gear and alternators must be
larger than need be. This means that the physical dimensions of the equipment must be larger and that advantage
is not taken of good design. The equipment is.als.0 more
costly.
(d) The 'Regulation ,-a term used for the 'sittingdown' of
the voltage of generating and transmitting plant, is
adversely affected by a low power factor. The lower the
power factor, the greater the internal voltage drop in this
1

equipment ie armature reaction and attendant effects are


worscnctl.
Electricity supply authorities encourage good power factor
oprntion by offering ndvnntngcous tnrith, Power fi~ctorurn Bc
improved by adding to existing loads, apparatus which will take
sufficient leading current to minimise the lagging current of the
load. Static capacitors are frequently used for this purpose.
Example 89. A 40kW load, operating at 0.707power factor
(lagging), is supplied from 500V. 50Hz mains. Calculate (a) the
capacitor value required to raise the line power factor to unity
(b) the capacitance required to raise the power factor to 0-95
(lagging).
kW 40
(a) Load k V A = -cos d 0.707
46000
- 80
800 = 113.15A
Load current =
0.707 x 500 - 0.707 =
Active component of load current I , = I , cos 4,
= 113.15 x 0.707
= 79.997A = 80A
Reactive component of load current I , = I, sin 4,
= 113.15 x 0.707
= 80A.
To nullify this reactive current, a capacitor can be fitted to
operate in parallel with the load. This capaoitor must pass a
similar value of reactive current as shown by the phasor diagram
(Fig 142a).
Thus I, must be 80A. Reactance X, of capacitor must be
= 6.25n
80

Fig 142

258

REED'S BASIC E L E C T R O T E C H N O ~ Y

Since X ,

(b)

,
;
,

For this part of the problem it will be seen that I, sin 4,


is not to be cancelled completely since, the line phase
angle is only to be reduced from 4, to 4, and line current
to a ncw vnluc I,. This is illustriitcd by the' diagram (Fig
142b). Sincc thc power or active component remains the
same, then for this condition V I , cos 4, = 40 000 as
before.
Also s i n k cos 4, = 0.95 (lagging) sin
I , sin 4, = 84.2 x 0.312 = 26.1A.

4,

0.3 12 and

The line reactive current component has now to be reduced


from 80A to 26.1A = 53.9A. This then must be the new value
of I , or a capacitor must be used which takes a current of 53.9A.
Thus X

- --500 - lo6 (C being in microfarads)


- 53.9 - 2Tcfc
- 53.9 x lo2
lo6 x 53.9
orC =
500 x 2 x 3.14 x 50 - 5 x 3.14
- 539 lo' = 343pF.
15.7
It will be noted that 343pF will bring the line power factor to
0.95 and that a further (510 - 363) = 167pF would be needed
to bring the value to unity. Since the cost of a capacitor depends
on its capacitance value and little advantage is gained by improving the power factor above 0.95, it is not always necessary
to achieve unity power factor working. In this way some saving
to thc consumer can be clt'ected. I t is important to note that,
although power factor is improved, an increased power output
is 1101 obti~ined fro111 tllc lontl. Students frcqucntly hnve the
erroneous idea that, if for example, the power-factor working ol'
a circuit supplying a 5kW motor is improved, then the motor
will then give iln output greater than 5kW. This is not the case.
All that is achieved is that, by connecting an additional item of
. ; t ; ~ t ~ c;\pp;Ir'ntus
'
cross the motor. the total line current is
rcduccd, ic a conditiori tan be attained when the minimum
\ul)l)lv c.t~r.r.crllrc~lt~irctl
101.
; I sl~ccificd powcr output is uscd.

A.C.CIRCUITS(continued)

AND SYSTEMS

259

This minimum supply current reduces all the disadvantages


already enumerated hut the motor crlrrent itself remains
unaltered.

kW, k V A and kVAr


As mentioned earlier in the chapter, the above terlllinology is
much used in practical electrical engineering and some revision
is necessary, before the full treatment is considered. The diagram
(Fig 143a and b) should be considered with the text. For a circuit,
where the current and voltage are out of phase, the phasor
diagram is as shown. Current I can be resolved into an in-phase
o r active component I cos 4 and an out-of-phase o r reactive
component I sin 4. I cos 4 is responsible for all the power dissipated by the circuit, since P = VI cos 4, and is also called the
power o r wattful component, whilst I sin 4 is responsible for n o
power, being at right angles to the voltage, and is called the
C
wattless o r reactive component.

From the expression P = V I cos 4 it is seen tha.t P'can be


the 'active' component of VI (symbol S--see Note), and if
the term volt amperes or kilovolt amperes is used for VI then
the k V A (kilovolt amperes) can be regarded as being resolved
into two components, one of which is the power component.
The term W or k W (kilowatts) can be used to describe this
component and the other component termed the 'volt amperes
reactive' or 'reactive kilovolt amperes' can be designated by
V A r or k V A r . If the current phasors of Fig 143a are multiplied
by V , the new condition becomes more apparent and leads to a
power diagram. The product VI (S) is shown as the volt amperes
VI
( V A ) or -= k V A and is referred to as the 'apparent power'.
1000
Since VI cos 4 = P then V A cos 4 = W and k V A cos 4 =
k W . k W is a measure of the 'active power', in line with, original
definition for power factor. ie the ratio of active power to
apparent power.

Thus: power factor or cos

kW
4=-

kVA
s&ilarly VI sin 4 (Q-see Nore), or k V A sin 4 is the 'reactive
power' o r volt amperes reactive designated by k V A r and from
the power diagram (Fig 143b), we have:
Apparent Pow'er = .\/True power2 + Reactive power2
Summarising k W = k V A cos 4.
k VAr - = k V A sin 4
kVA = z / k W + k ~ ~ r '
kw
k VAr
cos 4 = - sin 6 = k VA
k VA
Note. The symbols S , P and Q are recommended as subptitutes
for V I , VI cos 4 and VI sin 4 but it is probable that the units :'
kilovolt amperes, kilow:ttts nnd kilovolt amperes reactive will
continue to be used and shown on the phasor diagrams, since
this is the older, though basically inconsistent, practice of the
electrical power engineer. The appropriate alternative has been
introduced and shown where it is considered to be appropriate in
this chapter.
For summarising we have:
P = VI cos 4
Q = VI sin 4
and
S = VI
Q = S sin 4
Thus S = d m ' and P = S cos 4
P
Q
cos 4 = - and sin 4 = -.
S
S
It must be r e m e h e r e d that the k V A values of various loads
are not in phase and therefore cannot be added arithmetically.
kW values are all active components, are in phase and can be
added. k V A r values are reactive components, they can be inpbase or in anti-phase and can be added, provided due allowance
is made for the sign. This is shown by the following examples.
-'

I
I

#'

Example 90. Two loads are connected in parallel. Load A 1s


800kVA at 0.6 (lagging). Load B is 700kVA at 0.8 power

A . C . C IR C U IT S

(continued)

A N D SYSTEMS

26 1

factor (lagging). Find the total k W, k V A and overall power


factor of the joint loads.
Scc t l ~ ctlic~yro~n
(Fig 144),
For load A. cos 4, = 0.6 sin 4, = 0.8
Active power, PA = VIAcos 4,
800 x 0.6 480kW
Reactive power, QA = VIAsin 4, = 800 x 0.8 = 640kV Ar
For load B. cos 4 , = 0.8 sin 4, = 0.6
Active power, PB'= VIBcos 4B= 700 x 0.8 = 560kW
Reactive power, QB = VIBsin 4, = 700 x 0.6 = 420kV Ar
Total active power, P = 480 + 560 = 1040kW
Total reactive power, Q = 640 + 420 = 1060kVAr
7
.Total apparent powq, S =
1060
= lo3 J1@12
1.06~
= 1 0 3 m 5 = lo3 x 1.485
= 1485kVA
*
1040
Overall power factor = -- = 0.7 (lagging).
I t85
Problems involving a number of loads, may be best treated by
setting out the power-diagram components in tabular fashion as
shown. An arrow has been drawn in to illustrate the phasor
direction and to remind the student as to which columns can be
added arithmetically.

Example 91. A 220V, single-ghusc nltcrnator supplica thc


following loads :
(a) 20kW at unity power factor for lighting and heating.
(b) A 75kW induction motor having an efficiency of 90.5 per
cent operating at a power factor of 0.8 (lagging).
(c) A synchronous motor taking 50kV A at a power factor of
0.5 (leading).
Find the total k V A , current and the power factor of the
combined load.
Load (a) can be set into the columns directly as shown.

Load (h) Motor power output = rating as given = 75kW


75 = 82-9kW
Motor input active power = 0.905
82.9
Apparent power = - = 103.6kVA
0.8
Load (c) can also be set into the columns directly.

262

R EE D

Lord

h IfA 1 h lJA cos


(SIX
(P)

1 0 3 . 6 ~ ~ 82.9 --

a
I

B A S I C ELECTROTECHNOLOGY

k V ~ r o r 11

kWor-j

20

20-+

k V A sin

(Q,

cos

-+

127.9

1-

11

62.16 1 ,

43 3
-

sln

I$
1

1
-4

0.8

0.6

0.5

0,866

25

o
-

I
I

--

- 18.86 i

Total apparent power (S) = 2/127.9'+

18.9' = 129kV A

Total current = 129

loo' = 588A
220
127.9
Resultant power factor = ---- = 0.99 (lagging).
129
Note reactive component of inductive load predominates, hence
the resultant lagging power-factor condition.

(k V A method).
Treatment of problems follows lines, similar to those set Out
for the 'current method'. The diagram for the load condition is
built up by splif?ing the original load k V A into its k W and k V A r
components. Since the k W remains the same, then for a new
power-factor condition for the supply, the final k V A r value is
obtained by reducing the original k V A r by an amount equal to
the XC'Ar of the apparatus being added. Such apparatus must
use no power and the static capacitor is such an item of equipment. The added k V A r being leading, will reduce the lagging
X 1'.4r of the supplv. I t should be noted that if a synchronous
rnotc)r. 1s used to obtain a better overall power factor, then this
also contributes output power which must be taken into account.
. I ' I I I ~W ; I \ I I I U S ~ I ~ ; I I C ( I h y 1 1 1 ~prcvioi~sC X ; I I I I ~ ~ C .
Example 92. A 400V, SOHz, 20kW, single-phase induction
motor has a full-load efficiency of 91.15 per cent and operates at
;I powcl- factor of 0.87 (lagging). Find the k V A r value of the
capacitor to be connected in parallel to improve the circuit
pcnver klctor to 0.95 (I;~gpinp).Find also the capaciti~ncevalue
01' tllis c;~p;~citor.
Thc di;~gram(Fig 145), illustrates the problem
;111cI ~ o ~ l l t l o l l .

P O W E R - F A C T O R IMPROVEMKNT

A.C. CIRCUITS (conrinltcd) AND SYSTEMS

2 63

Fig 145
Motor output = motor rating as given = 20kW
20
Motor active input power P , = ------ = 21.94kW
0.9115
21'94 ---- 25.22kVA
0.87
Also, since cos 4, = 0.87 then, from tables, sin 4, = 0.493
Thus Q, or S, sin 4, = 25.22 x 0.493 = 12.44kVAr
Although the power factor of the circuit is to be improved to
cos 42,the power of the circuit is not altered :. P, = P,
or S , cos 4, = S , cos 4, whcnce
S, cos
.,
S, =
cos b,
Motor apparent power S ,

,
I

Again cos $, = 0.95 therefore, from tables, sin 4, = 0.3123


and Q, = 23.1 x 0.3123 = 7.21kVAr
Required Q value = 12.44 - 7.21 = 5.23kVAr. This therefore must be the rating- of the capacitor.
-5230
Capacitor current I, = -= 13.75A
400
Capacitor reactance

Ii

400 - 30.59i2
X, = -

or X,

13.75

30.59 =

lo6
2 x 3.14 x 50 x C

where C is the value in microfarads.


lo4
lo3 - 104pF.
Hence C =
30.59 x 3.14 - 961 -

POLYPHASE WORKING
The student who intends to have a good practical knowledge
of electrotechnology, must make himself thoroughly conversant
with the terms, relationships and theory of polyphase working.
The importance of the work now to be covered cannot be too
strongly stressed. ~ x p e r i e n c ehas shown that most students
consider this part of theory to be 'that little extra, which breaks
the camel's back' and accordingly give it insufficient attention a t
first. The result is that much hasty revision is necessary when
the v;~riouso.c. m;ichines iire to be studied later. Detailed attention to fundamentals will bring long-term advantages and,
although the next book will be devoted to more advanced a.c.
[ c c l ~ ~ ~ o l o ~g lyi,csul~jcclIlliltIcr now to bc considcrcd m u s ~bc
treated as bas~cand essential to such further studies. I t must be
both understood and rnemorised.
THREE-PHASE SYSTEMS

Universal practice has established 3-phase systems to be the


most advantageous for polyphase working. A single-phase
supply, as is usual for small installations, can always be obtained

A.C. C IR C U IT S
.

(continued)
-

AN D SYSTEMS
- - -- -

265

from a 3-phase system and in this way the relative advantage of


either system is available. The 2-phase system is only rarely used
und its unimportc~nccdocs not wi~rritnlits study u l this stuyc.
More than 3-phase arrangements, such as 6-phase, have relatively fewer and even more specinlised applications and here we
confine our investigation to 3-phase working only.
Consider a 2-pole magnet, as shown in the diagram (Fig 146).
to be rotated inside an external stationary armature or stator.
Three coils are shown equally displaced, with 'starts' and
'finishes' marked symmetrically to make .a regi1ar arrangement.
Induced e.m.fs. will result in each coil, which are identical in
magnitude but displaced in phase by 120 electrical degrees. A
phase sequence R-Y-B is assumed ie the rotor turns so that the
red-phase voltage reaches its maximum 120' before the yellowphase voltage reaches its maximum and the latter 120" before
the blue-phase voltage, as shown by the waveform diagram. A
sinusoidal distribution of rotor flux is assumed and that sinewave e.m.f.s are induced. The methods by which this is achieved
will be considered under the detailed study of the alternator.
The three separate coils can be used to supply three independent
single-phase loads, but advantages are obtained by interconnecting the coils or phase windings and the two important
methods are described as either the STAR or DELTA connection.
STAR OF Y CONNECTION

The diagram (Fig 147). shows the arrangement and it will be


seen that here, the three coils or phase windings are connected
so that either all the starts or,finishes are joined together to form
the star-point ie 'corresponding ends' are connected together.
Similarly the supply lines are connected to the free ends, remote
from the star-point.

Fig I47
The phasor diagram has been drawn in terms of voltage with
the red-phase voltage (V,) used as the reference. The notation
being used from now on should be observed. It will be seen that
the small letter suffix denotes the phase value, while the capital

letter denotes the line value. The lines have been identified withthe colours of the phases to whose 'starts' they have been connected. The double suffix such as V , - , denotes the voltage
between lines, the example being the Red to Yellow line voltage.
Assume the condition when the red-phase voltage wave is
positive and that the 'start' of the red-phase winding is +ve with
respect to the 'finish' or neutral point. Current will flow through
the lines and load as shown. For the example and condition
being considered this is possible because, for the yellow phase at
the same instant, its start will be - ve with respect to its finish,
since the yellow phase wnveform is in its -ve half-cycle.
Thus for the phasor diagram, the voltage between the red and
yellow lincs is,obtained by the phasor difference of Vr and V , .
Since a phasor din'erence is being considered, thc resultant I S
obtained by reversing one phasor with respect to the other and
completing the parallelogram. From the deduction set out
below, it will be seen that the line voltage is d 3 times a phase
voltage. This relation also holds for the other lines and the
associated phases. A further point of importance for the star
connection, is that the line current equals the phase current or

IL =

Ipp

Consider the phasor diagram. Let the line voltage V R - , = 2x


x
d3
But - = cos 30" :. x = --- Vr or 2x = 43 V ,
vr
2
Hence V R - , = .\/3Vr
or the voltage between lines = .\/3 x a phase voltage.
Thus V , = *d3 x V,,.
For a star connection the following must therefore be remembered.
Line voltage = .\/3 Phase voltage
or V = V' 3 V,, = 1.732V,,
Line current = Phase current or I = I,,
It will be noted that the subscript L , as in VLand IL, is omitted
when generalising. This is usual and both V and I can be
assumed to be line values. Ag;r~nthe relations deduced have been
derived for an alternator or source of supply but they also
rcl;itc to a stiir-conncctcd load a s the cxnmplc shows.
Example 93. Three 50R resistors are connected in star across
41 5V, 3-phase mains. Calculate the line and phase currents and
the power taken from the supply.
Since the load is balanced. the voltage across each resistance is
tllc correct phase volt;ipc. 'Thus V,,

415 - 240V
v'3

A.C. CIRCUITS

(continued) AND SYSTEMS

267

Phase current = line current or I,, = 1

?4!!
50

4,HA

Power dissipcited by one phusc of load


=

Iph1 Rph

= 4.82 x 50 = 2.4 x 4.8 x 100 = 24 x 48


= 1152W or 1.152kW

and 3-phase power from the supply


= 3 x 1.152 = 3.456kW = 3.5kW.
USE OF THE NEUTRAL

One obvious use of the star-connection is for distribution,


since two voltages are available to the consumer, one for
lighting and the other for power. Either I-phase or Iphase
loading is also possible and this is shown by the diagram (Fig
148).
c

Ibuu~

Fig 148
BALANCED LOAD

A 3-phase load is said to be balanced when the currents in all


three phases are equal and their phase angles are the same. If an
instant in time is considered, as shown, on the diagram (Fig 149).

Fig 149

it will be seen that the sum of instantaneous values of the


currents i, i,
i, = 0. Since these currents meet at the load
neutral point an'd the resultant flows tHrough the neutral line,
then the neutral carries no current and need not be used for
balanced loading,

+ +

UNBALANCED LOAD

A neutral must be used if the load phase currents are unequal

or if their phase angles are different. The neutral line will carry
the unbalanced current ie the resultant of the three line currents.
Since this ncutral current is a phasor sum, it can be obtained
graphically o r mathematically, as shown by the example.
Example 94. Thc loads of n 4:wirc. 3-phase system are:
Red line to neutral current = 50A, power factor of 0.707
(lagging).
Yellow line to neutral current = 40A, power factor = 0.866
(lagging)
Blue line to neutral current = MA, power factor = 0.707
(leading)
Determine the value of the current in the neutral wire. The
solution is worked with reference to the diagram (Fig 150).
I, = 50A lagging the voltage by 45" since cos 45 = 0.707
Iry = 40A lagging the voltage by 30" since cos 30 = 0.866
I, = 40A leading the voltage by 45" since cos 45 = 0.707
Resolving into horizontal and vertical components.
I,, = (50 x cos 45) - (40 x cos 30) - (40 x cos 15)
= (50 x 0.707) - (40 x 0.866) - (40 x 0.966)
= 35.35 - 34.64 - 38.64 = -37.93A
I, = - (50 x sin 45) - (40 x sin 30) + (40 x sin 15)
= - (50 x 0.707) - (40 x 0.5) + (40 x 0.259)
- - 35.35 - 20 + 10.36 = - 44.99

NOT TO SCALE

Fig I50

Current in the neutral is the resultant

cos 8 = - 37'93 = - 0.65'3


58.9
0 = 49.5".
Note. The - ve sign gives the quadrant in which I, lies.
This is shown on the diagram.
A (MESH) CONNECTION
The arrangement is shown in the diagram (Fig 151). For this
connection, the three-phase windings are arranged to form a
closed circuit by connecting 'uncorresponding ends' ie the start
of one phase to the ,finish of another phase. Thus R start is
connected to B finish, Y start to R finish, etc. T h e same reasoning as was introduced for Fig 148, is applied here, except that
the voltages are considered. Thus if the diagram represented the
three equal phase voltages, then it would be seen that, for any
instant of time, the sum of the instantaneous values of voltages
v, + v, + v, = 0. For the mesh or closed winding, since the
sum of the instantaneous voltages is zero, no circulating current
flows round the mesh. The lines are taken from the junction
points and for this connection, i t is obvious that the voltage
developed across a phase is the voltage provided for the conncctcd lines.
Thus V , or V = V,,
It will be noted that the lines have been identified with the
colours of the phases to whose starts they have been connected.
Assume the condition when the red-phase voltage is positive ie
DELTA OR

Fig 151
that the start of the red-phase winding ; + ve with respect to the
finish. Current will flow through fron R phase into R line as
shown. At this same instant the voltagc in B phase is negative ir
its finish is +ve with respect to its star . Thus it is also correctly
connected for feeding current into the i line and a line current

is thus obtained by considering the phasor d~ferenceof two


phase currents. The resultant line current is obtained by reversing
a phase current (I,) and combining it with I , as shown in the
diagram (Fig 151 ).
As before x = I , cos 30' = I,, cos 30"
or x = 1 . 3 Iph and as I, = 2 x = 2 t/31,h
2
Hence I, or I = t/ 31ph
Thus for a delta connection, Line voltage = Phase voltage
and Line current = 1 3 Phase current.
This rci;~rionshipcan bc deduced for any line and the connected phases and will give the same result. As before V and I
;IIT wed for line v;rl\lcs and I,'
and I,, for phase values.
L

Example 95. Three 5 0 0 resistors are connected in delta across


41 5V, 3-phase lines. Calculate the line and phase currents and
the power taken from the mains.
Voltage across I-phase resistor = 415V
41 5
Current in one phase of load = - = 8.3A
50
Since the load is balanced, line current = 11 3 x 8.3 amperes
= 1.732 x 8.3
= 14.78A
Power in one phase of load = 8.32 x 50 = 3445W
Power in three phases of load = 3 x 3,445 = 10.3kW.
&

THREE- PHASE P O W E R

For a Star-connected load. V = t'3Vph and I = I,,


The power expended in one phase = V p hI,, cos
and the power expended in three phases
= 3 v,, I,, cos 4
('onvert~ngto line values. the above becomes:

4
3 V I cos 4

Three-phase power = 3 --- I cos


\

or P = \
Dc>lt(l-conncctcdlo;~d1,' =. V,, and I = t'31,h
'l'he power expended in one phase = I.', I,, cos 4
and the power expended in three phases
= 3 v,, I,, cos 4
Converting to line values the above becomes:
I
Thrcc-ptlasc 1x)wcr = 3 1,' - - cos 4
b 3
Fclr

;I

(continued) AND SYSTEMS

A.C. CIRCUITS

---

-- -

--

- -

2,

Thus the general expression holds, irrespective of the type of


connection. nilmely for either ;I st;lr or delt;l connection
' I I I I C C . ~ ) ~ I ~ I ~ C~ ) t ) w o t i~
* ~ I V C I Ijy
I
\ -1 I''/ L ' L I ~(1
Example 96. A 75kW, 4MV, 3-phi\se, deltil-connected
induction lilotor has u I'ull-load ellic!ency ol' 91 per cent : ~ n d
operates at a power factor of 0.9 (lagging). Calculate the line
and phase currents at full load.
Output power = 75 x lo3 watts
75
103 100 wafts
Input power =
91
also P = 4 3 V I c o s 4
So 4 3 x 400 x I x 0.9 = 75 x lo5
91
75
lo4
750 lo'
amperes
and I =
1 . 7 3 2 ~ 4 ~ 9 ~ 19 . 17 3 2 ~ 3 6 ~ 9 1
*

or I = 7500
lo' = 132.2A
5674
132.2
Motor phase current = ---- = 76.3A.
v'3

k V A , k W AND k VAr
Since power factor can be defined as the ratio of true power to
apparent power, this can be applied to three-phase workin g .
Thus :
active
power
power factor = ---nppnrcnt p O W s r
Again from earlier deductions, we see that irrespective of star
or delta connection P = 4 3 V I cos d. Accordingly:
THREE-PHASE

cos

4 3 VI
It follows that for three-phase working, in order that the
definition for power factor should apply,
apparent power ( S ) = 4 3 VI
Note the introduction of d3,--distinguishing this condition
from single-phase working.
P or k W and, in exactly the
Again it is known that cos 4 = S
kVA
' vr
3
same way, it follows that; Three-phase k VA = ~loo0
This final deduction is most important.
Example 97. A 3-phase, 400V motor takes a current of 163A
when the output is 9kW. Calculate (a) the k V A input, ( b ) rhc
power factor, if the efficiency at this load is 89 per cent.

3 VI = 1 3 x 400 x 16.5
loo0
loo0
= 6.6 x 1,732 = 1 l.43kV A
( b ) Output power = 9kW
9.0
True active input power = -= 10.11kW
0.89
active power - 10.1 1
So power factor =
apparent power
11.43
= 0.88 (lagging).
Esnmple 97a. A 3-phase, star-connected alternator supplies a
delta-connected induction motor at 600V. The current taken is
40A. Find ( a ) the phase voltage of the alternator (b) the current
i n c i ~ c hp h i l ~ cof thc motor. Refer to the diagram (Fig 152).
( a ) k V A input

--

Fig 152

--

( a ) For a star connection V =


V,,
600
.'. V,, = v = --= 346V. This is the alternator phase
v3
\ 3
voltale
I -(b) For a delta connection I,, = - 40 = 23.1A
4 3
v'3
This is the current in each motor phase.
(c) I f the motor operates at a power factor of 0-8 (lagging)
and an efficiency of 88 per cent, find the k V A rating of the
t the motor.
alternator and nower o u t ~ u of

Motor ilpparent input power

~ ~ V l' 3
loo0
- 1.732 x 600 x 40
1000
= 41.6kV A
= ----

True active input power


Output power

41.6 x 0.8 = 33.28kW


33.28 x 88 - = 2 9 , 2 1 k W
IotK)

Example 98. A delta-connected load is shown by the diagram


(Fig 153). 11'thc supply volti~peis 40OV, 50111. ~ i ~ l ~ tthe
~ lredi ~ t
line current.
Assume the currents as shown and maintain the correct phase
sequence of R, Y and B. If the red-line current is assumed to
feed current into the red phase of the load, the blue-phase
current would be in the opposite sense or a phasor difference is
involved.
" b ~
b.

Fig 153

#YB

400
Here I, = - = 4A in phase with V,,
100
400
Also I, = - = 3.7A leading V,, by 90"
106
Reversing I, and using the modified Cosine Rule
21Jb CoS 30'
= dl,*

CHAPTER 11
PRACTICE EXAMPLES
1.

A coil consumes 300W when connected into a d.c. circuit


of 60V. It consumes 1200W when connected into an a.c.
circuit of 130V. What is the reactance of the coil?

2.

A circuit consists of two branches A and. B in parallel.


Branch A has n resistance of I 2 8 and a reactance of 3R,
whilst the values of branch B are 8R and 20R respectively.
The circuit is supplied at 100V. Calculate the current in each
hrnnch ;tnd t l supply
~
current.

3.

An inductive circuit of resistance SOQ and inductance


0.02H is connected in parallel with a capacitor of value
25pF across a 200V, 50Hz supply. Find the total current
taken from the supply and its phase angle.

4.

Two coils of resistances 8R and 10R and inductances


0.02H and O.05H respectively, are connected in parallel
across IOOV, 50Hz mains. A capacitor of capacitance value
80pF in series with a resistor of 20f2 is then connected in
parallel with the coils. Find the total current taken from the
mains and its phase angle with respect to the applied voltage.

5.

A single-phase motor has an input of 50.6A at 240V, the


power input being IOkW, and the output 9kW. Calculate the
value of the apparent power, power factor and of the
efficiency.

6.

A slngle-phase motor running from a 230V, 50Hz supply


takes a current of 11.6A when giving an output of 1.5kW,
the eflicicricy k i n g 80 per cent. Calculate the capacitance
required to bring the power factor of the supply current to
0.95 (1;1gging) C;llctrlntc ;~lso thc X-l'Ar rating of tllc
capacitor.

7.

The load taken from a slngle-phase supply eonsists of:


(a) Filament lamp load of lOkW at unity power factor.
( b ) Motor Io;~dof' 80kV ,A a t 0.8 power factor (lagging).
(c) hlotor load of 4OkV A at 0.7 power factor (leading).

a.

A . C . C IR C U IT S

(continued)

AND SYSTEMS

275

Calculate the total load taken from the supply in k W and


V A a n d the power factor of the comh~nedload Find the
'111111113' CUlI'CI11 I f 11\12 YU})I)!Y V O I L L ~ ~ 15
C 250V
In

8.
,-,bf

Three equal impedances of 10SI, ench with u phusc i~nplc


30" (lagging), constitute a load on a 3-phase alternator,
giving lOOV per phase. Find the current per line and the
total power when connected as shown. (a) Alternator in star,
load in star. (b) Alternator in star, load in delta. (c) Alternator in delta, load in delta. (d) Alternator in delta, load in
star.

9.

A 500V,3-phase, star-connected alternator supplies a starconnected induction motor which develops 45kW. The
efficiency of the motor is 88 per cent and the power factor is
0.9 (lagging). The efficiency of the alternator at this load is
80 per cent. Determine (a) the line current, (b) the power
output of the alternator, (c) the output power of the primemover.

10.

' A 400V, 3-phase system takes 40A at a power factor of


0.8 (lagging). An over-excited synchronous motor is connected to raise the power factor of the combination to unity.
If the mechanical output of the motor is 12kW and the
efficiency is 91 per cent, find the k VA input to the motor and
its power factor. Find also the total power taken from the
supply mains.

CHAPTER 12

ELECTROMAGNETISM (continued)
Earlier treatment of the electromagnetic circuit has introduced
fundamental relationships between magnetomotive force-F,
F l u x 4 and Reluctance-S. It will also be remembered that,
the m.m.f. per metre length is called the magnetising force,
magnetic field strength or intensity-H, whilst B (in teslas) is the
flux density.
F
For revision, we write Q, = -, and F = H x I where I is the
3
0
1
lc~igtllof tlic magnetic circuit. Also B = - and S = - where p
A
DA
is the absolute permeability and A is the area of the mcgnetic
circuit
PERMEABILITY OF FREE SP A CE ( p , ) . Earlier

work defined the term


permeability as the ratio of flux density in a medium to the
magnetising force producing it. Fur free space, a vacuum, air or
any other non-magnetic material, a value for the permeability
can be deduced as 4x x lo-' in the appr.opriate SI units or
p, = 4x x
microhenrys per metre (pH/m). Also since
p, = B : H then H =

4x x 10-

, amperes o r ampere-turns

per
metre.

Thus the amperesjmetre o r ampere-turnslmetre for air


flux density
permeability of free space'
From the foregbing i t can also be deduced that BIH = a constant or B ac H.
If therefore for air, values of B are plotted to a base of H , a
graph, as-shown in the diagram (Fig 154), would be obtained.
This would obviously be a straight line and if measurements of
flux density B were made, at a point outside, but near to, a long
straight current-carrying conductor, for various values of magnetising force H , by varying the current; i t being remembered that
I

H = -- where r 1s the radius from the point to the centre of the


2xr
c . o ~ l d ~ ~ i . l then
o r . tllc s l r ; ~ ~ g l ~ ~ - 11/11
l i r l c rcli~lior~ship
would bc

confirmed.
I . MAGNET!SIKG FORCE DIJE TO A LONG, STRAIGHT CURRENT('AHHYING CONDUCTOR
As mentioned above, the magnetising force outside, but

;~d~;iccnt
to. ;I current-cilr~.yingconductor, is given by /-I =

I
2xr

-- .

Fig 154

This expression was deduced earlier (in Chapter 5) and should


be revised. It should also be remembered that, H is the m.m.f.1
metre length. Magnetomotive force-F, is measured in ampereturns and thus the total m.m.f. for any magnetic circuit outside
the conductor can be found from F = HI.

2. MAGNETISING FORCE INSIDE SOLENOID


If a parallel field of flux lines is assumed inside a solenoid as
illustrated by the diagram (Fig 155), then its length can be taken
as I metres, the number ot'turns on the coil as N nnd tlic c u r r c ~ i l
passed as I amperes. The lines of flux are known to fan out at the
ends and for their return path they spread out into space. This
external return path has negligibly small magnetic reluctance
and the whole m.m.f. of the coil can be regarded as being utilised
in setting up the field inside the solenoid. Thus the m.m.f. per
unit length is, by definition, H-the magnetising force
F
IN
Thus H = - = - ampere-turns per metre.
I
1

I--AMPERES

1- METRE $
N TURNS

Fig 155

I
I
l
I
I

3. MAGNETISING FORCE INSIDE A TOROID


The diagram (Fig 156) shows this simple electromagnetic
arrangement. f t consists of a solenoid bent back upon itself so
that the lines of flux are confined to the inside of the coil. We
can now consider a non-magnetic ring to be wound uniformly
with a coil of N turns, carrylng a current of I amperes. The mean
circumference is 1 metres and since the flux is confined to the
inside and the path is uniform, the magnetising force or m.m.f.
per unit length is given by:
IN
H = - ampere-turns per metre.

Fig 156

Example 99. A wooden ring having a mean circumference of'


300mm and a uniform cross-sectional area of 400mm2, is wound
uniformly overau with 300 turns of insulated wire. If the current
is 3A, calculate (a) the magnetising force (b) the flux density
inside the toroid and (c) the total flux produced.
(a) The total m.m.f. produced = 3 x 300 = 900At
The mean circumference is 300mm = 0.3 metres
F 900
.: The magnetising force H = - = 1
0.3
= 3000Atlm
(h)Thc flus density is given by B = p, H
= 4 ~ lo-'
3000
12.56 x lo-' x 3 x 103
1.256 x 3 x 10-3
= 3.768 x
=
=

(c) The total flux produced

0.003 77T

= B x A
= 0.003 77 x 400 x
=
01. (D =

1.5 x
1.5uWb

webers

FERROM AGNETISM
I t is known that, when iron is used as tlie core of an electrom;lpnet. thc ficld is intensified or that a very much greater flux
results from tllc ~n;ignctising~ I I I ~ ~ ~ ~ C
01' -the
I Ucncryisiny
I . I I ~ coil.
must bedue to the
S'
reluctance S and if the dimensions oP the core I and A are kept
the same as for the air path, it follows that the permeability of
the iron must be very much greater than that of air. Thus we
can now make reference to the permeability of a magnetic
material which is termed relative peJmeabiliry.

Since the only ckilnye in the reliition

d,

(p,) This is the ratio of the flux densit)


which is produced in a magnetic material to tlie flux density
which would be produced in air by the same magnetmotive
force. For materials such as iron, nickel, cobalt, etc this value of
pr can be very large, ranging from 1000 to 200d or even more for
some special electrical steels. It can be quoted as the permeability
figure for the material but is not constant and varies with the
flux-density value at which the material is being worked. It is of
interest to mention that materials such as bismuth have a relative
permeability value of less than 1.
RE L A T I V E PERM E A BILIT Y

THE B-H OR MAGNETISATION CURVE


If a specimen of magnetic material is made up in the form of a
ring and is wound with an energising coil, measurements of flux
density for various values of magnctising rorcc can hc m:~tlchy
winding on a secondary coil and using the principle of transformer action. I t is not proposed at this stage to discuss the
manner in which the tests are made but it can be stated that this
is an accepted industrial method for determining the magnetic
properties of various materials.
It has already been seen thht; if the flux density B is plotted
against the magnetising force H for air, a straight line is obtained,
but for magnetic materials, curves as shown in the diagram (Fig
157) will result.
It will be noticed that, at first, the graphs.are approximately
straight lines, showing B to be proportional to H. Then the
curves begin to turn over forming a 'knee' and finally they
become horizontal and exhibit'little increase in B for a large
increase in H. In this state, the material is said to 'saturate.'an explanation for this has already been given when dealing with
the molecular theory of magnetism.
If the permeability (p,) is plotted to a base of B, curves as

MACNITISINC

FORC t . H (A S / m )

Fig 157
shown in the diagram (Fig 158) would result. The permeability
curve has a peak corresponding to the point on the B-H curve
where the tangent goes through the origin. Beyond this peak the
permeability value drops off fairly rapidly.
An examination of the B-H and p,-B curves shows how the
properties o f various magnetic materials differ. The effect on
machine design can also easily be seen,-lower working B values
necessitate larger section and greater mass to obtain a required
flux \,slue. The effect of high permeability materials is also

;lpparent and the shape of the B-H curve with the saturation
cll'cc.1, ~ I I O W lllc
~ I I I I I ~ ~ ~ Ior~ I IIIIIC'IIIIIC
~ I ~
ficI(I ' I ~ % ~ C I I I R
The magnetic properties are dependent upon the artu.CII COIIIpositiou ol' tllc subsrancc. T l l ~ s~n:~ng:~ncsc-steel i a p17i\cti~i\ll\
non-magnetic, but small quantities of carbon or silicon wlicn
added to steel vary the shape of the B-H curve and sheets of
commercial steel marketed under trade names, like Stalloy or
Lohys, are available to suit different design requirements.
( p ) .Since permeability is the ratio of the
flux density in a medium to the magnetising force producing it.
and for air it is known that B = p a , it follows, for a material
with a relative permeability of p,, that B = prp,H o r B = p H
where p = prpo.
Summarising, we can define absolute permeability as the ratio
of flux density in a substance (in teslas) to the magnetising force
(in ampere-turns per metre) which produces that flux density.
AB S O L U T E PERME A BI L IT Y

I
i

I
I

I
I
I
I
1

S). This term has been mentioned earlier.


It has been likened to the resistance of an electrical circuit. Since
flux is proportional to the m.m.f. and is restricted by the reluctance, further investigation will show that reluctance in turn is
proportional to the length 1of the magnetic circuit and is inversely
proportional to its area A . Furthermore it must be inversely
proportional to the permeability. since the greater the permeability of the material the greater the flux and hence the sm;lllcr

R EL U C T A N C E (SYMBOL

the reluctance. We can therefore write, S =

and point out


uA
hcrc tllac ;~hsolutcpcrmci~hilityis hcillg uscri. i~ being I . C I I I ~ I I ~ bered that p = p,po.
Calculations on magnetic circuits using magnetic materi:~ls
are now possible, but it should be noted that, unlike electrical
V
circuit calculations which use I = -, it is not always necessary
---

R
F
to use the comparable relationship of @ = - The solution of
S
most problems associated with the magnetic circuit can be made
without always determining the reluctance, and experience will
show the best method of solution. The following examples indicate the alternative way of treating typical simple problems.
Example 100. A solenoid is made up from a coil of 2000 turns.
carries a current of 0.25A a n d is Im long. An iron rod of
diameter 20mm. forms the core for the solenoid and is also 1.0m
long. Calculate the total flux produced if the Iron has :I permeability of 1000. Note. Here relative permeability is implied.
'

M . M . F . of coil is given by F = HI =

I = IN

xd2
3.14 x 400 x
Area of iron = - =
4
4
= 3.14 x
square metre
1
I
Reluctance of iron = --- = --~ LA

Flux 0 = - =
S

'0

PrPJ

;'944 = 5 x 3.944 x l o - ' weber


10

ALTERNATIVE SOLUTION

M . M . F . of coil. F = HI so magnetising force H = F


Also B =

j1H = ,LI,~,H
= 1000 x 4 x rt x lo-' x 500
=4
x 5
lo-' x 10' = 20 x
x
= 0i628T
Total @ = BA

0.628 x 3.14 x

webers
197.2pWb.
Example 101. A cast-steel ring has a cross-section of 400mm2
and a mean diameter of 240mm. I t is wound with a coil having
200 turns. What current is required to produce a flux of400pWb,
i f the permeability of the steel is 1000?
: \ I . ~ : I of'sttcl = 400 :, IO\squarc lnctrc
=

= 1.972 x

I
104
-ampere-turns per
104
4 x x
metre
I o4 x x x 240 x 10- 3
R1 hl.1- 01' rlrlg = 111 = - -4 x x

so H

--

- -

EL EC T RO MAG NET IS M

283

(continued)

600

If the coil turns are 200, then the current would be -= 3A.
200
ALTERNATIVE SOLUTlON

Iicluctancc of ring S =
or S =

-- ~rjlLV4
-

n x 240 x
lo3 x 4~ x lo-' x 400 x

6
4 x 10-~
= 1.5 x lo6 ampere-turnslweber
Since 0 = 400pWb = 4 x
weber
then required m.m.f. = 5'0= 1.5 x lo6 x 4 x
= 6 x lo2 = 600At
-

600

Required current = - = 3A.

200

THE COMPOSITE MAGWETIC CIRCUIT


THE SERIES ARRANGEMENT

Consider a magnetic circuit built up as shown in the diagram


(Fig 159). It is obvious that the sections are in series and that the
same flux passes through them.

Fig 159
Then total m.m.f. = m.m.f. across section 1 + m.m.f. across
section 2. If the flux is Q,
then @S = OS! -- OS,
where S is reluctance of the composite circuit. Thus:
S = S , + S,.
Summarising: Total reluctance = the sum of the individual
reluctances of the sections for a ser: : lrrangement.

THE PARALLEL ARRANGEMENT

Such a magnetic circuit is not frequently encountered but is


considered here, being complementary to the series circuit. The
arrangement is shown In the diagram (Fig 160).

Fig 160
If the different paths of the magnetic circuit are in parallel.
then the necessary m.m.f. is that which will produce the required
flux in each part of the circuit considered by itself. Let F = the
rn.1n.f. required to produce fluxes @,, @, @, etc. F also produces total flux @.
.-.
f14
-So @ =
Total Reluctance of circuit
S
and since @ = 0 ;+ cP2

.,@

=FA
Sl

+ 0,-

+F,+F,

S2
S3
But F,, F,, F, are the m.rn.f. across the same points of the
magnetic circuit and are equal to F.
. .

I .

1s referred to as the perrneance of a magnetic circuit and the

above can bc s u ~ i l ~ n a ~ . ~bys esaying


d
tllal; the reluctance ol' a

divided magnetic circuit (sections in parallel), can be found by


knowing that its permeance is equal to the sum of the permeances of the individual circuits.
Example 102. A n iron ring has a mean diameter of 200mm
, ~ n d;I C ' I ~ L ~ S ~ - S C C o~ I O' ~I O~ O l i l ~ ~ i.A11
' . a i l . gap of0.4rnm is made by
; I r:~tli;~l
.;;lu.-c.trl
;1cr-oss(lie r i n g . A s c ~ ~ m i n;I gpcnnc;~hilitvof 3000

(ronfinue~d)

E L E C TR O M A G N ET I S M

285

for the iron. find the current rewired to ~ r o d u c ea tlus of


250pWh, if tile cncrgising coil is wbund with'^^^) turns.
(I(
Z(K) n 10 ' )
( 0 4 * I 0 ')
Keluctance ol' Iron S
= 3000 x 4 x n x 1
0
m
fi

'

62.76
36 x rr .x
= 555.2 x lo3 ampere-turns/wekr
0.4
104
Reluctance of air gap S , =
4 x
x 1 0 - ' x 3 x 10-4

= 1061.5 x lo3 ampere-turnslweber


S, + SA = lo3 (555.2 + 1061.5)
lo3 (1616.7) ampere-furns/weber
Total m.m.f. = @S = 2.5 k l o p 4 x 161.67 x lo4
= 2.5 x 161.67 = 404.15At
404.19 = 0.674A.
Current = 600

Total Reluctance S

=
=

ALTERNATIVE SOLUTION

2.5 x
3 x 10-~
= 0.833T
Now H for air is given by:
0,833
- 0,833 x 10'
0.833
HA= -u,
4
lo-''
4 x n

Since 0 = 250 x

weber

then B =

Length of air gap = 0.4 x l o e 3 metre


Ampere-turns for air = 66 32 x 104 x 4 x
- 265.28At
H for iron is given by:
0.833
0.833 - - 66.32 x lo4
H I= py,
4 x
x 1 0 - ~x 3 x loT
a x lo3
-663'2
- - 221.066At/rn
3
Now length of iron path = (628 - 0.4)
= 627.6 x
metre
.Ampere-turns for iron 5 221.066 x 0.6276 = 138.74At
Total ampere-turns = 265.28 138.74 = 404.02
404.02 = 0.673A
Current = ------

600

I
I
1

For the three previous examples, alternative solutions have


been given in which the reluctances, as such, for the various
sections of the magnetic circuit being considered, have not been
found. This alternative' method of solution is especially of value
when the relevant B and H data for the magnetic material is
given in tabular or graphical form. The relative permeability is
not given as a specific value and would have to be found before
the reluctance could be calculated. Obviously any solution along
these lines would be tedious and the following example is
recommended to the reader as a n instruction on how to solve
the type of problem being discussed.
Exan~plc103. An iron ring of square cross-section has an
external diameter of 140mm, and an internal diameter of 100mm.
A r;ldi;~ls;lw-c~~t
through the cross-section of thc ring forms nn
alr gap ol I lnm. 11' the ring is unil'ormly wound with 500 turns of
wire, calculate the current required to produce a flux of 0.35mWb
in the gap.
Magnetic data of the material of the ring are given below and
shown in the diagram (Fig 161). Take p, as 47t x 1 0 - ' p H / m .

GAP 0.351Wb

Fig 161
0.65
0.89
1.06
1.18
Flux density (T)
Magnetising Force ( A P m ) 200
300
400
500
Solution uses the graph (Fig 162) obtained from the above
d;1t;1
: \ r c . ~ i ol'iron a n d air gap = 20 x 20 x
= 4 x IV4square metre
1.c11gtl1ol l1.011 - n >. ( I I I C ~ I I cl~;rr~~etcr)
I
- a l r gap
= ( n x 120 x l K 3 )- ( 1 x 10-3)metre
= (376.8 - 1.0)
= 375.8 x
metre
6 ,cngrli oi' ;ilr gap =- I x 10 ' metre

ELECTROMAGNETISM

287

(continued)

MACNETISINC FORCE

H (A Jrn)

Fig 162

From graph for the iron H = 290Atlm


So for Iron, m.m.f. F , = 290 x 375.8

lW3

= 108.88At

For air, since H

B
-

=,11"

and for air, m.m.f. F A =

0.875

4 x .n x 10-7

ampere-turns
metre

0.875 x lo7 x 1 x
4 x x

Total m.m.f. F, + FA= 108.4 + 696.7 = 805.6At


805.6 Current is deduced from = -- 1.6112A
500
Energising current = 1.6A (approx.).
8s-

Tlie diagram (Fig 163) illustrates the


manner in which flux is known to bridge an air gap, especially
if the latter is comparatively large
MAGN ETIC F RI N G I N G

Fig 163
I t will be seen t h ; t t the flux in air tends to occupy a larger area
than that of the iron, and the flux density is thus reduced. An
allowance can be made for this effect in problems when required,.
but unless a directive to this effect is given, the area of the air gap
can be taken as the area of the iron.
M A G N E T I C L E A K A G E . For some magnetic circuits, due to the shape
of the iron core and the positioning of the energising coil, a small
amount of flux leakage occurs as shown by the diagram (Fig
FLUX IN POLE

= VSIfVL

FLUX

If AKACf
COfFFlC IENT

164). Somc lines o f flux arc not confined to the iron and coni1)lrtr ! I I ~ I I ' 1>itt115 [li~,ougiiair. I-'or. pr.;rctic:tl purposes, a l'i~ctor
known as the In~X-rrgc.r,ocjSc,iettt may be given which, when used,
increases the required working flux value by a n amount sufiicient
to allow !.or' this leakage.
Thus: the required total flus = the useful o r working flux x
lea kagc coeff~cient.
Tlic Ic;~h;~gc
~ o c l i i ~ i c nm;ly
t hc hctwccn 1 . 1 ; ~ n d1.3.

Example 104. (a) A magnetic circuit has an iron path of


length 500mm and ;In air g a p o f lcnpth 0 , 5 m m , the whole being
0l' llnlf'0l'Ill SqULIrt: Cross-SCCIIOII, 100()111111' i l l ill'Cil. CilI~llIillt:lilt:
number of ampere-turns necessary to produce a total flux of
Irl~Wbi n the : ~ i r piip. Igllorc frilrpinp, :l[rrf :ISSUIIIC :I lc:~L:~yc
coefficient of 1.3. The B-H curve for the iron is given by the
following table :
H ( A t / m ) 100 200 300 400 600 800
1000 1200
B (T)
0.42 0.8
0.98 1.08 1.22 1.3
1.36 1.4
(b) A conductor is passed through the air gap at a speed of
1100mls. If the length of the conductor is greater than the length
of the side of the gap, calculate the e.m.f. induced.
The solution uses the graph, obtained from the above data, a s
shown in the diagram (Fig 165).

Fig 165
Area of iron and air gap

= 1000

= 10- square metre

Length of iron = 500 x


Flux density (B) for air

=
=

0.5m
=

I tesla.

B
1
For air, since N = - =
p0 4 x
10-7
- - l o 7 ampere-turns,metre
4 x x
lo' x 0.5 x lo-'
and m.m.f., FA = 4 x x
- - =1-o4
=
lo'
3988At
8x11

2.512

From graph f o r the iron, H = 800At;m for a B value of 1.3T


and m.m.f., F, = 800 x 0.5 = 400At
1 o t ; i I ;iliil~cre-turli~
required
398.8 + 400 = 798.8At
( b ) Induced e.m.f. given by E = Blv volts
S ~ n c ethe area of the alr gap is that of a square, the side of the
square is L. 1000 x
= v'10 x
= 3.162 K
metre
Thus E = I x 3,162 x
x lo2
= 3.162 volts
\'oic In the above B = 1 tesla. I = 3,162 x lo-' metre and
v = 1OOmls.

I R O N LOSSES
The efficiency of electrical machines and transformers is
lo~vered by tlg losses which occur in them. Apart from the
. C l c c ~ / ~ n t ~ Lasses
icnl
such as Friction and Windage and the Copper
Lo.\.srs, due to the resistance of the conductors, an additional
1qs.s occurs when 3 magnetic material is taken through a cyclic
iiiriation of magnetisation. This loss I S termed the Iron i o . s s
:ind 1s itself made up of two component losses which are ( 1 ) the
I!i~stc~rc~.si.r
L o x and (2) the Eddy current Los.s. The cause of
:hc\e two sources of power loss will next be considered.
r t l l - H Y S T E RE S I S L OO P

II'the m;lpnetlslnp force applied to an iron ci~rnpleis increased


exactly the sariic way ;I>
when making the test for a B-H curve, and is then reduced to
zero, ~t will be found that the new B-H curve, for decreasing
L.;IIUCS 01' 11, will lie ~ I ~ O Vthe
C original ascending curve, and that
when H is zero B is {eft at some value. The effect of the descendIns LXII.\C be111g ; I ~ o \ . ctlie ascending one. is said to be due to
' I ~ \ . ~ I ~ I . C \ I \ ' . since tllc R v:rlucs lag behind those for tlie corr.c.\l~o~icl~ng
11 \ll.engtll when increasing. T h e word hysteresis
11.0111 L ~ ' I . O to S O I I ~ C1ii;1~11iiu1ii
v;~lue,in

.
-

ELE C T R O MA G N ETI S M (continued)


.-

29 1

--

Fig 166
comes from the Greek meaning 'to lag'. The diagram (Fig 166)
shows the effects being discussed.
The value of B when H is zero is the 'remanence' and is a
measure of the residual magnetism. In order to demagnetise the
iron it is necessary to apply a negative magnetising force, known
as the 'coercive force'. If now, H is increased in the negative
direction to its previous maximum value, the curve will reach a
value equal to the previous maximum B and if H is next gradually
rccluccd to zero, rcvcrscd und incrcuscd to its arigini~l~nuxinlunl,
a closed loop will be traced. This is a hysteresis loop and is a
measure of part of the iron loss.
T o take the iron through the various stages represented by the
loop, an alternating magnetising force has to be applied. One
method of achieving this is by connecting the energising coil to
an a x . supply, when theiron will continue to go through the same
series of changes or magnetic cycles. To confirm that energy is
being expended, it will be found that the iron core will register a
temperature rise. Although it is not proposed, at this stage, to
prove ttie fact that the area of the loop is a measure of the powel
loss due to hysteresis, the loop can be regarded as an indicator
diagram. More advanced studies will show that the energq
absorbed per cubic metre per cycle, due to hysteresis, is given in
joules by the area of the loop, provided the scales used for the
graph are in the appropriate SI units. During the development
of the proof, it would be stated that the energy stored in the
magnetic field is represented by the area OABCDO (Fig 166).

When the field collapses, energy is returned to the s u p ~ l ywhich


is represented by the area DBCD. The area of the loop OABDO
represents the energy lost as heat through hysteresis and is
obviously the difference between the energy put into the
magnetic circuit when setting up the field and that recovered
when the field decays.

Fig 167

If the iron sample was non-magnetic, ie air, then the B-H


curve would be a straight line, as is shown in the diagram (Fig
165), and the energy stored in the field when it is set up, is
represented by the area of the triangle OBC. This energy is
recovered wlierb, the field collapses. No energy is wasted when
air is'qhe m e d ~ u mof a pulsing magnetic field.
For alr. area of triangle OBC = -1 O C x CB
2

where B, is thc maximum Ilux density value, which has been


attained for the H value which was impressed.
I
1
Bm - -Bm
But B = p,M. :. Area of triangle = - x Bm x -

Po

Sincc thc ;irc;i of thc tri;~nplcrcprcscnts thc cnergy stored in


; ~ r rper cubrc rnetlt ( i n joules), i t Ibllows that:
B",~
F o r air. Energy stored per cubic metre = - joules
2

The types of hysteresis loop, a s obtained from various


m;lgnctic m:rteri:~ls.can be grouped into one of three classes a s
sIio1~11by the diagr;rni (Fig 168).
I ("'1' I i \ I'or I I ; I I ( I 51ccl.Tlic I;irgc v:~luco f tllc cocrcivc h r c c

ELECTROMAGNETISM

(('Or?litlll('(/)

20 3

Fig 168

indicates that the material is suitable for permanent magnets.


The area however is large, showing that hard steel is not suitable
for rapid reversals of magnetism. Loop 2 rises sharply showing a
high p and a good retentivity (large intercept on B axis). The
loop is typical of cast-steel and wrought-iron, w b c h are suitable
materials for cores of electromagnets and yokes of electrical
machines. Loop 3 has a small arca and a high p . The material
(mainly alloyed sheet-steels) is suitable for rapid reversals of
ma g netism and is used for mnchine armatures, transformercores, etc
1. HYSTERESIS LOSS. Since this. is a function of loop area, the
effect of varying B on the area requires consideration. When the
value of H is increased eg doubled, the value of B is not doubled
and consequently the ratio of the loop area is not quadrupled.
1 1 is li)t~ntllo incrci~sc: ~ b o t 3~ ,ll 1i111csi ~ t ~ il'wc
tl
nsstlrllc Arci~crl'
Loop to be proportional to BmX then x lies between 1 and 2here B, is the maximum value to which the flux density has k e n
taken. Steinmetz originally found x to be practically constant
for all specimens of iron and steel and to equal 1.6 (approximately).
This figure is called the Steinmetz Index and we can now
write: power loss per cubic metre per cycle as PH cc Bm1''6
o r PH = constant x Bm"6 x f joules per second.
Thus P H = K H x B,"~ x f watts per cubic metre, where KH
1s a hysterrsls coeficicnt, depending on the material being con'sidered, and f is the cyclic frequency of magnetic alternation.
Summarising :
Hysteresis loss P, = K,B,' ' 6 f watts per cubic metre-an
empirical formula.

2. EDDY - C U RRE N T LOSS. When an armature rotates in a magnetic


field, an e.m.f. is induced in the conductors. Since the conductors
are let into slots, it is obvious that the armature teeth can also be

looked upon as conductors and that e.m.fs. will be induced across


them. Moreover. since the electrical circuit is complete for these
e.m.fs., currents will flow from one end of a tooth through the
armature end-plate. along the spider or shaft and back to the
other end of the tooth through the opposite armature end-plate.
These currents. known as 'eddy currents.' produce a power losr,
due to the resistance of the iron circuit, which is a PR
This eddy-current loss is also dependent on a &n
R
other factors.-which are considered below. Every attempt 3'
r ~ ~ ; ~ hy
c l crn;iclline designers to keep such loss to a minimum.
I llc prlncrpal methods by which this is achieved are (a) by
laminating the iron circuit and insulating the laminations from
each other by \,amish. cellulosr. or paper ( b ) b!, using iron with
a high specific resistance and (c) by keeping the frzquency of the
magnetic alternations or cycles to a minimum.
Since a generated voltage is proportional to flux and speed.
then E K Oh' or E K B,f where B, is the maximum. flux
density and f is the frequency of alternation.
E2
Again since power loss cr - we can write:
R
Power loss rc ~ , ' , f ~ o r PE = K ~ f Bwatts~ per~cubic metre,
whcrc K,: is a n ( ~ l ( l . v - ~ . i i r (r.(o, (~~~f i c i e n which
[
is dependent upon
the type of material belng used. its thickness and other dimensions.

4
??'

'

P U L L OF A N ELECTROMAGNET
It has been seen that the energy stored in a magnetic field in
BZ
air is given by - joules per cubic metre, where B is in teslas
2/43

Consider two poles arranged as shown in the diagram (Fig 169),

i?r
-X

Each has an area A square metres and let F be the force

ot'

r ~ l t r r ~ c t i o(in
n ncwtonu) hctwc.~ri t l i r ~ l c ~ l r u

Let one pole be m o v ~ da s l n i ~ l l~ I S ~ ~ I I .\I C( CI ~ w ~ I . c . \ ) i l g i l l l l h t [ I I C


force F. Then the work dorlc i s F\- nrw~on1nrlrt.s 01. io\~lrs.
The volume of the magnetic field has been increased by ,4.v
cubic metres and therefore the energy stored in the field is InB2
creased by -- x . 4 . ~joules. This is obviously equal to the worA
2 ~ 0
B 2 A.u
done In separating the poles so that Fu = 2/10

or F = B 2A newtons. where A is in square metres and B in


2/10
teslas.
Example 105. An electromagnet is wound with 500 turns. The
air gap has a length of 2mm and a cross-sectional area of
1000mm2. Assuming the reluctance of the iron70 be negligible
compared with that of the air gap. and neglecting magnetlc
leakage and fringing, calculate .he magnetic pull when the
current is 3 amperes.
N.M.F. ofcoil = 500 x 3 = 1500At
This m,m.f. 1s used to pass the flux through the air gap, since
the reluctance of the iron is negligible.
The magnetising force for the air, is glven by 'the ampere-turns
per metre' o r
Also the flux density in air is B where

= 0.942T

B2A
Now pull F = --2p0

~ h u Fs =

0.942' x 1000 x
2 x 4 x x lo-'

newtons

z:f

-newtons

and the pull would be 353.3N.


Example 106. A four-pole d.c. generator has a cast-steel yoke
and poles and has a laminated steel armature. The dimensions
of the component parts of the magnetic circuit are as follows:

Yoke. Total mean circumference 3 . 0 4 ~ 1


Pole Total mean length 0.24m
Air gap. Total mean length 2mm
Armature. Total mean path between poles
0.4m

'

CSA 0.04m2
CSA 0.065m2
CSA 0.065m2
CSA 0.025m2

Fig 170

The magnetisation curves are:


H(At m ) 400 800 1200 1600 2000 2400
1.37 1.43
1.3
1.2
Cast Steel
B(T)
0.45 1
1.63
Laminated S t e d B(T)
1
1.34 1.48' 1.55 1.6
Calculate the ampere-turns per pole. for a flux per pole of
0.08Wb in the air gap. The diagram (Fig 170) illustrates the
problem and the appropriate magnetic characteristics are shown
by the graphs of the diagram (Fig 171).
A I K (;AI'

Length 2 x 1W3 metre


Ar-e.10.065 = 6 .:

square metre

, ,.

Total ampere-turns o r M . M . F . for air gap IS given by:


1: = ..- 1 23-.- -- x 2 x 10 ampere-turns
A
4 x 1 1 4 ~10.'

(Cast Steel)
Length 24 x
metre
Area 0.065 = 6 . 5 x 1 0 r,ciu;lrc rnc!rr

P O LE

From graph, Up = 1370At/m or


Total Fp for pole = 1370 x 24 x

330At

(Cast Steel)
3.04
Length -= 0.76 = 76 x
metre (between poles)
4
or 0.38m magnetic length
Area = 2 x 0.04 = 0.08 square metre. (Note the
doubling of the area since full pole area has
been taken for the flux)
= 8 x 10-'m2

Y O KE

From graph H , = 800At rn


Total F, for pokc 800 x 38 x

304At

(Laminations)
0.4
Length - = 20 x
metre (magnetic length)
2
Area = 2 x 0.025 = 0.05 = 5 x lo-' sauare metres

A RM A T U RE

From graph H L = 2000At m


Total FL for armature = 2000 x 20 x lo-'
= 400At
Total M.M.F. per pole = 1955 + 330 + 304
= 9989~r.

+ 400

CHAPTER 1 2

PRACTICE EXAMPLES
1.

A brass rod of cross-section 1000mm2 is formed into a


closed ring of mean diameter 300mm. It is wound uniformly
with a coil of 500 turns. If a magnetising current of 5A flows
in the coil, calculate (a) the magnetising force (b) the flux
density and (c) the total flux.

An electromagnetic contactor has a magnetic circult of


length 250mm and a uniform cross-sectional area o f 4 h 2
Calculate the number of ampere-turns required to produce
a flux of 500pWb, given that the relatlve permeability of the
material under these conditions is 2500. ALSO po = 4x x
lo-' henrylmetre.

3.

In a certain magnet~ccircuit having a length of l m and


a uniform cross-section of 500mm2, a magnetising force of
500 ampere-turns produces a magnetic flux of 400pWb.
Calculate (a) the relative permeability of the material and
(b) the reluctance of the magnetic circuit. po = 4x x lo-'
H/m.

4.

An iron ring having a mean circumference of 1.25m and a


cross-section;tI are;i o f 1 500mm2, i5 wound with 400 turns of
wire. An exciting currenr of' 2.5.4 produces a flux df'
0.75mWb in the iron ring. Calculate (a) the permeability
(relative) of the iron (b) the reluct:tnce of the iron (c) the
m.m.f. of the exciting winding.

5.

A U-shaped electromagnet has an armature separated


from each pole by an air gap of 2mm. The cross-sectional
area of both the electromagnet and the armature is 1200mm2
and the total length of the iron path is 0.6m. Determine the
ampere-turns necessary to produce a total flux in each
air gap of I . l3mWb neglecting magnetic leakage and
fringing.
The magnetisation curve for the iron is given by:
B(T)
0.5
0.6
0.7
0.8
0.9
1.0
1.1
H(At/m)
520 585 660 740 820 910 1030

6.

A clrcular ring of iron of mean diameter 0.2m and crosssectional area 600mm 2 has a radial air gap of 2mm. It 1s
magnetised by a coil having 500 turns of wire. Neglecting

magnetic leakage and fringing, estimate the flux density in


the air gap. when a current of 3A flows through the coil.
Use the magnetlc characteristics as given by the graph of Q5.

7.

A built-up magnetic circuit without an air gap, consists of


two cores and two yokes. Each core is cylindrical, 5 h m
diameter and 160mm long. Each yoke is of square crosssection 47 x 47mm and is 180mm long. The distance
between the centres of the cores is 130mm. Calculate the
ampere-turns necessary to obtain a flux density of 1.2T in
tlic cores. Ncglcct magnctic lenkngc. The magnetic charactcristics of' tlic tnatcrial arc:
1.15 1.2
1.0
1.05 1 . 1
R(T)
0.9
310
380
470
f/(Ariln).
200
260
050

8.

A n iron rod 15mm diameter, is bent into a semi-circle of


50mm inside radius and is wound uniformly with 480 turns
of wire so as to form a horse-shoe electromagnet. The poles
are faced so as to make good magnetic contact with an iron
armature 15 x 15mm cross-section and 130mm long. ( a )
Calculate the current required to produce a pull of 196.2N
between the armature and pole faces. Neglect magnetic
leakage. ( b ) If the armature is fixed so as to leave uniform
air gaps O.5mm wide at each pole face, calculate the ampereturns necessary to obtain a flux density of 1 . 1 5T in the
air gap. Neglect leakage and fringing. Use the magnetic
characteristics as given by the graph of 47.

9.

Two coaxial magnetic poles each lOOmm in diameter are


separa ed by an air gap of 2.5mm and the flux crossing the
air gap is 0,004Wb. Neglecting fringing calculate (a) the
energy In joules stored in the air gap (b) the pull in newtons
between the poles.

10.

Calculate the ampere-turns per field coil required for the


air yap. the armilturc tcctli and the pole, 01.3 d.c. machine
working with a useful flux of O.O5Wb/pole, having given:
Illl'cctivc ;Ircq o S ; ~ igr ; ~ pM OOOmmZ.Mean Icnpth'of air gap
5mm. Ell'ec*tive area 01' pole 40 O m r n ' . Mean length ol'
pole 250mm. Effective area of teeth 25 000mm2. Mean
lehgth of teeth 45mm.
Magnetic leakage coefficient = 1.2. Magnetic characteristics 01' tllc materials i\rc:
R(T)
1.3
1.4
1.5
1.6
1.8
2.0
/ / ( A t ' I I I ) 1200
IS00
2000
3000
HSOt) 24 OW

THE ELECTRON THEORY, B A S I C


ELEC'I'KON IC'S A N L) ELEC"I'I~OS'1'A'I'IC'S
Electrical engineering is concerned with the passage of
electricity round conducting circuits and with the resulting
effects. The passage of electricity is referred to as t5e flow of
current but this in turn, is due to the movement of chargcs
round the circuits. Electricity is considered to be of two kinds,
namely positive and negative and in its smallest quantity consists
of +ve and - ve charges.
CONSTITUTION OF MATTER
Matter may be de'fined as anything that occu5es space. I t may
be'in solid, liquid o r gaseous form but basically consists of
molecules of the substance. A molecule is the smallest particle of
a substance that can exist by itself. Thus molecules have the
properties of the substance which they form but themselves
ccnsist of groups of atoms. As an example, a molecule of water.
written H,O, consists of 2 atoms of hydrogen and one of oxygen.
The atom is defined as the smallest particle that can enter into
chemical action, but is itself a complex structure consisting of
charges of electricity. A substance that contains only atoms with
tlie samc propcrtics is called iln clcmcnr, but one conluining
atoms of different properties is called a compound.
THE STRUCTURE OF THE ATOM

According to the electron theory, as propounded by eminent


scientists like Rutherford and Bohr, each atom has a core or
nltcleus surrounded by planetary c~lc~crrons.
The nucleus conslsts
of +vely charged particles or protons and uncharged particles
called neutrons but the arrangement is such that the nucleus has
a net +ve charge which is equal and opposite to that of the
planetary electrons. These electrons are considered to have - ve
charges and to revolve round the nucleus so constituting a
miniature solar system. The nucleus thus represents the sun and
the electrons represent the planets. Under normal conditions,
when the atom is said to be stable or unexcited, the planetary
electrons together neutralise the +ve charged particles of the
nucleus, so the complete atom itself has no electrical charge.
The diagram (Fig 172) gives an idea of the atomic structure as

it is assumed. The simplest atom is that of the element hydrogen,


consisting of a nucleus with one proton (having a +ve charge),
around which travels one electron in an orbit. The electron with
its - ve charge neutralises that of the proton. In the diagrams,
the electrons are denoted by circles, with their charges shown,
and are considered to be moving on dotted orbits. The nucleus is
shown with a full circle, has a net positive charge attributed to
the protons contained therein and these are shown by + marked
circles. The neutrons are shown by small circles with no charge
sign.
* - -

-.

ILtCTl.ON

a4

\
\

fltCTRON
#

NfUIRON
/

\
\

/
/

''

-4'-

\-or

,'

/',/

'.**'

HYDROCE N
ATOM

* - * -=\
-

.wc

Hf LlUM

f'.
-

.e'

OXlC f N

ATOM

ATOM

Fig 172

The next element considered is helium. This has 2 planetary


electrons and the nucleus consists of 2 protons and 2 neutrons.
The planetary electrons of most atoms are associated with the
nucleus in a definite manner ie the electrons are in groups
termed 'shells,' such that the planetary path of each shell is
difierent. This% shown if an oxygen atom is considered. This
has a nucleus of eight protons and eight neutrons. The planetary
electrons are eight in two orbits or shells-six in the outer shell
and two in the inner shell. For any one atom, the electrons in a
shell can be less than, but never more than a definite number.
Thus the first shell cannot have more than two electrons and the
second shell more than eight.
.',-* -9-- - - *\
,B'
/ ,/
-. \

/ ; l a \ a\!\
'

/o-.*

/'

' 4 -

~\

' -

,o.-

\*%\'\

/ /6 /

'\

' \
\

\
\

*\ ..

1 t0 \ 1

,d

1I

'

'

- - - D'

-..&-'

I#

,' /

P
,
/

/'

ELECTRON THEORY, BASIC ELECTRONICS, ELECTKOSTATI(3

303

Diagram (Fig 173) represents the atom structure of tivo


metals-lithium and sodium. In each case, and if other metals
;Ire considcrctl, i t will bc Fccn th:~ti111 I i i i v r cltic or- I W O c-lrc I r o r i ~
111 IIIC

OLIICI'IIIL)SI

511~11, 1'1115 I c ~ I I u I ' c

15

COII~ILICICLI

it>

~ I I C IC,I~OII

for metals hrtv~nggood cnnducting properties. sinc-c ~hracOUICI.most electrons are loosely held by the electrostatic field and can
be easily removed.
All the examples with the diagrams have shown atoms in their
neutral state; ie the total - ve charges on the planetary electrons
is equal to and balanced by the total +ve charge on the protons,
so that the complete atom has no charge. The neutral atom has
no electrical characteristics-it neither attracts nor repels other
atoms o r electrons. However, since the nlolecules of any substance are continually in motion, the atoms, which are part ol'tlic
molecules, also move about inside the molecules and such movement can result in an upsetting of the atomic structure.
IONS (Positive and Negative). An atom may lose or gain an
electron as the result of a disturbing action. It then becomes
electrically unbalanced, having acquired a charge and is called
an ion. Thus an atom minus an electron, exhlbits a +ve charge
and is a +ve ion. Similarly an atom w h c h gains an electron.
exhibits a - ve charge and is a - ve ion.
P

CURRENT FLOW AS ELECTRON MOVEMENT

Current flow, according to the electron theory, is due to the


movement of electrons from one atom to the next, each electron
c;~rryinpwith i t ;I - vc cll:~ryc.In s \ ~ l ~ s t ; 5~\ 1~c ~~l l: I~> c~si l c l : ~ l \ .
which are classified as good conductors of' electricity, the outer
orbital electrons can move comparatively freely between onc
atom and its neighbours. Such electrons, called 'mobile electrons,' move in a random manner from atom to atom, so that,
the transference of charge and therefore passage of electricity, in
any one particular direction, does not occur and n o current is
considered t o flaw. If an electrical force, in the form of a pressure
or potential difference or electric field, as it is sometimes called,
is applied across a good conductor then the mobile electrons will
move under the influence of this force towards the higher
potential or +ve terminal. The required electrical force can be
produced by a battery or generator which can be regarded as a
pump moving the electrons round the circuit. A stream or movement of electrons is said to constitute an electric current but. it is
stressed here that, attention must be paid to the difference between the direct~onof conventional current flow and electron
flow. Thus if a length of wire is connected to two terminals.

between which an electrical pressure or potential difference ( p . d . )


exists, then a current will flow from the +ve terminal through
the wire to the - ve terminal. Electron flow will be, however, from
the - ve terminal to the +ve terminal. This fundamental
difference between conventional current and electron flow must
always be remembered and is illustrated by the diagram (Fig
174a and b). It is also stressed here that, the electrical generator
or battery which maintains a pressure or potential difference
(p.d.) between the ends of a conductor, does not itself make
electricity but merely causes a movement of the charges o r
clcctrons wllicll arc ;~lrendyprcsent in the circuit.
FREE ELECTRONS I N
RANDOM M O T I O N

V
CURRNT

yo;
-qi,+?+~.ls.r-b-++;tb,
.
05
\ .

d
..

-<-,.:,

--q<f-Q<)

ELECTRONS
N O APPLIED

(a)

P. D .

APPLIED

Fig 174

P.D. AS

SHOWN

(b)

When an electron is made to leave a parent atom by


the application of some effect, such as the force due to an
electric field. or by the application of heat or light, it may
acquire sufficient energy to detach further electrons from any
other atom with which it may come into contact. Such action
causes the atom to become a +ve ion and, if electrons leave the
atoms tasterthan they can be regained by the atom, this state of
ionisation continues. Many electrical items of equipment, such
as fluorescent lamps, certain radio valves, and semiconductor
devices depend on ionisation for satisfactory operation and this
process will again be mentioned in gfeater detail when these
various pieces of electronic apparatus are considered.
IO N IS A T I O N .

ELECTRIC FIELD

This tcrm h ; ~ s;\lrcndy hccn used in association with the


potential diff'erence required to cause electron movement in a
circuit. I t will bc dc;llt with in mnrc dctnil under the heading of
Electrostatics, but must be given some attention here, since i t is
directly associated with electron or current flow.
Elemental bodies such as those made of metal, have the same
electrical properties of their atoms. If therefore, the atoms are
charged: in that they 11,ave become ions. either through the
removal of electrons making them +vely charged or through an
ovcr i~bundi~ncu
ol.clcctrons. making rhcm -vcly c h ~ ~ r g cthcn
d,

the main body of which they are composed, is +vely o r -vely


charged. Since atoms endeavour to remain neutral, ions will
always tend to acquire or discard electrons hy interchange with
nciyhbouriny utomH. I ' l ~ orrntno Ilropcrty will w i n 1 l o r c.lillrgrtl
bodies and if a +vely charged body (deticient in electrons) is
placed in contact with a -vcly charged body (excess clectrirns),
then electron flow will occur from the second to the first o r a
current will flow from the first body to the second, until both
bodies have the same degree of charge. Before the bodies were
placed in contact with each other, a force would be detected
between them and the adjacent space would show signs of such
a force. The space within which this force could be detected
would be an electricjeld. Thus a potential difference is said to
exist between the charged bodies, which results in an electric
field. When the bodies are placed in contact, an equalising of
charges takes place o r current flows and we have the basic
requirement that a p.d. must exist between two points before a
current will flow. It is not proposed here to consider how such a
potential difference is created, but earlier studies have shown
that an appropriate electrical device, such as a battery or
generator, functions by developing an electromotive force
(e.m.f.), resulting in a p.d. between its terminals and thus between
the two bodies being considered. If such an e.m.f. or p.d. is maintained continually by the action of the battery or generator, then
a continuous current will flow. Our related studies are concerned
with the broad field oT electrical engineering which deals with
clynnmic c~lc~ctric~iry
and its erects. and forms the main subject of
this book. If however, an e.m.f'. or p.d, is otlly maintained until
the current commences to flow and then falls to zero, in accordance with the dying away of current as the charges on both bodies
equalise, then our studies are concerned with the subject of static
electricity o r electrostatics, which forms the main subject matter
for this chapter.
Before concluding this sectlon on electron theory, the student
IS reminded that, although he has been introduced to the electron,
which is a unit of charge, this is much too small for practical
purposes. Experiment shows the -ve charge of an electron to
equal 1.6 x 10-l9 coulombs The coulomb is the practical unit
of quantity or charge and thus:
1 coulomb = 6.3 x !018 electron charges.
Again, passage of charges constitutes a current and the
practical unit of current is the ampere, which can be defined in
terms of the coulomb and the time taken for this to pass. Thus
if a charge of 1 coulomb takes 1 second to pass through a point

in a circuit, then the rate of flow of electricity is 1 coulomb per


second and the current is 1 ampere.
Thus 1 ampere (]A) = 1 wulomb/second.
Both the coulomb and the ampere are used as units of practical
engineering and have been defined at other appropriate stages in
this book. The properties of conductors and insulators are also
described elsewhere but can also be explained in terms of the
electron theory thus :
A conductor is a material, like metal, carbon and certain
liquids, which contains mobile electrons that move under the
influence of an applied potential difference and so allow the free
passage of c u m n t.
An insulator is a material which has very few free electrons.
Examples are materials, like rubber, glass, mica and most oils, in
which the electrons are bound strongly to the nucleus. Since
little movement of electrons occurs, current flow is negligible.

ELECTROSTATICS
Mention has been made in this chapter, and in Chapter 4, of
the existence of static charges but, for the reader who is consolidating his studies at this point, we can repeat the introduction
made before proceeding with further work. From earliest times
the presence of electricity was known because of its simple
attraction and repulsion effects. The Greeks for instance, knew
that a piece of amber, when rubbed, attracted light bodies such
as pieces of cork and fibrous material. The amber was said to be
charged with electricity and the phenomena to be discussed here,
are concerned with the presence of electric charges at rest ie
electrostatics.
Experiments show that the simplest method of generating
static electricity is by rubbing or friction. Thus a glass rod when
rubbed with silk is electrified and can attract pieces of paper, but
if a similarly treated glass rod is suspended by a thread, and
brought near the original charged glass rod, then a repulsion
effect would be noted. An ebonite rod rubbed with fur would
also be found to be charged and, if brought near-to the suspended
charged glass rod, attraction would be noted.
Summansing, we cnn aciy tlint the glnsa and cbonltc r~ccluirc
charges which can be of two types, termed positive (+ve) and
negative (- ve), and that like charges repel whereas unlike
charges attract. The allocation of the type, +ve charge to the
glass rod and -ve charge to ebonite, is purely arbitrary, but the
general theory is that all uncharged bodies consist of +ve and
- v e charges which neutralise each other. If these charges are

I?LI!CTRON THEORY, BASIC ELECTRONICS, I!I.ECTHOS.TATI('S

307

separated by some applied effort, then their presence becomes


detect:lhle ilnd if they ;Ire cilused to move from one body to
nnothcr then their movelncnt is cxplr~incd by the pr~asrrgc ol

current. It will be noted that these assumptions line up with the

electron theory ~lreodystudied nnd it i s nppnrent that o negw


tively charged body has an excess of electrons and a positively
charged body is deficient in electrons. In the uncharged state,
the atoms of the material are neutral ie the charges due to the
electrons and protons exactly balance.
THE ELECTROSCOPE. The deductions made from experiments in
electrostatics are fundamental to theory, but to assist in the
demonstrations a simple detector of charges is necessary. Such a
detector, termed a gold-leaf electroscope is generally used for the
investigations. This consists of two leaves of gold foil attached to
a metal rod, which in turn is contained in a glass jar from which
it is insulated as shown in the diagram (Fig 175). A metal disc
may be fitted to the rod and the container may be a metal boxlike frame with glass sides. The leaves of the electroscope are
arranged to hzng downwards when no charges are involved, but
if. a charge is imparted to the instrument the leaves are seen to
diverge as shown.
SULATtO
TOP

.MIlrL
FRAHf

CLAI J

IlDf 5

Fig 175
The action of the electroscope is apparent when the instrument
is studied. Assume a +ve charge to be given to the electroscope
by stroking the disc with a glass rod which has been charged by
rubbing with silk. The +ve charge imparted to the disc spreads
all over the insulated -metal and the leaves, having the same
charges, are repelled and diverge. The condition is as shown in
the diagram (Fig 175). If an ebonite rod, -vely charged by
rubbing it with fur, is brought near the electroscope, the leaves
will be seen to converge. The explanation is that the +ve
charges on the electroscope are attracted by the -vely charged

I
1
i

1
,I

rod and rise up to concentrate in the area of the disc. The charge
on the leaves thus diminishes and the repulsion force between
them falls. In the same way, a +vely charged. glass rod brought
near the instrument will produce a further divergence of the
leaves, since the +ve charges already present are repelled down
towards the leaves. The density of charge in this region thus
increases and increased divergence is indicated. The instrument,
though seldom seen in use outside the laboratory, is useful for
assisting with simple demonstrations and will be referred to
frequently.
POTENTIAL DIFFERENCE

If, when two bodies are charged, as described earlier, and are
brought into contact, a minulc current is notcd to flow bctwccn

them while the charges equalise, then in the isolated and


electrified state, a potential difference will exist between them.
For the bodies to maintain their charges they must be insulated'
from earth, ie mounted on insulating rods. In the case of the
electroscope, the charge given to it is with respect to earth; the
gold leaves, rod and disc of the instrument being one body and
the mass of earth being the other. Thus the leaves are charged
+ve to earth if a +ve charge is given to the electroscope.
Similarly a - ve charge given to the instrument means that the
leaves are -ve with respect to earth. It can thus be seen that if
two bodies are charged +ve and -ve, they are at a potential to
each other, ie a potential difference exists between them and
they are also a b a potential to earth. One body is +ve to earth
and the other -ve to earth, the earth mass itself being considered to be at zero potential.
ELECTROSTATIC CHARGING

The effect of friction has already been mentioned and in


practical engineering, it is the mo-st important cause of electrlc
charging. Tht: build-up of charges can be quite considerable and
prrcuutions must bc takcn in thc artificial silk, paper, rubber,
cable-making and associated industries to discharge the bales or
coils of mnterit~laftcr they hqve been processcd. Such processing
involves, kneading, rolling, drawing, etc and the friction effects
can cause large voltages to be generated, which would be uncomfortable and even dangerous to persons handling the
material. The electrostatic charging of aircraft and rnotorvehicles is a well-known hazard and in the case of the former,
because of the large voltages possible, a means of earthing is
necessary before persons can alight from the aeroplane. Electric-

ELECTRON THEORY , BASIC ELECTRONICS, ELECTROSTATICS


--- - ---- -- - - --

309

ally conducting rubber tyres have been developed to this end.


For motor-cars the problem is not as important, since the
cl\nryc# thet rrnult rtrc einnll Ncvrrthclc~s prcriiutions rlrc
necessary for special load-carrying road vehicles such as petrol-

tnnkers, which shoi~ldhe 'e:\rthed' before unloading or loading


of fuel is commenced. The action of the 'lightning-conductor'
will be mentioned shortly, but its use is concerned with the + vely
and -vely charging of clouds which results from atmospheric
activity.
C H A R G I N G BY I N D U C T I O N . lmag~nethe electroscope to be uncharged and a +vely charged body is brought near to the disc.
Then mobile electrons of the metal are attracted up into the disc
and +ve charged atoms or ions are displaced to the leaves as
shown in the diagram (Fig 176a). If now the disc is touched with
the finger (Fig 176b), electrons from the main earth mass flow
up and neutralise the ions or +ve charges and as a result the
leaves collapse or converge. If next, the finger is taken off and
then the adjacent +vely charged body is removed, the leaves
would be seen to diverge again slightly (Fig 1 7 6 ~ ) The
.
-ve
charges, previously held by the adjacent +vely charged body,
are allowed to spread all over the electroscope and charges of
the same polarity spread down to the leaves.

Fig 176
It will be noted that charging by induction results in a charge
of opposite polarity being produced. For the example, the
inducing charge was +ve and a -ve charge resulted on the
instrument. If a - ve inducing charge had been used, a +ve
charge would result on the electroscope.
DISTRIBUTION OF CJIARGE

The statements set out below are the results of experiments


with a charged electroscope and a proof plane. The latter is a

small metal disc fitted with an insulated handle. The proof plane
is placed in contact with the body being investigated and is
electrified to the same polarity. If brought near a charged
electroscope, movement of the leaves with the appropriate
interpretation will enable conclusions to be made. The following
are some deductions made as a result of such investigations as
illustrated by the diagram (Fig 177).

Fig 177
1. A hollow body such as that shown, is only charged on the
outside. Thus a proof plane contacted with the outer surface and
presented to a charged electroscope, will show a deflection. If
contacted with the inside, it will show no deflection.
2. If a sphere is charged, the charge is spread uniformly over
its surface and the surface density is uniform. If a charged body
is nop-spherical, the charge concentration is found to be
greatest in the region where the radius of curvature is the smallest.
The diagram (Fig 178) depicts what is stated, the charge distribution or surface density being represented by the dotted
envelope.
*

Fig 178
3. 11'n cliarycd body 1s shaped to have a sharp point, then the
charge concentrates at the point and the surface density may be
so great that dust or particles in the air, corning in contact with
the body, are charged and repelled. On moving away, each
particle removes a small portion of the original charge and the
effect of a point on a charged body is to discharge the. body. The
action of the lightning conductor can now be explained.

ELECTRON THEORY, BASIC ELECTRONICS, ELECTROSTATICS

311

Consider a +vely charged cloud in the vicinity of a high


building which is fitted with such a lightning conductor, consisting of a copper rod and conductor which is well earthed. The
building, and thus the rod, acquires u - vc clruryc by induction,
with the air particles becoming negatively charged by contact.
These particles move into spircc towirrds the cloiid tending to
neutralise it. Alternatively, the space between the cloud and
building being charged by the -ve particles, has its insulating
effect lowered until a break-down occurs and a spark (lightning
discharge) passes between cloud and earth. A current will flow
during the discharge as electrons pass from earth to the cloud
which is thus discharged with safety. The current is conducted
along a suitable path, having a low resistance and damage is
avoided, since minimum energy dissipation results.
ELECTROSTATIC FIELDS OF FORCE

When two bodies are charged, a force of attraction or repulsion is produced depending on the polarity of tfie charges. The
magnitude of this force relative to the charges can be investigated, especially as a conception of the conditions appertaining
is possible, if the existence of 'lines offlux' is assumed. Since a
force would be exerted on a small +vely charged body, placed
adjacent to a large +vely charged body, it can be stated that an
electric field of force exists in the space around the large
charged body. If the small body is considered to be small
enough to constitute a +ve charge then, if free to move, it
would travel in a definite direction in the electric field and the
pirth traced by i t would rcprcscnt n linc ol' f l u x . Tlltis thc I:~rpr
+vely charged body can be considered to have many lines 01'
flux passing out from it. The similarity with the representation
used for the magnetic field will be noted. In considering the
electrostatic fields shown below in the diagram (Fig 179), one

Fig 179(a)

important fact should be observed. Each line of flux terminates


at the surface of the charged body and does not pass through the
body itself to form a closed path, as in the case of a magnetic
line of flux. The medium through which the lines of electric flux
pass is called the 'dielectric'and the lines are shown to terminate
at the surface of another body where balancing charges of
opposite polarity appear. Thus Fig 179a shows a +vely charged

Fig 179(b)

Fig 179(c)

Fig 179(d)

metal sphere in the centre of a room. Lines leave the surface


perpendicularly, in all directions and planes, to terminate on the
earthed walls, floor and ceiling, the earth mass being negative to
the charged body.

ELECTRON THEORY, BASIC I Ll C 7 KONIC'S. I Ll C I KOS I A 7 ICS

--

--

317

Fig 179b, shows the field arrangement associated with two


h,)tlics wlicrl clli~rgcdin oppositiorl. ;tntl f 7 i ~170c shows the fieltl
with like charges on the bodies. Fig 17Yd, shows the arrangement
when an uncharged body is placed in the field and how induced
polarities result. As for the ~nagneticfield, the lines of' flux car1
be imagined to be elastic threads which tend to cohtract if
allowed to d o so.
ELECTROSTATIC

nux

As for the magnetic field so for the electric field, we can


introduce the term flux. The symbol is !P (the Greek letter psi)
and the number of electrostatic lines of flux, passing through a
particular medium, is called the flux. Since the practical unit of
charge is the coulomb, then in order to establish an electrostatic
unit we consider one line of flux to emanate frcqm 1 coulomb.
Thus a charge of 10 coulombs will have in association a flux of
10 lines, or !P will equal 10 coulombs. Since most practical
electrostatic work is concerned with capacitors which are made
up from flat plate-like conductors, placed adjacent and parallel
to each other and as the medium between the plates or dielectric
carries the flux, it is convenient to introduce the term 'rlrctric
r

,pzrx density ,-symbol D. Thus D =

'V

where A is the area of the


y or Q coulomb per
dielectric in square metres. Then D" = A
A
square metre.
ELECTRIC POTENTIAL

The basic idea of electric potential has already been intrnduced when it was seen that, if two bodies are charged and
connected together, then as a current flowed while the charges
equalise, there must have been a difference of potential between
them. Again, since the earth mass can be taken as being at zero
potential then, if a body charged with Q coulombs of electricity
is connected to earth, a current will flow. The current will be
from the body to earth if it is +vely charged, and from earth to
the body if -vely charged. The direction of the current is not
important but work is done during this period. If we consider
that 1 joule of work is done whilst 1 coulomb is passed, then
the potential difference or electric potential of the body must
have been 1 volt. Voltage is therefore used as a measure of the
potential difference between the body and earth.
The- reasoning already made can also be applied to the
practical capacitor, when the plates are charged with respect to
each other. The condition of a charged capacitor can usefully be

considered to enable further deductions. If a +ve unit charge is


placed in the field between the charged capacitor plates, a force
will urge it towards the - ve plate. The force is taken as a
measure of the intmsrr~!o r strength of the field. The symbol for
electric field strength or field intensity is 8 and the force is
rneasured in t i ~ w t o n sThus
.
a charge of Q coulombs placed in an
electric field of intensity 8, will experience a force of F = QQ
F
newtons o r Q = - newtons per coulomb.

An alternative method of considering the strength of the Seld


is to take it as an electric force o r electrical potential gradient,
when it can be measured in volts per metre. This is the more
usual unit.
v
Thus. 8 = - where I is the distance between the plates in
1
metres.
Both expressions for electric field intensity and potential
gradient o r electric force are numerically the same. If practical
units are substituted, the same amount of work would be done
by 1 newton of force acting through 1 metre distance between
the plates, as would be done by 1 coulomb being conveyed by a
pressure of 1 volt.
F
V
F
V
Thus. since E = - or - then - = - or Fl = VQ
Q 1
Q
l
Note.
1 Joule = 1 vo?t x 1 coulomb, or 1 joule = 1 newton x 1 metre.
THE CAPACITOR

Numerous references have already been made to the electrical


capacitor o r condenser, as it used to be called orignally. In its
simplest form, it consists of two metal plates separated from
each other by a film of air. As will be seen later, the area of the
pl;ltr.s, disti~nce of sepi1r:ttion and nature of the dielectric
(insulating medium), all influence the performance of a capacitor,
htlt tiic b:~sicf;~ctIcnrncd from any cxpcriments made with such
an arrangement is t h a t i t can store electricity. Thus if the plates
are connected to a source of supply through a sensitive milliammeter, then a current will be seen to pass at the instant of
closing the switch. The current will quickly fall to zero, because
the potential difference between the plates will be seen to rise, as
indicated by an appropriate voltmeter. The form of current falloff o r decrement, form: a separate item of investigation, but the
capacitor can now be assumed to hx1.e attained a 'charged' state.
If the supply is disconnected and the plates are shorted together,

a discharge current will be found to flow which will be opposite


in tlirrclion. ;in(! r ~ l t l ~ c ~ r l ) initinllv
rli
I:~rpr,will r;oon (ltcl~y: I W : I ~
to L6r.U. ' I IIC I I ~ I ~ I LV OI I~~ L I ~ C , i1ltl10~1p11~ I I O W I I I J J l l ~ rv i ~ l u c(11 I I I C
charging supply, will a $0 fall away to zero.
Expcrimcnts with :I si~nplcC ; ~ ~ : I C I ~ OwI .~ l lsoot\ cst;~\jliall~ I I C
basic relatioriship between quantity of electricity that can be
stored and the charging voltage. The former will be found to be
proportional to the latter or Q K V. Since this is direct proportionality, a constant can be introduced to give the expression:
Q = CV
C is termed the capacitance and a unit is possible by simply
defining it in terms of unit quantity and unit voltage. Thus the
unit of capacitance is the Farad and a capacitor is said to have a
capacitance of 1 farad if 1 coulomb of electricity is stored when
1 volt is applied across the plates.
Example 107. Find how many electrons are displaced when a
potential difference of 500V is established between the plates of a
4pF capacitor.
Since Q = CV
then Q = 4 x
x 500 = 2 x
coulombs
But 1 coulomb = 6.3 x 1018 electrons
:. No of electrons = 12.6 x 10'
CAPACITOR SYSTEMS

Such units can be connected in series or parallel and the


\ttldenl should compare [he expression\ glvlng ecl~riv;~lcnl
capacit;~nccv;~lucsw ~ t hlIlo\c. g~vlngC ~ ~ L I I L , . I ~I .CC \II~\ ( I; I I I C . C vi~luc\.
for comparable arrangements.
SERIES CO N N E C T I O N . The

arrangement is shown in the diagram

(Fig 180).

-----------+I

Fig 180
Let the capacitors have values of C,, C , and C , farads
respectively, and let the applied voltage V be dropped as shown.
Then since V = . V , + V ,
V,

and since V ,

c,

Q and V , = Q
V =2
2
c
2

c
3

we can write:

v-=a+&+&
c, C2
C3

If C is taken to be the equivalent capacitance of the arrangement


then

v = -Q
C

Q=&+&+e,
-

or

Cl
c
2
C3
but the same current flows through each capacitor for the same
time. :. Q = Q1 = Q2 = Q3 and the above can be simplified
to :
1
1
1 - -1
- - etc.

c c,

+ c2 + c3

P A R A L LEL C O NN E C T I O N , The arrangement is shown in the diagram


(Fig 181), from which it will be seen that the same voltage is
applied to each capacitor.

Fig 181

Then for each capacitor Q , = C ,V Q , = C2V Q , = C, V


If the total quantity of charge = Q then obviously
Q = C , V + C , V + C , V = V ( C , C , C,)

or

2v = c1+ C , +

C,

If C is taken as the equivalent capacitance of the arrangement,


then
Q = C V orCV=V(C,+C,+C,)
when C = C,
C,
C,.
Fxamplc 108. I f two capacitors of values 100pF and 50pF
respectively, are connected (a) in senes (b) in parallel, across a
steady applied voltage of 1000V, calculate the jolnt capacitance
(a) Series. Joint capacitance C is given by :

ELECTRON THEORY, BASIC FL.F(.TKONICS. EI.ECTHOSTATI(.S

(b) Parallel. Joint capacitance is given by C = 100


C = 150pF.
CAPACITOR C U R RE N T , F7rofnthe r c l u t ~ o nQ =
can also be deduced.
Since Q = 11 tllcrl It C ' I '

or I =

v
C-

317

50 or

( ' 1 ' . tllc I'ollow~r~g

The expression shows that current only flows when the voltage

across a capacitor is changing, since - represents a rate of change


t

of voltage. The current at any Instant can also be found, il' the
rate of change of the voltage is known at that instant. If however.
the rate of change is unifontg for a period of time, then a constant current will flow. This is illustrated by the example.
Example 109. The p.d. across the plates of a 50pF capacitor
r
varies thus
From time r = 0 to t = 1 millisecond. V rises uniformly from
0 to 200 volts
From time r = 1 to t = 3 millisecond. I'is constant at 200 volts
From time r = 3 to t = 5 millisecond, L'falls uniformly from
200 volts to 0
Illustrate the voltage variations on a graph and deduce the
shape of the current wave during the period of 5 milliseconds.

Fig 182

318

REED 'S BAS IC ELECTROTECHNOLOGY

Since Q = CV = It then It = CV or I = C t
(200 - 0) - 50 X 200 X
(a) I = 50 x 10-b x
0.001 1x
= 10A (charging)
= lo4 x

0 amperes
50 x

(0 - 200) - 50 X
x ( - 200)
0.002
2 x 10-3
= - 5A (discharging)
The required graphs are shown on the attached diagram (Fig

(c) I

1 82).
ENERGY STORED IN AN ELECTRIC FIELD OR DIELECTRIC

Consider the voltage to rise uniformly across the plates of a


capacitor, to a value of V volts, in a time of t seconds. The

avcrage value of the p.d. would then be - volts and the charging
2
current would be constant, of value equal to I amperes. The
average power supplied during the charging period would be

I watts and the energy fed in would be - X I x r joules.


2
This energy is not converted into heat, since a capacitor has
no resistance, but does work in establishing the electric field.
It is this enpgy which is stored, and can be recovered when
the field collapses as the capacitor discharges. Thus:

Energy stored = - It joules or = - Q joules


2
2
or alternatively, W = 4CVZ joules.
Example 110. Consider the capacitor arrangement of Example
108 and calculate the total energy stored for a steady applied
voltage of 1000V, for both the series and parallel connection.
(a) Series.
Energy stored is given by W = + C V 2 joules
= x 33.33 x
x 1ooo2
= 16.675
(b) Parallel.
Energy stored or W = x 150 x
x 10002
= 755,
I t will be noted that the equivalent capacit~ncevalue for each
arrangement has been used for C in the energy expression.

ELECTRON THEORY, BASIC ELECTRONICS, ELECTROSTATICS

3 19

RELATIVE PERMITTIVITY

Jf' a parallel-plate capacitor is made lip so that one plate I S


c11rt11i 1 1 1 d ~ I I C O I I I C I lo ; I I I C ~ C L ~ I ~ ~ X C[ IOI C~ c~ lCl c, ~0,1 ~
altering the nature of the dielectric can be observed. With air a s
[ l \ c I I ~ S L I I ; I [ I I \ ~I C ~ I L I I \\)C[\VCCII
\
[l\c ~ I : I [ C > ,tl\c c : ~ p : ~ i * ~ ~[ o\ ,~\ I, I1~
charged to a given value as would be indicated by the amount ol'
divergence of the leaves of the instrument. If now a sheet of
insulating material, such as a slab of paraffin-wax, was interposed into the air-gap, a converging effect of the leaves is produced, indicating that the charge appears to have been reduced.
This can be shown to be an incorrect supposition since, if the
insulation is removed the leaves again diverge to the original
extent. An alternative assumption to be made is that thc
capacitance of the arrangement has been increased, ie the capacitor is now capable of accepting a greater amount of charge
for the same divergence of the leaves. This is confirmed if the
capacitor is charged to an amount giving the original extent of
divergence of the leaves, and the wax insulation is'then removed.
The leaves would be seen to diverge to a far greater extent.
showing a larger charge to have been Imparted to the arrangement. The experiment shows that capacitance can be varied bq
the nature of dielectric or by a property which is termed its
permittivity.This property can be likened to the pemeability of
a magnetic substance, and at this stage is considered as relative
only. It can thus be defined quite easily, as the ratio of the
capacitance of a capacitor, with the material being considered
the dielectric, to the capacitance of the same capacitor with air--o r more truly a vacuum, as the dielectric. Another term for
relative permittivity is the dielectric constant. The symbol used is
E,, (the Greek letter-small Epsilon).
Typical values for relative permittivity are Air 1.0006,
Paraffin Wax 2.2, Mica 4.5 to 8, Glass 4 to 10.
C L ) I ~ I ~ C C10~ C ~

ABSOLUTE PERMITTIVITY

It is not proposed to d o much extensive work on dielectrics as


used for capacitors or cables, but the absolute value of permittivity needs to be considered, since E , as introduced is on14
relative to air. As for the magnetic circuit, we can writer
Absolute permittivity = relative permittivity x permittivity of'
free space
Thus E = E, x E,
PERMITTIVITY OF FREE SPACE

Some comparisons have already been made with the magnetic

circuit and permittivity can besr be understood by comparing- it


with perme~bility-itsrnagnetic equivalent.
Flux density
Since permeability was defined as the ratio
R
Magnetising force
Electric flux density
D
or&=Electric force
d
F o r air, permittivity is measured to be

Then similarly; Permittivity =

4rr . I
Thus

E,

farads per metre

-o r 8.85x lo-" farads per metre.

4~ x 9 x 109

Altllouyh other illtcrnativcs can be dcrivcd to allow estimation


of the value of E,, it will be seen that the following deduction will
give the required answer, if unity is taken for the dimensions of
area A and spacing 1 between the plates.
I
D =Q/A=Q. Then since E, = E
Vll
V
A

When 1 and A are made equal to 1 , E, = the capacitance value


of the arrangement. For a vacuum, the capacitance value of the
standard capacitor, using unity for A and I, will be measured to
be 8.85 x lo-'' SI units.
1
which is also expressed as
o r E; =
4lt x " 9 x lo3
8.85 x 1 0 - l 2 farads per mctre.
Note. Although a vacuum has been mentioned for the above
capacitor arrangement, air can be taken as the dielectric, since
the effect of the variation is small enough to be neglected.
CAPACITANCE OF PARALLEL-PLATE CAPACITOR

Consider the area of the plates to be A square metres and their


bc I mctrcs, ir this would be the thickncss of tlic
dielectric. The applied voltage can be taken as V volts, resulting
in a charge of Q coulombs. Since the charge Q is fissumed to bc
unil'ormly distributed over the whole area of' the plates, the
\17;1cillg to

electric flux density D would be -.Q


A
The electric force o r potential gradient d in the dielectric

4 volts per
1l

\\'auld be

metre and permi;+ivity E would be, by

D or

QiA - Ql
V/l
VA
C
'
1'1
C
'
I
Whence6 = --- = - o r C'= r , 4 but 1: = E,C,
L'A
A
1
,,I 1;" Lr
farads
SoC'=1
Example 1 1 1. A capacitor consists of two parallel metal plates,

Thus

f:

=-

--

each 300mm by 300mm, separated by a sheet of polythene 2.Smm


thick, having a relative permittivity of 2-3. Calculate the energy
stored in the capacitor when connected to a d.c. supply of 150V.
9 x lo4 x
x 8.85 x 10-l2 x 2.3 farads
C=
2.5 X
= 73.278 x l o - "

or 73.278 x
microfgrads
0.000 73pF = 730pF
Energy stored = :CV2 joules
= X 73,278 X 10-I X 1502
= 36.639 X 22 500 X lo-"
= 36.64 X 2.25 X lo- = 9.16 x 9 x lo-' = 8.244 x
joules.
=

Example 112. A capacitor of 5pF charged to a p.d. of 100V, 1s


connected in parallel with a similar uncharged capacitor. What
quantity of electricity would flow into the second capacitor and
to what voltage would i t he charged?
Consider the first capacitor designated by the letter A , lllclr
coulombs.
since Q = CAV, Q = 5 x l o w 6x 100 = 5 x
When the capacitor B is connected across A , charge will pass
from A to B until the potential of each is the same. The arrangement of capacitors can now be considered as a parallel connection or the joint capacitance is the same as that of 1 unit of
10uF.
Applying the formula Q = CV
Q - 5 x lo-"
Then T.' = - C
10 x 10-0
1
- lo2 - 100 = 50 volts
or I.'=
2 x lo-'
2
3
The final voltage will be 50V.
This could have been deduced more simply from the fact that
the capacitors are similar and charge will pass from A to B until
the potential of each is the same.

CHAPTER 13
PRACTICE EXAMPLES
I.

Two capacitors of 0.02pF and 0.04pF are connected in


serles across a lOOV d.c. supply. Find the voltage drop across
each unit.

2.

For the circuit shown, calculate the effective capacitance


bctwecn A and B. The capacitance values shown, arc in
microfarads.
20

20

3.

A variable capacitor having a capacitance of 1000pF is


l a r g e d to a p.d. of 100V. The plates of the capacitor are
then separated by means of an insulated rod, so that the
capacitame is reduced to 300pF. Find, by calculation,
whether the p.d. across the capacitor will have changed.

4.

A plate capacitor consists of a total of 19 metal-fcil plates


each 2580mm 2 and separated by mica O.lmm thick. Find
the capacitance of the assembly if the relative permittivity
of mica is 7.

5.

A p.dof' lOkV 1s ;~ppllcdto the term~nnlsof a cap:~citor.


consisting of two circular plates, each having an area ol'
10 000mm2, sep;~r;ltcd by a dielectric Imm thick. I f thc
capacitance is 3 x 10-411F, c;iIculate the elecfr~c llux
density and the permittivity of the dielectric.

6.

A capacitor consists of two paral~eimetal plates, each

300rnrn by 3001111~1,separated by a sheet of polythene 3~5mm


thick. having a relative permittivity o f 3.0. Calculate the
elicrgy stored in tl!c capacitor when conncctcd to ;I d.c.
supply of 300V.

I-LFCTKO4 T H E O R ) , BASIC 1 LI CTK0YIC.S. FLf (THOSTATICS

323

7.

Calculate thc capacitance value of a capacitor which has


10 parallel plates separated by insulating material 0.3mm
tlilck. Thc ilrcil o f r qc citle o f c : ~ c l ipli~tci s I 500mm2i ~ n dthr
rcli~tivcprrniiltiv~~yol' the diclcctrlc IS 4.

8.

Two ci~pacitorsA and B having ctlpacitances of2OpF ; ~ n d


30pF respectively are connected in series to a 600V d.c.
supply. Determine the p.d. across each capacitor. If a third
capacitor C is connected in parallel with A and it is then
found that the p.d. across B is 400V. calculate the value of C
and the energy stored in it.

9.

A d.c. voltage of 500V is applied to a 40pF capacitor. Find


the value of the charging current at the instants when the
voltage is varying as follows:
1
Time
second)

(m

0-1
Voltage values 0-100
10.

,.

1-2
2-3
3 4
100-150 150 const 150-50

4-5
50-0

A single-phase concentric cable takes a charging current


of 1A per kilometre, when connected to 11kV, 50Hz mains.
The diameter of the inner core is 12mm and the insulation
has a radial thickness of IOmm. Calculate the permittivity of
the insulating material.

CHAPTER 14

MISCELLANEOUS CIRCUIT CONDITIONS


AND METHODS OF SOLUTION
SPECIAL APPLICATIONS
Certain network problems may be much too difficult for
solving by the direct application of the fundamental rules for
series and parollel circuits. Solutions can then only be effected
by the application of one ol' several well-known Theorems o r
Methods of Solutions. These are based on first principles, but the
sirnplcfstonly arcconsidsrcd ; ~ n darc confined to d.c. applications
Although most of the methods are suitable ['or application to
instantaneous and phasor quantities. since impedances may be
involved instead of resistances, nevertheless due allowance must
be made for phase. and considerable experience is required.
before confidence is attained. For this reason the methods set out
below show d.c. examples only.
T o enable the more involved a.c. circuit problems to be solved,
the reader is introduced to Conductance, Susceprance and
Admittance. I t will be seen that the terms and methods of
resolving are based on well-developed principles. The reader
should regard the new work as an aid to solution only and as the
first of several methods, devised to facilitate progression into
more a d v a n a d theory and problems.

D.C. NETWORKS
1.

A PP L I C A T I O N O F K I R C HH O FF ' S L A W S

The laws are revised in association with the diagram (Figs


183a and b).

(3)

Fig 183

Law 1 states that in any closed circuit or mesh. the algchra~c


sum of ; ~ l l c.ni.f.;. is e q u ; ~ lto the i~lgebraicsum of thc ~ o l t a g c
~ I l ~ O ~ ~ \

I'llus I:, i I;, - I/(,


t /I\',
I Ili,,
i
Ili,,
a14 \IIO\\II 111 1 1 1 ~ .
dli~pr;\m
Law 2 states that tlic algebra~csun1 ol' ~ I I I rllc curI.cn[s :I[ ,I
polnt is zero. Current flowing to the point b e ~ n greckoned 21s
+ve and current in the reverse direction as -ve.
In the diagram I , + I , + I , - I , - I , = 0.
Problems are solved by the application of the above two laws
and by building up equ:rtions which are then solved ;ilpebraicaIIy.
The examples illustrate the procedure.
Example 113. Three cells each l i ; ~ v ~ n;In
g c..m.f. of 2 V ; ~ n d
negligible reslstnnce are connected in series. A resistor of 100R
and one of 25Q are connected in series across the battery. thc
iOOR resistor being connected to the +ve t e ~ i n a l Calculate
.
the current flowing in a 1 0 0 resistor connected between a
tapping o n the battery, 4V from the +ve end and the Junctio
of the .two resistors. The arran_rement is illustrated by t h
diagram (Fig 184).
2 VOLTS

4VOLTS

Fig 184

Let currents I , , I, and I , be assumed to flow in the directions


shown. Then I , = I, + I,. This involves applying Kirchhoffs
current law
Consider any closed circuit or mesh. such ;IS the right-hand
rectangle. Then applying the voltage law we have:
E = I R , + I R , etc o r 4 = ( I , x 100) - ( I , x 10) . . . ( ( I )
Note the - I , x 10. Since I , has been assumed in the direction
shown, the voltage drop in I , x 10 would be in the opposite
direction to the fall of p.d. as assumed for the closed circuit o r
mesh, hence the - ve sign.

326

R E ED ' S BASIC ELECTROTECHNOLOGY

Equation ( ( 1 ) contains two unknowns and thus a second equation is required to allow ,solution. This can be obtsined by
considering a second mesh. sucli as the left-hand rectangle.
T h e n 2 = ( I 2 x 1 0 ) + ( I , x 2 5 ) or 2 = ( l 2 x 1 0 ) + ( 1 , + I 2 ) x 2 5
. . . (b)
Solving ( ( 1 ) and ( b ) together we have:
1001, - 101, = 4 or 1001, - 101, = 4
251, + 3512 = 2
and 101, + 251, + 251, = 2
whence 1001, - 101, = 4
and 1001, + 1401, = 8
4
.'. I, =
I\!* sul>rr;~ction- 1501,
-4
150
Thus I, = 0.0266A and the 'tssumed direction of flow is
correct i t s 11 is i~ctui~lly
downwi~rdstowards [llc junction ol' tllc
resistors. Current in 10R resistor = 26.6mA. I , can be obtained
by substituting back in equation (a).

2.

M A X W E L L ' S C IR C U L A T I N G - C U RR E N T THEOKEM

For certain problems, the direct application of Kirchhoffs


laws may lead to a solution which may be laborious, and in some
instances this 'cyclic current' method may be used to advantage.
I t will be seen that to each closed mesh is assigned a clockwise
c11:ulating current and that the voltage law is applied to build
up a set of simultaneous equations. Note. The current flow must
be indicated and assumed to be the +ve direction. Voltage drops
In this direction will be + ve, in the opposite direction, - ve.
Consider Exajnple 113, solved by this method. Then thc
diagram (Fis.185) would be as shown.
2 '10-TS

4 VOLTS

Fig 185
('onsider the right-hand mesh, then 1001, + I O ( 1 , -1,) = 4 ( ~ ) .
I t will bc seen that the current in the IOR resistor has been
t:~kcn ;rs I , - I : . Nest considcr the left-hand mesh, then
10(1, - I , ) + 251, = 2 f h ) . Here cyclic current I2 being clockWlhC. I \

~;II\CII; I 4 I

VC.

S e t t ~ n gdown equ'itlons ( ( 1 ) and ( h ) glves


0 0 , I I
- 1
or
1101,
101,-4
A I\( I
IO(1, - I , ) t X I 2 = 2 01 - 101, i M I z - 7
-

I.ll\l\ \\C I l , l \ L S

1 101, - 101,
~ n d 1101, + 3851,
Adding gives. 3751,

=
=
=

or I,

4
21
26

26

lo-'
3.75
Subst~tutingback; then 351, - 101,

O0694A

2
(35 x 0.0694) - 2
or (35 x 0.0694) - 101, = 2 and I, =
10
giv~ngI, = 0.0427A
The current in the 10fl resistor is then given by:
0.0694 - 0.0427 = 0.0267A (in the downward direction)

3.

THE SUPER-POSITION OF CURRE>IT THEOREM

Chis is based on the fact that, the current in any branch of a


network is the sum of the currents due to each battery o r
generator taken separately, all other generators or ba:terltl>
being replaced meanwhile by their internal resistances. Thls
theorem avoids the use of simultaneous equations. The magnltude and direction of current for each section is calculated.
assuming one supply source, and all other sources having t h e ~ r
e.m.f.ssuppressed. Their intern;~lresist;~nceshowever. ;Ire t:~kcn
into account. This IS repeated for each source o!' supply i ~ n dr l l t
current. in magnitude and direction through each resistance. I S
finally obtained by adding algebraically all the ;ipprc>pri;~tc.
values.
The solution, shown below, refers to Example 113 and should
be studied in conjunction with the diagram (Fig 186 a , b. c and
d).
2 VOLTS

Fig 186(a)

10 OHMS

2 VOLTS

2 5 OHMS

100 OHMS

Fig 186(b)
4 VOLTS

Fig 186(c)
1
0 OHHS
*VOLT5
100 OHMS

25 OHHS

Fig 186(d)

-I
i
I

I
I

Condition 1 . 4 V battery e.m.f. suppressed. Fig 186a.


To illustrnte procedure, the diagram has been set out
shn~
Fig 1S6b.
L
= resistance of parallel section
1 I
- " or I{ = 9 090

' '

H - 100 10
106
The ~ 6 t a clrcult
l
resistance would be 25

;IS

+ 9.09 = 34.090 and

the current flowing =

= 0.0587A
34.09
Voltage drop across the parallel section = 0.0587 x 9.09volts
0.0587 x 9.09
~
in the 1 0 9 resistor =
and t h current
= 0,053312.
10

Condition 2. 2V battery e.m.f. suppressed.


Let R = resistance of p:rrallel section
1
1
I - 4 t lo
14 ,,,.
l llf 11
I0
I00
1 00
I.!
25

,{

'

and the current flowing =

lo
14

--

7.14fl

'

0.037 35A
107.14
To illustrate procedure, the diagram has been re-arranged a s
shown in Fig 186d.
The voltage drop across the parallel section = 0,037 35 x
7.14 volts.
0.037 35 x 7.14 = 0.0267A
Current In 10R reslstor =
10
The current In the 10R resistor is given by the joint effects of
conditions 1 and 2. Thus I = 0.0533 - 0.0267 = 0.0266A.
Current flow is towards the junction of the resistors- as
shown for the alternative solutions.
Example 1 14. The diagram (Fig 187) shows a Wheatstonebridge network. Find the current, and its direction, in the
galvanometer circuit.
-=

Fig 187
Method 1 . Application of Kirchhoff s laws.
After allocating currents as shown. we apply the voltage law
to :
Mesh ABD 21, + 501, - 512 = 0
Mesh BCD 4(11 - I,) - 8(12 + I,) - 5013 = 0
Circuit ABC and battery 21, + 4(I, - I , ) = 6

. . iu)
. . (b)
. . lc,j

330

REED'S BASIC ELECTROTECHNOLOGY

S~mplifying( u ) , ( h ) and (c), we have:


21, - 5/, + 501, = 0
. . . (a)
and 41, - 41, - 81, - 81, - 501, = 0
. . . (d)
or 41, - 81, - 621, = 0
also from (c) 21, + 41, - 41, = 6
o r 61, - 41, = 6 giving I , = 1 + $1, o r I, = 1 + 0,661,
Substituting in (a)
2(1 + 0.661,) - 51, + 501, = 0
or 2 + 1.321, - 51, + 501, = 8
giving - 51, + 5 1.321, = - 2

. . . (ej

Substituting in (d)
4(1 + 0.661,) - 81, - 621, = 0
o r 4 + 2.641, - 81, - 621, = 0
giving - 81, - 59.361, = - 4
Solving (e) and (f) Multiply(e) by 8 to get
- 401, + 410.561, = - 16
Multiply (f) by 5 to get
- 401, - 296.801, = - 20
Subtract ( h ) from ( g ) then 707.361, = 4
4
o r I, = 70'7.36 amperes
Thus I ,

lo-'
0.707

..

ff)

..

(g)

. . . (h)

5.65 x IF3= OW56A


(downwi~rds).

Method 2. Application of Maxwell's Circulating-Current


Theorem
0

VOLTS

Fig 1'88

CIRCUITS : hfETHODS OF SOLUnON : SPECIAL APPLICATIONS

33 1

Mesh ABD
21, + 50(1, - 1,)+ 5(11 - I,) = O
or 21, + 501, - 501, + 51, - 51, = 0
and 571, - 501, - 51, = 0

...

Mesh BCD
41,
8(I, - I,) + 50(1, - I,) = 0
or 41, $- 81, - 81, + 501, - 501, = 0
and - 501, + 621, - 81, = 0

. . . (b)

(cij

Multiply (a) by 8 and (6) by 5


Then 4561, - 4001, - 401, = 0
and - 2501, + 3101, - 401, = 0

. . . (c)

Subtracting 7061, - 7101, = 0


Mesh ADC and battery
5(13 - I,)
8(13 - 12)= 6
or 51, - 51, + 81, - 81, = 6
and - 51, - 81, + 131, = 6

Multiply (a) by 13 and (d) by 5


Then 7411, - 6501, - 651, = 0
and - 251, - 401, + 651, = 30
Adding 7161,

. . . (e)

6901, = 30

Solving ( c ) and ( e ) Multiply (c) by 6.9 and ( e ) by 7.1


Then 6.9 x (7,061, - 7.1I, = 0)
or 48.7141, - 48.991, = 0
and 7.1 x (7.161, - 6.91, = 0.3)
or 50.8361, - 48.991, = 2.13
Subtracting - 2.1221, = - 2.13 or I, = 1.003 77A (downwards)
706
I - --- I, = 0.994 x 1.003 77 = 0,998 16A (upwards)
- 710
:. Resulting current = 1.003 77-0.998 16 = 0.0056A (downwards).
I t will be seen that the Super-position of Current Theorem
cannot be applied to this network since there is only one source
of e.m.f.

CONDUCTANCE, SUSCEPTANCE AND ADMITTANCE


( F O R A.C. CIRCUITS O N L Y )
SERIES CIRCUIT
REVISION

p, +

V,

V,

Fig 189(a)

Fig 189(b)

The circuit diagram (Fig 189a) and phasor diagram (Fig 189b)
summarise the c2nditions. from which it can be deduced that:

Since

v=

Jm

+ VRI f VR, . . . ) 2 f ( VG + V X Z+ VX,. . .)'


= ~ ( I R+,I R 2 + I R , . . .), + ( I X , + I X , + I X , . . .),
VR,

= /(

= I J ( R ~ + R ~ + R ,. . ) 2 + ( X 1 + X 2 + X 3 . . .)'
1i

But

L- Z--where Z is the equivalent impedance


I

Then
= J ( R+, R , + R , . . . ) 2 + ( X I + X 2 + x 3 . . . 1 2
Also since R , = Z , cos 4, A', = 2,sin 4, etc
and R , = Z 2 cos 412 X 2 = Z 2 sin 4, etc

PARAI.Lti1. CIKClIIl'

I<cv~niorli - - i, I 1, + I , ,
The circu~t d ~ i i g r ~(1;lg
~ ~ u 100~1),111d l > l ~ ~ i \ do r~ i i g r ~(1:1g
~n~
190b) summarise the conditions, from which ~t can be deduced
that:

CIRCUITS: METHODS O F SOLLTION : SPECIAL APPLICATIOSS

fl+1,1= dla,
+ l a , +I,, ) + I I
v
v
= J($ cos ml + - cos 42 + - cos d3.
2,
z3

iI

I=

.I2

'

I'

v
I.'
- sin 4, + - sln 4, +
z2
z,sln 8 ,
v \ / il ( cos
~ 4, + 1 COL 4, + 1 cos B3 l2
2 2
-- 2 3
I .
1
sln $, +,
rln 4, + - sin 9,
/z3

iz",

or I =

j2

G1

--

)'

But if Z is the equivalent impedance, then Z =


whence

I ( l
v z ci z,

V
I
1
or -, = l
l
Z

1
1
dl + cos 4, + - cos 4, .

I - -I -

sln

333

4,

- sin

z,

1
3 :'

R I Xa

)I

z
3

T,

<

Flg 190(a)

Fig 190(b)

I
4, + sln 4,

+
7

)&

334

REED 'S BASIC ELECTROTECHNOLOGY

From the expressions for the equivalent impedances of series


arid parallel circuits. as deduced above, we see a certain similarity
and note that the resultant is obtained from the summation of
in-phase and quadrature components.
For the series circuit, the components involve the known
values of resistance and reactance and thus no difficulty is
presented for obtaining the resultant.
For the parallel circuit, the reciprocals of impedance are
required and it would appear that, if a new name was given to
these reciprocals of impedances and to their in-phase and
quadrature components then, some simplification of the r e l a t i ~ n ships is possible. Furthermore. since the new terms are concerned
with rectangular co-ordinates and their resultant, they must
follow 'known relationships and the pattern of the right-angle
tri;inpltt. This will be shown, once the new terms have been
defined.
1

I,

c;illcd thc .I,/rrrrtttr,rc.c, 01. fhc circuit. cvprcsscd in siclncns

(symbol S) and is denoted by Y. Just as impedance 1s resolved


into the two components, resistance and reactance, so admittance
is resolved into Conductance, denoted by G and Susceptance
denoted by B. Again 'the siemensiis the unit for both components.
The Conductance of a circuit is the extent to which a circuit
admits the power component of current.
- I cos 4
Power component of current -Voltage
V
R
1
G = - cos 4 =

Thus G =
or,

The Susceptance of a circuit is the extent to which a circuit


admits the wattless component of current.
Wattless component of current -I sin 4
---------Voltage
V
1 .
X
or, B = - s 1 n 4 = Z
z*
The Admittance is the extent to which the circuit admits
current hcing , = curreni = -I = -1
\~clltnpc I '
%
Thus B

then

zr1 ('cos 4)
=

(i

sin

4)

CIRCUITS: METHODS OF SOLUTION: SPECIAL APPLICATIONS

!35

and

Y =

JcT2

'

Also, from the expression deduced for the parallel clrcu~t

+ -cos
1

22

4,

+ - 1c o ~ b , ) ~+
z3

($sin 4, + z21 sln 4, +


-

or Y =

--

sin

4,f

z3

d(G1+ G1 + GJ . . . ) 2 + ( B 1 + B2 + B3 . .

.)2

etc

Since there is a right-angled relationship between G , B a n d 1.


it follows that this can be represented by an admittance triangle.
a s shown in the diagram (Fig 191a and 191 b).

TRIANCLf

TRIANGLE F 0 9
CI RCUIT

CAPACITIVE

INGCTIVE
(a)

FOR
CIRCUIT

Fig 191

It will be noted that as reciprocals are involved, and if ille


phasor diagrams of Fig 192a, 192b and 192c, are studied, then the
deduced current triangle when modified by dividing all sides b)

1"

Kv

Ir

- _ - - _ _ _ CURRENT D I A G R A M

Fig 192(a)

- -m
1
3
.

CURSENT

TRIAA

Fig 192(b)

V, will give the admittance triangle. This. for a parallel circuit


containing resistance and inductive reactance, is inverted when
compared with the impedance triangle. This is shown by the
diagram (Fig 19 l a and 19 1 b). For parallel circuits made up from
resistance ar?d capacitive reactance branches, the opposite is the
case and due allowance must be made for the sign of B when
solving problems. This will be illustrated in the examples which
follow, but it must be stressed here that:
the condlrctance, susceptanco ~ 1 1admittance
d
rnethod o f solution
is onlv applicable to parallel circwirs o; to the parallel section of
cr series-parullel circtrit.
Example 115. Consider the parallel circuit shown by the
diagram (Fig 193). Find the total current and its power factor.
Here Z 1 2 = R 1 2 X I 2 = 22 + 32 = 4 + 9 = 13
z~~= R~~ X Z 2 = 32 + 42 = 9 + 16 = 25
Z 3 2 = R3'
x~~= 32 + 22 = 9 + 4 = 13

+
+
+

171-1O H M S

1-x=1

Also

Fig 193

Total G

0.505s

0.237s
+ 5.62
= 10-1~5.6=0.56S
Total B =

1' = \/;C1.505~ 0,2372 = 10-'J25.5


=

10-'dm

CIRCUITS: METHODS OF

SOLUTION: SPECIAL

APPLICATIONS

337

The lagging condition is determined from the fact that the


inductive c f f c c l o f the circuit prctlorn~n;~tcs.
;IS indic;ltcd by the
total value of B = - 0.237S.
Exiin~ple 116. A serits-p;in~llc.lcircuit ARC is m i ~ d eup :IS
follows: a 552 resistor connected, in seiies with a 0.02H inductor with a 100pF capacitor connected in parallel across
these components. This constitutes the parallel section of the
circuit. The series section AB consists of a 1052 resistor in series
with a 0.02H inductor, which in turn is connected in series with
the parallel section. The complete circuit is connected across a
100V. 50Hz supply. Find the voltage across the parallel section
BC. The problem is illustrated by the diagram (Fig 194).

Fig 194
Branch 1

X1 = 2 x 3.14 x 50 x 0.02 = 6.2852


R -50
z,' J 5 % + 6 6 . 2 ~ ~25 + 39.5 = 6 4 . 5

Branch 2

Total G = 0.077%
Total B = - 0.0661s
Thus :
Y = J0.0775~ + 0.0661' = 10-~1/7.75'
6.61'

Z of parallel section

1 -lo - 9.8452
0.1017 1.017

Equivalent R of parallel section = 9.842 x 0.0775


= 97 x 0.0775 = 7.52n
Equivalent X of parallel section = 9.842 x 0.0661
= 97 x 0.0661
.-= 6.42R (inductive)
Total R of whole circuit = 10 + 7.52 = 17.52R
Total X of whole circuit = 6.28 + 6.42 = 12.70R
Total Z of whole circuit = d l 7.522 + 12.7'
= 10 J 1 . 7 5 2 ~ + 1 . 2 7 ~
= 10J3.07
1.61 = I O ~

~ B

loo - 4.62A
Circuit current = 21.65
Voltage drop across section BC = 4.62 x 9.84 = 45.4V.

T H E WHEATSTONE BRIDGE
Although this network has already been introduced as an
example for 'methods of problem solution,' it should be noted
thnt thc condition of no current in the centre arm is of particular
Importance. The bridge In this basic form has been adapted for
methods of measurement and control but the original application, used by H. Wheatstone-a railway engineer, indicated the
conditian of unbalance and was used for signalling. The now
best known adaptation enables resistance measurements to be
made and is considered here in connection with the diagram
(Fig 195).
R

Fig 195
MEASUREMENT OF RESISTANCE

The bridge consist. of a network of four resistors wl~icli.by


the choice of values for three, can be used to measure the v.iilue
of the fourth resistor. R , , R , and R , are the known resistance
valucs. R , is the unknown resistor and G is a centre-zero
ammeter o r milliammeter, called a galvanometer. A cell, battery

CIRCUITS : METHODS OF

SOLUTION: SPECIAL

APPLICATIONS

339

or 'mains' power unit can be used us the supply source. Tllc


bridge, as stated, is used in the 'balanced' mode and, for this
condition, gives a measurement in terms of the known resistors
and is accurate irrespective of variations of the supply voltage.
To understand the theory, letters have been given to the network and currents I, and I, are assumed to flow. The bridge is
balanced by varying resistors R,, R, and R,. Under the balanced
or 'null-deflection' condition, connection BD carries no current.
Thus the assumption of only currents I, and I, is valid, even
though these current values may be dissimilar. Further, it is seen
that since there is no current between B and D , then these points
are at the same potential. Also point A being common to
branches AB and A D then :
the voltage drop across AB = the voltage drop across :AD
. . . (a)
or I,R, = 12R3
Again point C being common to branch el^^ and DC then;
the voltage drop across BC = the voltage drop across DC
or I,R, = 12R4
. . . (b)
Dividing (b) by ( a ) , we have :
IlR2 =
IlRl
I,&,
R
or R, = R , x 2
R1
It will be seen that, if R, is an unknown resistor.-now termed
R x , resistor R, can be made a variable standard whilst H , and
R2 can also be standard fixed resistors of decade value. R ,
and R2 are frequently termed the 'ratio arms' and, by suitable
choice, the value of R4 can be accurately determined for a wide
range.
The Wheatstone-bridge method of measuring resistance
1s utilised for commercial instruments which may appear in
various forms. Generally the resistance value of the variable
resistor R3,-now termed Rs (say 1000R) is shown against a
calibrated scale, whilst R , and R2 values are switched together
to give ratios of say 1 : 100, 10 : 100, 100 : r00, 100 : 10 and
100 : 1 ie times 0.01, 0.1, 1, 10 and 100. Thus, if the switch is on
Yimes 0.01' and balance occurs at 340R, then the unknown
resistance = 0.01 x 340 = 3 . 4 0 .
The range of measurement is thus extended by using ratio
arms of three o r four decades and its accuracy is dependent only
on the accuracy of the components used in the bridge. The range
resistors R , and R, and the variable resistor Rs would be wound
from constantan wire and would be calibrated with precision.
Great care would be taken to minimise the contact resistance of

the sliding contact on Rs and at the terminals of the selector o r


range switch.
Whilst considering the Wheatstone-bridge principle, it should
be noted that other forms of 'direct-reading' ohmmeters are also
available. These show an ohmic value direct on the meter scale
and a variation can employ the bridge network. Thus if the
terminals for connecting Rx are open circuited, the value of Rx
is described as 'infinite'. The meter would be arranged to sive
full-scale deflection-a side-zero instrument is used for filis
application. If standard resistors of known decreasing value are
substituted for Rx,the meter pointer deflection would decrease
and the scale could be marked accordingly until a minimum is
shown when Rx is zero. The scale would not be linear but the
meter would be suitable for everyday work and factory production 'checks.' More accurate'direct-reading ohmmeters, employing electroni~principles,have now been devised, but the bridge
'out-of-balance' mode is still favoured for other measurements
siicli a s temperature, gas-content, strain etc. Note that for thi:
arrangement, accuracy is dependent on the voltage of the
~UPP~Y
T E M P E R A T U R ~ M E A S U R EME N T

Although it is not pI"oposed to describe here, the m'any


m e t h ~ d savailable for measuring temperature, it is necessary to
point out to theyenweer that present-day pethods utilise either
a resistance-thermometer probe or a theirnocouple probe. The
former only is destribed here, as the 'thermo-electric' effect has
not been introduced as yet.
The fact that the resistance value of a resistor varies with
temperature is already known, the-relationship being a straight
line--ref Fig 20. If therefore, a thermometer 'head' ie probe is
connected into a Wheatstone-bridge arrangement, then the
resistance value for any temperature can be measured and either
reference can be made to the appropriate graph o r the scale of
&can (E directly calibrated in temperature units. The resistance
thermgmeter head can consist of a spool of platinum wire
wound onto a ceramic former, the whole being enclosed in an
envelope of special steel. Various forms of 'head' or probe have
been evolvcd for cxtrcmc conditions sucli as furnacc or rcSrigerator work and it should be noted that, thermistors have
illso bcun introduced ; I S altcrna tivc probc clcrncnls lor tcnlpcrilturc mrasurcmcnl.
tiere again, the 'out-01'-.balance' bridge method can be used in
-conjunction with a direct-reading instrument. The scale is

CIRCUITS : METHODS OF SOLUTION : SPECIAL APPLICATIONS


.-

341

calibrated in temperature ullrts und by meiltls o f sclccrclr


~ ~
stvrrr'll
various probes can be connected-in and readings taken. As
before the readings are dependent on i\ constant voltnpc hcinp
available to the bridge and, to check accuracy, a standardising
facility is usually provided. Thus, by means of a two-position
switch, a known standard resistor can be connected in place of a
probe. The pointer should then come to rest at a calibration line
on the scale: If not, a variable resistor in series with the battery
o r power-pack can be adjusted to achieve the required condition.
The switch can next be moved from the 'test' to the 'run' position
when readings, as obtained, can be judged to be accurate.
T H E POTENTIOMETER
The word is derived tiom 'potential meter' and thus the
instrument performs the same functions as a voltmeter but with
greater accuracy because, when a 'reading3% being taken, no
current is being drawn from the potential or e.m.f. source which
is being measured. The potentiometer is not an indicating meter
in the true sense, but consists of a resistance arrangement,
galvanometer and cells which can be used for measurement,
once the apparatus has been set up. Like the Wheatstone bridge,
commercial versions are available but here also measurements
must be made rather than 'readings' taken.
Considcr the dingrrlm (Fig 196). A n i s

rcsistor of h ~ p h

accuracy provided with a sliding contact C AB is wound with a

Fig 196
length of uniform resistance wire so that, when a current flows
through it, the potential dropped across a portion of it is
proportional to the length of the wire contained therein. Thus
if 2V is applied across AB then, if the sliding contact C is placed
a quarter of the way from B, the potential across CB would

be 2 x 4 = 0 . W . Similarly with C midway between AB, the


potential CB would be 1V and so on. The polarity of the
supply across AB is most important. Thus if A is positive, with
respect to B, then C for the quarter way position would be 0.5V
above B. For the half-way position, as shown, C would be 1V
above B.
Consider next, an e.m.f. source of standard voltage value to
be connected across CB, strict attention being paid to polarity.
Such an e.m.f. source is available,-an example being the
Weston Standard Cell. This cell is manufactured with extreme
care, only the purest of materials being used for the electrodes
and electrolyte. The construction and chemicals used are not of
importance here, but the cell can be recognised, in any type of
potentiometric indicator or recorder, by noting that it is enclosed in a sealed glass envelope and mounted so that it is
subject to the minimum of disturbance. A constant e.m.f. of
I.018V can be guaranteed under no-load conditions. Referring
to the diagram, with the change-over switch in position 1, the
standard cell is connected across. CB, a galvanometer being
included in the circuit. The position of sliding contact C will
then determine the deflection on the meter. If C is midway and
the p.d. across CB is assumed to be IV, due to the current from
the main energising battery then, as the standard cell is applying
an ,opposing e.m.f. of 1.018V, a difference of 0.018V will exist
and a current will result, flowing from the standard cell through
the meter to p o p t C, down the resistor-usually called the
'slide-wire,' to point B and then back to the cell. The resistance
section CB will thus carry a current additional to that already
due to the applied voltage across AB. If C.is moved up the slidewire resistor, the meter deflection will decrease until a condition
of nodeflection or balance is attained. This means the potential
across CB is now equal to 1.018V and, if an evenly graduated
scale is provided, this can be marked in voltage, both at the
standardising point and also over the whole scale range in values
proportional to length. Note that the diagram (Fig 196) shows
series resistor R in the slide-wire circuit. This can be varied to
adjust the main current and so alter the potential drop along the
wire and thus bring the scale markings to their correct values.
We can now consider the normal operaiional procedure when
working with a potentiometer. The first step would be to
statidardisc tllc slidc-wire. Tlic chiingc-over switch is put in
position 1 and the slide-wire contact C is set against 1.018V on
the scale. The meter deflection is noted and R is then adjusted
until a balanced condition i s obtained-the scale markings, in

CIRCUITS: METHODS OF SOLUTION: SPECIAL APPLICATIONS

343

vpltiige, are then correct. The next operation would be to move


the switch to position 2. Another external c.m.f. can now be
measured, provided its polarity is corrcct with rcspcct to tlic
slide-wire. A typical application would be the measurement of a
~hermocouplee.m.f, a n d llcncc i t s tcmpcri~turc.
A description of the thermocouple and its associated theory
follow later in the chapter but, it can be taken that, when the
arrangement is used as a thermometer, an e.m.f. is produced
which is proportional to temperature rise. If this e.m.f. is.
measured accurately, the temperature can be determined. Thus
in the diagram (Fig 196), the thermocouple shown, can be taken
to relate to a temperature indicator or recorder and is connected
in when the switch on the instrument is moved into the position
2 or 'run' setting. Position 1 would be the 'test or standardise'
position. Since, for a thermocouple, the relationship between
e.m.f. and temperature is straight line, the scale of the .potentiometer can be marked in X and is accurate,jf standardising is
checked periodically. It should be noted that, a potentiometric
type of temperature indicator is calibrated directly for s use with
thermocouples made up from specified metal pairs, such as
copper and constantan, and such thermocouples must always be
used. Most modern multipoint indicators and recorders operate
on the potentiometer principle, the slider contact being motor
driven, the drive power coming from an amplifier which is fed
from the error voltage. Thus the greater the unbalance the larger
thc powcr drivc : ~ n dthe slider C is driven fast to the balance
point. Once this is reached the drive ceases. Standardising is
carried out automatically at regular intervals to maintain
accuracy.
T H E THERMOCOUPLE
A method of making temperature measurements by electrical
means, has already been considered when the resistance ihermometer was described. It will be noted that for such an arrangement, a battery or power-pack is needed to energise the
Wheatstone bridge network. The advantage of using a thennocouple is that, for fundamental operation, no such .additional
e.m.f. source is required and basic instrumentation consists of a
thermocouple and voltmeter. For accuracy however, the
thermocouple is usually used with a potentiometric indicator.
TEMPERATURE MEASUREMENT

The thermo-electric effect was discovered by Seebeck in 1526.


He joined together one end of a piece of iron wire and a piece of
copper wire by soldering and connected the free ends to a

sensitive galvanometer (Fig 197). O n heating the copper-iron


junction a current was shown to flow ie an e.m.f. was generated.
S e e k c k experimented with various metals and arranged them

GAS

MATER

Fig 197

in an order which showed the current across the hot junction


from the earlier to the later metal. Amongst these metals are:
bismuth, platinum, copper, lead, tin, silver, zinc, iron, antimony
etc. For practical purposes thermocouples are classed into two
main groups (a) rare o r precious metals and (b) base metals.
Because of the obvious reason of cost, the latter are used most
extensively in industry and, with cxperience, alloys havecbeen
dcvclopcd which stl~id'y tllc rcq~~ircmcnts
of a working temperature range. Thus a common usage for measuring the temperature
of electrical machines is the copper/constantan o r co per/
eureka couple. The latter is an alloy of nickel (40 per cent and
copper (60 per cent), the temperature range is 0-40OUC. An
iron/constantan couple is suitable for a 0-800"(? range, whilst
nickel-chromium/nickel-aluminium alloys, when used for a
thermocouple. can cover temperatures up to 1200C.
In practice the hot (common) junction is kept in good electricaI and thermal cbntact by welding together the thermocouple

THERMOCOUPLE
(HOT JUNCTION)

CIRCUITS : METHODS OF SOLUTION : SPECIAL APPLJCATIONS

345

wires and the free ends are kept at a common temphrature, as is


the measuring instrument. The thcrmocouplr m t ~ yhe usrd here
but for most applications i t is inserted into a metal pocket which
must be strong enough to withstand corrosion effects, heat and
pressure. The arrangement is shown by the diagram (Fig 198).
The e.m.f. generated is a function of 'the temperature difference
between the hot junction (H.J.) and the instrument terminalscalled the cold junction (C.J.). The graphs (Fig 199) show the
relat~onshipsto be linear over a part of the temperature range.
Since the linearity is based on the assumption of a constant
cold junction temperature, allowance is made for this by
circuits which compensate for C.J. temperature variation o r by
the provision of bimetal components in the instrument movement.
r
b0 -

nV

,
,

400

,,
/

I 2C.m

e00

TEMPERATURE

LC

Fig 199
C ~ M P E N S A T I N GLE A D S OR C ABL ES . Consider the first of the
diagrams (Fig 200). Since all parts of the ldtrument and
terminals B and C are at the same temperaturs, this section
constitutes an electrical connection only and the cold junction
can be considered to be at point B. If the instrument is located
near the hot junction then the cold junction -temperature will
rise and the temperature difference between A and B will be
reduced thus giving an error in measurement. The obvious
method of reducing this, is t , ~mount the instrument at a point
remote from the hot junction. This is also an ad~jantagefor
instrumentation since the indicator can be placed qt ;I convenient point or grouped with other instruments.

'.

WIRE

Fig 200
If the leads shown in the second diagram of Fig 200 are made
of copper, the cold junction would still be at B and any improvement is only achieved if the thermoco~plemetals are extended
by making the leads of appropriate metals. The leads between
BC and DE are thus known as 'compensating leads' and vary to
suit the thermocouple. Thus a copper/constantan couple will use
copper and constantan wire in the compensating leads. Similarly
iron and constantan are the wires fur an ironlconstantan couple,
r N s m u M t w r A T l o N . A S was stated carlier, measurement can be
made by using either a meter or a potentiometer.
the
former, a sensitive milli-voltmeter of the moving-coil type is used
since the e.m.f. is unidirectional and a thermocouple has
polarity. At the cold junction, the wire from which the current
flows. is called the positive element and that into which the
current flows is the negative element. Polarity is also given to the
compensating leads.
For a copper@nstantan couple; the +ve element is co'pper
and the - ve element is constantan. For the compensating leads
the +ve wire is copper and the - ve wire is constantan. T o
achieve accuracy, the thermocouple responses need to be
registered by a highly sensitive instrument movement ie one
whi,h requires only a small current for full-scale deflection.
Resistance voltage drops are thus minimised and measurement
errors are reduced. Such indicators are delicate and not extensively used for marine work. With the advent of reliable
semi-conductor amplifiers however, the sensor signals can be
magnified and applied to a more robust instrument with a more
substantial movement. The indicator would be calibrated
directly in temperature units and i f the e.m.f. is to be a true
rnc:isurc of thc tc~npcrnturcdiffcrcncc (7;,- Tc), betwccn tllc
hot and cold junctions. i t is essential to keep T , constant o r to
compensate for variations of 7',. Such nuton~aticcold junction
compensation is a feature of direct-reading indicators and is
achieved by introducing a bimetal corrector into the movement
suspension system.

o or

CIRCUITS : METHODS OF SOLUTION : SPECIAL APPLICATIONS

347

Tlic potcntiometric or null-deflection method has already


been dcscrihccl (I:lp 100) and hiis the advan1;lpc l h ; i t ' ; i t b;r l;~ncc.'
i t does not dritw current and hence registers tllc true c.~n.l'. ; i l l
resisti~ncevoltilge drops being eliminated. The method. \vhcn
oper;~[cdmanually, is mucli used in ilidus[rial I;~lx~l';~~ol'ics
!NII
I S not favoured for marine applications. However, when
adapted to function atuomatically, it forms the basis of man)
familiar ship-board installations incorporating well-known
makes of recorder and controller.
Basic theory states that, provided the slide-wire is of uniform
section and material then, the voltage drop along it is proportional to its length. Once the current has been adjusted
to give a known voltage drop over a length BC (Fig 196), the
wire can be scaled over its length in voltage and even in temperature if the thermocouple characteristic is known. The calibration
and operational procedure of a potentiometer have also been
described but, it should be noted that, for temperature measurement, certain additional refinements are introduced into the
instrument. Thus automatic temperature compensation for the
cold junction can be achieved by special circuitry and in line with
modem electronic developments a zener diode arrangement can
be used to replace the standard cell.

CHAPTER 14
PRACTICE EXAMPLES
1

The sides of a triangle ABC are made u p as follows:


AB, resistance of 3Q, AC, resistance of 8Q and BC is a
battery with a constant e.m.f. of 10V and an internal
resistance of 1R. The points A, B and C are joined to a
common point D by resistors BD = 6R, D C = x R , A D =
yR. Find the value of x and the battery current when the
current in A D is zero.

2.

Two batteries X and Y, having e.m.fs. of 8V and 4V and


internal resistances of 1.5R and 3R respectively, are connected in parallel across a resistor of 6Cl. Calculate the
current flowing through each battery and the terminal
voltnge.

3.

A 10V battery of resistance 2R is connected in parallel


with a 12V battery of resistance 3Q. Both batteries are
connected to a 100 resistor. Calculate the current flowing in
the resistor and the current supplied by each battery. Solve
by (1) Maxwell's Cyclic-Current Theorem ( 2 ) Superposition
of Current Method.

4.

A network .,is arranged, as shown by the diagram. Calculate the value of the current in the 8Q resistor (by
K i r c h h o f s laws).
5 OHMS

6.

10 OHMI.

12 OHMS

For lhc circuit shown below, calculiire the value a n d


direction of the current in resistor A.

CIRCUITS : METHODS OF SOLCTION : SPECIAL APPLICATIONS

349

7.

A parallel a.c. circuit has two branches A and B. A consists of an inductive coil, of which the resistance is 3052 and
the inductance is 0.1H. B consists of a 30pF capacitor.
Calculate (a) the joint impedance at 50Hz. ( b ) the power
taken from 230V mains.

8.

A circuit consists of two sections AB and BC in series.


AB consists of two parallel branches, the first being formed
by a 6052 resistor in series with a 50pF capacitor, the second
by a 6052 resistor in series with an inductance of 0.25H.
BC consists of a 100f2 resistor in series with a 0.3H inductor.
If the frequency is 50Hz and thc volt;\gc itcross AC is 5 0 0 V .
find the voltage across BC.

9.

- A parallel circuit consists of three branches A , B and C.


Branch A is made up of 3R resistance in series with 0.03H
inductance. Branch B is made up of 10052 resistance in
series with 400pF capacitance. Branch C is made up of 7R
resistance, 0.02H inductance and 300pF capacitance of all
in series. Find the impedance and equivalent resistance and
reactance of the complete circuit as offered to a 50Hz
supply.

10.

A parallel circuit consists of the following three branches


all in parallel :(a) A non-inductive resistor of 20R
(b) A coil of inductance 0.05H, resistance 552
(c) A capacitor of 50pF capacitance
The whole circuit is connected across 100V, 50Hz mains.
Find the total current taken from the mains and the operating power factor.

CHAPTER 15

ELECTRONICS
A knowledge of electronic fundamentals is now considered an
essential requirement for all engineers, and the student is
reminded that, although electronic engineering is, on its own, a
fast developing technology nevertheless, with the advent of semiconductor devices, its impact on power engineering is a factor
of major consequence. Large solid-state motor starters, controlled rectifiers and static inverters have been introduced into
the ship's engine-room, as items of electrical equipment to
complement the smaller but highly sophisticated apparatus
relating to instrumentation and control such as; monitoring
and alarm systems. supervisory units and data loggers. Because
ol' lllc conlplexity of' such cquipmcnts, specialist knowledge is
required for major breakdowns and malfunctioning but the
manufacturers have gone some way to assist in everyday faultfinding and repair by evolving procedures for checking by
substitution and methods of routine maintenance. Even such
tasks would only be performed effectively by an engineer with
some basic knowledge of the art and, it is to this end that, the
author would encourage an approach to the study of electronics.
An introduction to electronics is made in this chapter, with a
consideration of ?he thermionic diode, related devices and some
semiconductor theory. Further reading of t h e subject in Vol 7
is advised.

THERMIONIC DEVICES
ELECTRON EMISSION

The electrons associated with the atoms constituting a metal,


are considered to be normally confined to the shells, as was
described earlier in Chapter 13. At the surface of a metal, the
'free' electrons in the outermost shells may, by virtue of increased velocity or deflection by collosion, tend to, leave the
surface. They are howeve: immediately attracted back by the
iinbalnnce of the electric field which is caused and tends to set up
it potential barrier. It' the free electrons acquire sufficient
velocity to ovcrcoinc thc ;~tlr;~ction
n f tlic electric ficld a n d tllt~s
break through the potential barrier, the process is termed
c,lcc,tron c~rnission.Electron emission can result from various

iind I S tlicrcl'orc dcscribctl by classifici~tionundcr one of'


the following headings; ( a ) thermionic emission. ( b ) cold
emission. ( c j S C C O I I ~ ~ emission,
I~Y
(J) photo-electric eri~ission.
Tliermionic emission will be considered here in detail. Tlie
others will be mentioned only when such reference is required,
since more detailed study will lead into the general subject
matter of electron~cs,wliicli is much too large to be dealt with
in this book. When a metal is heated, energy is transferred to i t
and tlie electrons are considered to acquire increased and
random velocities which may be sufficient for some to break
through tlie surface potential barrier. Such electrons can be
likened to tlie vapour globules given off from the surface of
water when it is boiling and it is found that, unless some provision is made for drawing off the electrons as tliey are emitted.
tliey will lose their velocities and tend to collect together to form a
SPLIL'P ('ltul-ge. Such a cloud of electrons o f space charge will.
in effect, give rise to a - ve charge which will tend to repel further
electrons and we can thus visualise electrons leaving the surface
of the metal but, being repelled by the space charge which has
built up, returning to the metal.
causcs

T H E V A C U U M DIODE
If a plate is placed near to the source of electron emission and
is m;~clc positive to tlic I;~ttcr,the elcctrons emitted will be
attracted to i t becai~sc01' the +vc ~ ~ t ~ ~ l tii1lC1
~ i l 1115
l , \I)iICC
charge will not accumulate. This prinziple is made use of in the
simple radio' valve and discharge lamp. The valve in this b a s ~ c
form is called a vuc~uu~n
diode. It consists of two electrodes sealed
into an evacuated glass envelope and constitutes a simple form
of electronic device. One electrode is made in the form of a wire
which is heated by a current flowing through it. This results in
thermionic emission from the wire or ,filainent: Tlie other
electrode is in the form of a cylinder which is rriade to surround
the filament and is called the anode. The anode is made +ve with
respect to the filament or cathode and attracts the emitted
electrons. Thus we have current flow from anode to cathode
inside the valve as long as the anode is made +ve with respect
to the cathode, but it should be noted that if tlie anode is made
- ve with respect to tlie cathode, tlie emitted electrons will be
repelled and current will stop. T h m the diode is a unilateral
conducting device ie it will .allow current flow in one direction
only and functions like a one-way valve-hence the term. The
diagrams (Fig 201a and b) shows tlie simple construction and
tlie equivalent electrical circuit.

[g

LOAD

R f SISTOR

tVACUATID
CLASS

CA1Hm

ANODI

lop

EMITTED

BATTERY fOR

THEATINC
CATHOOF

\VALVE
PINS

(a)

Fig 201

(b)

In practice, the cathode can be heated either by passing a


separate current through the filament electrode itself o r through
a heater placed adjacent to the cathode. The diagrams (Fig 202)
show the circuit symbols of the diode valve and its position in
t h c circuit.

t
HtATtD

i t i l c t l TtNsloN
I

VALVE

BATTERY

I
I

VALVE

Fig 202

7
1

1
1

For the indirectly heated arrangement, the cathode consists of


nickel tube on to wllicli is sprayed barium or strontium oxide.
Tlie Iiec~tcr'wire i s Iool7cci insidc i t : \ t i t 1 \~~it:lhIy
insuli~tctlI'rij111i t .
For a.c. radio sets, ;I low-voltage heater-transformer is used to
.nergise this heater and those of otiier valves'.
7'he diagrams (Fig 203) show the test circuit used to determine

- i ~

353

ELECTRONICS

the characteristic of' the diode valve. As already stated, conduction is only possible when the anode is +ve with respect to the
ca~llodca n d tliat the Inore + ve the anode is made, the greater
the number of electrons reaching it ie the larger the anode
current. Tlie anode current cannot increase indefinitely, since
there are only a finite number of electrons leaving the cathode
per second. The anode current thus reaches a saturation value
which is shown on the characteristic at the point where, when
anode voltage is increased bejond a certain value, no further
increase of anode current is obtained and the curve bends over
to become horizontal. T o obtain more anode current, the
cathode temperature must be increased to give an increase of
electron emission.

K
A N O D ~ V O L I A C ~ V,

Fig 203
The valve characteristic ie relationship between anode voltage
V , and anode current I,, is obtained by varying the high-tension
voltage applied to the anode, by means of the adjustable contact
on the potentiometer resistor. The corresponding readings of the
voltmeter and milliammeter are noted and plotted to give the
graph shown. Certain factors for the diode can be derived from
the characteristics in order to use the valve for a particular
application. Thus the diode can be used as a rectifier, but its
internal resistance in the conducting direction would require to
be known. This value can be obt,iined from the characteristic
but it is necessary, at this point. to distinguish between srntic and
dyt1~11lic
operating conditions. In the circuit of Fig 203, the valve
anode is not connected in series with a load, such as a resistor.
The voltage across the anode and cathode is set to a known
value when doing the test-a condition different from that when

the valve is loaded or operating dynamically. This second conditlon will also be examined and so the characteristic is considered as (a) static and (5)dynamic.
( a ) S T A T I C CHARACTERISTIC . The departure from Ohm's Law, in

[lie behaviour of the diode, shows that its internal resistance is


not constant but varies with voltage. If an enlarged part of the
static characteristic, as obtained from the test circuit, is examined it is seen from the diagram (Fig 204) that, for any
particular anode current value there is a definite ratio of voltage
to current. This is the unods (I.(.. rr.ristunc,e. Thus for point P the
BP
OA
d.c. resistance = - - or = - This value would also be
PA
OB'
obtained from the reciprocal of the slope of the line from P

I=.

tlirough the origin is I O B Point P is known as the 'operating


'quiescent point' and the d . c . resisti~nccvalue is rclativc
to this point.
~ > o i t l tor
'

ANON

VOLTAGE

Va

Fig 204

Altliouph oper;ttion of the diode under a.c, conditions will not


be considered to any extent. it is appropriate, at this stage to
introduce the term a.c. rcsistcrt1c.c~or slops rr~si.rrancc~.
This term-symbol r,, will bc rcconsicicrcd whcn opcrc~tionol' thc triodc
valve is being described, and here we merely consider it in
relation to the d.c. value.
Assume that the alternator, shown in the diagram (Fig 205),

355

ELECTRONICS

generates a sinusoidal voltage of maximum value Em,which is


small comparcd with the battery high-tens~on voltage, a s
applied to the anode of the diode.

H.T.
bAllLRI

Fig 205
The battery is assumed to offer negligible resistance to the a.c.
voltage and so an alternating current will flow which is superimposed on the direct current, and the totaldurrent fluctuates
between the limits of a maximum and minimum value. As will be
deduced from the graph (Fig 206), the valve offers a resistance
value which is different to that for the d.c. working. Summarising, it can be said that, if the diode is used in a circuit with
a steady d.c. current then the anode d.c. resistance is used, but
if ihe anode current is varying between limits then the a.c. value
is used. Using the characteristic, note the limits 0 and Q

Fig 206

va

between which the operating point oscillates. The changes in the


voltage and current values can be found by drawing the small
triangle XYZ. Hypotenuse ZX is the tangent to the curve at
point P.
The slope of the graph would give a conductance ---

(::;)

but it is more practical to use resistance o r the reciprocal of the


slope. Thus the reciprocal of the slope o r the a.c. resistance value

zy-.

(r,) is given by - 1.e.

XY

Thus r.
= the ratio given by

small change in anode voltage


resulting small change in anode current

6 1'
o r ra = 2.
5' 1,
The sign 6 is used here to signify a small change and in the
limit it can be written as d Va
(/I,,
I he a.c, resisrirncc is less than the d.c, resisluncc: over [hc
working range and the following example shows comparative
values.
Example 117. The anode current-voltage static characteristic
of a diode can be drawn from the following test values. Deduce
the values of a.c. and d.c. resistance for anode current values of
5mA and 10mA.
P.

Anode current (mA)

Anode voltage (V)

30.5

6 1 8

46 58

68

101
77.5

The I,/ V , characteristic is plotted as shown by the graph


(Fig 207).
Then for the 5mA value
523V --

52.5
5mA
5 x 10-3
= 10.5 x lo3 ohms o r 10.5kO

D.C. resistance-Point A =

A.C. resistance. Assume an a.c. voltage of 2.5V superimposed on


the 52.,5Vfor a standing current of 5mA. Then the anode
voltage varies between 50V and 55V and the corresponding
currcnts would bc 4.6 and 5.5rnA.

ELECTRONICS

357

For the lOmA value:

A.C. rcsist:~ncc. As bclhrc, assume :lri 3 . c . v~11:igc01 2 5 V . bar a


standing current of lOmA the corresponding voltages would be
77.5 $ 2.5 = 80V and 75V. The currents would be 10.6 and

Fig 207
(b) D Y N A M I C CHARACTERISTIC-LOAII
LINE. A valve 1s U S U ~ ~
used with a load which, in the simplest form, can consist of a
resistor in the anode circuit. It is evident that, even though the
high-tension voltage V is kept constant, when the anode current
varies the voltage drop across the resistor R will vary and the
resulting voltage, as applied to the anode will vary. The valve is
now operating under dgnamlc conditions ie with a load and, in
order to investigate the correct circuit conditions, it is necessary
to use the dynamic characteristic rather than the static characteristic. The former can be obtained from a test circuit similar
to that shown by Fig 203 except that a load resistor ~ o u l dbe
included and the voltage drop across this would also be measured
for each value of anode current.
A constructional method can also be used to enable the
dynamic characteristic to be obtained from the static characteristic. Thus refer to the diagram (Fig 208). Let P be a point on

~ Y

the static characteristic. Then I, = OB and the valve voltage


drop equals OA. For the same current, the resistance voltage
drop = I,R. Call this V,, then the high-tension voltage V would
be V, + V , and the point P' would be obtained on the dynamic
characteristic. If the procedure is repeated, the complete
characteristic can be obtained and then used to find the current
and voltage values for any operating condition of high-tension
voltage. It must be noted that, there is a separate dynamic
characteristic for each value of R and that the characteristic
approximates to a straight line as the load-resistor value is
increased.

A'

-VOLTAGL

Fig 208
LO AD LINE . A S shown above, the voltage-drop conditions in the
anode circuit of a diode with a resistor in circuit, can be expressed as V = Va + VR o r Va = V - V R . Since the voltage
drop across the resistor will be proportional to the anode
current. the relationship can be represented by the straight-line
graph which would result from the expression:

If the line is superimposed on the static characteristic (Fig


209). it is seen to have a 'negative slope' ie it slopes in the opposite direction to tlic vi~lvechnrncteristic bcciluse a n incrensc in I,
means a decrease in anode voltage. Such a line will therofore cut
the static characteristic to give a point which shows the respec-

'

ELECTRONICS

359

live volti~gc-dropcondit~onsiicross the valvs and rcslstor I'or tllc


indicated current value. The str~l~ght-line
graph or 'lai~ciline' \\,ill
be dift'ercnt for various vulucs ol' H and can bc used to :~dvantagc
for a specific problem such as that shown below. Thus when the
loading condition for one particular value of R at a specified
high-tension voltage is required. it is not necessary to construct
a dynamic characteristic but merely to draw the load line which
is obtained as follows.
Referring to Fig 209. point A is obtained by assuming 1, = 0.
Under this condition the volti~gedrop across R would be zero
and V, would be the full high-tension voltage V . Point B is
obtained by finding the value of current which would flow if the

Fig 209

anode voltage was reduced to zero ie if the diode developed a


short-circuit between anode and cathode. Under this condition

anode current would be given by I, = -. Example 118 shows


R
the treatment in full and with Fig 209, the two methods of
solution.
Method 1 has involved plotting the static characteristic and the
load line to give the required answer.
~ e t h o d2 has involved deducing the dynamic characteristicits \ d u e s being determined from the .table compiled. Tliis
method is obviously of more'value if the operating conditions
for various values of high-tension voltage are to be considered.
Example 118. Using the test viiues of Example 117. deduce
rlic dynamic characteristic for a load-resistor of 4kQ. Find-the
circuit current for a high-tension voltage of 1 lOV and the
\.olt;~gcdrop ;\cross rhc resistor.
Method I . I f the dynamic characteristic had not been rcquired, this method would give the required answers. Conside
Fig 209. Plot the static characteristic and obtain the load li @
thus:
/
Point A at 110V, since I, = 0

lo = 27.5 x
= 27.5mA. Here the val?.e
4 x 10
resistance is assumed to be zero. The point of intersection gives
the answers. Circuit current = 9.lmA. Voltage drop across
resistor = 9.1 x
x 4 x lo3 = 36.4V. Also V R = V - V a
= 110 - 36.4 = 73.6V.
As read from the graph: V R = 74V and Va = 36V.
Method 2. This involves deducing the dynamic characteristic
which is shown plotted. One point only is considered. Assume a
current of 4mA. Then Va = 46V and voltage drop across the
resistor = 4 x l o p 3 x 4 x lo3 = 16V. High-tension voltage =
46 + 16 = 62V ie a value for the dynamic characteristic. The
following table can be deduced.
Point B =

'

Anaclc c.urrent (mA)


--

Anode voltage ( V )
Ker~storvoltage drop ( V )
Il1sh-ten4lon ~ o l t ' i s er V )

For a h~gh-tens~on
voltage of 1 IOV, the crrcult current rs seen
to be 9 I m A . Thc volt,tge drop acro\s the rcslstor = 1 I0 - 74
= 36V.
L)IOI)L A S A I U : C . I . I I . I E K . The most common usage ol' the vacuum
diode is as a rectifier valve for a.c. 'mains' radio sets. A radio
receiving set can be divided into four basic parts: the R.F.
(radio-frequency) unit, the Detector unit, the A.F. (audiofrequency) unit and the Power Pack. U p to now the high-tension
battery has been considered as the voltage source for the valve
anodes but, for 'mains' sets, it is possible to obtain the d.c.
voltage from a power pack, and for most general purposes a
double-diode in one glass envelope is used. The operation of this
arrangement will be considered in Volume 7 but, i t can be
mentioned in passing that, there is a limit for the electrical
loading to which the normal vacuum diode rectifier can be
o p erated and that, prior to the more recent deyelopments in the
rectifier field, it was possible to extend the working capabilities
of the diode valve by the introduction of gas or vapour. The
inert gas o r mercury vapour, considerably altered the diode
characteristic but allowed i t to function more efficiently by
enabling it to carry a heavier anode loading and reducing the
internal voltage drop and thus the valve resistance. Our studies.
are confined here to the consideration of vacuum valves only,
since the use of hot-cathode, gas-filled valves has decreased
considcritbly will1 tllc dcvclopmcrll of' semiconductor rcctificrs
with superior characteristics. Simple gas-filled devices still in
constant use will, however, be studied in this book.
IONISATION

Earlier in Chapter 13, ionisation was mentioned, when i t was


stated that if an electron is removed from a n atom, the latter
acquires a +ve charge and becomes a +ve ion. The ionisation
of a gas atom is accompanied by a n emission of light, which is
monochromatic ie the colour o r wavelength of the light depends
on the gas being ionised. It is unlike that given off by the tungsten-filament lamp which covers the wide spectrum of visible
white light, consisting of seven colours. Ionisation of a gas is
mainly the result of the collision of fast, free-moving electrons
with gas atoms. We have seen how electrons can be given off
from the surface of a metal by thermionic emission, and how
these electrons can be made to travel in a stream with considerable \felocity towards an electrode, if the latter is made +vc to
the emitter. The higher the p.d. between anode and cathode. the

3 62

R E E D ' S BASIC ELECTROTECHNOLOG)

greater the velocity of the electrons, and if these collide with gas
atoms with a velocity sufficient to remove further electrons from
the parent gas atoms. ionisation takes place with the accompanying of light. The additional electrons, being subjected to the
electric field between ano.de and cathode, accelerate towards the
anode, joining the electron stream and collide with further gas
atoms to maintain the ionisation. We then have a stream of
electrons making their way to the anode or a current flowing
from anode to cathode. The e'lectronic device, whether in the
form of a gas-filled valve o r discharge tube, is thus dependent on
ionisation for its conducting properties. When the electrons
rcacli tlic anode, they displace fucther electrons in the conducting
metal of the circuit and this process explains the current taken
from the mains. which is necessary to maintain the device in
operation. The + v e ions of the ionised gas, bcing vcry much
heavier than electrons, move slowly to the cathode to combine
with the electrons moving round the circuit. The current stream,
in an ionised gas, thus consists of electrons moving from cathode
to anode and + ve ions m0vir.g from anode to cathode. It should
be noted that the $.ve ions, in moving to the cathode, neutralise
any space charge effect and, if the voltage across the electrodes is
too large, they may bombard the cathode with sufficient velocity
to destroy it rapidly. It should be remembered that the mass of a
proton, which constitutes the major part of the +ve ion, is alone
some 1850 times that of an electron and that, in a discharge tube,
the heat generated at the cathode by ion bombardment, under
conditions of ~ o r m a loperation, is sufficient to maintain the
temperature high enough for electron emission to take place,
even though the filament heating current has been switched off.
Sin:e electronics now plays such an important part in practical
everyday electrical engineering, it is essential that some knowledge of related devices is acquired. Above we have introduced
the basics of thermionic emission, ionisation and conduction in a
vacuum and in a gas. A fuller treatment of the operating action
of any practical device is left to books devoted to their detailed
technology, construction, operation and maintenance; the
descriptions which follow are intended to provide fundamental
knowledge only.
DISCHARGE LAMPS
The operation of a discharge lamp depends on the ionisation
of a gas, which can be accomplished by thermionic emission o r
cold emission. The latter is sonietiri~escalled field emissinr a n d
1s ;~cliic\,cdhy the ;~pplic;~tian
o f :I high potenti::' :;;rC:s .::-F

3 63

ELECTRONICS

lamp electrodes. The lamps are thus of two basic kinds, known
as (a) hot-cathode types (b) cold-cathode types. Since the light
given off is mainly in the ultra-violet range of the spectrum. the
tube is coated with a chemical powder coating or phosphor. This
phosphor glows or 'fluoresces' when subjected to the ultra-violet
radiations from the ionisation.
(Low-pressure)
The diagram (Fig 210) shows the basic construction. The
lamp envelope or tube is long c~mparedwith its diameter.
Lengths vary from 300mm to 2.4m, with respective diameters
from 15mm to 38mm. The inside of the tube is coated with the
phosphor, and mounted at each end are the electrodes in the
THE HOT-CATHODE DISCHARGE LAMP

OXIDE COATED
FILAMENTS

T
L0llil-i
TUBE

INTERNALLY

COATCD

%\'/IlCt4

Fig 210

form of small tungsten wire spirals. These spirals may be oxidecoated to assist electron emission, and additional guard electrodes may bk fitted to form the roots for the main discharge.
The tube contains a small quantity of mercury and argon gas
which, when warm under working conditions, is at about
bar or 0.1 Pa, The low working pressure.allows the lamp to
run at a temperature which does not affect the phosphor coating.
The argon gas helps to initiate the discharge t h r ~ u g hthe low
pressure mercury vapour, and although the amount of white
light given off is small, some 65 per cent of the input energy is
converted into radiant energy in the ultra-violet range. The
ultra-violet energy acts upon the fluorescent powder in such a
way that the latter radiates visible light.
At start, current is passed through both electrodes, thereby
heating them, making them suitable for electron emission and

3 64

REE D ' S B A S I C ELEC TRO T EC HN O LO G Y

reducing the voltage value required for starting. The passage of


current ~rlsosets up a small potential between tlie two ends of
each filament, ionising the gas and vapour in the vicinity of each
electrode, thereby assisting strik~ngof the main discharge. When
the electrodes are hot.. the heating current is .broken by the
starter-switch and a momentary surge voltage of 70&1000V is
set up by the choke-coil. This voltage is sufficient to start the
main discharge, which is then maintained by the normal mains
voltage. It will be seen that the choke-coil serves a dual purpose.
1niti:tlly i t gives the high-voltage impulse to ciluse the tube to
strike, and then acts as a voltage dropper to maintain the
correct p.d, across the tube..
Altllougll 'instant-start' fluorescent lamps arc widely uscd
ashore, marine systems still mainly prefer circuits which employ
a starter-switch. The starter-switch may be of the 'thermal' o r
'glow' type but its function is basically to allow preheating of the
lamp electrodes, and then to apply the striking voltage. Action
of the lamp and stsrter-switch may therefore be summarised a s
follows. When the control-switch is closed, the supply is applied
to the circuit and as the starter-switch is closed, the current
flows through the filaments which give copious electron emission
and start tlie ionisation. At a pre-determined time, the startersw~tchopens automatically to interrupt the main circuit. The
collapsing field around the choke induces a high-voltage across
the tube, causing i t to strike. Once the main discharge current
flows, the volt?ige drop 'across the choke results in the correct
p.d. being applied across the tube, and a sta.ble running condition.
It is not proposed to describe the action of the starter-switch
here, since this is more a matter of engineering knowledge than
basic theory. A measure of the efficiency of a lamp may be made
by comparing the lumens put out for every watt put in. Although
i t is not nccessnry here, to define 'the lumen', the efficiency of the
fluorescent 1iin-l~can bc compared wit11 that of tllc tiingstcn
filament lamp, when its efficiency is given as 40-50 lumens/watt.
'The figure for ~ h filt~nicnt
c
Ii~tnpc n n bc tnkcn ;IS 14.- 16 lumens1
watt.
(High-pressure)
This type of lamp is more compact and is mainly used for
out-door instnllntions. I t is not used to any great extent for
m;~rine work ;111d the description given here covers hesic
I'undamentals, 'l'he lanlp rrlay bc ol'(a) the Mercury-vapou~typc
or (h) Sodium t!'pc'. Although the latter are used extensively for

Tt1II HOT-CATHODE IIISCHARGE LAMP

365

ELE C T R O N IC S

road illumination and can be recognised by their golden-yellow


light. they have, to date, had no marine applications apart from
the lighting of docks, wharfs and quny arcns. Since the light
given out is monochromatic, a fluorescent coating would not

achieve c o l ~ u rcorrection. The high-pressul-c n~cn.ury-vnlx>ur


lamp has been used on board ship, to a l~miteddegree, to
illuminate deck working-spaces and is briefly described below in
conjunction with the diagram (Fig 21 1 ) .
CHOKE

m
CARBON
SERIES
RfIISTOR

AUXILIARY
ELECTROD(

----

L-(

Fig 21 1

Since this lamp will operate at the ~lonllal~llilinsvoltage, no


step-up transformer is required. The inner tube is made of
quartz glass to withstand the high operating temperature and
contains mercury with a small amount of neon o r argon gas. The
inner tube is contained in an evacuated, outer, glass envelope
which may be coated on the inside with a fluorescent phosphor.
There are two main electrodes and a starting electrode adjacent
to one of them.
When the supply is switched on, discharge occurs in the inert
gas (neon o r argon) between the main and auxiliary electrode.
This preliminary discharge is sufficient to 'trigger' the discharge
between the two main electrodes by ionising the mercury
vapour. Once the main discharge strikes, the electrodes are
maintained at their working or emitting temperature by the
bombardment of the +ve mercury ions. Having started the
lamp, the auxiliary electrode has no further effect, the current
flowing through it being kept to a minimum by a series resistor
of high ohmic value.
a

A stabilising choke in the main circuit prevents excessive build


up of current after discharge commences. The light emitted is
mostly in the ultra-violet range, hence the need for colour
correction by the fluorescent coating. The main electrodes,
though not heated by a preheating current, are usually made in
the form of a wire spiral which contains a pellet of barium o r
strontium oxide. This assists in electron emission when the
electrodes are heated.
The lamp takes some 10 minutes to warm-up, as the mercury
vi~poriscsand thc pressure builds up to about ititndard atmospheric pressure ( 1 bar). A disadvantage of the high working
pressure is that the lamp will not strike immediately when
wllchrcl oll', and must be allowed to cool down, t o crii~hlcthe
pressure to fall, before the discharge will re-start.
THE COLD-CATHODE DISCHARGE LAMP

The basic principle concerning the operation of this lamp is the


use of a high voltage gradient across the gas to produce electron
flow and the necessary collisions to cause ionisation. The
electrons are initially extracted from the cathodes by the large
applied voltage between anode and cathode and, to produce the
initial ionisation, a 50 per cent higher voltage than the running
voltage is required. When operating, a potential gradient of I to
1.3kV per metre is used, and since the total supply voltage for
some lamps and signs can be in the region of lOkV, a mains
transformer is rtsed. This transformer is designed to have a high
internal reactance so that the voltage 'sits down' to the required
running value, once the lamp has struck.
The lamp, in its simple form, is used for advertising .urposes
and the colour of the discharge is governed by the tT-)e of gas
used. For illumination purposes, mercury vapour is used together with a fluorescent coating on the inside of the tube. The
long tubes (2.4m) with three o r four in series are difficult to
accommodate and the lamp has only limited applications for
marine work. I t is however, used for the illumination and
decoration of saloons and public-rooms on some of the larger
passenger ships.
A more familiar type of cold-cathode lamp is the small pigmy
or sign lamp used for indicator purposes. This is arranged to
work at mains voltage with a resistance in series, the value of
which is some 10 000R. Neon gas at high pressure is used and
the discharge is usually in the fonn of a glow which surrounds
the electrodes. The tiiagriirn (Fig 212) sliows tile colnlnon
iirrallgclnent.

ELECTRONICS

367

CARBON RESISTANCE

01 ?It4W I N HOUND
ON AIbI$TOS BRAID

Fig 212
T H E CATHODE-RAY OSCILLOSCOPE
This instrument has now been developed to suit a wide vari ty
of applications but it should be appreciated that, in its fundamental form, the cathode-ray oscilloscope (C.R.O.) is a device
for measuring voltage both in magnitude and time. By displaying
a luminescent, mobile spot on a fluorescent screen, a trace of the
deflection and/or. waveform can be observed and measured-a
graticule suitably scaled, is usually provided for superposition on
the screen. By the use'of ancillary equipment such as amplifiers
and transducers, any measurable quantity can be converted into
a voltage and is so displayed. Suitable calibration will then allow
the quantity, being checked, to be measured directly in its
appropriate units.
THE C ATH OD E- RAY TUBE (c.R.T.)

The construction of a cathadequy tube is described in rclatian


to the diagram (Fig 213). It may be considered to consist of four
distinct parts :
(1) A system of electrodes, to produce an electron beam of
high velocity and a means of controlling the beam
intensity.
(2) A means of concentrating the beam, by electrodes to give
a sharply focussed spot on the screenb
(3) A deflecting system, to move the focussed beam to any
part of the screen.
(4) A screen, coated with fluorescent material, which emits
light when energised by a stream of high velocity
electrons.
The whole arrangement is enclosed in an evacuated glass
envelope. The system of electrodes and the focussing electrodes
ie parts. 1 and 2 are sometimes referred to as the 'electrode gun'.
A coating of colloidal graphite is connected to the final anode

and terminates at the screen. Its purpose is to allow the path for
electrons to be completed back to the supply source. once they
through the glass and screen phosphors from the
values, shown for the resistor chain, are
do however, indicate the order of such

' X'PL ATES

GRID
OXlM

CATHODE

ANODES

COATING

HEATER

63

ac

fc

kt='

- --

Ai
-

- - - - -

SCREEN

Fig 213
A S for a themionic valve, electrons are produced
from a heatekcathode C, and attracted to an anode A , to which
a high, +ve voltage is applied. The cathode is oxide-coated and
heated by a separately energised heating element. Interposed
between the'cathode and anode is a cylindrical electrode called
G. The name has been borrowed from the
the grid o r r~~odularor
triode valve where the grid is in the form of an open-wire spiral
or mesh and here, although a complete cylinder, it performs a
similar function. The grid, which is used to control the number
o f clcctrons lc:~ving the cathode, is given a highly negative
potential with respect to the cathode. Electrons arc repelled by
the -ve charge on the grid. the degree of repulsion depending
O I I 111c ~ ~ ~ i ~ g ~ 01'~ i lllc
l i ~nsg:~tivs
clc
poiclllii~l.Cotiiplctc 'cut-oll"
ol' the electron beam can t h u s be itch~evedand varying the grid
potential therefore controls the beam intensity i.e, its 'Brightness
or Brilliance'.
The m o d e A , can be in the form of a disc with a small central
hole. hut the more usui~larrangement consists of the disc being
built into the end of :I small cylinder. This cylinder assists in

O PER A T I O N .

ELECTRONICS

369

forming the electrostatic focussing field which will be described


shortly. The electrons, emitted by the cathode, are attracted to the
anode by its high +ve potential and some pass through the
hole at high velocity, thus constituting a beiim. If uncontrolled,
the electrons, being - ve charges, would repel each other and
would diverge to reach the screen in a spreading beurn tlnd
would give a relatively large, dull spot. The beam, after emerging
from anode A , is therefore to be focussed, and this is achieved
by the second and third anodes (A, and A,). These are at high
but differing d.c. potentials, and produce an electric field, of
such a configuration, that the electron beam is compressed and
thus focussed. The high d.c. voltages on electrodes A, and A,
further accelerate the electrons.
An introduction was made to fluorescence when discharge
lamps were considered. As for the lamps, the screen material
consists of phosphors which emit visible light-when excited by
electron bombardment. The electron beam thus results in a
luminous spot which draws the pattern to be observed. Due to
the 'after-glow'-the time-lag in the extinction of the fluorescent
glow, and the persistence of vision of the human eye, the light
spot, when moving, appears to trace a line and thus produce an
image which can be observed. Such an image can be a straightline deflection, a waveform or a recurring figure.
FOCUSSING. The full function of anodes A, and A, can now be
examined. In Chapter 13, electrostatic fields were considered
and it was seen that their forms co'uld be depicted by lines of flux.
The shape of such fields can be altered by varying the potential
differences between adjacent electrodes. When an electron moves
in a region of changing potential, a force acts on it, at any point
in its path, which is perpendicular to the planes of equal potential.
Such planes can also-be depicted by lines, similar to the flux lines
and, if drawn, the overall effect will be to reveal an electrical lens
pattern which functions in a manner similar to an optical lens
system focussing a beam of light. Thus the three anodes A,, A,
and A, form a converging lens in the gaps between the electrodes
and, by adjusting this lens, focussing can be carried out in this
region. It will be seen that the appropriate potentiometer in the
diagram (Fig 213) is marked 'Focus'.
There is also a control marked 'Astigmatism'. This works with
the focus and brightness controls and allows adjustment of the
spot so that its size, intensity and regularity is uniform in any
position on the screen. The quality of the trace and the resulting
picture is thus capable of adjustment.

T O display a picture it is necessary to deflect the


beam in both a vertical and horizontal direction. The object of
the X and Y plates will now be seen. These deflector plates are
arranged in parallel in pairs, each pair being at right angles and
located between the 'gun' and the screen. Viewed along the axis
of the C.R.T., the area enclosed by the deflector plates is square
in cross-section, the length of the sides ie width of plates, being
sufficient to allow a full-screen deflection of the electron beam
on both the X and Y axes. The beam is deflected by the electric
field between the plates when a potential is applied across them,
-the + ve plate attracting, the - ve plate repelling. An nlternating voltage applied to the Y plates will deflect the spot up and
down to give a vertical line.
In ordcr [ o cxilmrnc wi~vcforms and transient chnngcs In
circull cond~tions,i t is also necessary to move the spot sideways.
A deflective effect occurs if the horizontal X plates have a d.c.
potential applied across them. This is achieved by providing a
time-base.
DEFLECTION .

TIME-BASE

This can be regarded as a repetitive pulse of voltage which


can be adjusted. The time-base impulse is usually provided by a
saw-tooth voltage sequence, built up by special circuitry. This
voltage increases uniformly with time until it reaches a value
when it collapses, to restart building up again. The spot accordingly moves uniformly across the screen, flies back a n d again
commences mouing at uniform rate. Additional circuitry allows
'synchronising', by feeding some of the Y .amplifier's output to
the X time-base. This arrangement allows a trace pattern to be
held stationary.
The time-base for a modern C.R.O. is a specialised, involved
item of circuitry, arranged in rnodular form to allow difkrent
time-range requirements to be met. The criteria would be
stability, reliability and accuracy but a simple circuit to illustrate
the principle can be considered. This would be made up fr'om a
small cold-cathode lamp o r tube connected across a capacitor
in series with ;) resistor. When a d.c, voltage is applied across the
combination, the capacitor charges up, with the volta e across
i- e breakthe capacitor plates rising uniformly with time, until t!
down voltage value of- the tube is attained. Thedtu6e t.hen strikes
and the capacitor discharges; with its p.d.-falling to zero. The
process of charge and discharge recommences and a graph, i f
plotted with time, will be of a sawtooth'shape. Thus the voltage
rises unifc>rrnly, I'r~lls to zero, rises again, falls and so on. I S

ELECTRONICS

37 1

connection is made to the X plates, the spot will advance


across the screen from left to right, flies back, moves to the right
and returns, the sequence giving a horizontal straight line. A
y~nusoidalvoltage applicd to the Y plrltes will give :I vertical
deflection and by adjusting the 'time-constant' of the time-base
circuit, to equal the periodic time of the alternating quantity for
an exact multiple of the periodic time, the waveform will be
displayed upon the fluorescent screen as a graph of voltage or
current against time. The time-constant of the time-base circuit
can be adjusted by varying the size of capacitor. and series
resistor.
SOLID-STATE DEVICES
SEMICONDUCTORS

In this chapter, an introduction is made to the semiconductor


diode. This device is comparable to the thermionic diode and as
such is required to function in a manner prescribed by the
associated electronic circuit. It should be appreciated however,
that semiconductor materials have properties which allow their
usage for devices other than those mentioned in this chapter and
to-date they have been developed to respond to the effects of
temperature, pressure, stress, light-energy, magnetic and electric
fields. As such they can be used as sensors and transducers.
In general the term 'semiconductors' can be taken to include
all solids whose electrical conductivity lies between that of metals
and that of insulators. Germanium and silicon are the most
Important semiconductor mnteriuls but before proceeding to
investigate their electrical properties, it is of interest to recall the
history of semiconductor development for electrical work.
The rectifying properties of certain substances were known
during the last century and, early in this century, the advent of
radio saw both the thernlionic valve and rectifying crystal being
used. The latter was a form of natural semiconductor diode. but
was displaced in favour of the valve which, in time had been
evolved into its triode form and, could be used for detection.
rectification and amplification. The crystal again became
popular as a detector in the early days of broadcasting because
of its cheapness and the device using it, was most commonly
known as a 'cats-whisker'. Use of the crystal lapsed again as
valves became more readily available and were adapted for
'mains use, but it was redeveloped for use in radar during the
last war. The crystal diode as used till then was known as the
"oint-contact rectifier' and the first 'junction' diode way ~volved
9

in 1941. Impetus was given to the use of semiconductors when


the crystal triode was developed in the Bell Telephone Laboratories in America in 1948. Since then intense research projects
investigating semiconductor theory, development and applications have been launched by all electrical firms and scientific
bodies, resulting in an ever-expanding field of usage. To-date the
most remarkable achievements have been in the fields of electronic automation equipment, electrical power control devices
and in the miniaturisation of electronic apparatus.
To understand the action of a semiconductor device it is
necessary to repeat work already covered in Chapter 13, since
reference will be made to the electron theory and the conception
of an electric current as being due to the movement of electrons.
The csscnti;~lsof the fundamental concepts are therefore summarised as an introduction to the basic semiconductor theory.
BASIC THEORY

The atom of a material consists of a nucleus (+vely charged)


and planetary electrons ( - vely charged). The nucleus has nearly
all the mass of the atom and consists of protons and neutrons.
Neutrons have no electrical properties, but since forces of
repulsion exist between the +ve charged protons, then the
neutrons ha* the function of holding the nucleus together. An
atom. contains equal numbers of protons and electrons with
charges cancelling to make the atom neutral. The atoms of
various elements contain different numbers of protons, neutrons
and electrons a t d the greater the number of protons and
neutrons in the nucleus, the greater the density of the substance.
The electrons move in orbits and for the larger atoms, these
electrons are arranged in shells. The chemical properties of an
atom may be explained in terms of the number and grouping of
the planetary electrons. The shells--called 'quantum shells,' are
considered to be concentric and seven in number, distinguished
by the letters K, L, M, N,0 , P, Q. The K shell is closest to the
nucleus and the appropriate maximum number of electrons
possible in each shell is 2, 8, 18, 32, 18, 13 and' 2. Since germanium will be of particular interest in our studies, i t should be
~~<)tccl
tllat it:, 32 clcctruns i\rc arr-iinged in the K, L, M and N
shells thus: 2, 8, 18 and 4. Silicon, also of interest, has 14
electrons arranged in the K , L and M shells thus: 2, 8, 4.
A solid in its srnallcst particle form, if investigated, is found to
consist of a large number of crystals joined together and the
crystals are built up from a regular structure of atoms which
rcpe;\t.; ~ t s c lcontin~l;~lly
f
to fornm ;I Iiitticc. The electrons in the

373

ELECTRONICS

innermost shells of an atom do not appear to play an active part


in the crystal lattice structure, but those in the outer shells are
important, in that they decide the electrical and chemical
properties of the substance and are known as 'valence electrons.'
The chemical nature of an atom is revealed by the manner in
which it combines with other atoms to form molecules. Valency
is a chemical term and is, explained as, the property of the atom
of an element which enables it to enter into chemical combination
with another atom. Thus when the molecules of a substance are
formed from two atoms, this is done by the electrons in the
external quantum shells or by the valence electrons. The valency
of an element is determined by the electron disposition in its two
outermost shells. Note that two shells are concerned because a
combination of atoms is achieved by the gaining o r losing of
electrons, so that it appears that all shells are complete. Thus we
see that the planetary electrons of an atom can be divided into
two classes: (1) valency electrons o r those aflilable for linking
atom to atom and (2) the core or those which retain their configurations in all compounds of the element. These are not
affected by the formation of linkages.
CO-VALENT BONDING .

One way in which an atom may combine


with other atoms and bring about a change in the number of
atoms in its valency shells, is by co-valent bonding. This means
the sharing of a pair of electrons by two atoms, each atom
contributing one electron to form a shared pair. A powerful
bond between atoms thus results.
Consider hydrogen which tins one plnnet;~ry or valence
electron. The K shell electrons should be 2. Thus the hydrogen
molecule contains two atoms, with the nucleii linked by the
valency electrons to form a pair. The arrangement is illustrated
by (a) the first diagram of Figure 214.

8
'

- - - e - ---

Fig 214

O N M
~ OL~CULE

(b)

374

REED 'S BASIC ELECTKOTECHNOLOGY

Consider germanium where the outermost shell of an atom


contains four electrons. To obtain stability, the atoms of the
element build up into a molecular structure giving combinations
as shown by (b), the second diagram of Figure 214. It will be
seen that one valence electron of the four goes towards making
up a 'shared pair' between two adjacent atoms.
The shared pairs of electrons can be depicted by double lines
as shown by (a), the first diagram of Fig 215 o r alternatively
the crystal structure which leads to a regular arrangement of
atoms throughout the lattice, (in three dimensions) can be
represented by (b), the second diagram of Fig 215. Germanium
is considered us an obvious cxnmple.

(a)

Fig 215

(b)

CONDUCTION CONTROL

Germanium and silicon, in the pure form, have crystalline


structures and the atoms, arranged in a crystal lattice,-as
depicted in Fig 215b, show these materials to be similar to
carbon in its diamond form. The arrangement, as illustrated, is
frequently called a 'diamond lattice' and it is seen that there are
no free electrons except for any shaken free by the application of
energy in the form of heat or light. This effect comes under the
general heading of ionisation, which is considered later. The
conductivi\y of pure germanium, for example, &thus poorcscept when hci~ted,but i t can be improved by injectigg an
impurity. For senliconductors the fact that conduction is a
ftinction o f (;I) tcmpcriltiirc and (b) impurity content, is a most
important property. In general an increase of (a) o r (b) will
increase conductivity which is now considered with these effects
in mind. At -273'C pure germanzurn, (s:iil
being taken as an example), has no free electrons since all tilt.
valence electrons are attached to the% respective atoms. If !i;c

I N TRI N S I C CO N D U C T I V I T Y .

ELECTRONICS

37 5

crystal is supplied with energy such as heat or light, the bound


electrons can absorb some energy and escape from the atom.
Through this 'diusociution,' clectrons leave the_,atoms and
wander around the crystal and conductivity will incrcuse us more
energy is supplied. The places from which the electrons leave are
known as holes. If thc crystal is subjccrcd to a yotentinl gradient,
a current will flow, the electrons and holes acting as charge
carriers. The idea of the holes moving will be considered shortly,
but using this assumption, the holes are regarded, as + ve charge
carriers moving towards the negative terminal while the electrons,
being -ve charge carriers, move towards the positive terminal.
Thus although electrons move in the opposite direction to that
of conventional current flow, hole movement is in the same
direction. Intrinsic conductivity is therefore associated with the
movement of equal numbers of oppositely charged carriers.
IMPURITY (EXTRINSIC) CONDUCTIVITY. A S akead y stated, the
conductivity of a pure semiconductor material can be controlled
by the addition of a known impurity such as an'timony o r
indium. The process is known as 'doping' and gives a result
which can be considered under one of the 'two following
headings.
N-TYPE GERMANIUM . When a pentavalent (5 valence electrons)
atom, such as that of antimony o r arsenic is introduced into the
pure germanium (or silicon) crystal lattice, one of the electrons
of the impurity atom is free to become a - ve charge carrier.
This is shown by diagram (a) of Fig 216. The pentavalent atom,
when bonding with germanium, only utilises 4 vulcncc clectrons
and thus one electron of the impurity atom is not held by
covalent bonding. This electron has sufficient energy to migrate
through the lattice structure as a charge carrier. The impurity

Fig 216

atom is a donor o r an N-type impurity since it provided a n


electron. The adulterated germanium is known as N-type
germanium.
If a trivalent impurity atom such as that of
indium or aluminium is introduced into the crystal lattice, then
bonding is imperfect in that a 'hole' is left in the lattice. The term
+ve hole, is given to the deficiency which is created in the
bonding arrangement, since the valence electrons of the impurity
atom only pair up with three of the electrons in the neighbouring
4 germanium atoms. A hole exerts an attractive force on and will
'cnpture' any electron which. having been liberated from a
gcm~aniumatom by some ionising effect, happens to pass near
the hole. Thus, as an electron moves from the orbit of one
gcrnli~niu~nittom to fill a deficiency in another, i t leaves a
similar 'hole' in its original orbit. The hole will then appear to
move in the opposite direction to the electron and as the process
is continuous throughout the germanium it constitutes a n
effective flow of + ve charge carriers. The arrangement is shown
by the diagram (b) of Fig 216. A 'hole' has the characteristics
of a +ve charge and behaves as such and can be regarded as a
free, mobile, +ve charge. Trivalent impurity atoms are called
oc,c,eptors. since they accept electrons from covalent bonds and
in doing so they create holes o r +ve charge carriers. They are
termed P-type impurities and the germanium, containing such
an acceptor impurity, is known as P-type germanium.
I t slio~lldbe noted that what has been said about germanium
also applies to siliton since the latter also has 4 valence electrons.
P - TY P E G E R M A N I U M .

IONISATION. In addition to the charge carriers ma'de available by


'doping', the reader is reminded of the fact alreadydiscussed, that
electrons can be liberated by the action of heat, light o r other
radiations. This process comes under the general heading of
ionisation. I t must be remembered that the total charge in a
given piece of' material is always zero, since for every charge
carrier produced by 'doping' or ionisation, there is an incomplctc atom or ion carr\,ing a charge of opposite sign. Some
dissociation or breakdown of the covalent bonds can occur for
I,otli 1'- and N-type materials, even at room temperature and
thus a few holes could occur in N-type material with a corresponding number of extra free electrons. Free electrons would
thus predominate in N-type material and are called the ~ n r r j o r i r ~ .
carriers, whilst the holes are known as the minori[v carriers.
S111111;1r.l>
I'or P-type material, holes would be the mi~jority
c,;lrrtc*r\ .lnd any frcc clcctrons the minority carriers.

377

ELECTRONICS

T H E P-N JUNCTION
By such a junction is not meant the joint between two pieces of
germanium, one being P-type a n d the other N-type. T o produce
i i ~ n ~ to snsurc
an clTcctivs junction, one piece c ~ l ' g c r ~ i ~ i ~isnyrclwn
a suitnble continuous crystal lattice. During the growing procyss.
P- and N-type zones are created by 'doping,' with a detinite line
of demarcation between them. Since one side has a preponderance of holes and the other of electrons and, because both sides
are electrically neutral, there is no tendency for electrons to flow
and neutralise the holes. Majority carriers will however tend to
drift across the junction in an attcmpt to even out the concentration. As soon as electrons flow from the N-type to the
P-type material or holes pass from the P-type to tlie N-type
material, the latter becomes positively charged and the P-type
material becomes negatively charged. This effect immediately
prevents any further flow of charge carriers aeross the junction
and can be looked upon as the build up of a potentizl barrier.
This potential barrier can be likened to a battery connected in
tlie sense shown by the diagram of Fig 217.

7
P lYPt

-rely CHARGED

N lYPt

POltN71AL

- ~L~CIRONS

ACROSS
BARRIER

+ HOLES

Fig 21 7
For the arrangement shown, few charges can cross the barr~er
unless they have an energy level high enough to overcome the
potential. The +ve charge of the N-type will however attract the
minority carrier electrons from the P-type side and the - ve
charge will attract the minority carrier +ve holes from the

N-type side. The resulting current I = IF - I? will be zero and


since there is no external voltage applied, the junction will be in
equilibriun~with charges and potentials as shown.
THE JUNCTION DIODE

There are two types of semiconductor diode, the junction


diode and the point-contact diode. An example of the latter was
the crystal---cat's whisker. as used in early radio receivers and,
although a modern version of the arrangement is used, it will not
be considered in any detail as its applications are specialised.
Attention here, is therefore given to the junction diode only.
FORWARD BI A S . (Good conduction). If an actual battery is connected across the junction with opposite polarity to that of the
barrier, tllc t , i ~ r r ~ e~r ) o ~ u n t i is
u l rccluccd, t l ~ ccllctivc rcsisli~frcc
decreases and a current of several milliamperes can Aow. A
forward voltage of 0.2V producing 4 to 5mA. The barrier
potential having been reduced, the majority carriers in the
P-type material are reinforced by the +ve potential and the
forward current increases, overcoming the reverse current. The
forward current is thus considerably larger than the reverse
current or I = IF - IR.The conditions being discussed are
shown by the diagrams of Fig 218.

POlCNTlAL

DI51RIBUl I O N
!

Fig 218
(Very poor conduction). If the external battery I S
rcvcrsed ic connected across the P-N junctiori so as to assist the
barrier potential, then tlic latter will be increased. As shown by

RE V E R S E BIAS .

ELECTRONICS

379

the diagram (Fig 219). the action is equivalent to increasing the


effective resistance of the junction and it is 'reverse' o r back
biased by the battery. The reverse current is only of the order of
a few microampcrcs duc to clcctrons rclcased by ionisation ic
Yntrinsic conductivity'. For this reverse-bias connection, the
t,
minority carriers are reit~l'orccd2nd the verse c i i r r c ~ ~:~ltl\ougl\
small, predominates and remAins at a substantially constant
value for a wide range of impressed reverse potentials. The
ability to withstand a high reverse potential is an important
feature since this Peak Inverse Voltage value may be required to
be in the region of 1000V.

Fig 219
Avolding involved deta~lsof the junctlon diode action. ;i clcar
explanation 1s possible if we consider one side of the junction to
have only holes as charge carriers and the other side to have on]!
electrons.

Fig 220

The arrangement is shown by the diagrams (Fig 220). If a


potential is applied so that the P-end is + ve with respect to the
N-end then a current due to this potential consists of either
electrons moving from right to left or holes moving from left to
right. Since the N-side contains free electrons and the P-side
contains holes, current readily flows across the junction.
If the applied voltage is reversed, the appropriate direction of
current would occur either by electrons moving from left to right
and there are no electrons available, o r by holes moving from
right to left. There are no such holes available and therefore
currc'nt c,innot floiil,
I ) I O I > ~ cH,iuAc..ri:nIs.rIC..

11' ;I tcst circuit is arranged, as in the


771;1. ; I I ~tlic results c ~ l ' i~pplicdvnlti~pe;ire
plotted agalnst diode current, tllc ~e~nicontluctor
junctlon is seen
to act like a thermionic diode when the latter is conducting. The
general shape of the current/voltage characteristic is also shown
by Fig 221b. It resembles that of the vacuum diode, but it
should be noted that when the applied voltage is reversed, a
minute reverse current will flow. As mentioned earlier this
reverse current is due to the release of electrons by ionisation ie
the thermal agitation gives the P-type side a few electrons and
the N-type side a few +ve holes. If the reverse voltage becomes
too great. there is a breakdown of the covalent bond structure
and the current increases very rapidly. This is known as the
Zener qffrc,r.
Nore. For a erni icon duct or diode, the electrode to be connected to the +ve terminal of the supply for forward conduction, is always clearly marked. As for the diode valve. the terms
anode
cathode are used accordingly
-I lie and
characteristic shown is typical for a small semiconductor
diode. Note the change of current scale for the reverse graph.
c l i : ~ ~ t . ; ~()I'
~ l i Fig

which shows the limit where the saturation value of the reverse
current is broken down-the '7ener effect.' and the semiconducror hccomcs ;I conductor. IJnlc55 rhc cfrc'cct is lilnitcd thc
material is ruined
thc crystal srructul.~of' I l ~ cycnl\:~niu~n
is
destroyed. Unlike the \,acuurn diode this reverse operation of t h C
~emiconducrordiode I S csploilcd Ii)r L . ; I S I O ~ I ~C I S C I I I [ S IIIIJtllc
Zener Diode, as it is called, is used for voltage regulation and
stabilisation. for meter protection, cut-off, limiting and clipp~ng
circuits.
RECTIFIER OPERATION

Since the junction diode requires no heater supply, it can be


introduced directly into circuit arrangements to allow either
half-wave o r full-wave rectification. The characteristic shows the
device to be suitable for a rectifier and if an alternating voltage
of about 1 volt (peak to peak) is applied to P-N junc:ion. the
potential barrier is alternately strengthened and weakened to
allow the rectifying action. The reverse current can be considered
as negligible. Silicon is used in preference to germanium for
power rectifiers since i t can carry large currents and can operate
at liigller temperatures. Its reverse current is also lower than that
for germanium with similar forward current values.
The ad\,anrages of sem~conductorrectifiers over the earlier
type of 'metal' rectifiers such as the copper-oxide and selenium.
lie chiefly in their smaller size, longer life and greatly improved
regulation and efficiency. 7 he latter features are the result of t h e
low I'orui~rdr-csist;~ncc;111dvoll;~gcdrc~p;lnd t l l c I ' ; I C ~ t11;1t I'c\\cI
elements are required In series to handle a given voltage.
Example 119 The folloulng vnlues refer to il gcni1;inlum
diode.
Forward current
(mA)
0 0.1 0.18 0.22 0.4 0.6 0.8 1.1 1.6 2.3 3.45
Forward voltage
( mV)
100 140 160 180 200 220 240 260 280 300 320
- 0.3 -0.4
- 0.4
-0.2
Reverse current (PA)
Reverse voltage (mV)
- 200 - 400 - 6,
- 800
Plot the anode characteristics for the above diode and
determine from i t ( a ) the 'forward' d.c. resistance when the
current is 3mA. ( b ) the forward anode voltage when the d.c.
resistance is 2000.
( a ) From the graph of Fig 322 the forward d.c. resistance

( b ) Construct the 200R d.c. resistance lines thus. Assume a


current of ImA. Then the applied voltage would be
I x 200 = 200mV. Plot this point P. Draw the voltage
drop line to cut the curve. The forward anode voltage
would be 272mV.

Fig 222
It has been seen from the
I, I', curve that, as for the themionic diode. there is a definite
ratio of voltage to current for any particular anode current
value. A \ shown by the preceding example, this is the anode d . c .
r-e\15tancc,determined for any one point on the characteristic by
d~\.~ciing
the anode-voltnge value by the corresponding ;\nodecurrent \slue. Operation of the semiconductor diode for a.c.
ccjnd~c~ons,
;IS when used ; I S ;I 1i.F. demodulator or detector,
will 1101 be considered liere but, mention is made in passing that.
the slope or a.c. resistance value for any working range can be
determined. a s for tlie \~acuumdiode, by the riitio:
small
change in anode
voltage
- ---result~ngsmall change in anode current
\\'l~en u w d a s ; I rectifies. the diode is loaded or is oper;lting
I I I I I ~ I I I I t I \ therel'c)re apparent that a d\,namic char-

S T A TI C A N D DY N A MI C O PER A TI O N .

acter~sticshould be obta~ned e ~ t h e r by direct testlng o r 17)


dcductlon In a manner c~milarto that u ~ e df o r thc d ~ o d e\ , ~ l \ c
Y i ~ i c ( *;I s r p ; l r ; ~ f c( I y n ; ~ r ~ i ~i c~ l ~ i ~ r ; ~ Ic\ tr c qr ~~i ~ rt v~/.or
( Ic c;1<.1i\,ti I I ~ ,
OI I<
[ l i e l o ; ~ d ~ C \ I ~ ~ ; I I I C C , 1 1 1 ~ 1 1 LI\C 01' ;I 'III:I(~ IIIIC ~ \ o ~ i i IIC:( l

oitcn more approprlatc for sol\,~ngsome problems. 'I 1115 I \


illubrr.;~rcdh y t llc I ; ~ l l o \ v i n g c \ : ~ ~ i ~ l ~ l r
Example 120. The characteristic of a germanium diode I \
shown by Fig 223. If the value of load resistance is to be IOOR
and the average value of the applied voltage is to be 2V, find tlie
average value of the terminal voltage and the load current.

20

IS

6
U

;lo
a

1 .5

K)

VOLTAGE

(v)

The characteristic is plotted as shown and the load line drawn


for lOOn thus:
With n o current flow the full 2V would be applied across the
diode and point A is obtained. With a diode d.c. resistance of
zero, the maximum current which could flow would be
- - O.02A or 2OmA. Thus point B is obtained. The polnt of
100
intersection between the load line and characteristic shows that
. . .
. .. . . .
-

CHAPTER 15
PRACTICE EXAMPLES
1.

The anode dissipation of a diode valve is 336mW. The


anode to cathode voltage is 160V. Find the anode current.

2.

Example 117 shows that when an anode voltage of 58V is


applied to a two-electrode valve the anode current is 6mA.
Cnlculnte the powcr dissipated at the anode.

3.

When the linear part of the I,] V , characteristic of a diode


v ; ~ l v ci s cx:rmined. the slope is found to be 1 ~ 6 4 m A ] 8 ~ 1 3 V .
I-'~ndthe a.c. resistance 01' the valve.

4.

The I,/ V , characteristic of a diode valve shows that a n


increase of anode voltage from 75V to 129V increases the
current from 12mA to 22mA. Find the a.r resistance of the
valve and the d.c. resistance for each voltage condition.

5.

The I,/ V, curve of a diode valye is given by:


5
10 15 20 25 27.5
Anode voltage (V)
0.6 2.0 4.2 7.25 10.5 12
Anode current (mA)
Plot the curve and find the a.c. resistance over the straight
line region.

6.

A vacuum diode has the following forward characteristic :


Anode-cathode voltage (V)
0 10 20 30 40 50
Anode current (mA)
0 25 60 100 145 200
This diode is placed in series with a 3 0 0 0 resistor load
and the combination is fed from a 60V d.c. supply, connected so as to make the diode conduct. Determine the
current that flows and the power dissipated in the load
resistor.

The current in a certain thermlonic vacuum dlode I S


related to its anode-cathode voltage by the expressron
I, = V , 3 i 2 .The current 1s in milliamperes when V , is In
volts. The diode is connected via a 1kR resxstor ta a 4 :
supply. Plot the dynamic characteristic showing t L e zriat~on
of anode current against supply voltage for ~ c e1-8,
0-10V What 1s the value of anode current whez tne ;;

b
.

. ~ c r o \ \ 11112 C I ~ C I I II\ ~ XV7

8.

A diode \~rlvchas the following cliaracteristic:


Anode current
( mA )
0 5.5 13 23 33
43
52
59
03
Anode volt:~pc
(V)
0 25 50 75 100 1
3 150 175 200
Tllc virlvc I S c o ~ ~ n c c ~111c sdc ~ . ~ wi~ll
c s :I I . C S I ~ I01'O I10h11
'
lo
a 240V d.c. supply. I f a reslstor of 50kR is connected between the anode and cathode o f the diode. determine the
current through the diode.

9.

On test. a f u l l - w a ~ e ,s111con semiconductor rectrfier 1s


found to glve a constant forward voltage drop of 0.7V,
independent of current. Calculate the power dissipated in
the rectifier diode for a d.c, current of 5A. If the rectifier
dissipates heat, at the rate of 0 . I J's,"C rise in temperature,
find the maximum d.c. output current ratingif the temperature rise is to be limited to 70-C.

10.

The characteristics of a typical semiconductor junction


diode is as follows:
Forward voltage ( V )
0 0.25 0.5 0.75 1.0
Forward current (mA)-at 2.5 C 0 0.5 1.5 4.8 8.75
Forward current (mA)--at 60 C 0 1.0 3.0 6.5 1 1.5
Reverse voltage (V)
0 0.2 0.4 0.6
Reverse current (PA)--at 25?C 0 1.5 1.5 1.6
Reverse current (PA) at 6 0 C
0 10
10.25 10.5
( ; I ) I f ii voIt;ipe of I V I S applied ;Icroys t he diode :inti :I
scrics rcsislor ol' 100R. find tllc loid currc~i[varlatlotn
between the temperature limits shown by the graph.
(b) Find the variation of its d.c. resistance over the ri1n.p ol'
25C to 60C for the above load condition.

SOLUTIONS T O PRACTICE EXAMPLES


CHAPTER 1
1.

Let R be the equivalent resistance of the parallel arrangement

Volt;~gcdrop iicross thc ;lrr;1ngemcnt = 8.6 x 0.952


8.19

8.19V Ana

Current I, in 2R resistor = - = 4.095A

Ans.

8.19
Current I, in 4R resistor = ---- = 2.0475A Ans.
4
8'19 = 1,638A Ans.
Current I, in 5R resistor = 5

Current I, in 10Q resistor = 8'19 = 0.819A Ans.


10
Check. Total current = 8.6A Ans.

Fig I
2.
For Group A . Let R A = the equivalent reslsrance, then
I - -1 + -I + -1 = 6 + 4 + 3 o r R
24
4 =
= issn
H , 4 6 8
24

;,

387

$011JT10N9 TO PRACTTCF FXAMPLFS


-

- - --

For G r n u p B: Let R ,
1 - -+-=
I : - .' -+ I
~R,
7
6
6

--- - --

--- . -

the equ~valentresistance, then.


3
= - 0 r R = 2R
6

Voltage d r o p across group A


Volt:tgc d r o p across group B

=
=

1.85 x h 23
2 x (I 23

=
=

1 1.53L' Anb
13.46L' An,

CI:cl(.X.Total \,olta_ged r o p = 33.99 = 24V

An5
4 OHMS

1
6

OHMS

OHMS

-t

b OHys

8 OHMS

A
24

VOLTS

Fig 2
Current in resistors. G r o u p A

11.53

Current In

2.88A

Ans.

Check 6.23A
Group B

Ans.

resistors.

12.46
= 4.153A
3
Check

6.23A

Ans.

Ans

3.

F r o m Q1 the e q u ~ v a l e n tresistance R of the load = 0.95R


T h e total resistance of the circuit = 0.95 + 0.65 = 1 . 6 0

388

REED 'S BASIC ELECTROT ECHNOLO G Y

12
3
= - = 7,5A Ans
1.6 0.4
The term~nalvoltage
- = 7.5 x 0.95 = 7,125V Ans
7 125
Current In 5 0 reslstor =
= 1.425A Ans
5

The circuit current

--

--

4. ( a ) Ammeter with shunt.


Voltage drop across parallel arrangement for full-scale
= 0.15-4
deflection = 10 x 15 x
Current to be carried by shunt
6
= 25 - (15 X 1 0 - j )
= 25 - 0.015 = 24.985A
0.15 - 15 x
Resistance of shunt = ---- 24.985
2.4985 x 10

= 0.006R Ans.
(b) Voltmeter \vifli wries resistance.
I<csist:~~lcc
ol' instrument circuit to drop 500V

.:

3 3333 x loJ = 33 333Q


Ser~esresistance to be added = 33 333 - 10
= 33 323R Ans

5. Parallel section BC has

:I resistance,

glven b)

FOLUTlONS TO PRACTICF FXAMPI FS


- ---

-- -

3 8'9

Total resistance of network = 60 + 20 = 80kR


240
Current t,~kenby network =
= '4 x l o - ' dmpere\
XO OM)

= 3 x

10 ' A

(11

3mA

Volt;~gcdrop ,)cross wctlon RC


=
=

10 ' 20
\,)I[\
3 X 20 = 60V
re'id~ngon \oltmeter 4115

Vote. When the voltmeter reslst3nce I S comparable with that of


the resistor across which it is connected. then the above reading
will give a false indication of the actual voltage drop in the
working circuit. Thus. if the 60ki2 and 40kR resistors were part
of a resistor chain in a radio set, the actual voltage drop across
40
RC would be - x 240 = 96V and a \pec~alboltmeter, t;rk~ny
100

In erect no current such a s a v a l v e \,oltmetcr. \\,auld hc uhccl


The problem however. is applicable to t h ~ sctiapter's work.
bcing a network problcm a s such.
6.
Let E = e.m.f. of the battery and R i its Internal reslstancr.
then E = 0.18 (10 + R i )
..a
and E = 0.08 (25 + R i )
. . . b.
E q u a t ~ n g( a ) and ( b l 0.18 (10 + R , ) = 0.08 (25 + R , )
or 1.8 0.18 R , = ? c 0.08 R i
(0.18 - 0.08) R i = 2 - 1.8
or 0.1 R , = 0.2
or R , = 2 0 4 n s
Subst~rur~ng
In E = 0.18 (10 + 2 )
= 0.1s x I:!
= 1.16V Ans.

7. PD across 4 R resistor = 4 x 1.5 = 6V


T h ~ sis also the volti~ged r o p across the other resistors in
group A
Current in 2 0 reslstor =

2
6

= 3A

Ans.

Current in 6 0 reslstnr = -

1A

Ans

6
-8

0.75A

Ans.

Current in 8 0 reslstor

70 ~ o l t , ~d&r o p acres, g r o u p B

urrent in 1 0 0 reslstor

6 x 6 25 = 37 5V A n \
37 5
- = 3 75A
Ans
10

37'5
- = 2.58
Current in 15R reslstor = 15

Ans.

I
t
Total current = 6.25A Ans.
Voltage d r o p across g r o u p A = 6V Ans.
Supply Voltage = 6 + 37.5 = 43,SV Ans
b.

8. Open-circuit voltage of generator= 11OV


Voltage d r o p in generator for 75A = 110 - 108.8 = 1.2V
1.2
Internal resistance s f generator = - = 0.016R Ans.
75
Voltage d r o p in cables = 108.8 - 105 = 3.8V
3.8
Kcs~stanceof cables = - = 0 . 0 5 0 7 0 Ans.
75
0 1 1 ">llort-cir~~l~
tile
~ t "OI~I!, l i ~ l l ~ l ; ~ tto~ o1n1 1 ~cllrre111, 1s I ~ c
res15tancc 01' the gcncrator. ;ind tllc cables.
= 0 . 0 I 0 1- 0.0507
0.0067R
I I0 -~ ' I
= I 05 x 10' ;ilnj?crc.
S o S.('.current =
0.O6h7
6.67
= 1650A Ans.

9 Meter voltage d r o p for full-scale deflection = 1 x 0. I.! volt\

0.13V
I?OmV.

39 1

SOLUTIONS TO PRACTICE EXAMPLES

Since shunt voltage drop for 300A is 150mV, then the meter
resistance would have to be ~ncreaccdby an external resistcr of
v;iltlc 0.03R. 0ht;llnccl I'rom I ;Imp
(0.03 t 0.12) o h m
1 x 0.1.5
0. 15V or IOlnV.
Ilndcr this cc>ndition rhc ;1ctt1;11
current ~ c I I . ; ~ncrcrc~l
,
\vot11d
be 301A. 300A would pi~ssthtougl~the .;llii111 : \ t d I t \ tllrougll
the ammeter.
.Yore. The resistor would have to be rated for this current rc
I amp. Thus l 2 x 0.03 = 0.03 w a t t s -d e a l t with in Chapter 2 .

Fig 5

PD across BC = 34 + 6
M V , since E is 6V above C and
in turn is 34V above E.
Now the p.d. across AE and E D is proportional to their
resistance values.
'I lius p.d. across A E = - 0 - x 90 =. 54V w ~ t hA + v c t o I :
15

6
and p.d. across ED = - x 90 = 36V with E +vc to 1).
15
Since A is positive to E by 54V and B is positive to E by 34V
(voltmeter reading), then A is +ve to R by (54 - 34) = 20V.
Similarly since B is above E by 34V and E is +ve to C by 6V
then the p.d. across BC = 34 + 6 = 40V. Again E is above D
by 36V and above C by 6V so C must be +ve to D by (36 - 6 ) =
;0v.
AB = 20V
Thus the p.d. across the resistors are BC = 40V Total 90V.
C D = 30V 1
Also as the branches are series circuits. the ohmic values are
proportional to the p.ds.
l o x 40 = 2 0 a
p.d. across AB 10 or R , = -------Ans
p.d. across BC
R,
30

iind

p.d. across C D - R ,
- - or R, =
p.d. across A B
10

20

30 = t 5 R

Ans.

90
- 90
= 2A Ans.
Current in branch ABCD = 10 20
15- 3
90 - 90
Current In branch A E D = ----- - = 6A Ans
15
9 + 6
Supply current = 2 + 6 = 8A 4 n s .

Mass to be lifted = (2 + 0.25) x lo3 kilogrammes


Force to be exerted = 2.25 x lo3 x 9.81 newtons
Work to be done = 2.25 x 103 x 9.81 x 30 newton
metres
= 66.2175 x 104Nm or 662.175kJ
work done (joules)
Power output of hoist =
time (seconds)

1,

Power Input = 220 x 50 = l I OOOW = 1 1 kW


7.36
Efficiency = --- = 0.6689 or 66.9O,, Ans
11

2. Since a battery voltage in the region of 20V 1s required. the


cells must be connected in a series-parallel arrangement. I t is
apparent that 10 cells in series would give 22V and that this
would be the e.m.f. of the battery irrespectike of the number of
identical parallel banks.
Thus an arrangemcnt of I0 cells in serles with three such banks
in jara all el would be ;i possible comhin;~tion.Thi\ can he triccl.
Em.!'. of' battery = e.m.1'. 01' I bank = 2.2 x 10
= 22v
Internal resistance of 1 bank = 0.3 2 10 = 3R
Internal resistance of battery =

= IR

v2 202
Resistance of 1 lamp = - = --- = 40R
P
10
P
10
o r lamp current = - = - = 0.5A
v 20
20
and lamp resistance = - = 40R
0.5

40
Resistance of 3 lamps in parallel = - = 13.33R
3
Total resistance of complete circuit
Circult current =

LL

---

14.33

= 1.54A

1.54
(b) Current taken by 1 lamp = - = 0.513A Ans.
3
( a ) Voltage drop in battery = 1.54 x 1 = 1.54V
Battery terminal voltage = 22 - 1.54 = 20.46V Ans.
Power loss per cell = (current in 1 bank)2 x
(c)
resistance of a cell
= 0 . 5 1 3 ~x 0.3 = 0.079W Ans.

3. The equivalent head of water can be obtained thus:


A pressure of 15 bars is 15 x 10' newtons per square metre
Ypcclfic wcight of water is 103 x 9.81 newtons er cubic metre
1 5 x los = 15 x --10 metres
Then head of' water =
lo3 X 9.81
9.8 1
= 15 x 10.19 = 152.85m
Force required to lift 12 700 litres or 12.7 x lo3 kilogrammes
IS 12.7 x 103 x 9.81 newtons = 124 587N
Work to be done per hour = 124 587 x 152.85 newton
metres
= 12.46 x 1.53 x lo6 = 19.064kJ
19.064 X lo6 190.64
-,03
Power output of pump =
3600
36

Input to pump o r output of motor

Input to motor = ---6'465 - 7.275kW


0.89
7275
- 363'75 - 33.1A
Motor current = -- -220
11
4

Ans,

E.m.f. of battery = 4 x 2.2 = 8.8V


-1'crnlinal voltage of battery = voltage drop across resistor
= 5 x 1.4v= 7v
Voltage drop in baitery = 8.8 - 7 = 1.8V
1.8 9
Internal resistance of battery = - = - = 1.290
1.4 7
1.29
Internal resistance of 1 cell = --- = 0.32R
4
For parallel working:
lnternill resistance of bi~ttcry= 1
'2
= 0.08f2
4

-.

395

SOLUTIONS TO PRACTICE EXAMPLES


-

E m.f. of batter) = e.m 1. of 1 cell = 2 2V


Total clrcult resistance = 5 + 0.08 = 5.080
( ' ~ r c u ~currcnt
t

--

O47A

5 08

5. Output o f winch = 5 x lo3

An\

36.5 ncb\\,tonmt.rrr.;
per minute
= 4.905 x 36.5 x 10' joules per minute
-

4'905

9,S!

36'5

l ' joules per second

60

179.033 x 103 watts


6
= 29.84kW
Since the winch is 75 per cent efficient.
100
the Input must be 29.84 x --- kilowatts
75
4
= 29.84 x - - 39.78kW
3 'Input to winch = output of motor
:. power rating of motor = 39.78kW Ans.
If the motor efficiency is taken as 85 per cent,
39.78
the electrical input would be ---- = 46.8kW
0.85
46 8 0 - 2340
Current taken from the mains = -- 220
11
= 212.7A Ans.

1,1gl1t111g
lo;~d = IOU
100 - I0 OOOW .IIICJ
2 0 0 x 6 0 - 12oooW
= 10 + I' = 22kW
Heating load = 25kW
Miscellaneous loads = 30 x 220 = 6600 = 6.6kW
Total load = 22 + 25 + 6.6 = 53.6kW
Generator output = 53.6kW
53.6 = 63.1kW
Generator input = 0.85
Now generator input = engine output.
So engine must develop 63.1kW Ans.

6.

7.

0 . C . e.m.f. of battery

O.C. e.m.f.,'cell =

4.3V
4.3

I .43V

4.23
Value of load resistor = - = 10,575R
0.4

Voltage drop in battery = 4.3 - 4.23 = 0.07V


0.07
Internal resistance of battery F -= 0.175R
0.4
0.175
Internal resistance of 1 cell = -= 0.058R
3
With a cell reversed, the e.m.f. of two cells cancel eack other
and effective e.m.f. = that of 1 cell = 1.43V.
Let I be the current under this condition.
1.43
Then 1.43 = I( 10.57 -+ 0.175) :. I = -= 0.134A Ans.
10.75
Note. For the solution, the internal resistance of a cell has been
assumed to be the same in both the forward and reverse direction,
which is in order in the absonce of any further detailed informauon.

Fig 6
40 = 0.363A
Current for 40W lamp = --110
110 = 303R
Resistance of 40W lamp = 0.363
150 = 1.5A
Current for 150W lamp = Resistance of 150W lamp

100
100
--- = 66.66R

1.5
Parallel circuit has to carry 1.5A
:. Current in shunt resistor = 1.5 - 0.363 r- 1.137A
Voltage drop across shunt = 1 IOV
Resistance of shunt = -- = 96.8R Ans.
1.137
110V is dropped across the parallel circuit
IOOV is dropped across the series lamp
.: 230 - 210 = 20V must be dropped across the s e x $ *
resistor which carries 1.5A
20
40
:. Resistance value of series resistor = - = - = 13.3fl .Ans
1.5
3

SOLUTIONS TO PRACTICE EXAMPLES


--

397
-

E.m.f. of battery = e.m.f. of 1 cell for parallel working


= 1.46V
Let R i = the internal resistance of the battery
and R, = the internal resistance of 1 cell.
The total resistance of the circuit = 0.525 + R i
Circuit voltage drop = 0.8 (0.525 + Ri)
= applied e.m.f. = 1.46V
. , 1.46 = (0.8 x 0.525) + 0.8Ri
P

Now since the cells are in ~ a r a l l e lthen


,

and internal resistance of I cell =

5.m

Ans

r---l

-:

T
I.SV/CELL

1 0 . 2 2 5 OHM
1

-L

/CELL

4 CELLS

IN SERIES

I-

2 OHMS

-L
2 . 5 OHMS

Fig 8

398

REED 'S BASIC ELECTROTECHNOLOGY

Battery e.m.f. = e.m.f. of 1 bank = 4 x 1.5 = 6V


resistance of 1 bank - 4 x 0.225
Battery res~stancs= 3
3
Load resistaxe = 2.5 + R (resistance of parallel section)
1 = 5- o r R = 6- = 1.2Q
here-1 = 1- + R
3
2
6
5
Load resistance = 2.5 + 1.2 = 3.7R
Resistance of complete circuit = 3.7 0.3 = 4 0
6
Circuit current = - = 1.5A

Volt;~pcd r o p in hatter, = 0.3 x 1.5 = 0.45V


Battery terminal voltage = 6 - intcrnitl voltage drop
= 6 - 0.45 = 555V Ans.
Power rating of 2 . 5 0 resistor = 1 2 R = 1.52 x 2.5
= 5 625W Ans.
voltage drop - total current x R
Current in 351 resistor = resistance
2

Power ratlng of 2R resistor = 0.92 x 2 = 1.62W Ans.


Current in 3Q resistor = 1.5 - 0.9 = 0.6A
Power ratlng of 3R resistor = 0.62 x 3 = 1.08W Ans.
Energ) conversion = energy in external resistors + energy in
battery
= time (total wattage of external resistors
battery resistance power wastage)
= r (5.625 + 1.62 + 1.08 + 1.52 x 0.3)
= 3600 (8,325 + 0,675) = 3600 x 9 = 32 400 joules Ans.
7 his could also be obtained thus:
Energy = e.m.f. x current x time
= 6 X 1.5 X 3600 = 9 X 3600
Ans.
T 32 400 joules or 32.4k.l

17 x
x lo6 = 170 Ans
'b) Area of plate = 100 x 100 = 1C 4mm 2
10 x 103 = Irnm
Thickness of plate = -=

,,

This is the length in the expression R = @


.:

R = l7

I0l6
10

I ohms = 1.7 x iO-3irQ Ans,

Alternatively using p = 1.7 x l o - @ ohm-metres for ( a t - a s


an example.
=

1.7 x 10 = 17Q

RO(l x20) and R60 = Ro(l + a60)


R"(1 c *fa)
K o ( I tct20)
Hz,[I + (60 x 0.004 28)]
[I -t ( 2 0 x 0 (XM'X)]
90
90 x 1,2568 ohnls
R,, = ---( l + -0.2568)
---- 1 + 0,0856
1.0856

2. S ~ n c eR2, =
-Jllcn
wt o
and R,, =
or

Ans.

230
Current taken by coil at 20'C = --- = 2.56,4
90
At 60'C to keep the current constant, the toltage must be
2 56 x 104 4 = 267 26V So the voltage must be ra~sed bh
267.26 - 230 = 37.26V Ans.
3. Assuming I litre of water to have
Mass of 0.75 lltre of water = 0.75 x
Heat required = 0.75 x
= 3.15 x
= 296kJ

a mass of I kilogramme
I = 0.75kg
4.2 x (100 - 6 )
94

220
11
The current taken by the heater is --- = - = 1.83A
120
6
a n d the power r a t ~ n gof the heater = 220 x 1.83
= 403.3W
Since the heater is only 84 per cent efficient, only 403.3 x 0.84
watts are available to heat the water.
time of heating = 296
lo' seconds
403.3 x 0.84
- 296
0339 = 873s
873
= -- minutes = 14min 33s Ans.

.:

60

Since R

4.

P1
=A

Since
.'

2=

R,

R2

RA
then 1 = -

2.99 o r 3m (approx) Ans.

R, (1
cx T,)
R,(I+rT,)
R, (1 + aT2)
I + cc T,-

and r T, = '1.2 + 0.0750 - 1 = 0,2756


0'2756 - 65.6cC
thus T , = ----0,0042
Tcn~peraturerlse = 65.6 - 15 = 50.6'C A n s

40 1

SOLUTIONS TO PRACTICE EXAMPLFS


--

6.

L-

I50 m

SUPPLY A
240V

I50

-.-. --

rl

200 A
LOAD

Fig 9
Since the resistance of a cable core 880m long and 50mm 2 in
area is 0.219R
then the resistance of a cable core 88Om long and 150n1rn2 ~n
0.219
area = -= 0.073R
3
and the resistance of a cable core 150m long and 150mm2 in
area = 0.073
150 = 0.0124n
880
e
Current In length AB = 300A
Resistance of length AB = 2 x 0.0124 = 0.0248f2
Voltage drop in length AB = 300 x 0.0248 = 3 x 2.48
= 7.44V
Voltage at 200A load = 240 - 7.44 = 232.56V Ans,
Voltage drop in section BC = of that in A B since the current
3
1
is -, all else being the same
3
7.44
: Voltage drop In BC = ---- = 2.48V
1

Voltage a t IOOA load

it. at

7
C = 2 3 2 . 5 6 - 2 . 3 8 = 130.OXV All\.

1
7. Resistance of l m m diameter cable R - P1
- A.

Let R2

Then R ,

resistance of 800 x
x 840

lo3

x x 1.5~

105
100

-=

840m of 1.5mm diameter cable

= 2.47 x 8.4 x

- 20.75 - 9.22Q
2.25
Since there are 19 strands in parallel, the resistance of the
complete cable, being inversely proportional to area, will be
reduced by 19
9.22
.: Resistance of cable = -= 0.485R Ans.
19

8 . Since R = R, ( 1

+ a T) or 15 = 10 [ l + (a x

100)]

= 1 0 + (a x 1000)
or 5 = 1000a and a = 0.005 Ans.

or using the definition


increase of resistance per OC rise in temperature
a =
resistance at OC
- l 5 - 1/10 = 5 = 0.005 Ans.
- 100
Also since R = R, (1
a T ) then 30 = 10 (1 + 0.0057')
and 30 = 10 + 0-05T
20 = 20 x 100 degrees
or 20 = 0.05T and T = 0.05
5
Celsius
= 400C Ans.

Heat required by brass = 500 x 0.39 x (910 - 15)


kilojoules
= 5 x 39 x 895
= 195 x 895.
= 1.95 x 89.5 x lo3
or energy required by brass = 174.525kJ
174.525 x lo3 x 100
Energy taken from supply =
80
9.

= 21.816 x lo4 kilojoules


Time taken to expend this energy at the rate of 2OOkW
- 21,816 x lo4 - 21.816 x 10' second,
200
2
= 10.908 x 10' = 1091s
= 18. I8 min o r 1-8 min 1 1s Ans

S O LU T IO N S TO PR A C T I C E EXAMPLFS

10.

Electrical energy used

403

372 units = 372kW h


0.2
= 372 x 3600 kilojoules
Energy passed to heat watcr = 372 x 3600 x 0.8 kilo.joulcs
= 1071 360kJ
Heat energy received by water = 1.07 x 10' kilojoulcs
Temperature rise of water = 82 - 16 = 66
1.07 x lob - 1070 x lo3
So quantity of water =
kilogrammes
66 x 4.2
277.2.
= 3.86 x lo3 = 3860kg
Assuming 1 litre to have a mass of 1 kg then quantity of water
used = 3860 litres Ans.
=74'4
-=

CHAPTER 4
Input to accumulator = 6 x 18 amp_ere hours
Output from accumulator = 3.5 x 28 ampere hours

1.

28 = 0.907
Ampere-hour efficiency = 3'5
6 x 18
= 90.7 per cent Ans.
Mass of deposit, m = zlt
o r (19.34 - 14.52) x l o - ' = 330 x
4.82 x
or l =
amperes

2.

50 x 60 x 330 x
30--.x-j.j 4 . 8 2

x~-io7

x I x 50 x 60

10-9

499
x2

4.869A
Error in reading = 5.1 - 4.869
= 0,231A (high) Ans.
This may be better expressed as a percentage thus:
- Difference between false and true reading x 100
true reading
=

4.75 per cent (high) Ans.

E.m.f. of 6atter) = 40 x 1.9 = 4 x 19 = 76V


o r E, = 76V
Internal resistance of the battery = 40 x 0.0025
= 4 x 0.025 = 0 . 1 0
Total resist;lrlce of circuit = I + 0.1 = I . 1R = R
For charging V = E, + IR
Thus 90 = 76 + ( I x 1 . 1 )
o r I = ------90 - 76 - l 4 amperes
1.1
1.1
- 140 = 12.72A Ans
I1

3.

4. Area of nickel deposited

x x 100 x
x 150 x
3.14 x 15 x
47.1 x
= 0.0471m2
Volume of nickel deposited = 47.1 x
x 0.5 x l o - "
L
: 23.55 x
10- cubic mctrcs
=

=
=

40 5

SOLUTIONS TO PRACTICE EXAMPLES

x 8.6 x lo3
Mass of nickel deposited, m = 23.55 x
= 0.202 53kg
= 202.53 X'
x I x 8 x 3600
Now m = zit so 0.202 53 = 302 x
or 202.53 x 10-"
3302 x H x 36 x 10-" I
202.53 x lo3
giving I =
30.2 x 8 1 3h
- 202.53 X lo3
241.6 X 36
202.53
lo3 -202.53
8.698
2.416 x 3.6 x lo'
= 23.3A Ans.
Discharge ampere hours = 6 x 12 = 72
Charge ampere hours = 4 x 22 = 88
9
72
*
Ampere-hour efficiency = - = 88
11
= 0.82 o r 82 per cent Ans.
Discharge watt hours per cell = 6 x 12 x 1.2 = 72 x 1.2
= 86.4
Charge watt hours per cell = 4 x 22 x 1.5 = 88 x 1.5
= 132
86.4 - 21.6 7.2 = o,65
Watt-hour efficiency = -- - - 132
33
11
or 65 per cent Ans.
5.

6. Battery voltagc at s t a r t ol'cli;~rgc-

HO

Ebl

Eb2

I X

144V

Battery voltage at end of charge = 80 x 2.4 = 192V


No battery resistance is given and is therefore neglected.
Let R , = control resistance at start of charge
Then V = Eb,+IR, or 230 = 144 + 5R,
Thus 5R, = 230 - 144 = 86
172 = 17.2R (maximum value) Ans.
R, = 86
- = --5
10
Let R, = control resistance at end of charge.
Then 230 = 192 + 5R2 o r 5R2 = 230 - 192 = 38
38
76
R, = - = - = 7.6Q (minimum value) Ans.
5
10
60
Charging tlme = - = 12 hours (approx)
5
Say 13h to allow for losses Ans

7. Area of deposit = 2 x 50 x 150 = 15 000rnm2


Volume of deposit = 1.5 x lo3 x
x 0.05 x
= 0.75 x 10+rn3
= 1.5 x 0.5 x

Mass of deposit, m = 0.75 x


x 8800
= 0.75 x 8.8 x
= 6.6 x
= 0.0066kg
m
Now m = zIt so I = zt

Thus I = ll.1A Ans.

8.

Discharge ampere hours = 4 x 40 = 160


Charge ampere hours = 8 x 24 = 192
160 40 5
Ampere-hour efficiency of battery = - = - = - = 0.833
192 48 6
or 83.3 per cent Ans.
Discharge watt hours = 4 x 40 x 1.93 x 40
charge watt hours = 8 x 24 x 2.2 x 40
4 x 40 x 1.93 x 40
Watt-hour efficiency of battery =
8 x 24 x 2.2 x 40
-9.65
- 5 x 1.93 6 x 2.2
13.2
= 0.731 or 73.1 per cent
Ans.

Voltage drop in leads = 10 x 1 = 10V


Voltage drop due to battery internal resistance
= 10 x 0.01 x 3 0 = 3V
At start of charge, if R, is the external resistance
10R,
Then 200 = (30 x 1.85) + 10 + 3
or 200 = 5 5 . 5
13 IOR,
131.5 = 13.15R
and 200 - 68.5 = 10R, thus R, = 10
At end of charge, if R, is the external resistance
Then 200 = (30 x 2.2) + 10 + 3 + ]OR,
or 200 = 66 + 13 + IOR,

9.

and 200 - 79 = IOR, thus R, = l 2 I - 12.1i-2

10

At start 13.1 5R are needed Ans.


At cnd 12.1R are needed Ans.

SOLUTIONS TO PRACTICE EXAMPLES

407

4.2 x lo-'
seconds
330 x
x 3.5
420000 4 x lo4
- 60000 - 12 x lo4 --Thus r =
33 x 3.5 33 x 0.5
33
11
= 3636s = 60.6 min Ir 60 min 36s Ans.
From the second law of electrolysis or by proportion:
Mass of hydrogen liberated - Chemical Equivalent of hydrogen
Mass of copper liberated
Chemical Equivalent of copper
1 x 4.2
Thus mass of hydrogen = ------- = 0.1321g Ans.
31.8

m
10. Here m = zlt or r = =
zl

CHAPTER 5
1.

F = BII newtons
= 0.25 x 100 x 1 = 25 newtons per metre length

Ans.
2. Magnetomotive
force F = 4 x 250 = lOOOAt
F
(a) Magnetising Force H = - = m.m.f, per metre length
1

loo0
- 500 x 1 0 - 3
= 2000At/m Ans.
(b) Flux Density B =

x H = 4 x n x lo-' x 2000
= 8 x rc x
teslas
square metres
Cross-sectional area of ring = 400 x
:. Flux @ = B x A = 8 x rr x
): 400 x
= 25.L2 x 4 x lo-'
11,

0.002 512 x 4 x

= 0.010 048 x
= 1.0048 x

webers
or 1.0048pWb Ans.
3. Magnetomotive force F produced = 3200 x 1
= 3200At
3 200
The maynetising force H or m . m . f . ' m = -F =
..
1
800 x
= 4000At/m
Also since B = 1, x H
B = ~ ~ ~ ~ I O - ~ X ~ C M X
= 16 x x x l o W 4
teslas
n x 8 - n x 202 x lo-' =
Area of solenoid = ------ lo-4
4

So @

B x A = 16 x x x
x n x l o w 4 webers
= 16 x n 2 x
= 16 x 9.86 x lo-' = 157.76 x
= 1.58 x
webers or 1.58pWb Ans.

4. Magnetising force H of a long, straight conductor

I
--

2000
2 x x x 0.8 ampere-lurns/metre
or H at conductor X, due to current in Y,
2xr

SOLUTIONS TO PRACTICE EXAMPLES

4(K,

and B at conductor X due to current in Y = p, x H


x I O - ' ~ 1000
- 4
x
08 x x
-

104

---

- In

2 x lo-'
So F' = U l l Ilcwtorl\

' tssi,is

- -l o - '

x 2000 x I = i newtonlmetre length


Ans
5. Current to give full-scale deflection
- 50
= 5
10-1*
10
Force exerted on 1 conductor
= BII=O.I
5
25
10-3
= 12.5 x
newtons
Force exerted o n all conductors on both s i d p of the coil
= 100 x 2 x 12.5 x
= 2500 x
newtons
Torque exerted by coil
= Force x radius

= 37.5 x
newton metres
Therefore the Controlling Torque of the spring
= 37.5 x I O - ~ N
m Ans.
o r = 37.5pN ni Ans.

0.05

6. Flux density B in air gap = ~650 x

5 x lo2
4 x rr x lo-'
500 lo1
26 x n
81.64
= 6 12 x 10' ampere-turns metre
Air g a p = 3mm = 3 x
metrec
Requlred ampere-turns = 6 12 x 10' x 3 x lo-"
= 18.36 x lo4
= 183 600At Ans

Also B = p, x H

B=
:H = 6.5 x
p
5
x
lo9
- -

7. F = BNnewtons = 0.6 x 150 x I = 90N/rn Ans.


Assuming current flowing away from the observer, then the
force acts from right to left to move the conductor horizontally.

8. F = BII newtons
= 0.5 x 25 x 400 x
= 0.5 x 10 = 5N Ans.

= 0.5 x 100 x l o - '

9. Force on 1 conductor = 0.6 x 8 x 250 x


Force on 800 conductor = 0.6 x 8 x 250 x
= 1.2 x 8 x 10'
= 9.6 x 10' newtons
Torque on armature = 9.6 x 10' x 1'00 x
= 96N m Ans.
27t NT
Powcr dcvcloped is given by - watts
60

x 8 x lo2

9'6 kilowatts
6
6.28 x 1.6 = 10.05kW Ans.
6'28

10. Magnetising Force H of a long straight conductor


I
= - ampere-turnslmetre
2 ~ r
250
2 x 3.14 x 25 x
- 25 x loJ
2 x 3.14 x 25
so&=p, x H
10 4
o r B = 4 x x x lO-'x2 x K
= 2 x
teslas
x 250 x 1
Again F = BIl = 2 x
= 5 x
x 102newtons
Mutual Force per metie run = 0.5N Ans.

1 . Dynamic Induction

BI,)

40 x

4 x 14 x
-

14
-

100 x lo3
3600
volts
lo3
-

x 1.4 x

36

10-3

= 1.55 x 10-3 volts


=

0.001 55V .jr 1.55mV Ans


P

2. In 1 revolution. the flux cut

6 x 64 x
webers
loo0 - 100
The number of revolutions per second = - - 60
6
1 02
.: Flux cut per second = 6 x 64 x 10-.3
=

= generated voltage = 6.4V

The conductors in series per parallel path


= 4687
8
6
So voltage p e r parallel path
6.4 x 78 = 499.2V
= e.m.f. of machine
Currcnt per conductor = current per pi~s:\llrlp : ~ ~ l i
= 50A
Current in six parallel paths = 50 x 6 = 300A

So power developed = 499'2


300 = 49.92
1000
= 149.76kW Ans
Z
@
N
P
Check E = - x 60
A
Note. This formula
shown here.
'Thus E =
=
=

IS

introduced In Chapter 8. Its usage

468 x 64 x

60
78 x 6.4
499.2V Ans

x 1000 x -6 = 468 x 6.4

I\

12

REED ' S BASIC ELECTROTECHNOLOGY

weber = 28miVb
3. Change of flux = (30 - 2)
Induced e.m.f. = rate of change of flux-linkages
2000 x 28 x
- 2 x 28
0.12
12 x 1 0 - I
2
X 7 X lo2 - 14 X lo2
3
3
= 4.666 x lo2 = 466.6V Ans.
4. Diameter of armature = 0.2m
Circumference = rtd = 3.14 x 2 x l o - ' = 0.628111
500
revolutions
In 1 sccond thc ;\rm;iturc turns
60
500
I n 1 xecond ;I cot1 stde travels ---- x 0.628 metres
00

So E = Blv = 1.2 x 2 X 0.4 Y 5.233 volts


= 2.4 X 4 X 5.233 X l o - '
= 9.6 x 5.233 x l o - ' = 5.024V Ans.
j__

5. Let El 0 , and N , be the values under the original conditions


and E , @, and N, be the values under the final condition.
Here Y l and N, are the speed conditions.
Also for a generator E K 0 N o r E = k 0 A'
We know that 0 x B and N x 1' thus the more general form
I.: 7 @A' (introduced later) can be used, k is a constant.
E @, N2
E2 - kQ2Iv2
.. - o r E, = l
El
k0,hTI
01N I
x 1 1 0 0- 2 0 0 x 19 x 1 1
200 x 19 x
and E, =
20
1 0 - ~ 1000
20 x 10
= 19 x 1 1 o r E l = 209V Ans.

Diameter of coil = 0.2m


6.
Circumference of coil = n x 0.2 = 0.628m
1200
Spced = -- = 20 revis

60

\cloctt\ ofcotl \ ~ d c- 20 x 0 628 = 4 x 3 14


:
12 56mj5
Now E = Blv volts
= 0 0 2 x 0 3 x 1256
= 2 x l o - , x 3 x l o - ' x 12.56 volts per
conductor
T o t , ~ c.l ni f = 9- x 3 x lo-' x 12 56 x 400
0 , 1 356 x 4
24 X 1 256 = 30 144V
1

S O L U TI O N S TO PR A C TI C E EX A M P L E S
-.
. -- ---

--

413

The above is based on the conductors cutting the field :it rig11t
angles and therefore the maximum value of e.m.f. generated I \
10.144V Ans.
r l n ~ lbr
c 1 rcvoiotlon = I \ c c o n d s in OIIC r c v o l i ~ t i n ~:I ic y l r
20
2 0 L~\(~IC,
IVI
I S pcncr;\rcd. So I'rcq~~c~lc.!,
of pcllc.r;~r~-d
c 111.I'.
second or. in S1 units. I'requency of generared c.m.1 = 70 Ilcrrl
'

iz

11s

7.

Average e.m.f. In voltc = rate of change of flu\Ilnhages


Change of flux = (4 - 1.5)10-3
L 2.5 x
webers
Time for the change = 0.04s
2.5 x 10So rate of cha,lge of flux-linkages = 1200 x
4 X 10-I
0.25
= 1200 x - =
3 x 25
4
= 75V Ans.

"

8. Flux per pole = 0.09 x 0.92 = 8.28 x


webers
In one revolution a conductor cuts 4 x 8.28 x
= 33.12 x
webers
Also in one second the armature revolves

600

-=

60

10 tlmes

So by 1 conductor, the flux cut per second


= 3 3 . 1 2 x 10F2 x 10
3.312Wh
and the induced voltage per conductor = 3,312V An5
The number of armature conductors is 2 x 210 = 420
These are arranged in 4 parallel paths
420
There are thus - = 105 conductors in series per parallel
4
path
The e.m.i. of 1 parallel path = 105 x 3.312 = 347.76V
The generated e.m.f. of the machine = e.m.f.of 1 p:~rallclp:~rii
= 347.76V A n s
9. ( a ) Ampere-turns of solenoid = 400 x b = 7400A1
The magnetising force H at the centre = ampere-turns metre
-

= 1600Atm
1.5
The flux density B a t the centre of the solenoid and small coil
tesla
= p,H F 4 x rt x l o - ' x 1600 = 64 x rt x

Area of small coil =

nd4 --

So flus linked = 64
= 16

(10

l ~ - ~ ) ~

lo-' square metre

*n*
X

n2

lo-' x
X

'

lo-' weber
4
= 0.158 X
webers

0.158pWb Ans.
(b) Average induced e.m.f. = rate of change of flux-linkages
2
- 50 X 16 X x X
50 X 10-3
= 1.6 X rr2 X
= 15.8 X
volt
= 0.158mV Ans.
=

10. Coil A. Associated flux 0 = 18 x l o p 3 weber


Associated flux-linkages during reversal = turns x flux
decrease t o zero and then its build up to full value @ in
reversed direction.
= 1000 [0.018 - ( - 0.018)] = 1000 (0.018 + 0,012)
= 1000 x 2 x 0.018 = 2 x 18 = 36 weber-turns
Time of reversal = 0.1s
and induced e.m.f. = rate of change of flux-linkages
- 36 - 360V (Average Value) Ans.
0.1
Coil B. Only 80 per cent flux is associated and turns are 500
.: Associated flux-linkages during reversal
= 500 X 0.8 X 2 X 0.018 = 0.8 X 18
= 14.4 weber-turns
Induced e.m.f. = 14'4 = 144V (Average Value) Ans
0.1
Alternatively:
Proportion of e.m.f. in Coil B to e.m.f. in Coil A
= 3 ( j o x -'0- - 180V, if full flus is associated
1 000
I:or onl}, XO per cent flux, c.m.f. 1s rcduced in proportion
= 180 x

0.8 = 144V

Ans

I
CHAPTER 7
I A scale of lOmm = 1A 1s used and I, is the reference
phasor d r a w n h o r i ~ o n t i ~ l l ly' h c d ~ : ~ g r : ~dmr :, ~ w n g c o ~ r ~ c t r ~ c , ~ l l v

to scale, shows the solution which gives I. the r r s u l t ~ ncurrent


t
value 9 23A lagging I, by 6
If the above is checked mathematically
IH = 4 cos 0 + 6 cos 30 + 2 cos 90
= (4 x 1) + (6 x 0.866) + (2 x 0) = 4 + 5.196 = 9.196A
I, = 4 sin 0 - 6 sin 30 + 2 sin 90
= ( 4 ~ 0 ) - ( 6 ~ 0 . 5 ) + ( l2) ~= O - 3 + 2 = - l A
= 9.24A Ans
I = 4 9 . 1 9 6 ~+ 1 = j 8 4 . 6 4 + 1 =
9.2 = 0.995 and 8 = 6' ( a ~ p r o x ) Ans.
1s 8 =

Fig 10

'

4 'a

2. Since v = V , sin (2xft)

200 0.707
282.8
Wow the angle whose sine is 0.707 = 45'
.'. 2nft = 45

.'. sin (2nft)

= -=

= 5 x
= 0.005 seconds = 5ms
( a ) The first time is 5ms after zero value Ans.

1
4
( b ) Time for 1 cycle = - or - = 0.04s

25 100
Time for 4 cycle = 0.02 second
The second time is 0.02 - 0.005 = 0.015 seconds or 15ms
after zero value Ans.

416

R EED ' S BASIC ELECl'ROTECHNOLOGY

3. A simple phasor diagram, as shown, illustrates the problem


and the mathematical solution.

Fig 1 1

Horizontal component V , = 100 + 80 cos 60


= 100 + 80 x 0.5 = 140V
Vertical component V , = 0 - 80 sin 60
= 0 - 80 x 0.866
= - 69.28V
Resultant V = .\/1402 + 69.28'
= J19 600 + 4800
= d'24 400 = 156.3V Ans.

Since maximum values have been used for the phasor the
resultant is a maximum value, which lags the l00V values by
26" 26' Ans.
4. The waveform is plotted as shown in the diagram. Erecting
mid-ordinates. measuring and squaring these gives the following
columns.
I,
=0.22
i,'
=0.05
i, = 0.60
i Z 2 = 0.36
i, = 0.92
i,2 = 0.85
Total of i2 = 19.1'9
i, = 1.25
',i
= 1.56
19.17
- 1.55
i,$' = 2.40
Avcr:~gcof i 2 fC!
i6
= 1.8
'6i
= 3.24
i7
-. 1.97
i.,
-- 3.87
= 1.317
.a = 1.92
i g 2 = 3 68
r.m.s. value = \ , T 4
l9
= 1.56
i9 , = 2.44
- i , j nL.,X
c
AT?',
I,, = 0.85
i l o - 0 . 7 2 Pois'er dissipated obta::,cc ir:x:-.
['I? - \.01: X p - ' < l L , ' . i
'IF..
1 5

:,

41 7

SO L U TI O N S TO P R A C TI C E E X A MPLE S

Fig 12

5. The phasor diagram I s drawn as shown, but is not to scale


since the resultant is obtained mathematically. For convenience,
the second current has been used as the reference along the
horizontal.

....\ ,

Sum of horizontal components


1, = 17.32 cos 0 + 20 cos 60 + 10 cos 90
= (17,32 x 1 ) + (20 x 0.5) + (10 x 0)
= 17.32 + 10 + 0 = 27.32A
Sum of vertical components
Iv = 17.32 sin 0 - 20 sin 60 + 10 sin 90
= (17 x 0) - (20 x 0.866) + (10 x 1 )
= 0 - 17.32 + 10 = - 7.32A
Resultant I = \/'27.322 + 7.322 = .\/746 + 53.4
= m
4= 28.32A
27.32
cos e = -= 0.965 SO e = 15" 12'
28.32
Siticc pcnk or m:iximum vnlucs wen: used for the individual
phasors, the maximum value of the resultant current is 28.32A
and this lags 105" 12' behind the 10A current. Ans.
6. For a sine wave voltage applied to a resistor, a sinusoidal
current results, and the maximum value of this current is at the
instant of maximum voltage.
maximum voltage 340
Maximum current =
resistance
24
= 14.14A
R.M.S. value of current = 0.707 x 14.14 = 9.996A
= 10A Ans.
7. The phasordiagram is drawn as shown. For the construction
a scale of lOmm = 25V is used. The phase relation between the
various phasors are determined more clearly thus:
e , = 100 sin wt
e, = 50cos wt = 50sin
=

e, = 75 sin

+ 90")
ot + - = 75 sin (wt + 60")

50 sin (cot

125 sin

i ;'i
(-1

E)

= 125 sin (wt - 30").


The diagram is drawn using the maximum value E,;, of the
first voltage as reference. For the diagram E,, = IOOV

419

SOLUTIONS TO PRACTICE EXAMPLES

Fig 14

E2, = 50V

E3,

75V

E,,

125V

For the resultant E = 100.5mm= 252V


The angle 8 = 12" (approx)
180
IT
Now 12" = or radians (leading)

15

15

The required expression can be written


cl,

+ c, +

P,

+ e4 ore = 252 sin

Ans.

8. (a) Alternators in step V = 100 200 = 300V Ans


(b) When phase dispacement is 60"
V=
+.-20o2
+ 2 x 1 0 0x 2 0 0 o ; 60 - - -= d l 0 000 + 40 000 + (40000 x +) = 1\70000
= 100 t / 7 = 264.8V Ans
( c ) When phase displacement is 90"
.v = \ 10O2+2OO2=~50OOO=100~5
= 223 7V Ans
( d ) When phase displacement is 120'
V = / 5 0 0 0 0 + 4 0C ~ 120
S
= \'50 000 + 40 000 ( - cos 60)
--= t/50000 + 4 0 0 0 0 ~ = t'50000 - 20000
= 130000 = 1 0 0 ~ =
3 1 7 3 2 V Ans.

v'no2

-+

(e) When phase displacement is 180cos a = - I


1' = d50000 - 4 0 0 M = \/ji0000 = lOOV
The above is obvious ie since the voltages oppose, the
resultant is the arithmetical difference ie lOOV Ans,
9. The waveform, when plotted. is seen to be of stepped shape
but each half wave is regular and similar to its other half, except
that it is reversed. The r.m.s, value can be obtained by considering a half wave only. since the reversal mentioned, will not
;ilfcct t h ~ svnluc.

Fig 15
the time interval 0 - 8 is considered as the base of the half
.c, cipllr ~nici-ordini~tcs
can be used, giving:
I , = 4 and i , 2 = 16
i , = 6 and i12 = 36
The sum of i 2
= 1 12
i, = 0 and i,' = 36
112
1, = 4 and i,2 = 16
The mean of i Z - - - J i 4
- 8
i , = 2 and ',i = 4
1 , = 0 and i,2 = 0
The r.m.s. value = ~'14
i, = 0 nnd i7' = 0
= 3.75A
i , = - 2 and i f 1 2= 4
Ilc rcqu~rcdvalue o1'd.c. would be 3.75A An5

'

SOLUTIONS TO PRACTICE FXAMPLES


-

--

---

--

--

---- -

42 1

10. The problem calls for a solution by phasor construction and


i s therefore worked by drawing; the scale chosen being IOmm =
2 V and [he first c.m.f. OA being uscd ; ~ . sthe reference.

Fig 16(a)

Fig 16(b)
The resultant O D o r Em is measured to be 15.6V and
n
6 = 15- o r - radians

12

Thus e = 15.6 sin jwr

+1
)Ans.
12

The above diagrams 5how alternative soiut~ons.Thus ( a ) u$c$


t l ~ cp;tr;tllclogram ~nctliod.( b ) uses Ihc polygon mctllotl.

CHAPTER 8
ZON
P
1. E.m.f. generated = - x - volts
60
A
Here Z is 144 x 6 = 864
N = 600 rev/min
P = 4 and A = 4 since this is a lap winding
.'. 216 = 8 6 4 x 0 ~ 6 0 0X -4
4
60

216 webers
864 x 10
If the armature is wave wound A = 2
Substituting the value of 0
2 16
600 4
then E = 864 x
864x 1 O X 6 o X 2
= 21 6 x 2 = 432V Ans.
or@=

2.

Voltage applied to shunt field = 220V


220 = 1.25A
~ u ; r e n t through shunt field = 176
Armature current = 250 + 1.25
= 251~25A
Voltage drop in armature and series field = 251.25 (0.05+0.015)
= 251.25 x 0.04
= 10.05V
Total voltage drop = 10.05 + 2V (brush
voltage drop)
lnducede.m.f. E = 220 + 10.05 + 2
= 232.05V Ans.
3. I t should be noted that a change of speed is involved in t h ~ s
problem and solution cannot bc afl'ected before the O.C.C. at 900
rc\, min is obtained. Since E .x h', tlie new values can be
o t ~ ~ ; ~ i nhyc d~nliltiplyinpthe oriplni~lhy 900 = 31200 4

423

SOLUTIONS TO PRACTICE EXAMPLES

The table shows the adjustment for the 900 rev mln condition

1 F r c ~ t ; ~ t ~Cl~rrcnt
on

, 0X

04

; I, (amperes)

11

900rev/min
E (volts)

11 ~
+--- -1- 1 1
1

E.m.f. at
1200 rev/m~n
E (volts)

1 E.m.f. at

1h

I 2

I
1 9 6 , 226 , 2 4 4 2541
I
i
I
I
I

15x3 88x3
112; 6 6 T l ~ ~ 5 1 4 1 6 5 8 9 0 5 ~

The field voltage drop line is drawn by taking any current


value and multiplying it by 90R. Example : 2k x 90ll = 180V.
Join this point R to the origin. The required answer 185V is
obtained from the point of intersection as shown.

0.5

1.0

--+

1.5

If

Fig 17

20

172.72A

I , = 172.72 + 4.4 = 177.12A


Also E = I / + /,R,
= 220 + (17712 x 0.1) = 237.72V
Z@h P P
Again E = --- x 60

0.01 28Wb o r 12.8rnWb Ans.

250 000
Full-load current = ------ = 520A
480
Total voltage d r o p on full load = 520 (0.01 + 0.005 + 0.005)
= 520 x 0.02 = 10.4V
So e.m.f. generated on full load = 480- + 10.4 = 490.4V

= 0.0633Wb o r 63.3mWb

Ans.

6. On no load. the e.m.f. generated is caused by the shunbfield


:impere-turns = 7900. These give a no-load voltage of 500V.
But since the load voltage rises to 550V, the shunt-field current
w ~ l rise
l
and the shunt-field ampere-turns, on n o load, increase to

Actually o n full load it is found, from a separate test, that


I I ZOOAt ;Ire required ; ~ n d the cxtr;l (;~hovc 8690At) m u s l
~llerel'orebe supplied by the series field.
Series field must supply I 1 200 - 8690 = 2510At

500 OoO - 9lOA


Now the full-load current = -----5 50
2510
:. The series turns required = ---- = 2.76
9 10
ic. 3 turns A n s

SOLUTIONS TO PRACTICE EXAMPLES

7. Here Z .= 90 x 6 = 540
540 x 0.03 x 1500
.: E =
60

42 5

---

*4

9 x 3 x 15 = 9 x 45 = 405volts Ans
If I, = 25A. The armature voltage drop = 25 x 1.0 = 2 5
volts. Since the same field flux and speed iire to bc assumed, tllcn
the same e.m.f. is being generated or V = E - I,R, and
V = 405 - 25 = 380 volts,
380
So shunt-field current = - = 1.9A
200
Machlne output current = 25 - 1.9 = 23.1A Ans.
=

Let I, = the load current


Then I , x 40 = V (the terminal voltage)
Also V = E - I,R,
= 380 - I , x 1.0 also I, = I&+ 1,
... V = 380 - 1.0 ( I , + 1,)
o r 4 0 x I'= 380 - I, - I ,
and 41 x I , = 380 - I,
V - -----40 x I , -I,
also I - 200 - 200
5
So the above becomes
IL
41 x 1, = 380 - 5
205 x 1, = 1900 - IL
1900
206 x I,, = 1900and I, = 2m= 9.22A A n s .

'-

8. ( a ) The 1000 rev/min O.C.C. is plotted and cut by the lOOR


field voltage-drop line which is plotted by drawing a
straight line through any deduced point and the origin.
Thus consider a I, value of ]A, then a field voltage-drop
line point would be 1 x 1000 = 100V. Join this point
to the origin.
The point of intersection at 108V is the answer
required.
(b) The tangent is drawn to the 1000 revlm~nO.C.C. The
critical resistance Rcis determined by taking any voltage
value on this tangent and dividing by the current.
145V
Thus Rc = ---- = 161.1R. The critical resistance for
0.9A
t h ~ speed
s
I S 1610 (approx.)

Fig 18
(c) The 1100 revlmin O.C.C. is obtained by multiplying the
original 1000 revlmin values by 11 - 1.1

to give the new table:

10 -

'

I
I

i
L

427

SoLunoNs TO PRACTICE EXAMPLES

This 1100 revlmin characteristic when plotted is cut by the


lOOQ field voltagedrop line at 127 volts. This is the required
answer.
= -m = 2 1 f i 9 ~

output current = M X 1 o a ,

9.

230

23

230 -46 - 4.18A


Shunt-field current = 55
11
Armature current = 217.39 4.18 = 221.57k
Armature voltage drop = 221.57 x 0.034 = 2.2157 x 3.4
= 7.53v
Induced emf = 230 + 7.53 + 2 = 239.53V
Electrical power required to be generated
= 239.53 x 221.57 watts
= 239.53 x 0.2216 kilowatts
*
= 53.18kW
Total input power = electrical power input + mechanical loss
= 53.18 + 1-6 = 54.78kW
Thus input power = 54.78kW Ans.

10. Since the answers required are at a different speed condition,


the new O.C.C. at 400 revlmin is obtained by multiplying the
original values by 400
-= 2. Thus:
200
Excitation
currcn t
I, (amperes)
E.m.f. at 200
revlmin
E (volts)
E.m.f. at 400
rev/min
E (volts)

10 38 61 78 9 3 106 115 123 130 135


,

'-'

'

20 76 122 156 186 212'230'246 260 270


:I

(a) E.m.f. to which machine self-excites = 243V Ans.


This is the point of intersection between the 400 'revlmin
O.C.C. and the 3651 field voltage-drop line.'The latter is
drawn by taking any current, say 5A and finding the
~ivoltagedrop 5 x 36 180V, and joining this point to
,
the origin.
q

q '

(b) Draw the tangent to 400 rev/min O.C.C. This may be


difficult. Disregard the bottom part of the characteristic
which is due to residual magnetism and assume that the
characteristic would pass from the second point given,
(76V at 1A) through the origin. A straight line drawn as
shown through this point and the origin, would be
sufficiently tangential to give a suitable answer.

Consider a point such as X. Then critical resistance

(c)

For the e.m.f, to reduce to 220V. the field voltage-dmp line


~ I I O L I I C I C L I I L I I C O.('.C'.i\L (111sI ) ~ I I ~ 'l'lle
[ .
field current would

SOLUTIONS TO PRACTICE EXAMPLES

429

then be 5.3A. So field-circuit resistance would be


220 - 41.5R. Thus additional resistance required =
5.3
41.5 - 36 = 5.5R Ans.
( d ) This answer is obtained by assuming the machine slows
down and is best obtained by trial and error. Multiply [llc
O.C.C. values by various fractions of the speed to obtain
a Magnetisation Curve which makes the 3 6 0 voltage-drop
line a tangent. The 200 revlmin O.C.C. isa suitable starting
point and is plotted. It is seen to be cut at the bottom by
the 36R line, which therefore is not quite tangential. The
required O.C.C. must be at a lower speed and 400 x 0.45
could be tried to give values of 9, 34.2, 54.9, 70.2, etc. If
plotted this is seen to be approximately correct and the
answer would be 180 revlmin (approx.).

CHAPTER 9
1. Inductive reactance of circuit X, = 2~cfL
= 2 x 3.14 x 50 x 0.01
= 3.140
(a) Circuit impedance =
= JGT%3

=Jirssi
= 4.3422

(b)

ic)

I'owcr I'UC~OT =

-z

Ans.

4.34

Power absorbed = I Z R =

(I)R=?~~

= 572.3W

Lamp resisiance =

100

---- =

= @6P (lngging) Anr.


P
R

Ans.

lOOR

( a ) Total resistance to give 1A with 220V applied

- 220 = 220R
I

.'. Series resistance = 220 - 100 = 120R

Ans.
Power absorbed by circuit = 12R = l 2 x 220 = 220.W
Ans.
( b ) Wticr~11 coil (re:ictor) is used for voltage dropping
220
Impedance of circuit Z = --- = 220R
1

Reactance of circuit =

dm---%?
= 100 J E T

=loo

J ~ = 1 0 0 , / m
= 100 x 1.96 = 196R -

-i2

= 0.624H Ans.
Power absorbed by circuit P = I * R = l 2 x I100 = lOOW Arts-

SOLUTIONS TO PRACTICE EXAMPLES

3.

Power absorbed, P = VI cos 4


P = 2500
:. cos i$ = VI

2 4 0 x 15

43 1

= 0.694 (lagging)

Ans.

Current in the circuit I = 15A (this data i s given).


Then since P = I' R

. R = T r = T2500
= - i T 2j -5 0 0 - l l - l R

Ans.

X = 4 1 6~ 11.12 = J256 - 123.21


=
= ll.6R Ans.
In Fig20 V, = IR = 15 x 11.1 = 166.5V
V, = IX = 15 x 11.6 = 174V
I = 15A
V = 2 4 b ~ Ans.

Fig 20
4. RA = I 2 W
R B = 100Q
X, = 2xfL
X, = 2xft
=2 ~ 3 ~ 1 4 ~ 5 0 ~ 2 5=
0 ~2 ~1 03 .- 1~ 4 ~ 5 0 ~ 10-3
400~
= 314 x 250 x
= 3.14 x 40
= 78.5Q
= 125.m
Z, = 100 6 1 . 2 +~ 0.785~ Z , = 100
= 100 41.44 + 0.615
= 100 17-/
= 1 0 0 r n 5 = 14352
= 1 0 0 m = 160.552
Total circuit R = 120 + 100 = 22052
X = 78.5 + 125.6 = 204.152
Z = J2202 + 2 0 4 . 1 ~= 100 d 2 . 2 2 + 2 . ~ 4 ~
= ioo J4.85 + 4.1 8 = I 00 m
3 = ~OOR
230
(a) I = - = 0.766A Ans.
300

Jm2

2 20
(b) cos d, = - = 0.733 (lagging) and 4 = 42" 46' Ans
300
(c) Voltage across ,4 = 0.766 x 143 = 10936V Ans.
Voltage
- across B = 0.766 x 160.5 = 122.9V Ans.
120
( d ) cos 4 - --- = 0.838 (lagging) o r 4, = 33" 7'
A 143
CoS

OB = loo
-- = 0.623 (lagging) r r
160.5

= 51 " 27'

Tht~zpl~itsedifference 4 = 51" 27' - 33" 7'


= 18" 20' Ans.

140 = 70R
100 = 50R
2, = 2
2
.Y, = J
= d49OO - 100
= tm= 10 JG = 69.3R
X , = J502 - IS' = J2500 - 225
- = &m = 10 m 5 = 4 7 - 7 ~
Since X is proportional to frequency
Therefore at 50Hz X, = 69.3 x = 86.6R
XB = 47.7 x 2 = 59.7R
For the total series circuit R = 10 + 15 = 2 5 0
X = 86.6 + 59.7 = 146.3S2
So Z = J252 + 146,32 = 10 d 2 . 5 2 + 1 . 4 . 6 3 ~ohms
= 10 ~ 1 6 . 2 5+ 214 = 10 JEEB = 10 x 14.81
A.C. condition Z ,

4
.

Current I =

230
148'1

= 1.55A

Ans.

p
- 750 looO = 2 8 4 ~
3 3 0 x 0.8
I/, cos d,,
Resistance voltage drop in the line, I.', = l R = 284 x I

6. Current in the line I =

Reactance

-.

,,

.%

,. , I',

= 1.Y

=S

284 x 2 . 5

= 0.710kV

From the phasnr diagram


(Fig 2 1 )
~.
C ' = J(3.3 x 0.8 + 0 , 2 8 4 ) ~+ (3.3 x 0.6 + 0 . 7 1 ) ~
=
0.28 4 i T T - T F - T
= \/'2,9142 + 2.69
= dR.55'.1--7rE = f l 5 3
= 3.98kV
q o \oltase a t the generator = .3,98kV AIIS.
~-

dm:+-

SOLUTIONS TO PRACTICE EXAMPLES

433

2.924
Generator power factor cos 4 = - = 0.73 (lagging) Ans.
3.98
3980
x 284 x 0.73kilowatts
Generator output =
lo00
= 3.98 x 284 x 0.73
= 825kW Ans.

Figs 21

64 = 8A
Current in 8R resistor = 8
= current in circuit
(b) Power absorbed.in resistor = I'R = 8' x 8
= 64 x 8 = 512W Ans.

7.

LOAD
C - - - - - - -

-----.

Figs 22
From the deduced diagrams (Fig 22)
0 A 2 = O C 2 + C A 2 + 2 x OC x CA x c o s 8
or 1002=
48'
2 x 64 x 48 x cos 6
10 000 = 4096 + 2304 + 128 x 48 x cos 8
loo00 - 4096 - 2304
COS e =
128 x 48

- 1OOOO - 6400 - 3600 - 300


128 x 48
128x48
32x16
= 0,586
( d ) Power factor of load = 0 , 5 8 6 (lagging) Ans.
Voltage drop in resistance of inductive load = V,,
= V , cos B = 4 8 x 0.586 = 28.128V

(a) Power absorbed by Ioad = 1 2R, = 8, x 3.52


= 64 x 3.52 = 225.3W
(c) Total Power = 512 + 225.3 = 737.3W Ans.
P
( d ) Power factor of circuit or cos 4 = VI

- 737.3

737.3
--

1 0 0 x 8 - 800

= 0.92 (lagging)

Ans.

8 . Let X I ohms = the reactance at 4 0 H z


Then Z , =. 4200 = 3 0 . 3 0
and Z l = Y
6.66
Let X, ohms = the reactance at 5OHz
then Z , = d m 2
200
and Z , = - = 2 5 R
8
Hence 3 0 . k = R,,
x,'
and 25, = R,, + x , ~
but, since R , = R, then:
Subtracting 30.3, - 2 5 , = x12- X,,
o r (30.3 - 25)(30,3 + 2 5 ) = ( X , - X 2 ) ( X l X2)
hence 5.3 x 55.3 = ( X I - X , ) ( X ,
X,)
o r 293.09 = ( X I - X 2 ) ( X l + X,)
1
k
1
Also since X = --- .'. X = - o r X a -

+
+

2rrfC

XI -50 and X I = -5 X , or X , = 1.25X2


Hence x2
40
4
Substituting

293.09 = ( 1 . 2 5 X 2 - X 2 ) ( l . 2 5 X 2 + X,)
= 0 , 2 5 X 2 x 2325X2
- .---2 2 5 X Z 2 o r 2 9 3 0 9 = 0,5625X2'
4

Ans.

SOLUTIONS TO PRACTICE EXAMPLES

Whence X,'

29309
and A',
0.5625

= a

43 5

22.W

Thus R' = 25' - 22.8' = 625 - 520 = 105


R= m
5 = l0.25Q Ans.

1o6
- lo4 - 10 000
- - -22.8 x 3 . 1 4 ~lo2 72
72
= 139pF Ans.
For the diagram Fig 23, as an example:
Resistance voltage drop = IR = 6.66 x 10.25 = 68.27V
Reactance voltage drop = 1 X , = 6.66 x-28.5 = 190Vetc.

Thus C

Fig 23
9. At resonance 2rcj'L =

---

2nfC
1

1
and f Z = ( Z ~ ) Z L C
or/ = 2n d7.Z
v
Here I can be obtained from I = -

xc

or I = -= V2xfC
1
2xfC
Thus 1 = 100 x 2 x 3.14 x f x 10 x
= 6.28 x f x
B
= 6:28 x
x 2 x 3.14CC

436

REED'S BASIC ELECTROTECHNOLOGY

For Fig 24, if values were required then as an example:


V, = IR = 0.446 x 60 = 26-76V

X, = X, =

1oov.

Fig 24

= 148.3R
Impedance of circuit

z = Jm2
+ (X,- X,)
/400'

+ (148.3 -

1 O6
2 x 3.14 x 50 x 40

'i2

SOLUTIONS TO PRACTICE EXAMPLES

I = - 200 = 0.495P
405
1 o6
V c = 0.495 x
2 x 3.14 x 50 x 40
- 0.495 x 10' - 495 - 39.5,, Anr.
3-14 x 4
12.56
Impedance of coil Z , = d400'
148.3'
= 1o2JFTTW
= 10~JlCiTi3= 1 0 2 m
= lo2 x 4.265 = 426.5R
V,, = IZ,
= 0.495 x 426.552
= 211V Ans.

--+

437

CHAPTER 10

1 . 7.5kW = 7500W This is the output.


Input is 7500

85

'Oo = 8824W

8824 = 80.2A
Motor current = 110
E,, = V - I, ( R , + R,,)
= 110 - (80.2 x 0.12)
=- 1 10 - 9,624 = 1 W 4 V

Ans.

Power input = 90 OOOW


90000 900
C:urrent input to motor =
= --- = 180A
500
5
500
Shunt-field current I, = 00 = 5A
7
-.

Armature current = 180 - 5 = 175A


Back e.rn.f. = E, = V - IaRa
= 500 - (175 x 0.1) or 500 - 17.5
= 482.5V Ans.
3 No load

Full load.

Back e.m f EbU= I' - IaoRa


= 460 - (10 x 0.025)
= 460 - 0.25 = 459.75V Ans
B:$k e.m.f. E,, = V - I,, R,
= 460 - (300 x 0.025)
= 460 - 7.5 = 452.5V Ans.

40
4. ( a ) Current in 1 parallel path = - = 10A
4
Current in 1 conductor = current per parallel path
So f'orcc on I conductor is given by B / l
= 1.2 x 10 x 0.4
= 4.XN Ans,
i

= O72N m

Ans
( b ) T o t ~ torque
l
due to ,111 cond:qctors
= 240 x C, - L
= 172 SN m A n >

SOLUTIONS TO PRACTICE EXAMPLES

439

80 x 172.8
6
= 502.4 x 28.8 watts
14 469W = 14.5kW Ans

- 6.28

'X

- (25 x 0.2) = 220 - 5 = 21 5V


Also, since torque is unchanged, and TI = T 2
and since T cr. Q I , o r T = k o l , , we can wrlte:
k @ , l a , = kQ21az
But Q z = 0.9@,

5. On full load E,, = 220

Also slnce

E
Eb,

k 0 2N2
kQIN1

= ---

= 804 revlmin

Ans.

6. As a generator. 50kW at 250V gives a load current of


50 000
I, = ---- = 200A
250
Field current I , = 250
- - = 5*
50
:. Armature current I, a IL+ If = 200 5 = 205A
General voltage E = V + I,R, + brush voltage drop
or E = 250 + (205 x 0.02) + 2
= 250 + 4.1 + 2 = 256.1V
50 000
As a motor. Input current I, = -= 200A
250
250
Field current I, (as before) = -= 5A
50
.: Armature current I, = I, - If = 200 - 5 = 195A
Back e.m.f. E, = I/ - I,R, - brush voltage drop
= 250 - (195 x 0.02) - 2
= 250 - 3.9 - 2
or E, = 250 - 5.9 = 244.1
Again since E and Ebare proportional to flux and speed then.
E = k @ N and E, = k@N

E
5x400
Thus - =
or 256.1 - 400
E,
5 x N
244.1
N
400 x 244.1
or N =
= 382 revlmin Ans.
256.1
Vote. Since flux can be assumed to be proportional to the
field ampere-turns, and hence the exciting current, then the
current value has been substituted for the flux values 0.
7. On no load.

I,

10s
90

--- =

1.1 7A ILo= 3.5A

lao = 3.5 - 1.17 = 2.33A


Ebo = 105 - (2.33

0.25) = 105

2.3'3

--

105 - 0.58 = 104.42V


On full load. Output = 3 x 1000 watts
3 x 1000-x-100- 1000 x 100
Input = 82
27.33
= 36.6 x 100 = 3660W
3660 = 34.86A
Input line current I,, = 105
la, = 34.86 - 1.17 = 33.7A
Eb, = 105 - (33.7 x 0.25)
= 105
8.43 = 96.57V
N o w E a @ N o r E = k@N
k 0 , No
= and we can write
E,,
k Q 1, N l
104.42 x 1.17 x 1000
o r No = Ebo x @, x N , Eb, X 00
96.57 x 1.17
-= 1080 revlmin Ans.
Again, since T is constant and T cc @Ia,we can write
T , = k 0 2 1 a 2and T I = kcb,la,
k@ 3
I
B U ~T , = T ,
or T_1= 2
T,
I
and 0 , = 0 ,
=

I f R is the added resistance to reduce speed then


I:',> = 105 - 33.7 ( R + 0.25)
Also since Eb a @ and N, since flux is constant
800
then % = 800 o r E,, = 96.57 x Eb.
lo00

Back e.m.f. (at reduced speed) = 77.26V

SOLUTIONS TO PRACTICE EXAMPLES

441

Note. As for No 6. Flux being proportional to field current,


If is substituted for @.
8. Cold condition
Iao = 50 - 11.15 = 48.85A
E,, = 230 - (48.85 x 0.2)
= 230 - 9.77 = 220.23V
Hot condition
Temp rise = 60 - 15 = 45C

Similarly R,,= 200


=
=

200
200

+ (200 x 45 x 0.004)
+ (90 x 0.4) = 200 + 36 = 236Q

/,I
= 50 - 0.975 = 49,025A
E,, = 230 - (49.025 x 0.236)
= 230 - 1 1.57 = 2 18.43V
Again since E, cc CD and N and since CD K If
we can write E,,o = kZfoNO
and Eb, = k l f ,N 1
i r N I = Eb, X Ifo XNo
or!%=

SO

& &
If,

No

and N ,

Ebo

Ebo X

218.43 x 1.15 x loo0


220.23 x 0.975

= 1 160 revlmin

9. (a) V = E,

Ifl

Ans.

+ ZaRa

o r E, = V - IaRa
= 230 - (200 x 0.35) = 230 - (35 x 2)

and E, = 230

70

160V

Z@N P
E,
60,4
Also since E, = -x - then N = - x 60
A
Z
P

(b) Again Torque is given by:


P
T = 0.159 x 2 0 1 , - newton metres
A

230
10. I , = -- = 2.2A I, = 5 - 2.2 = 2.8A
104.5
Also since I f I = 2.2A then I,, = 50 - 2.2 = 47.8A
Again Ebl = 230 - (47.8 x 0.4) - 2
= 230 - 19.12 - 2 = 208.88V
And Ebo = 230 - (2.8 x 0.4) - 2
= 230 - 1.12 - 2 = 226.88V
O

Eb2= k@l Nl
( a ) Since Eb,,
k@O'VO
Thcn Vd = Ebo N~ asrumlng constant flux.
Eh .

(b) At 600 revimin E,, = 208.88V


24ssumrnga constant flux. then for 500 revimin
E,, = 208.88 x 2.

.:

o r Eh, = 1044'4
-- - 174.07V
6
The voltage across the armature has to be reduced by
230 - 173.07 = 55.93V
or since V = c, + I , ( N ,+ R ) + 2
then V - E, = I; ( R , + R ) + 2
o r 55.93 = I, R, + 1,R + 2
SO I,R = 55.93 - 2 - (47.8 x 0.4)
= 53.93 - 19.12 = 34.81

SOLUTIONS TO PRACTICE EXAMPLES

443

(c) Under the new condition, @ is to be altered,-hence If


E
kcP N or E,, = 230 - (30 x 0.4) - 2
. . -!a = -2

Eb,

= 228 - 12 = 216V

Thus current-and therefore flux, is to be reduced to


1.82 = 0.827 = 82.7 per cent Ans.
2.2

CHAPTER 11
1 For the d.c. clrcuit, P = VI or 300 = 60 x I
300
so d.c. current = --- = 5A

60
L'
60
and d.c. resistance = - = - = 12R
I
5
For the a.c. c~rcuit,P = 12R
:. 1200 = I2 x 12
o r l2 = 100 and I = 10A
S~ricccurrcnt taken hv the a.c. circuit F 10A

Thus X = f i 9 - 144 = f l 5R
Reactance of coil = 5 ohms Ans.
2. Impedance of Branch A . ZA=
Current IA

=
-

= --

ZA

100

--

12.4

a= 12.4i2J F12K Z 9
=

8.08A. Also cos $, =

--

12.4

= 0.968 (lagging)

and sin
.1

Impedance af Branch B. Z ,
=

d m i

-= ( X . 0 8

7N 1

+ 400

0.172)

@B

(8.08 x 0.242) - (4.64 x 0,928)


- 1.96 - 4.3 = - 6.26A
- .7
- - - -L/'9.53- + 6.26' = J91
39 =
I IA
Arls.

t4c11c.cI

d64

dlB

0,968) + (4.64
1 7 3 = 0 53A

React~\,ecomponent of current:
ir = - I,, sin qb,, - 1" sin
=
=

3
12.4

= - = 0.242

= 21.652

m
4

. . \ c ~ i \ c cornponcnts o f current.
1 , -: 1, cclh d)A t 1,)

4,

SOLUTIONS

445

M PRACTICE EXAMPLES

3. Let the inductive circuit be circuit A, then:


XA = 2xfL = 2 x 3.14 x 50 x 0.02 = 6.28R
RA = 50n
J~Q
-t- '6.28'
d250 i- 398.5 - 10JSS
ZA
= 10 x 5.05 = 50.5R

..

RA
c~~+A=
- --- =50
--

ZA

50.5

I0 - 0.99 (lagging)
10.1

Let the capacitive circuit be circuit B, then


127R
r
200
I - - = 1.575 cos 4, = 0 sin 4, = 1
' -127
Then I, = (3.96 x 0.99) + (1.575 x 0) = 3.92 + 0 = 3.92A
I, = - ( 3 . 9 6 ~ 0 . 1 2 4 ) + ( 1 . 5 7 5 ~ 1 =
) -0.491+1.575
=

= 4.075A
3'92 = 0.962 (leading)
Total current = 4.075A cos 4 = 4.075
Phase angle 4 = 15" 50' Ans.

4. Let the branches be A , B and C respectively. Then:


X, = 2 n f L = 2 x 3.14 x 50 x 0.02 = 6.1HR
Z A= Jg2
6.28' =
5- J
=5 -

= 10.2R

and cos

4,

8
10.2

= - = 0.785 (lagging)

sin

4,

6-28
10.2

= - = 0.616

'10
15.7
cos 4, = = 0.537 (lagging) sln 4, = --- = 0.845
18.6
18.6

446

REED'S BASIC ELECTROTECHNOLOGY

Then Z , = J202 + 39.8' = J400 + 1584 = Ji-984


= 44.54R
20
39'8 = 0.894
cos ip, = ---- = 0.449 (leading) sin 4, = 44.54
44.54
Aclding the active and reactive current components.
I, = I, cos ip, + I, cos ip, + I, cos 4,
= (9.8 x 0.785) + (5.37 x 0.537) + (2.24 x 0.449)
= 7.7 + 2.88 + 1.01 = 11.59A
I , = -- I , sin $, - lBsin 4, + I , sin @ c
= - (9.8 x 0,616)-(5.37 X 0.845)+(2.24 x 0.894)
= - 6.04 - 4.54 + 2.04 = - 8.54A
l'hrrl I = v/li2 + 1,' = dl ] ' . S O 2 +
= d7747TT7293 =
= 14.38A Ans.
cos 4 =

--

14.38

0.805 (lagging)

36' (approx) Ans.

5. Apparent power = VI = 240 x 50.6 x l o F 3 kilovolt


amperes
= 12.144kV A Ans.
true power
10
Power factor =
12.144
apparent
power
..
= 0.823 (lagging) Ans
9kW
output (power) -- P(tic.~cnc.!=
Input ( p o i i r )
lOkW
or q = 90 per cent Ans
.1

6. Output from motor = 1.5kW

Efficiency = 80 per cent

"0
Input to motor = - x 100 = 1875W
80
Also power input.to motor, V l cos q5 = 1875W
: 1875 = 230 x 1 1.6 cos d
Po\\.cr. colnpc7ncnt of input current. I cos rf, = 1 1.6 x 0.7
= X.12A
I s m ip = 11.6 x 0,714
= 8.28A
A t the neu power fi~ctor.the power component of current
S I?.\
= I , c o \ (),. Also since I, cos 4, = I cos (p
8.12
I , x 0.05
8 . 12 a n d I , =- - - - = 8.55.A

..

React~ve ..

;=

.,

0.95

447

SOLUTIONS M PRACTICE EXAMPLES

Note. cos 4, = 0.95 sin 4; = 0.327


The reactive component of input current, at the new power
factor = I, sin 4, = 8.55 x 0.327 = 2.8A
So reduction of reactive current = 8.28 - 2.8 = 5.48A
and capacitor current = 5.48A
230
1 O6
Capacitor reactance = ---- ohms = 5.48
3x1C

Rating of capacitor, = 230 x 5.48 x


= 1.26kV Ar Ans.
active power 10
-7. Load (a) Apparent power, S, =
power fdctor
1

= lOkVA

Reactive power, Q, = S, x sin

4 = 10 x 0

= OkV Ar
Load (b) Apparent power, Sb =: 80kVA at a power factor
of 0.8 (lagging)
Active power, Pb = 80 x 0.8 = 64kW
Reactive power, Qb = 80 x 0.6 = 48kV Ar
(kwng)
Load (c) Apparent power, S, = 4OkVA at a power factor
of 0.7 (leading)
Active power, PC = 28kW
~ e a c t i v epower,-^, = 40 x 0.7143 = 28.57kV Ar
Total power tnken from the supply, P = 10 + 64 + 28
= lO2kW Ans.
Total reactive power, Q = 0 - 48 + 28.57 = - 19.g
Total apparent power from supply, S = v'1022
IS

Power factor of combined load, - =


S

Mains current

= 104kVA Ans.
102
- = 0.98 (lagging)
104
Ans.

"O0 - 416A Ans.


250
\
$(a) ha& voltage = I ~ O V1rnped;ince per phase of load = !OR
=

100

a.

:. Load current p r phase = -- = 10A

10
Line current = ~ d a s current
e
= 10A Ans.
Total power, P = 3 VI cos q5

1/

But V = d3V,, = 1.732 x 100 = 173.2Vand I = 10A


v'3 x 173.2 x 10 x cos 30
:. P =
loo0

or total power, P
(b) L ~ n evoltage = t/3 x 100
Voltage per phase of load

2.598kW Ans.

3 x 100 volts
x 100
ZPh
10
= t 3 x 10 amperes
L ~ n current
e
= phase current =
x \ 3 x 10
= t

v,-- v 3
Current per phase of load = -1
=

l ot,tl power, I' =

30A

v.3

An\
1

1 ~ 0 11)
4

t 3 x z 3 x 100 x 30 x 0 8 6 6
1 000
= 3 x 3 x 0.866 kilowatts
= 7 794kW Ans.
-

(c)

Line voltage = 1OOV


Voltage per phase of load = l00V
Current ,,

- -=

10A
10
Line current = t / 3 I,,, = 1,732 x 10 = 17.32A Ans.
Total power. P = \, 3 V I cos 4
3 100 \ 3 10 x 0.866
1000
= 3 x 0.866 kilowatts
= 2.598kW Ans.
,

,,

(d)

Line Voltage = lOOV


100
Voltage per phase of load = t'3
100
- 10
10 - 7 3 Z'mPeres
Current
,, .. ,. \,

Line current = Phase current =


10
1.732
Total power, P = v 3 VI cos 4
-

-=

0.866kW Ans,

10
~ / 3

5.77A Ans.

SOLUTIONS TO PRACTICE EXAMPLES

449

9. Output from motor = 45kW = 45 OOOW


Efficiency of motor = 88 per cent
100
Input to motor = 45 000 x 88

Since P =

(a)

3 VIcosc#J
51 140
Line current, I =
t 3 x 500 x 0.9
\

or I =

51
= 65.6A Ans.
1.732 x 450
(b) Output from the alternator = input to motor
= 51.14kW Ans.
51 140 x 100 watts
(c) Input to alternator =
80.
.
51 140 x 5
or output power of prime-mover = 4 ~1000
= 6 4 k ~Ans.

10. Input power to the system, P, = v/3 VI cos c#J


% / 3 x 400 x 40 x 0.8
1000
= 22.2kW
Power output from the motor at 91 per cent efficiency
= 12kW
:.Power input to the motor, P,,, = 12 x 100 13.18kW
91
~ ' x3 400 x 40
Apparent power of system, S, = 1000
= 2 7 . 7 1A
~
Reactive power of system. Q, = S, sin 4
= 27.7 x 0.6 = 16.62kV Ar
T o improve the power factor to unity, the reactive power of
the motor must be equal to the reactive power of the system, so
reactive power of motor, Q, = 16.62kV Ar.
Apparent power to motor, S,,, = /-l
= J13.18' + 1 6 . 6 2 ~
= d173.8
276.3
= 1= 21.21kVA Ans,
113.18
=Power factor of motor, cos &,, =
s,
21.21
= 0.62 (leading) Ans.
Total power taken from the mains = power supplied to the
system
power supplied to the motor = P, + P,
= 22.2
13.18 = 3538kW Ans.

The phasor diagram shows the method of solution. Even


though the problem is a 3-phase one, the diagram, as drawn, can
be a ppl~ed.slnce balanced conditions can be assumed.
L V A O f MOTOR

I /

TOTAL kW

Fig 25

CHAPI'EII 12

Total m.m.f., F = 5 x 500 = 25OOAt


Mean circumference = xd = 3.14 x 300 x
= 0.942m
F = --2500
So magnetising force, H = 1
0.942
= 2654Atlm Ans.

1. (a)

(b) Since po =

then B = po H
and B = 4 x x x
x 2654
= 4 x 3.14 x 2.654 x
= 12.56 x 2.654 x
= 33.33 x
tesla
= 0.0033T = 3.3mT Ans.

(c) Total flux,

Q, = BA
= 0.0033 x 1000 x
= 3.3 x 10-6Wb o r =

weber
3.3pWb Ans

5
B = -Q,= 500 x
7 = - = 1.25T
A
400x104
Also since B = p H = / L , 11, H then

2.

Length of iron = 250 x 10-3 metres


X 1.25
So total m.m.f., F = 250 X
x

o r F = .250

10-3

ampere-turns = 99.7At

Required ampere-turns = 99.7, say 100. Ans.


400 x
400 = 0,8T
500 x
= %%
total magnetomotive force
Also, as H is given by
length
F 500
Then H = - = -- = 500Atlm
i
I
Q,

3. (a) B = - then B =
A

452

REED'S BASIC ELECTROTECHNOLOGY

Also. since B = p H = p, p , H then

or relative permeability = 1275 Ans.

(b) Reluctance

Length Arzi~ p,,/l, x ,+f

ampere-turnrlweber

/I Y

1.25MAiWb Ans

F = 400 x 2.5
4. (a) H = ampere-turns per metre = Thus H =

B = 0.5 x 1.25
pHorp = H
loo0
p = , ~ ~ ~ p "

Again B
Also

I
1.25
Ioo0 - 8OOA t/m
1.25

I =
( h ) Rcl\lct;lncc, .S -- -/,,,l

--

0 027

I --25x

I 500

0.625
loo0

- --

103
v

1 (I

(I

ampcrclurns/weker

453

SOLUTIONS TO PRACTICE EXAMPLES

square metres
11.3
= - teslas
12

5. Area of air gap = 1200 x


1.13 X
.'. BA = 12
also since B

=I

then H for air =

p,H

B
=
p
12

11.3
x

4x x

lo-7

- 113 x

lo6

48x
metre

The magnetomotive force for the air gaps is given by:


75 x lo4 x 2 x 2 x
ampere-turns
= 300 x 10 = 3000At
Area of iron is the same as for the air gaps
.: B value of the iron is the same
Using the graph of Fig 26, we see that, for atflux density of
0,942T. the ampere-turns per metre length of the iron = 850.

I. 0

<

-I

w
fi

0.5
0

XX,

Fig 26
Since length of iron path = 0.6m
:. M.M.F. for iron = 0.6 x 850 = 6 x 85 = 510At
Total magnetomotive force required = 3000 510 or
3510At Ans.

6. Circumference of flux path = rr x 0.2 = 0.628m


= 0402m
Length of air gap = 2 x
.
.. iron = 0,6261~1
This problem I S best solved by trial and error thus:
Assume a flux density of 0.5T in the air gap and iron since these
;ire of the same cross-sectional area. Then using the graph of
Fig 26:
M . M . F . for iron = 520 x 0.626 = 326At

Total m.m.f. would be (326 + 795) = 1121At. Thus too low


a flux density has been assumed.
Again. assume ;I flux density 0.6T. then:
M . M . F . for iron = 585 x 0.626 = 365.21At

M.M.F. for air

E6*4x

l
10-

3 or

55

of tl~.at required
for 0.5T
6
= - x 795 = 1.2 x 795 = 954At
5
would be 365.2 + 954 = 1319At-still
too
=

Total n1.m.f
low
Assume 3 flu3 density of 0.7T. T h e n :
M . M . F . for iron = 660 x 0.626 = 411.16At
7
M . M . F . for air = - x 795 = 1.4 x 795 = 1113At
5
1526At
Total m.m.f. would be 413 + 1 1 13
Thus for an exciting ampere-turn value of 3 x 500 = 1500.
the estimated flux density in the air gap would be a little less
than 0.7T Ans.

7 . Since the B value in the cores is to be 1.2T. rlicn tlie A t l n l


requlred u.ill be 650. This is seen from the graph of Fig 27. Tlic
total ~ n a g n e t o ~ n o t i vI'orce
e
for thc cores will be 2 x 160 x
l o - ) x 650 = 32 x 6.5 = 208At.
In the \.ekes, the flux is the same a s t h a t for the cores a n d the
flux density will therefore be different. as the areas are different.

Thus flux,

Q,

= 1.2 x

rr x 50' x

weber

SOLUTIONS TO PRACTICE EXAMPLES

455

Fig 27

From the graph, the At/m for a density of 1.066T is 330


Mean lcngth of flux path in yokes = ( 2 x 130) + (2 x 47)
= 354mm = 0.354m
Total m.m.f. for the yokes = 330 x 354 x
= 3.3 x 35.4 = 116~8At
Total m.m.f. for complete magnetic circuit
= 208 + 116.8
= 324.8 say 325At Ans.
8. (a) Pull of magnet = 196'2N or 98,1 N per contact face

B ~A
B*A
Also Pull = newtons
2 ~ , 2 4~ x l o - ?
B~A
. ~ 98.1
,
=
8~
10-~
98.1 x 8n x lo-' x 4
Whence B~ =
x x 15 x 15 x

f i3

and B = \
m
6= 1.185T
From the graph of Fig 27, for a B value of 1.185T,
the H value = 560Atlm.
n
square
Area of one contact face = - x 1 5 x~
4
metre
n
Flux, @ = BA = 1.185 x - x 225 x lo-' weber
4
= 0.296 x 3.14 x 225 x lo-"
= 0.296 x 3.14 x 2.25 x
= 2.09 x 10-4Wb
Since B for horse-shoe magnet
:I I X51' tlic11,llro1111:ig 27, the i f vulue = 560
At/m
Length of magnet path
115 = 180.5mm = 0.1805m.
= rt x
&

Note. Mean circumference of ring


= 27t x (radius of ring)
= 2x x (50 + radius of rod)
= 2rt x (50 + 7.5) = rt x 2 x 57.5
= n x 1 1 5 millimetres
Then m.m.f. for magnet = 560 x 0.1805
= 101.1At
Flux - 2.09 x
2.09
B value for armature = -6= -Area
1j2
x
10
2.25
d
F 0.9321
and H value = 215At/m (from Fig 27)
Length of armature path = (115 15) = 130mm
= 130 x
metre
So m.m.f. required = 215 x 130 x
= 13 x 2.15 = 27.95At
Total magnetomotive force required = 101.1 27.95 =
129
129At
Cusrcnt = -- = 0.268A Ans.
480
In the air gap B = 1.15 teslas
(h)

:. Flux, @ = 1.15 x

rt

2.03 x
2.03 x
Flux density in core =
n x-. 152
-4

x 1 5 x~ lo-.'

weber

=
x

lo-"

457

SOLUTIONS TO PRACTICE EXAMPLES

Length of core path = x x 57.5 = 180.5mm


= 0.1805m
M.M.F. for core = 0.1805 x 470 = 84.8At
2.03
Flux density in armuture 152-4--j-6-6
- x lo-* a 2.03
-j:j5
3
:

= 0.905T

From curve H value =


Length of armature path =
M.M.F. for armature =
Flux density in I air gap =
1.15
B
... H = - =
4~
lo-'

205At/m
130mm = 0.13m
0.13 x 205 = 26.65At
I.15T, But B = p,H
1.15 lo7
ampere12.56
turnslmetre

M.M.F. for 2 air gaps


- 2 x 0.5 x
-

x 1.15 x lo7
12.56

9.125 x lo2 ampere-turns = 912.5At


Total m.m.f. for circuit = 84.8 26.65 + 912.5
= 1023.95, say 1024At Ans.
=

9, (a)

10-6

n1m2
Area of air gap = 4

- 314
-

Volume of air gap =

l o ' square metre

4
3.14 x 10''

2.5 x 10-3

= -3'14 x lob4 cubic metre

16

~ i u dinsity
x
in gap

T[ X

0.004 x 4
loo2 X

Energy stored in joules =

x
B~

-x
2

lo' -

tesla = 0.508T
lt

Volume

0.5082
3.14
10-4
X 2 4~
lo-'
16
- 0.508' x 10' - 0.258 x 103joules
8 x 16 128
258
-= 25 Ans.
128

458

REED 'S BASIC ELECTROTECHNOLOGY

( b ) Pull (newtons) =

B 2 -~ 0.508'
----- -

x 3.14 x
X 10- 'X 4

2p,

2 x 477

806N

Ans.

Useful flux = 0.05Wb/pole


0.05
Flux density in air gap =
60000 x

10. .Air gap.

Also B = p,H
B
. . H value lor alr = - =

M.M.F. for air gap

=
=

0.833

- 833
x 104
---

4nx10-~
471
66.2 x lo4 ampere-turnslmetre
66.2 x lo4 x 5 x 10-3

SOLUTIONS TO PRACTICE EXAMPLES

459

Total flux = 0.05 x 1.2 = 0.06Wb


0.06
Flux density in pole =
- 6 teslas
40 000 x
-4
o r B .= 1.5T
From the magnetlc characteristic, plotted for Fig 28. a tlus
density ( B ) of 1.5T gives an H value of 2'CKX)~tlm.
Thus m.m.f. required for pole = 2000 x 250 x lo-' = 500At
Pole.

Teeth. Total flux = 0.05Wb (same as the gap)


0.05
Flux density in teeth =
25 000 x
= 2T
From the characteristic, a flux density of 2T, gives an H value
of 24 000At/m.
Thus m.m.f. required for teeth = 24 000 x 45 x
= 24 x 45 = 1080At
Total field-coil m.m.f. = 3310 500
1080+
= 4890 say 4500At Ans.

CHAPTER 13
1. For a series combination, the equivalent capacitance is given
1
1
1
2+13
by C, where - = -- + --- = -- C 0.02
0.04
0.04
0.04
0.04
o r C = ---- = 0.0133pF Also Q = CV
3
Q = 0.0133 x
x lo2 coulombs
1 3 3 x lo-" = 6 6 ~ 7 Vrind V , = -.-.-1.33 -.-.
x
l'I1c11 I =
0 4 2 ' ~lo-';
0.04 x lij-"
= 33.3v.
'I'he voltage drops are respectively 66.7V and 33.3V Ans.

',

2. The final two parallel 5 p F capacitors are equivalent to one


unit of 10pF.
The capacitance C of the branch, consisting of 20pF, 1OpF
and 20pF in series, is given by:
1 + 2 + 1 = - o 4r C = 5 p F
-- - + - + - =
c - 2 0 10
20
20
?O
This series circuit is in parallel with a 5pF capacitor, making
the equivalent capacitance = 10pF
The final arrangement between A and B is now equivalent
to a 20pF, 10pF and 20pF capacitor in series. The equivalent
capacitance is g v e n by:

3. Since Q = CP'. :. quantity of electricity received initially


is given by Q = 1000 x
x 100 = lo5 x
= 10- coulombs.
Since the plates are separated by an insulated rod there is no
loss o f charge a n d hence Q remains the same.
Under the new condition since, as before, Q = CV then
lo-'
lo5 - 10' -- -1 0 0 = 333
' I 1 r 1 I' - Q 3oux 10--~=300- 3
3
Thus the potential difference will have ~ncreasedby 333 3 100 = 233.3V Ans

,"

4. 1-he c;ipacltor \vould be made up from 10 plates In p;~r:~llcl,


rnahlng one nsscmblq. interleaved with 9 plates in parallel
li)r.~~llrlg
tllc olllcl. plalc assembly: Tlicrc will be I S I I I ~ C ; Ihcp;i,--

46 1

SOLUTIONS TO PRACTICE EXAMPLES

ators or 18 electr~cfields and the total capacitance will be 18


times the capacitance between one pah. ,f 7lates.
E,E, A
Thus C of one pair of plates = EA
- =1
I

= 88.5 x 7 x 2.58 x 10-l2 = 1.6 x


farads
or with eighteen units in parallel
C = 18 x 1.6 x
= 28.8 x
farads
= 28.8 x l o w 9 x lo6 microfarads
= 0.0288pF Ans.

5. Since Q = CV, then Q = 3 x


x 1 0 i 6 x 10 x lo3
= 3 x
coulombs
3 x
- 3
Q :. Flux density, D = A
10OOOx10-~-~
= 3 x l o v 4 coulomb per square metre
Ans.
electric
flux
density
D
-Also, permittivity, E =
electric force
d
And electric force, d =. - = lo
lo: = 10 x 106volts per
1
1 x 10metre
3 - ~1 0 - ~
Hence
E =
~ I S O E = E , x E,
= -C =

and

&

or

E, =

6.

E,

+ 3

30
= 3-39
8.85
EA

= - where

!= 3.5 x

and
Hence

E = E,

x 1 0 4

1 0 x lo6 x 8 . 8 5 ~lo-''

- 3 x /OH
8 . 8 5 ~10'

Ans.

A = 6 x lo4 x
= 6 x
metres

square metres

square metres

E,

= 8 . 8 5 x lo-'' x 3
8.85 x 10-l2 x 3 x 6 x
C=
3.5

Energy. W = ;Cv2 joules

1,264 x 16.2 x

= 20.477 x

joules = 2 0 . 4 8 ~ 5 Ans.

7. A 10-plate capacitor I S made up from two 5-plate assemblies


intr.l.lr.;~\cdwith ei\ch other and separated by the dielectric.
I I ~ r arc
c tl1115 nine electric tields or the final capacitance is nine
tlrnc4 t h ; ~ tof one plate arrangement.
t lcrc

('

1,

/I

wliere A

I500 r 10 - " square metres

0.3 x

metres

F: = Eo X 6 ,

8.85 x

lo-'*

x 4 x 15 x
3
10-4
= 2.95 x 1 0 - l 2 x 60 = 2.95 x 0.6 x 1 0 - l o
= 1.77 x lo-'' farads = 1.77 x
microfarads
Total capacitance = 9 x 1.77 x
= 15.93 x
= 0 ~ 0 0 1 6 g F Ans.
(' =

8. Let

of' the series arrangement.


1
1
1
3 + 2
5
then- = - + - = - = - o r C = 12pF
C
20
30
60
60
The clii~rgestored is given by Q = C V = 12 x
x 600
= 72 x
coulombs.
7 , 2 ~ 1 0 - ~ 3.6
f'.L). across 20pF capacitor A = --------20 x
=
= 360V Ans.
I'.D. across 3OpF capacitor B = 600 - 360 = 240V Atis.
If' P . D ;Icrc>ssF3 i.; 400V thcn P.D ;Icross p;~r;~llcl
;lrr;lnpcmcnt
ivould Ilc 7 0 0 V . A l w tllc cquiv:~lentcapacitance must bc 60lrI:
(double) since the voltage is half that across B.
(' mlrst be 30jrF, bcinp in parallel wit11 A
A i w the energy stored, W = -fCV2
=
x 40 x
x 20O2
= 4 x 2 x 10- joules
'Thus LC' = 0.85 Ans.
(' = capacitance

'

SOLUTIONS TO PRACTICE EXAMPLES

463

9. Since Q = CV and Q = Ir then I1 = CV


V
o r I = C - where V = the voltage change
I

50
amperes
1 x 10= 2 x lo-' x lo3 = 2A Ans.

ii. I = 40 x

iii. I = 40 x

O
1 x 10-

iv. I = 40 x

lM) amperes = 4A Ans.


1 x 'lo-

v. I = 40 x

50
amperes = 2A Anr.
1 x 10-

amperes = OA

Ans

The accompanying graph (Fig 29) shows the current and


voltage conditions.

Fig 29

1 1 000 = 1 I 0000
10. Reactance of cable per kilometre = ----1
6
lo6
I
0
Also X, = ---- ohms
2nfC
2 x 3.14 x 50 x C

whence I I x lo3 =

lo'
andC=
lo
microfarads
3.14 x C
1 1 x 3.14
or C = 0.289pF
electric flux density
Again since permittivity =
electric force

Bur Q
I)

= C'l'
r

'I'

-.

/1

"Y

lo-'

II

lo'

coulomb per \qu.,rc


metre

1 1 ~ 1 0 ~
10 x 1 0 - ~
= I l x lo5 \,olts per metre
The mean diameter of the insulation = 10 + 12 = 22mm
The area of the dielectric
= mean circumference x length
= rrd x 1000 square metres
= 3.14 x 22 x
x lo3
= 3.14 x 22 = 69.1m2
Also, from the above,
x l l x 10' coulomb per
n = CV = 0.289 x
,.A
3.14 x 22
square metre
- 3.179 x lo-'
69.1
D
0~289x11~10-~
Again E = - =
8 3.14 x 22 x 1 1 x 10'
- 2.89 X
3.14 x 22
Also E = E , x E ,
dnd & =

.'

F
'

- =

=- -

2.89 x

lo-'

3.14 x 22 x 8.85 x lo-"

I . Draw tile circuit diagram


Let I 3 amperes = current in battery towards junction C
,,
,, C,D away from
,,
,,
,. I ,
.,
,,
,. CA ,. .
12
I\'otr. There I S no current between A D .: I , flows in A B and
I , in D B .
Applying Kirchhoff s laws l3 = I l + 1,
Also point A is at the same potential as point D
:. Voltage drop in A B = .oltagc drop in DB or 31, = 61,
giving I2 = 21,
Also voltage d r o p in CA = voltage drop in CD o r 81, = . y I ,
Substituting in the above 8 x 21, = .uI, o r s = 16.Q Ans.
Also, if R = the resistance between points C and B with no
current in Y

T h u s 1,

3,

3,

10
7.33

- -l o

8.33

= 1.2A

Ans.

2 . Let I , in current in battery X and I , = current in battery Y


Applying K i r c l ~ h o f f slaws, we can build up the equation\
. . . (a)
8 = 1 . 5 / , + 6 ( 1 , + 1,)
7 . 5 1 , + Of2
. . . (b)
and 4 = 31,. -t 6 ( 1 , + 1 2 ) = 61, + 91,
Solving (:I) and (b) we have 24 = 22.51, + 181,
8 = 121, + 1812
16
Subtracting, 16 = 1 0 ~ 5 1o, r 1, = -= 1.524A
10.5
Also from ( b ) 91, = 4 - 61, = 4 - 6 x 1.524 = 4 - 9.144
= - 5.144

Thus the assumed direction for current 1, was wrong and


battery Y is being charged.
Current in battery X = 1 - 5 2 4 A (discharge) Ans.
,,
,, Y = 0 . 5 7 1 A (charge) Ans.
Total current = 1,524 - 0,571 = 0 . 9 5 3 A in 6R resistor
So terminal voltage = 6 x 0.953 = 5 72V Ans.

3. ( 1 ) By Maxwell's Method. Refer to diagram below (Fig 30).

Fig 30
Equatlng voltage drops to e.m.fs. then:
21, + 3(I, - 12)= 10 - 12 o r 51, - 312 = - 2
o r - 31, + 1 3 1 2 = 12
3(12 - I,) + 1012 = 12
giving 151, - 91, = - 6
and - 151, + 6512 = 60
54
Adding, 5612 = 54 or I2= - = 0.964A
56
So 51, = - 2 + 3 x 0.964 = - 2 + 2.892 = 0.892
0,892
o r I, = -= 0.1784A
5

Current I, in 10Q resistor = 0.964A Ans.


..
I, in 10V battery = 0.1784A = 0.18A Ans.
. I I
in I
,
= 0,964 - 0,1784 = 0.7876A
= 0.79A Ans.

(2) By Superposition of Current Method. See the attached


diagrams (Figs 31a and 31 b).
2 OHMS

- - - ---

J OtcMI

I,

2 OHHI

I
0 OHMS

I,
A

467

SOLUTIONS TO PR A C TI C E EXAMPLES

(a) Only IOV battery is effective. If R = resistance of parallel


circuit
1 = --1 0 + 3
13
30
t h e n -1 = -1 + = - o r R = - = 2.3150
R
3
10
30
30
13
Total resistance = 2.3 15 + 2 = 4.31 5 0
So current I, = -- 2.325A
4.315
Voltage drop across R = 2.315 x 2.325
2.315 x 2,325 - 5.35 =
Current I2= --- - 3
3
,,
1 3 = - =5.35
0.535A
10
(b) Only 12V battery is effective. If R = resistance of
parallel circuit
Total resistance

+ 1.666 = 4.666R

l2 =-

2.575A
4 66
Voltage drop in parallel section = 2.575 x 1.666
= 4.29V
Current I S = 4'29 - 0.429A
10

So current I4

4'29 - 2.145A
Current I6 = 2
So current in 12V battery = I, - I,
= 2.575 - 1.783 amperes
= 0.79A Ans.
,,
,, IOV
.. = I, - z6
= 2.325 - 2.145 amperes
= 0.18A Ans.
Current in 10R resistor = I, + I,
= 0.535 + 0.429 amperes
= 0.9644 Ans.
4. Let the currents be as shown in the diagram (Fig 32)

SOHMS

12 OHM$

1
1
0 OHMS

OHMS

VOLTS

Fie 72

WLTS

Taking .the mesh formed by the 4V battery, the 5R. IOQ


and 12R resistors and the 6V battery.
4 - 6 = 51, + IOI, - 121,
(a 1
o r - 7 = 51, + 10lz - 1113
Taliins the mesh formed bl, the 4V battery, the 5R reslstor
n ~ i d15R resiqtor.
4 = 51, + I S ( / , - I*)
. . . (b)
o r 4 = 201, - 1512
T a k ~ n gthe mesh formed by the 6V battery, the 12R resistor
and the 852 resistor.
- 6 = - 1213 - 8(12 + 13)
or 0
0 1 , t - 812
. . . (c)
- 8 = 201, + 401, - 481,
Multipl! ( a ) by 4.
\ I I I I I I ,IC.I (11)
4
?Or, - 1 512
glLlng - I2 = 551, - 4XI3
. . . (d)
M u l t ~ p l >( c ) by 7.4. 14.4 = 4813 $ 19.21,
- 12 = - 481, + 551,
Adding ( d l
..- -- -2.4

Thus 1,

74.21,
3.4
- = 0.0324A
74.2

S u b \ t ~ t u t ~ for
n s I, In ( c )
then h = 701, + 8 x 0 0324 o r 6 = 201, + 0 2592
6 - 0.2592 =
glvlng 1, =
20
Current in 8 R resistor = I, + I, = 0.0324 + 0.287
= 0.3194A o r 0.32A Ans.
.d

5. W ~ t hon11 the 4V battery eff'ective as hhown by the d ~ a g n i m


( F I 33)
~
S OHHI

10OHHS

I2 OHMS

8OHMS

VOLTS

I5 O H M S
'vvvvv

Let R be the resistance of the 12Q and 8 R resistors In ptirallel

S O L U ~ I O N STO PRACTICE EXAMPLES

469

Resistance of branch A = 10 + 4.8 = 14.8C2


Resistance of branches A and B in parallel = R .A.R
1
1
1
29.8
222
= 7.43R
Then -= - + - = --- or RAB=
RAo
14.8
15
222
29.8
Total resistance of circuit = 5 + 7.43 = 12,4352
4
Circuit current = --- = 0-322A
12.43
Voltage drop across A = 0.322 x 7.43 = 2.59V
2.59
Current in A = --- = 0.1615A
14.8
Voltage drop across 12 and 8 0 resistors = 0.161 5 x 4.8
= 0.775V
0.775
Current in 8C2 resistor = -= 0.097A

With only the 6V battery effective, as shown by the diagram


(Fig 34).
p

5 OHMS

10OHMS

12 OHMS

Fig 34

Let R be the resistance of the 5R and 1 5 0 resistors in ~arallel


Resistance of branch A

10 + 3.75 = 33.750

Resistance of branches A and B in parallel = RAB


li = 1 + -1 = 8 + 13.75 21.75
-Then --RAB 13.75 8
110
110
Total resistance of circuit = 12 f 5.06R = 17.06R
6
Circcit current = -= 0.352A
17.06
Voltage drop across branch A = 0.352 x 5.06 = 1.78V
1.78
Current in 8C2 resistor = - = 0.2225A
8

Currpnt due to 4V battery


= 0.097A from junction of IOR and 12R resistors to
batter) - ve tcrminal
Current due to 6V battery
= 0,2225A from junction of ]OR and 12R resistors to
battery - ve terminal
These currents can be added. thus:
Current In 8R resistor = 0.3195A or 0.32A Ans.
6. Consider thc top junction or a p e s of the network (Fig 35).

rl

Fig 35

Let 1, amperes flow in the 2R reslstor towards this junction


Let(,
.. ,. resistor A away from this junction
Let 1,
,,
. ., 6R resistor in the direction left to
right
Then using Kirchhotl's laws, we can build up the following
equations.
~~~~~~~~~~I I S I 21, t 41, - ()I2 = O
(a)
Top circuit
21, + 4(11 - 1,) = 12
. . (b)
I l l l - l l l l l l 1 1 1 1 I
I , ) ( I t I , ) - 4 = 0 . (c)
C s ~ n g( a ) and ( b )
21, + 41, - 612 = 0
6 - 41,
= 12

47 1

SOLUTIONS TO PRACTICE EXAMPLES

Substituting in (a)
211 + 41, -'6(1.3311 - 2) = 0
21, + 41, - 7.981, + 12 = 0
41, - 5.9811 = - 12
Substituting in ( c )
41, - 41, - 812 - 81, - 41, = 0
411 - 812 - 1613 = o
41, - 8(1.331, - 2) - 161, = 0
41, - 10.641, + 16 - 161, = O
- 6.6411 - 161, = - 1 6
or 6.641, + 161, = 16

(d)

. . (e)

Multiplying (d) by 4 and solving with (d) and (e)


161, - 23,921, = - 48
161, + 6.641, = 16
Subtracting
- 30,561, = - 64 or I, = 2.09A
using (b)
611 - 12 = 41, or I, = 1.51, - 3
thus I, = (1.5 x 2.09) - 3 = 3.14 - 3 = 0.14.A
Current is 0.14A (downwards) Ans,
7. XA = 27tfL = 2 x 3.14 x 50 x 0.1 = 31.4R
Z A 2 = 30' + 31.42 = 900 + 986 = 1886
A'so '

1o6
- 1o3
- ---- ohm
x~ 3.142 x 50 x 30
9.42

9.447

Joint G = 0.0159s
Joint B = - 0.007 035
Y = d 0 . 0 1 5 9 ~+ 0.007 032 = I O - J~ 1 . 5 9 ~+ 0.703'
= 1 0 - ~ J 2 . 5 2 5 + 0.50 =
Jm
= 10- x 1.74 siemens

'

z= lo' = 57.6Q
1.74

Joint impedance = 57.6R Ans.

The equivalent resistance = G x z2= 0.0159 x 57.6'


= 52.8R

The equivalent reactance = B x z2= 0.007 03 x 57.62


= 33.18 x 0.703 = 23.320
230
Circuit current = - = 3.99A
57.6
Power = 3.992 x 52.8 = 844W Ans.
8. Branch 1 of section AB
,ind Z,' = 602 + 63 5 2 = 3600 + 4050 = 7650
Branch 2 of cection AB
7 x 314 x 50 x O 2 5 = 3 1 4 x 25 = 7 X 5 R
\,
'ind L,' = 60'
78 S Z = 3600 + 6150 = 9750

Total (; = 0.01 4S
63.5
Also B, = - = 0,008 31s
7650
78 5
B2 = - -= - 0.008 06s. (Note the
9750

I-e sign)

Total B = 0.000 25s [ r B = 0 s


+ 0 02s2 = lo-' x 1.4 = 0.014s

and

1. = 10-2J].42
1

Z =

= 71.5R
8.014
OR
Equivalent R = 71.5R Equivalent X
Total R = 100 + 71.5 -. 171.50
X = 94.2 + 0 = 9 4 . 2 0 .
,
Note ,'A = 2 x 3.14 x 50 x 0.3 = 94.20
.. Z = fi71.5' + 94.2'
= 1 0j 1 7 . 1s2 + 9.322
= 10~'?94.!2 + 88.74
= 10\/?83-06 = 10 x 19.7 = 197R
8

Impedance of scctlon BC' = v/ 10O2 + 94.1'


i H 840
137.4
or ZBc = 1.17.4C)and \olt:~gcd r o p = 2.54
= 348.99V
7'hu\ 3 4 0 V will I>c tlic vo1t;lgc ;Icro\s \cctiorl I%('

SOLUTIONS TO PRACTICE EXAMPLES

a n d ZA2=

j2

9.422 = 9

+ 88.74

473

= 97.74

a n d ZB2= 10O2
7.8' = 10 OOO + 60 = 10 060
Branch C X = X, - X C
1o6
= (2X 3.14X 5OX 0.02) 2X 3.14X 50X 3 0
Zc2 = 72 + 4.322 = 49 + 18.66 = 67.66
3
-9.42 = -0.0964s
Then GA - ---- = 0,0307s
B A = --- 97.74
97.74

Total G = 0.1436s
Y =
=

~
lo-"

Total B = - 0.031 8 s
o - ' J
10-'J2,062
~ o
+ 0,101
T
= l o - ' X 11.471 = 0.1471s

z=--1

- 6.798R
0.147 1
T h u s circuit impedance Z = 6.798R Ans.
E q u i v a l e n t resistance R = G x Z 2 = 0.1436 x 6.79g2
= 0.1436 x 46.44 = 6.5R An5
Equiv;~lcntrc;~ct;incc(intluctive, sincc total R i s -- v t )
X 2
' = 0.0318 X 6.798'
= X, =
= 0.03 18 x 46.44 = 1.48R Ans.

%, = R, = 2 0 0 X A = OR
X = 2 x 3.14 x 50 x 0.05 o r XB = 15.70R
Z R- 2 = 52
15.7' = 25 + 246.49 = 271.49
1o6
Circuit C X =
2 X 3.14 X 50 X 50

10. C i r c u ~ A
t
Circuit B

Then

GA = -R~
7 -

Z,

-,

20i
= - = 0.05s
20
20

Thus total G = 0.0684s

~ =

474

R E E D ' S BASIC ELECTROTECHNOLOGY

0
20
15.7 = - 0'0578s
---271.5
1
-= 0,0157S
63.65

BA = 7
XA - - T =OS

Similarly

z,

X7
B
BB = =
ZB
B

Xc
ZC2

Total B = - 0.0421s
Hence Y = 10-2J6,842 + 4 . 2 1 =
~ 1 0 - ~ 2 / ~ m 2
x 8.05 = 0.0805s
- 10-~\/64_51
=
I
.
.
I'}
IOO h 0.0805 :H,OSA A I ~ s .
-

(-.irctilt p o w c kictor is plvcn by


tllinu

;lnd is lagging. since 6 is


'

o r riel ~ n d u c l ~ v c :

: cos

0.0684
= 0.85 (lagging) A n s
0.0805

CHAPTER 15
1 . Anode current (amperes) =

power d i s y y t e d (watts)
--.
anode to cathode voltage (voli,)

Thus anode current = 2.1mA Ans.


2. The d.c. resistance of the valve, for the condition, is given
- anode to cathode voltage
anode current
58
= 9.666 x lo3 ohms
thus d.c. resistance, R =
or9.67kQ
Power dissipated is given by: P = V,I,
Thus P = 58 x 6 x l o p 3 = 348 x

or P = I , ~ R
watts or 348mW
Ans.
o r P = (6 x 10-3)2 x 9.666 x lo3 = 36 x
x 9.66
= 347.76 x
watts o r 348mW Ans.

3. Resistance, r, =

1
conductance

or r, = 1
=

1.64 x

- 8.13
-

x lo3
1.64-

4.957 x lo3 ohms = 4.96kQ

Ans

- change in anode voltage

4. A.C. resistance =

change in anode current


129-75
- 54 x 103
o r ra =
(22 - 12)io-%10
= 5.4 x lo3 ohms o r 5.4kQ Ans.
D.C. resistance, R (for 75V condition)
75
= 6.25 x l a 3 ohms
12 x 10= 6.25kQ Ans.
D.C. resistance, R (for ,129V condition)

129
= 5.864 x 1030hms
22 x 10= 5.86kR Ans.

5 . T h e c h ~ ~ r a c t e r ~ sat pi cp e a r s t o be straight line between t h e


10.5 a n d 4.2rnA values a n d , o v e r this region, t h e 2.c. reclstance \vould he
95 - 15
rS i ,' ra =
(51,
(10.5-4.2)lO-"

- .l o
6.3

lo'

1 5 9 L R An,.

Fig 36
6 Tlic c l i ; ~ r ; ~ c t e r ~ 1s
\ t ~plotted
c
a s s h o w n a n d the load line 1s
d r a \ \ n In t I ~ u \ .
I
A h \ r ~ ~ nI:,c = O T h e n voltage o n a n o d e ~ v o u l dbe 6 o V t o
! ? I \ C poll11 ,\
II

. \ \ \ L I I ~ ~il:'
I~

\.II\c'

resl\l;lnce 10 be o t ' r n l n ~ m u balue.


~ ~ i Thc

I \ l l l ~ l \ o l ~ t < ~ l l l, 1c1 1~
k I l 111%
g l \ c \ tI1c lo:l(l I l l l C 1'01., I
l o a d - r e \ ~ \ t a n c e v:~lueof' 30OR.
Tlic p o ~ n to l ' ~ n t c r s e c l i o nwith t l ~ ec l i a r a c t e r ~ s t ~Ic\ I' a n d rhc
standing current 15 IOOmA. A n s .
T h e power dissipated in t h e load resistor
= (100 x 1 0 - 3 ) 2 x 300 wiitts
=
I0
r 300 = 3W Anh.

l J 0 l l ~ lI{

'

477

SOLUTIONS TO PRACTICE EXAMPLES

- - ...~.~ .
~

.. .

-. . -.-. -

- -

- -.

200

IS0

,-.5

-z

$100
3
V
4

cl
z

50

1
0

K,

X)

ANODt V O L l A C t

43
VO(VOLTS)

Y,

Fig 37
Alternatively (from the load); for polnt P, V , = 30V
. . Voltage dropped across load resistor
= V - Va = 60 - 30 = 30V
Power dissipated = voltage across resistor x anode current
= 30 x 100 x
= 3W Ans.
7 . Tablc of results.

Sincc I, = Va3i2 thcn:


Anode voltage ( V )
0 1
2
3
4
5
Anode current ( m A )
0 1
,.
,,
(mA)
0 1
2.83 5 . 2
8
1 1.2
Load voltage (V)
0 1 ' 2.83 5.2
8
11.2
Supply voltaze (V)
0 2
4.83 8.2
12
16.3
Specimen calculation, for the 2V anode voltage condition :
- 1 8 = 2.831nA
1, = V a 3 , ' 2 , So I 33 2 - (23 ) 1 ' 2 - The supply voltage for this condition
= load voltage drop + valve voltage d r o p
= /,R + 5 = (2.83 x
x lo3) + 2
o r V = 2.83 + 2 = 4.83 volts.
The deduced valve static and dynarn~ccharacteristics :\re
plotted,, as shown below. with I, to a lia and V base. Fro111
the latter, the value of anode current, for an applicd voltage
o f 8V, 1s seen to be 5.1mA.
Note that this result could have been obtained by drawing

i
I

--

--

VOL1 A C k

Fig 38
a load line on the static characteristic for a resistor of I kR
and a supply voltage of 8V. This is shown dotted and the
current is seen to be 5.1mA. For this problem, since the
dynamic characteristic has been deduced the load-line
method is unnecessary.

8. The problem is solved by plotting the valve characteristic


and drawing the load line for the resistive circuit. Solution
will be assisted, if the circuit is drawn out.
Consider the valve on open-circuit ie non-conducting.
Then the current through the resistive circuit would be
,111~1 [tic

\oI[,~gedrop across the anodc rcslstor woulci hc


4
10
lo3 = 40v
Thc p.d. between anode and cathode 31' the calve, l o r
zero I, would be 240 - 40 = 200V
This gives a kalue for point A on the load line.
Next consider the valve 'short-circuiting' i c ~witli no p . d .
across it. The full 240V would be applied across the anode
',

resistor and the current would be --i40


10 x 1
F = 24mA This is

479

SOLUTIONS TO PRACTICE EXAMPLES

for a zero anode-voltage condition and gives point B on the


load line. Join AB and the intersection point shows a value of
60V applied to the valve and an anode current of some 17mA
Ans.
A s a check, we have: For 60V across the valve, ii current

= I.2rnA through the 50k22 shunt circuit. With


50 x 10
180V would be dropped
60V across the valve, 240 - 60
180
across the anode resistor ie a current of 10 lo-"
=8mA
:. Valve current = 18 - 1.2 = 16.8mA. a s indicated by
the graph. Ans.

of

2 6 0

J
t-

IY
3

40

a
V

:2 0

z
4

100
ANODE

200
VOLTAGE

250

v~(v)

Fig 39
9. P = V I , where V is the forward voltage of the diode for ;I
current of 5A.
:. P = 0.7 x 5 = 3.5W k n s .
The rate of heat dissipation per 'C rise is 0.1 J, s o r 0.1W
Thus the rate of heat dissipation for a 70C rise is
0.1 x 70 = 7W
If the voltage drop is constant a t 0.7V then the diode

7 = 10A Ans.
current is given by I, = 0.7
10. Since a number of examples involving the use of a load
line have now been illustrated. this problem should present
no difficulties. T h u s :

( a ) F o r a n o anode-current condition, 1, = 0 amperes and


V , = 1 volt. Thus P o ~ n A
t is obtained.
F o r a zero diode-reblstance condition.
a p p l ~ e dvoltage
I, =
load resistance
1
= O.0lA o r lOmA SOI. IPa = 0 ~ 0 1 t h
100
(acrohs dlode
Point R is then o b t a ~ n e d
Tile rcsulr~nglo,td liric Intersects w ~ t htile c u r \ c s to g l \ e
points X a n d Y with tile r e q ~ ~ i r eanswers
d
At 25'C the I o ; ~ dcurrent is 3.4mA
Ar 00' C' rllc Ir):~dcut.t.cnr is 4. 1 t11A
The current \ariation would be 0.7rnA Ans.
0:

1, =

(1)) I1.C'. rc.si\tancc I'or point X

-D.C. resistance So: point Y =


=
=

Fig 40

0.66- - - - -

3.4 x 10-3
0,194 x lo3
!94R Ans.
0.59
4.1
0.144 x lo3
1440 Ans.

SELECTION O F TYPICAL. EXAM INATION

QUES'TIONS
SECOND CLASS
1.

A thin rectangular plate 350mm by 250mm 1s totally


covered on both sides with nickel 0.12mm thick.in 8.25h.
The current required would, if supplied to a voltameter.
cause 0.0805kg of silver to be deposited in lh. If the E.C.E. of
nickel is 304 x 10-9kg/C, calculate the density of nickel in
kg/m 3 , if the E.C.E. of silver is taken as 1 1 18 x 1 0 - ~ k g / ~ .
C

2.

A 1 IOV d.c. lighting system comprises six 150W and


forty 60W lamps. Calculate the inductance of a choke-coil of
negligible resistance which, when placed in series with this
system, would enable it to be operated on 230V, 50Hz mains.

3.

The resistances of the armature, field coils and starter of a


220V shunt motor are 0.2, 165 and 9.8Q respectively, the
field being connected across the first stud of the starter and
an armature terminal. Calculate (a) the field current a t the
instant of starting; (b) the field current when running, and
(c) the total current taken by the motor at the instant of
starting, considering the armature as stationary.

4.

If the instantaneous value of a current is represented by


i = 70.7 sin 5201, calculate the current's (a) maximum value,

(b) r.m.s. value, ( c ) frequency, (d) instantaneous value


0.001 5s after passing through zero.
5.

A 220V furnace uses a current of 4.5A to melt 5.5kg of


lead in 12 min. If the initial and melting temperatures of the
lead are 16C and 327C respectively, its specific heat is
0-1278kJ/kgC, and its latent heat is 22.72kJ/kg, calculate
the furnace efficiency.

6.

The maxiniurn value of a sinusoidal current wave is 170A.


Find g r a ~ h i q l l y the instantaneous current value after
0-OOls, 0.003s, 0.006s, and 0.008s after zero and increasing
positively. Take the frequency as 50Hz.

7.

The open-circuit voltage of a cell, as measured by a voltmeter of 10052 resistance. was 1.5V. and the p.d. when
supplying current to a 10R resistance W L I S I.25V. me:isured
by the same voltmeter. Detennine the e.m.f. and internal
resistance of the cell.

An nlternat~ngcurrent series circuit consists of a coil '4


that has an inductance of 0.3H and negligible resistance and
,I resistor B of 100R. Tile supply voltage is 200V with a
frequency of 50Hz. Determine ( a ) the impedance of the
circuit. ( b ) the current flowing, ( c ) tlic power factor.

Wli;~ric ~nr:lnthy rllc lrnii 'h;~chc . n ~ . f .; I' S ;~pplicdto ;In


electric motor? A 40kW, 220V shunt motor has a full-loiid
efficiency of 90 per cent, an armature resistance of 0.075R
and a shunt-field resistance of 5 5 0 . When 'at starting', the
starter handle is moved onto the first stud, it is desired to
limit the current through the armature to 1.5 times the value
which i t has when the motor is on full load. What must be
the total value of the starting resistance? If, on overload, the
speed falls to 90 per cent of its normal full-load value. what
would be the armature current? Neglect the effect of
armature reaction.

10

Determine the instantaneous value of a sinusoidal e.m.f.


of frequendy 50Hz and lOOV maximum value, 0401 seconds
after it has passed through its zero value. Determine also the
time that elapses before the voltage reaches 50 per cent of its
maximum value.

11.

State Lenz's law. An iron ring is wound with a coil of 84


turns and carries 0.015rnWb of residual magnetism. When
the coil is excited. the magnetic flila increases to 0.3mWt-1in
0.12s. C';~IcuI~te
tlie ; I V C ~ ~ Iviilue
~ C
ot'tlie e.111.l'.wliicli will bc
self-induced in the exciting coil wliile the fluu is incrcasin~.
and state the direction in wliicli ~t u~llact rcliitivc to tlic
supply voltage, giving reasons.

12.

Find the impedance and power fiictor of an 21.c. clrcuit


consisting of 1\vn picccs of :lpp:rratus in series. Piece A Iias a
resist;ince of 2R i ~ n dinductive reactance of 14R. a n d piece R
II;IS ;I rcsi5t;incc 01' 100 and ;I c ; ~ p ; ~ c i t ireactance
\~c
of 6i2.

SELECTION OF TYPICAL EXAMINATION QUESTIONS

483

13.

Define the temperature coefficient of regstance of a conductor. Name a conductor which has a negative temperature
coefficient. When first switched on, the field winding of a
200V shunt motor takes 2A. After running for two hours
the field current is observed to have decreased to 1.7A. tllc
supply voltage having remained constant and the setting of'
the shunt regulator not having been altered. It' the ambient
air temperature is 15"C, calculate the average temperature
rise within the windings. Temperature coefficient of resistance of copper is 0.004 28 at 0C.

14.

A 440V single-phase motor is rated at 7.5kW and operates


at a power factor of 0.8 (lagging) with an efficiency of 88 per
cent. Find the current taken from the supply.

15.

Explain how the value of a current can 'be measured by


the deposition of a metal from a copper sulphate solution.
Find the quantity of electricity (in coulombs) which will
deposit O.Olkg of copper from a solution of copper sulphate.
What current would be required if this process took one
hour? What mass of silver would be deposited from a silver
nitrate solution if the same current flowed for the same
length of time? E.C.E. of copper = 330 x 10-9kg/C.
E.C.E. of silver = 1118 x 1OV9kg/C.

16.

A coil consumes 300W when the voltage is 60V d.c. On a n


i1.c. circuit the consumption is 1200W whcn Ihc volt;~gc1 5
130V. What is the reactance of the coil?

17.

A motor has four poles, its armature is 0.36m in diameter


and has 720 conductors whose effective lengths are 0.3m.
The flux density of the field under the poles is 0.7T. Each
conductor carries 30A. If the armature is turning at 680 rev/
min, find the torque in newton metres and the power
developed if only two-thirds of the conductors are effective.

18.

Define the average value and r.m.s. value of an alternating


quantity. Calculate the average r.m.s. value for the stepped
half wave given.
Time (ms) 0-10, 10120, 20-30, 30-40, 40-50, 50-60, 60-70
Steady current (A)
2
4
6
8
6
4
2

d4

REED'S BASIC ELECTKOTECHNOLOGY

19.

T w o 200V lamps are connected in series across a 400V


supply. O n e lamp is 75W. the other is 40W. Wliat resistance
would have to be connected in the circuit so that each Iainp
gives its correct i l l u m i n ; ~ t ~ o nW? h a ~would be the power loss
in the iesistance? Wliat is tlie extra cost per week. if electricity cost 0 3 p per un:t?

20.

A series circuit consists of a capacitor of 5 0 p F a n d a coil


of inductance I.5H a n d resistance 3000. Find the total
impedance when u o r k i n g on a 50Hz supply. Find whether
tllc cu~.rcntIc:~d\01. I : I ~ tllc \oltagc.

21.

A OV battery is connected to polnts A<' of' a Whe;itstc>nc


1$1,1cIgcA l i ~ ' l > Side\
,
A l i , LIC' ;11ii1 A I ) 01' t l bridge
~
Iii~~~c
re\l\ti~nce value5 oI' I ,511. 212 a n d IR respcctivcly. If tllc
gi~lvanometer is connected across BD a n d shows no
deflection, find tlie \,slue of resistance of side DC.

22.

Explain the term 'power factor'. A n alternator supplies


560kW a t a power factor o f 0 . 7 (lagging). W h a t extra power
would be available if the power factor is increased to 0.8
(lagging) for the same kV A output'?

33

1 lie ;~rln:itur-ewinding o l ' a 5 1 - p o l e . Ii~p-woundgcncr;r[or


1s made u p from wlre 250m long and 7n1m2 cross-section:~i
;ire;\. It' tllc >lxxlfic resistance ol' cnppcr is 1.7 x 1 0 % ~ m ,
find the rexlstancc of' tile armature.

24.

IOOV a.c. I S applied to a circult of 3R resistance a n d 4R


rc:Ictiince. F ~ n d( a ) the current in the circuit, ( b ) the a c t i x
e.m.f. (resistive voltage d r o p ) of the circuit, (c) the e.m.f. of
self-inductance (re:tctive voltage d r o p ) .

25.

A p c n c r : ~ t o ~is. o v e r - c o i n l ~ o i ~ ~ i d;111d
e d designed to 4~11~1~1y
tlic cll.cuir 1oi1J ; ~ t770C'. 11' tllc 111;1clliiletcrlnin;il ~ o l t ; ~ gI \ c
270 2 V u llcn tllc l o ; ~ di~~irrcrit1 5 1 OA. wI1;1t will be the
r ~ ~ a c l t r rtcririrnal
~c
voltage w11c1it l ~ cIoi~dI \ WOA'!

36

A coll ol' 115R ~ m p e d a n c eIi;is :I resistance of 1 0 0 0 wllcn


conncctcd ,\cross :I SOH/ hupply. Find its inductance. II' tlic
impcdiince fall5 to 1 3 0 . 6 0 when tlle I'requcncy is varied, finl{
tllc IlCLk I'l-ci~ucllc),L ,1Iuc.

SELECTION O F TYPICAL EXAMINATION QUESTIONS


--- ---- - --

--

485

27.

4.5 litres of fresh water at 17 C is heated to boiling point


in 15 min. If the h a t e r is 80 per cent ellicient and the supply
voltage is 220V, find tlie current taken I'rom the mains and
the resistance of the heater. Take the density of water a s
I kgllitre and thc specific Ileal c a p ~ ~ c i t;IS
y 4.2kJ:kg ( '

28.

An alternating voltage of r.m.5. value IOOV is applied to :I


circuit with negligible resistance and an inductive reactance
of 25R. Determine the r.m.s. value of the current flowing.
Show graphically the var~ationof current and voltage durlng
one cycle of applied voltage. What is the value of the current
when the voltage is at its maximum value?

29.

A generator has eight brush-arms, eirch with six brushes.


30mm long and cnch with :I bearing surb~ceof 301nm b
20mm. The current density is 0 . 0 5 4 ~ : m min~ tlie brushes
Find the sectional area of tlie cables, if the leads to the
switchboard are each 9.2m and the current density must not
exceed 1.0A/mm2. Find the power lost in the brushes and
cables. The resistivity for carbon and copper is 2550 x
and 1.7 x 10-sQm respectively.

30.

A coil is connected in series with a capacitor of 60pF


across a 200V, 50Hz supply. The current is 3A and the
power absorbed is 144W. Calculate ( a ) the p.d. across the
c;~p;lcitor,( b ) thc rcsistilncc and induct;lnce of the coil. ( c )
the power I'uclor 01' l l i u coil, and ( d ) l l ~ epower I'i~clor01 111c
whole circuit.

31.

An iron conductor and an aluminium conductor are connected in parallel to a supply. The iron conductor is 10 per
cent longer than, and half the diameter of, the aluminium
conductor. Given that the ratio of the resistivities of iron to
aluminium is 40 to 13, find the ratio of the currents in the
two conductors.

32.

Two currents I, = 14.14A and I, = 8.5A with a phi~se


difference of 30G,are fed into a common conductor. Find
the resultant current and the heating effect in joules when it
passes through a resistor of value 4S1 for a period of 2
minutes.

48 6

REED'S BASIC ELECTROTECHNOLOGY

33

A six-pole d.c. generator has 498 conductors. the e.m.f


per conductor being 1 3 V and tlie current in each IOOA. Find
the e.ni.l'. and current ciutput of tlie annature. if it is ( a ) lap
wound. ( b ) \+:a\,e\\,ound.

34

A c~rcuittakes a current of 10A from 220V. 50Hz malns


at a power f'actor of 0,866 (lagging). Find the value of the
current when tlic ~.oltageis ( a ) passing through its maximum
~ , ; ~ l and
u e ( b ) 0.005 seconds later.

Tlie filament of an electric lamp is 600mm long and 0.04


in cii;1liic1cr.Tlic resistivity of the material when cold is
65 x 1 0 - ~ R mIf. tlie resistance of the wire when hot is 5
tirncs ~ l i crccij~;~ricc
when i t is cold, wlli~t would he tllc
work~ngcurrent taken by tlie lamp when placed in a circuit
having a supply voltage of 1 IOV?

111111

36.

An inductance coil has a resistance of 19.50 and when


connected to a 220V, 50Hz supply, the current passing 1s
10A. Find the inductance of the coil.

37.

State F a r a d a ) ' ~ and Lenz's Laws of Electromagnetic


Induction. A 4-pole. 250V motor has its armature removed
in order to test the continuity of the field windings which are
connected in series and consist of 2000 turns each. What is
tlie average e.m.f. induced when the current is switched off.
if tlie flux .falls from 0.026Wb to 0.001 Wb in 0.2s?

38.

A choke when connected across 206V a.c. mains, passed


a current of 10A and dissipates 500W. If it is connected in
series with apparatus having an impedance of 5 0 and a
capac~tivereactance of 4R, find the impedance and power
f'rtctor of the complete c~rcuit.

39

;\ p~ecco I ' c o 1 ~ ~ wire


~ e r I S belit 1 0 t'os~ii;I circle and ;~notlicr.
plece ol' the same wire is placed across to Sorm a diameter.
; ; I 1 I lic 111r1ct
ori is hclng clcctric;~llyconncclcd. I T t hc rcslbL ; I I I C C 01 t l ~ ch [ r i ~ ~ gw
l ~~ts cI \ 211. find [!it LO{;II cusre1it 110~411ig
when a p.d. of 220V is applied across the Junctions.

40.

Tlie average value of a sinusoidal waveform is 125A irnd


the frequency is 60Hz. Calculate tlie first time from zeri,
when the instantaneous value of the current is 95A. Find
~ I I ~ C tilt
) r.111.s.\ , i l I t ~ t 01' t l i t C L I I . ~ C I I ~ ,

SELECTION OF TYPICAL EXAMINATION QUESTIONS

487

41.

An 15kW motor of efficiency 90 per cent ic supplied ar


240V by a 2-wire system. The supply cables are 500m long
and are of diameter 5mm. Find the current taken by the
motor, the voltage at the supply point and the effic~encyof
the distribution system. Take the specific resisrancc 01'
copper as 1.7 x 10-8Rm.

42.

If the impedance of a circuit is 20R, the resistance I S 16R


and the inductance 0.047 75H, find the frequency of the
supply.

43.

Differentiate clearly between the kilowatt and the kilowatt-hour. A heater with an efficiency of 85 per cent develops
lOMJ in 30 minutes at 200V. Find the energy consumption
in kilowatt hours and the current taken. Find also the
length of wire in the element if its resistamce is 0.26 ohm per
metre.

44.

An electric heater of resistance 6 . 3 2 is connected in series


with a choke of inductance value 0.1H. If the mains Srequency is 50Hz, find the vol?age to be applied to the arrangement in order to maintain' 110V across the heater. I f the
frequency was increased by 5 per cent, keeping the applied
voltage constant, find the voltage across the heater.

45.

An ammeter is tested with a silver voltameter. The welght


of the cathode before deposit was 04238kg and after a
steady current was passed for half' an hour, the we~ghrW;I\
0.039 25kg. The reading on the ammeter was 7.3A. Find the
error of the ammeter, taking the E.C.E. of silver as 1 1 18 x
10-~kg/C.

46.

A resistor of ohmic value 3R is connected in series with a


coil of inductance 0.1H and resistance 1R. If lOOV at a
frequency of 50Hz is applied to the circuit, find the current
flowing.

47.

A 500V, d.c. shunt motor has a full-load armature current


of 20A. Three per cent of the input power is dissipated a s
heat in the armature. What would be the current on starting
if.5OOV is applied across the armature. Find also the value of
starting resistance required to limit the starting current to
twice the full-load current.

48.

Find the total efyective reactance of a 50Hz circuit made


u p from a coil of inductance 100mH, in series with a
capacitor of 20jtF. I f the coil has a resistance of 10R, find
the impedance of the circuit.

49

The nrmature resistance of a 200V. shunt motor is 0.4Q


a n d the no-load annature current is 2A. When fully loaded
and taking an armature current of 50A. the speed is 1200
revlrnin. Find tlie no-load speed and state the assumption
rn:~dcin tlie caIcu1;1t1(7n.
*

50.

A c o ~ lti~hes5A wllen connected across a 220V. 59Hz


.;t~pplv.The power tahen is 420W. Calculate ( a ) the p o ~ v c ~ .
I:tctol.. ( 1 ) ) tile I I I ~ ~ > C ( I~Y ~
~ I I\ [I~~
I IC
) CC
Crci1cti1ncc
,.
:111d1nc1~1cLance of' thc ccil.

SOLUTIONS TO TYPICAL SECOND CLASS


EXAM INATION QUESTIONS
1 . When used ,in the voltameter i m = z l t

Wllcn used Ibr pli~ting:


Area of coating = 350 x 250 x 2 = 175 000mm 2
Volume
,,
= 175 x lo3 x 12 x
= 2 1 000mm3
Also mass of nickel deposited
= 304 x
x 20 x 8.25 x 3600
= 0.304 x 16.5 x 36 x
kilogrammes
0.304 x 16.5 x 36%
So density =
21 000
IO-~
- 304 x 16.5 x 36
21
= 8600kgjm3 Ans
( 6 x 150) + (40 x 60) = 900 + 2400
3300W
System current = --3300 = 30A
110
230
I m p e d a ~ c erequired on 230V a.c. = - = 7.660

2. Totalwattage

=
=

30

Keslstance ol'lamps =

l lo

---

30

= 3,660

Reactance of choke coil = J7q66' - 3.662


7= J 5 8 . 6 8 - 13.4 = d 4 5 . 2 8
6.73
- 6.73 x lo-'
3.14
2 x 3.14 x 50 = 2.14 x
= 0.0214H Ans.

Inductance =

220
1.35A Ans
165
(b) When running, the starter resistance is inserted into the
field circuit by vi,rtue of the position of the contact ami.
Field-circuit resistance = 165 + 9.8 = 174.80
220
Field current when running = -= 1.26A Ans.
174.8

3. ( a ) Field current at instant of starting =

--- =

( c ) Armature-circuit tesistance, at instant of starting


= 0.2 + 9.8 = 10R
220
Annature current = - = 22A
10

..

Total current taken by motor = 22

+ 1.33 = 23.33A

Ans.

4. ( a ) Maximum value of current = 70.7A Ans


( b ) The current is sinusoidal
:. r.m.s. value = 0.707 x maximum value
= 0,707 x 70.7 = 49.94A Ans.
(c) i = 70.7 sin (520 x 0.0015) is in the form i = I,,, sin ot
wllcrc 1 , ) = 520 r;~di;ins/sccond.Also (11 cquals 2rtf
o r 2n/' = 520
520 - 520 = 82.8Hz Ans.
Thusf = -2 x rr
2 x 1.14

( d ) Again i = 70.7 sin 2xft. and if degrees are used for the
angle, then
i = 70.7 sin (2 x 180 x 82.8 x 0.001 5)
= 70.7 sin (3.6 x 8.28 x 1.5) = 70.7 sin 44.65
= 70.7 x 0.703 = 49.65A Ans.

220
12 k .~ l o w a t thour
4'5 x loo0
60
= 0.198kW h
= 0.198 x 3600 kilojoules
= 712.8kJ
Energy received (by lead)
= (5.5 x 31 1 x 0.1278) + (5.5 x 22.72) kilojoules
= 218 7 + 125 = 343.7kJ
Energq put out by furnace into lead = 343 7kJ
343.7
Etticiency =
= 0.482 = 48.2 per cent Ans
7 12.8

5. Energy input to furnace =


d

Plot

;I

sinc u.;I\.e on gr;tpli paper with

;I

masimum v ; ~ l r ~ e

I>;I\L* 01' 0 01 5 . ,A I'I~CLILICI~L~!01. 5 0 1 I / give\ 1 1 1 ~ time of a hall' wave a s 0.01seconds.


The wave can be plotted from a phasor of length equal to

01. l70A

, I I I ~;I

170A o r by usc of tables to obtain ordinates. Thus'for 30


the insti~ntanenusi.alue o r ordinate would be 170 x sin 30
= I 70 x 0.5 = 85A. SO for 45 i = I 70 x n.707 = 120.2A
1 0 0 ' i
147,09A. F o r 00 i T 170A. C I C .

SOLUTIONS TO TYPICAL EXAMINATION QUESTIONS

49 1

Answers from the deduced waveform:


When time t = 0.001s.
i = 57.5A.
When time t = 0.003s,
i = 132A:
When time t = 0.006s,
I = 162A.
When time r = 0 . 0 0 8 ~ ~ i = 102A.

7. Let E = the e.m.f, of the cell and R , = the internal rcslstance.


With the voltmeter across the cell terminals only,
1.5
Current taken by voltmeter = -. = 0.01 5A
100
. . . (a)
Then E = 1.5 + (0.015) x Ri)
With the voltmeter and resistor across the cell terminals,
1.25
Current taken by resistor = - = 0.125A
10
1.25
Current taken by voltmeter = -= 09125A
100
Current supplied by cell = 0.1 25 + 0.0125 = 0.1 375A
. . . (b)
Thus E = 1.25 + 0.1375Ri
Solving (a) and (b) then E = 1.25 + 0.1 375Ri
and E = 1.5 + 0.015Ri
Subtracting 0 = - 0.25 + 0.,1225Ri
0.25
o r R i = -= 2.04R Ans.
0.1225
and E = 1.5 + (0.015 x 2.04) = 1.5 + 0.031
Thus cell e.m.f. = 1,531V Ans.
8. R A = OR

XA

2xfL. ~2 x 3.14 x 50 x 0.3


= 31.4 X 3 = 94.2R

R, = 1000 Z = d 1 0 0 2 + 94.2' =. 1 0 . J M 2
= IOdlOO + 88-74 = l o d m 4
= 10 x 13.74 = 137.4R Ans.
200
Current I = -= 1.45A Ans.
137.4
R
100
Power factor, cos 4 = - = - = 0.73 (lagging) Ans
Z
137.4
9. Output = 40 x 1000 watts
100 400 x lo3 watts
Input = 40 x lo3 x - =
90
9
Input current =

44 444 - 2222.2
- -= 202.02A
220
11

492

R EE D ' S B A S I C E L E C T R O TE C H N O L OG Y

220
- = 4A
55
Armature current = 202.02 - 4 = 198.2A
Armature starting current = 198.2 x 1.5 = 297.3A
220
Resistance of armature circuit = ---- = 0.74R
297.3
Resistance to be added = 0.74 - 0,075 = 0,665Q Ans.
On normal load E, = 220 - (198.2 x 0.075) volts
= 220 - 14.87 = 205.13V
On 90"" speed E,, = 0.9 x 205.13 = 184.62V
/\s~ii;~rurc
\olr;~gedrop = 220 - 184.62 = 35.3XV
35.38
Annaturecurrent = ----- = 471.7A Ans

Shunt-field current =

0.075

10 \ u h \ t ~ t u t ~ ning r = I', sin 2nlr


I = 100 x s i n ( 2 x 180 r 50 x 0 0 0 1 )
= I00 s ~ n
(18 x 1 ) = 100 x sln 18 = 100 x 0 3 0 9
= 30 9V Ans.
If = SOV, then 50 = 100 sin ( 2 x 180 x 50 x r )
50 = sin (2 x 180 x 50 x r )
and --100
4 = sin 0, whence 0 = 30'
or
1
3
= _ x 10-2
Thus 30 = 18 x 10'1 or r =
18x10'
6
= 0.166 x 10- seconds
Tlic i.cquired tinie = 166nis A n s

--

84
-

--

lo-'
12

(2 85)

2 85

I()

-2

The induced voltage will oppose the appl~cdsilpply voltage.


~ I I L I S ~.cducirig[tic I . ; I [ C 01' C L I I . I . C I I ( g1.0w111
12. Total resistance of circuit = 2 + 10 = 12f2
Total reactance .. ,, = 14 - 6 = 8R (inductive)
From impedance relationsliip:
-7 = \ 122 t X 2 = "'144 + 6 4 = b/20t( = 14.4R
I'hc c ~ ~ . c impedance
u~t
is 14.40 A n s

--

SOl,llTIONS TO TYPICAI. EXAMINATION Qt1I:Sl'IONS


- -----

493

T h e power factor = 0.832 (lagging) Ans

13. Carbon has a negative temperature cocllicient.


200
Resistance of field (cold) = R , = - 3.- = IOOR

'00
Res~stanceof' field (hot) = H , = - = 117.64R
1.7
Then 100 = R, ( l + 0.004 28 x 15)
and 117.64 = Ro ( 1 + 0.004 28 x 7J
o r I ,1764 x 1.0642 = 1 + 0.004 28 T
and 1,2519 - 1 = 0.004 28T
Temperature rise of winding = 58.85 - I5 C
= 43.85'C Ans.

14. M o t o r output = 7.5 x 1000 = 7500W


100
M o t o r input = 7500 x --- = 3750 x 25 = 8523W
88
II
P
8523
:
.
/
=
-----amperes
But V I cos @ = P
440 x 0.8
V cos q5
8523
or I = - = 24.21 A
352
kilogranirnes o f copper are deposited b!, I
15. 330 x
coulomb
0.0 1
- -l o x 103
:. 0.01 kg of copper will require
330 x
0.33
= 30.303 x 103
= 30 303C Ans.
If time was 1 hour, since Q = It
Q - 30 303 - 303.03
... I = - - ----- - --t
3600
36
o r current required = 8.418A Ans.
Mass of silver would be in proportion to the E.C.E
0.01 x l l I8 x
0.01 x 1 1 1.8
So mass = 330 x
= -- 33
= 0.01 x 3.387
= 0,033 87kg o r 33.87g Ans.

V2
V2 60 x 60
R
P
300
When on a.c. P = 1200W also P = 1*R

16. When on d.c. P = - :. R = - = ------ = 1 2 0

V 130
The impedance Z of the circuit = - = - = 13R
I
I0
So reactance X

v'132 - 12' = d m 4 4 = f l 5
5R
T h i ~ sreactance of coil = 5R Ans.
=

17. Force on 1 conductor is pven by F = Bll newtons


or F = 0.7 x 30 x 0.3 = 6.3N

2
No of conductors in the field at any instant = - x 720

= 480

Total force = 480 x 6.3 = 3024N


Torque = force x radius = 3024 x 0.18 newton metres
= 544.32
So torque exerted = 544Nm Ans.
2x)YT 2 x 3.14 x 680 x 544
Power developed = --- 60
60
= 3.14 x 68 x 181.33 watts
= 38.7kW Ans.
d

18.

I f plotted, t h ~ swavefonn w ~ l lbe found to be made up of


seven rectangular blocks. the mid-ordinates of which are 2.
4. 6. etc. as given

= 4.57A

Ans.

SOLUTIONS TO TYPICAL EXAMINATTON QUESTIONS

495

75
19. 75W lamp. I = - = 0.375A
200
40
40W lamp. I = - = 0.2A
200
With lamps in serics 40W lamp will only p;~ss0.2A
.: (0,375 - 0.2) amperes must be piissed through il sliunt
resistor connected across the 40W lamp. This resistor is i i l > r )
to be suitable for 200V, and its resistance value must
- -200
=
1143R Ans.
0,175
Power loss in this resistor = 200 x 0.175 = 35W Ans
35 - 4 9 x 1 2
Units used per week = 7 x 24 x ---- lo00
100
Cost = 588

0'5p = 2.941, Ans,


100
*

20. Here L = 1.5H so X, = 2 ~ f L= 2 x 3.14 x 50 x' 1.5


= 471R

X, - X, .'. 471

407.3R
(inductive)
=
J30o2
+ 407.3~
Impedance o f circuit, Z =
= 1 0 0 J 3 ~ + 4.0732
say 100/3~ + 42 = 500R
The circuit impedance is 500 ohms . Ans.
Curre,nt will lag the voltage because the circuit is inductive.
Ans.
Total reactance X

21.

63.7

With no current in BD, points B and D are a t the same


potential, and since A is common, then:
P.D. across AB = p.d. across A D .
Similarly since C is common, then:
P.D. across BC = p.d. across DC.
Let I, = current in tog branch and I, = current In bottom
branch
Then I, x 1.C =: I, x 1 and I, x 2 = I , x Rx
2 x Il
Written as a rat o Rx x IZ1x1,
1.5~1,
2
4
'
Thus Rx = - = - = 1.33R Ans.
1.5 3

49 6

RE ED ' S BASIC ELECTROTECHNOLOGY

or hilo\oll iilnperes ( S ) =
Thus S

14

kilowatts (P)
power kictor (cos 4)

560
0.7

.= ---

800kV A

This would bc the rehistancc of'~ l l cleng~h01' wirc. H u t ~ 1 1 t h


;I 1'1p-bound generator. u.1tl1six parallel patha in thc ;irm;i-

0"07 - 0.101 16R


ture. -Phuh re\ict;~nceof 1 parallel path = ---6
H u t there are 6 p;~thbIn parallel so the equivalent resistance
0.101 16 - 10.116
Ic-2
is one-sixth of thc above = ----------

= 1.686 x

= 0.01686R

Ans.

( a ) Current I

1'
100
=- =
= 20A
I

Ans.

Z
5
( b ) R e i i s t ~ ~i.o!t:~gedrop
c
I,', = I R = 20 x 3 = @JV Ans.
( c ) liei~ctivevolt:cge drop b', = I X = 20 x 4 = 80V 4 n s .

25

M'llcn t l ~ cloact current is 10A. t l ~ ctc~lt:rgc d r o p in tllc


( . ; ~ t > I v \ I>c.~ir,c,c*n
.
ni;~chineterminals and load = 220.2 - 210
= 0.2v

By propnrtlon the voltirge drop for 600 ampere\


600
= 0 2 x - - = 0.2 x 60 = 12V
10

SOLUTIONS TO TYPICAL EXAMINATION QUESTIONS


497
-- - -- - - - -.- .
- - -- ---. --

26. Reactance X, at 50Hz

Also X, = 2rcfL .: L

~t

So

=
=

= 1

0
0
d
1 = loom6

75
XL
- -2 x 3.14 x 50
100 x 3.14
-0.75 = 0.24H Ans.
3.14
=

new frequency X, =

new frequency
50
o r new frequency

d-2
100 J

;/m~@

= 100 J1.206~ - I
= 100JFEC-1
= l o
= 100 x 0.674 = 67.4Cl
67.4
--

o m 4

75
50.
4
67.4 x - = 33.7 x 75
3
= 11.233 x 4 = 44.93 hertz
The new frequency value would be 45Hz I n s .
=

27. Mass of water = volume x density = 4.5 x 1 = 4.5kg


Heat received by water = 4.5 x 4.2 (100 - 17)
= 19.9 x 83 kiloioules = 1569kJ
100
Electrical energy supplied to heater = 1569 x 80
- l 5 690 - 1961kJ

8
1961
Power rating of heater =
- -time
15 x 60
196
1
- -= 2.18kW
9W2180 - I09 - 9.gA
Current taken from mains = -- - 220
11
Mains current = IOA (:ipprox) Ans.
Resistance of heater = ,-220 = 22.252 Ans.
9.9
energy
_-

28. Here R = O R and Z

25l2

V
100
:. r.m.s. value of current I = = --- = 4A

Z
25
The graphical solution consists of a sinusoidal voltage
wave with a sinusoidal current wave lagging it by 90G,since
the circuit is wholly inductive. Thus when voltage is maximum current is zero. When voltage has fallen to zero, the
currer-t has risen to its maximum value and as voltage rises
to its negative maximum the current falls to zero.
When V is a maximum, current value is zero Ans.

29.

e 1 brush = 30 x 20 square millimetres


Bearing s ~ r f a c of
= 600mm2
With a current density of 0 . 0 5 4 ~ / m mthe
~ , current carried
by one brush = 600 x 0.054 = 6 x 5.4 = 32.4A
With 8 brush arms, there are 4 positive and 4 negative
brush arms. Also since there are 6 brushes per arm, the
number of brushes in parallel carryng current into o r out of
the annature = 4 x 6 = 24 brushes, o r total current carried
by brushes = 32.4 x 24 = 777.6A.
Current density in the cable is limited to 1.6A:mm2
777.6
= 486mm2 Anh.
. . ('able nrc to carry 777.6A =
1.6

3.4 x 9.2 x
- 31.28 x
486
486
= 0.643 x
ohms
Voltage drop in cable = 777.6 x 0.643 x 10- = 0.499
= 0.5V
Power loss in cable = 777.6 x 0.5 = 388.8W Ans.
P 1 where' 1 is the
Resistance of the brushes is given by R =-

length of a +ve plus a - ve brush and ,4 is the area of half


the total number of brushes.
2550
2 x 30 x
- 25.5 x 60 x W 3
Thus R 4
108 600 x 1 0 - 5 24
600 x 24
- 255 x
= 0.106 x
ohm
24
Power loss In the brushes is given by 1 2 R
= 777.6' x 0.106 x
= 7.776 x 7.776 x 1.06 watts
Thus power loss in brushes = 64.1 W Ans.

v 200
If Z is the circuit impedance then Z = - = - = 66.6R
I
3
Now only resistance is responsible for power dissipation.
1 44
= 16R
then P = I' R or 144 = 3 ' ~ .Thus R =

SOLUTlONS TO TYPICAL EXAMINATION QUESTIONS

499

For this circuit X is made up of inductive reactance X, and


capacitive reactance X , which nullify each other.
The resultant reactance X = d m 2= d66.6* - 1 62
= 44436 - 256 = m
0
= 647R
But X, alone is 53R :. X, = 64.7 + 53 = 117.7R
Thus ( a ) p.d. across capacitor = 53 x 3 = 159V Ans.
(b) Resistance of coil = R = 16R Ans.
1 17.7
- 1.177
Inductance of coil = % =
2nf
1 0 0 ~ 3 . 1 4 3.14
= 0.38H Ans.
(c) Impedance of coil z,-=J
162T1 17.72 = J14109
= 119R Ans.
R -16
Power factor of coil = cos $, = 119
3
= 0.1346 (lagging) Ans.
(d) Power faotor of circuit = cos

=-

-16 = 0.245 (lagging)


66.6
Ans.
Note. Both power factors are lagging, the circuit being net
inductive.

31.

The equations for t h e iron and aluminium conductors can


P ili

be written as R i= - and R,
Ai

and di=

@a

.: A , =

-2since

pa'a
=-

Aa

area

diameter 2

40 x 1.1 x I, x A, x 4
13 x I, x A,
Ra
160
x
1.1 176 - 13.54
13
13
1
Thus, since the resistance ratio of the iron to the aluminium wires are 13.54 to 1, and as the wires are in parallel.
the currents in the wires are in the ratio Iron :Aluminium =
1 : 13.54. Ans.

R i=
So-

+ (2 x I1x I2 x cos 30)


32. Resultant current I = JII2 +
=J14.14~+8.5'+(2~ 14.14~8.5~0.866)
= d20O
72.25
208.17
= J48042 = 21.92A
Resultant current = 21.92A Ans.
Power dissipated = 1 2 R = 21.92' x 4 watts
= I2Rr joules
Energy at heat
= 2 1 , 9 2 2 x 4 x 2 x 60
= 480.42 x 480 = 230 6025 Ans.

total conductors - 498 = 83


-parallel paths
6
1I.1n.l'.ol' I parallel p;~thr- c.rn.1'. ol'machinc
= 1.5 x 83 = 124.5V Ans.
Current per parallel path = current in 1 conductor
= 160.4
Current of 6 paths in parallel '= 6 x 100 = 600A Ans.
( a ) WaveJwound. A = 2
498 = 249
total conductors - Conductors in series = ---parallel paths
2
E.m.f. of' 1 parallel path = e.m.f. of machine
= 1.5 x 249 = 373.5V Ans
Current per parallel path = current in 1 conductor
= IOOA
C u r r e n ~o f 2 paths in parallel = 2 x 100 = 200A Ans.
Conductors in series =

34.

The power factor of this circuit is 0.866 (lagging) o r cos 4


4 = 30G,where 4 is the angle of lag
between the voltage and the current-the latter lagging the
former.
The values of 10A and 220V a s given, can be assumed to
be r.m.s. valucs. So the maximum value of current is given by
I = 0,707 I,,, (here I,,, is the maximum value). Sine-wave
working is ;~ssumcd
= Q.866 (lagging) and

Also V

0.707V , (maximum value). Sine-wave working

15

assumed.

Thc voltage and current can be written as


L' = L',
sin cot
and i = I, sln ((!)I - rb) 4 15 In radians

1
50
(a) When the voltage is at a maximum, the time is for; cycle
1
or
t = ------ - 0.005 seconds
4 x 50
Current at this instant is given by substituting in
i ,= 14.14 sin (2x50 x 0.005 - 4)
or i = 14.14 sin (2 x 180 x 50 x 0.005 - 30). x and 4
in degrees
= 14.14 sin (90 - 30) = 14.14 sin 60"
= 14.14 x 0.866 = 12.25A Ans.
(b) At an instant 0.005s later t would be 0-01s
:,
i = 14.14 sin (2 x 180 x 50 x 0.01 - 30)
or i = 14.14sin (180 - 30) = 14.14sin 150"
i = 14.14 sin 30" = 14.14 x = 7.07A Ans.

At a frequency of 50Hz, time for 1 cycle =

- seconds

35. Resistance of filament (cold)


65 x lo-'' x 600 x 7 x 4 - 273mohms
22 x 16 x lo-''
88
273 x 5 x 10' ohms
:. Resistance when hot =
88
110x88 . 968
Working current =
273 x 50
273 x 5 x 10'
--- - 0.0709 = 0.071A Ans.
273
36. From the information given, the impedance Z of the
.
v
220
circuit is = - or Z = - = 22R
I
10
The resistance R is 19.5n. Therefore the reactance X is
obtained from X = d m 2= J22' - 19.5'
= J484 - 380.25 = J 1 i 5 = lO.lsR
10.15
- 0.1015
Also X = 2xfL :. L =
2 x 3.14 x 50 - 314
= 0.032H Ans.
37. From Faraday's law.
E*" = N ( @ I - @,)
or R =

-2000 x 4(2.6 x lo-' - 0.1 x


0.2
- 80 x (2.6 - 0.1) - 80 x 2.5 =
,, 0.2
0.2
Induced e.m.f. = lOOOV or 1 kV Ans.

25
volts

500
Also since P = 1 2 R . then R of coil = --r = 5R
10
Thus reactance X of coil = V'Z* - , R ~
= Jw-Ti
o r X = \;424.36 - 25 = d m 6
= 19.98R
Z ofadditional apparatus = 5R
X
,
,,
= 4R (capacitive)
.: R
,,
=
=
= 3n
Total resistance of circuit = 5
3 = 8R
Tor;ll reactance
,,
= 19.98 - 4 = 15.98R
Note. The inductive and capacitive reactances have been
9

Jm2

st1 ht r a c ~ c d .
*I otal impedance ofcircuit

J82

= J64
=

Since cos
Thus power factor
inductive Ans
39.

+ 1 5.5)g2
+ 255.36
= 17.880 Ans.

= 0.44 (lagging)
4=17.88

0.44 (lagging), slnce circuit

15

net

Since the circuit is built up from wire of the same material


and
area, then the resistance of various parts of the
. sectional
.
clrcult are proportional to length.
The resistance of the diameter = 2 ohms
:. The resistance of the circumference = x d = 2x ohms.
i'he resistance of circumference = x ohms
The circuit is made up of a diameter and two circumferences in parallel. :. if R is the circuit resistance

With 220V applied across R , the current would be

SOLUTIONS TO TYPICAL EXAMINATION QUESTIONS

503

40. For a sine wave:


Average value = 0.6365 x maximum value
av value - 125
:. Maximum value of current I,,, = -0.6365
0.6365
or I, = 196.4A
Also as current is sinusoidnl, then i = I,,, sin 2 x f t
or 95 = 196.4 sin (2 x 180 x 60 x t )
95
and sin (18 x 12 x lo2 x t ) =- = 0.483
196.4
Now from sine tables, the sine of 28.9" is 0.483
Thus t = 0.134 x lo2 = 0.001 34s.
First time from zero = 0.001 34s or 1.34rn.s A n s .
R.M.S. value of current = Q.707 x 196.4 = 138.86A Ans.
r

41. Output of motor = 15kW


15 .
Input to motor =- kilowatts = 16 666W
0.9
16 666
input current o r current in cables = 240
= 6 9 4 A Ans
pl
Resistance of cable is given by A

Voltage drop in cable = 69.44 x 0.866 = 60.14V


Input voltage at supply cables = 240 + 60.14 = 300.14V
Ans.
Power output from cables
Efficiency of distribution =
Power input to cables
240
x 69.44 - 240
300.14 x 6944 - 3oO14
= 0.799 = 79.9 per cent Ans
42., Here Z = 20Q and R

16Q

:. X = J2iGF-S'
=6-4J
=

m =1 2 n

Also X = 2nfL = 2 x 3.14 x f x 0.047 75

= 12

- 40Hz Ans.
29.987
The frequency of the supply is 40Hz Ans
-

43 Output of heater = l 0 M J
'ilso IkLV h = 3600 x 1000 = 36 x lo5 loule5
10 x lo6
100
Nou c.ncrg\ o u t p u ~o f Ilc,rtcr = -= 17s
36 x lo5 = 36
= 2 78kW 11
O L I I ~ L I I - 7, 711
1-rlcryy 1111)111 =: 3 27h W 11 / \ I ] \
ellic~ency 0 85
energy - -Power lnput = ---27 = 6 54kW
time
05
I
C2
Also. since P = 1 2 R = I x R x I = I/ x - = -R
R
'-

L '2
R =P
- 40000--- - 6 1 2 Q
TIius R = 2002 6340
6540
resl51,lnce
- 6 12
Length c ~ elenlent
l
, ~ tO 26R m = -- -- ohms Der metre
0.26
= 23.54;
Ans.
P 6540
Current taken = - = -- = 32.7A An\.
v 200
4

44. If 110V is maintained across the heater. the current would be

In~ped~ince
Z of complete clrcult

&?-+
3142

= J42 25
985 96
= dl028 Z I
=

10fi0-

3 207 = 32 07Q
Appl~ed\ o l t L y efor 16 92A = 16 92 x 32 07 - 542 6V An5
If frequency rljes 5 per cent. reactance rlses 5 per cent N e u
~ c ~ ~ c t . ~ r l c7 cl 4 r 1 0 5
1 2 07R
= 10 x

+ 32.97'

= J42.35
1 0 \ M 3
10 x 3.36 = 33.6R
542.6
Circuit current = ----- 16.15A
33.6
Voltage across heater = 16.15 x 6.5 = 104.97
= 105V (i~pprox) Ans.

New impedance = J6.52


=
=

J1129j

+ 1087

45. Mass deposited = 39.25 x 10- - 23.8 x 10 = 15.45 x


kilogramrnes
True current is obtained from m = zlr
= 7.68A
Error of ammeter

7.68 - 7.3 = 0.38A (low) Ans.

o r as a percentage error

0.38
---- = 0.049
7.68
= 4.9 per cent low

Ans.

46. Total resistance of circuit = 3 + 1 = 4 0


Reactance of circuit is given by X = 2njL
or X = 2 x 3.14 x 50 x 0.1
= 31.40
= d l 6 -+ 985.96
Circuit impedance Z =
= d m 6 = 1 0 m 2 = 31.650
C~lrrcntflowing = --100 = 3.16A Ans.
3 1.65
47. Neglecting the field current, since i t will bc sm;~ll
Input power = V I
= 500 x 20 = 1OOOOW
3
3 per cent of the input power = - x 10 000 = 300W
100
This is dissipated as heat in the armature, ir it is a copper o r
r2R loss.
:. I ~ R ,= 300 o r 2 0 2 ~ =
, 300
300 3
and R, = - = - = 0.75a
4
400
The starting current with only armature resistance to limit
500
the armature current = IaS= --- = 666.66A Ans.
0.75
Twice full load current = 20 x 2 = 40A

:. Total resistance required in the armature circuit to limit

starting current to 40A =

500
= 12.50
40

----

S ~ n c cfi, = 0.75R
Series resistance \vc>uld be 12.5 - 0.75 = 1 1 , 7 5 0 Ans
48 Induct~vereactance Y, = 2xJL = 2 x 3 14 x 50 x 100 x
or Y, = 3 14 x lo4 x
= 31 4 0
lo6 1O6
C , ~ p a c l t ~ reactance
ve
X
- -'=2xfC
2 x 3 1 4 ~ 5 0 ~ 2 0

Total e f e c t ~ c ere'lctance X = 159.23 - 31 4


or \
127 X3R ( c . ~ ~ l , ~ c ~ t (111,
~\c)

\/'I 0-+- 16 340

~/16440
= 1 0 2 d 1 . W= lo2 x 1,283
= 128.30 Ans.
=

49. On no-load: Ebo = I . - I,R, = 200 - (2 x 0.4)


= 200 - 0.8 = 199.2V
On full-load: E,, = 200 - (50 x 0.4) = 200 - 20
= 18OV
As this is a shunt motor, constant field current and therefore the same constant flux can be assumed for the no-load
ltnd full-load conditions.
Since Ebo = k@,Y0 and E,, = k @ , N , and 0,= 0,

' 9 9 2- = ---3984 - 1328 revimin


T h ~ sgives N o = 1200 x 1 80
3
Ans

AlsoPowerP

1 2 R o rR

P
J2

450

450
=

2T

Hence I<= 18iL


Also since Z 2 = J F i 2then X = d ~ 2 - R ~
or .Y = \'44' - 18' = d l 9 3 6 - 324

Hence X = m
j = 10Jm
= 40.14Q
('1)

Powcr f'lctnr c o j

d)

= I X-

44

17

10 x 4.014
041 il.~py~ng)

( b ) l~npedanceZ = 44R Rcsht,\nce R = ISR


Reactance X = 40 142
Inductance L is given by: X = 2 x f L
Inductance, L = 0.127H Ans.

An\

SELECTION O F TYPICAL EXAMINATION


QUESTIONS
FIRST CLASS
1.

2.

The rudder motor of a Ward-Leonard type steering gear


is a compound-wound machine details, of w h c h are given
below. From this information find: (a) armature current,
( b ) the torque developed by the motor, (c) output power of
motor. Armature-volts 90, resistance 0.0288Q revimin 370.
I R drop o\,cr ;~rmaturcm:ly be taken :is 8.5V. Shunt ficld
scp;u;~relyexcited from 110V \upply, rcsisri~ncc6.50,
per pole 1000. Series field-separately excited by primc
lnovcs I~nccrlr.rcnt 01' 325A, t u r n s per polc I I . Torclric
Nrn -- (asrnaturc currcrlt ( A ) x I l u x (kwh) pcr polc i
10Y4 x 6.8) - 15.
M . M F per pole ( A t )
3000 3500 4000 4500 5000 5500 6000
Flux per pole (kWb)
2500 2700 2860 2990 3100 3190 3275

A 500V installation consists of a synchronous motor


taking 50kW working in parallel with a load of 90kW
having a power factor of 0.6 (lagging). If the power factor
of the combined load is 0.8 (lagging), find the power factor
and reactive kVA of the motor.
3. The electrical circuit of an explosive gas sampling device
is shown below. The resistance R is heated by the gas being

SELECTION OF TYPICAL EXAMINATION QUESTIONS

509

ignited and causes a current of 80mA to flow through the


galvanometer. If the element is of platinum, to what
temperature has it been raised if the instrument was
originally balanced at 15.55OC? (Increase in resistance per
ohm per "C for platinum = 0.002 43l-2.)
4.

A I75kVA, 6600/440V, single-phase transformer has iin


iron loss of 2.75kW. The primary and secondary windings
have resistances of 0 . 4 0 and 0.001 5R respectively. Calculate
the efficiency on full load when the power factor is 0.9.

5.

A shunt-wound generator has the following open-circuit


characteristic. If the actual field-resistance value is half that
of the critical field resistance, above which the machine will
fail to excite, find the open-circuit voltage. The e.m.f., when
the generator is operating at a load of 200,A, falls to 135V.
Find the terminal voltage and the armature resistance.
Field current (A)
0.5
Open-circuit voltage (V)
55

1.0

2.0

3.0

4.0

5.0

90

133

160

179

193

6.

What are the units of length, mass acd force in the SI


system of units?
A current of 12A produces a magnetic flux of 0.4mWb
in a coil of 60 turns. What e.m.f. would be induced in the
coil if the current of 12A was reversed i d second? What
is thc induct;~nceof thc coil in Iicnrys?

7.

A two-wire ring-main, 2km long, is supplied a t a point 'X'


with 220V. At a point 'Y' situated 400m from 'X' there is a
load of llOA. The resistance of Ikm of single, main conductor is 0.03212. Calculate the current in each section of the
main, and the voltage at the load.

8.

In a supply, the voltage and current vary sinusoidally at a


frequency of 50Hz, the r.m.s. values being 31 1.2V and 70.7A
respectively, at a power factor of 0.866 (lagging). Plot o n
one chart the graphs, for a single cycle, of the voltage and
current in their correct relative positions. From them, d e r i ~ e
the instantaneous values of voltage and current at 3, 6. 1 1
and 18 milliseconds from the time that the voltage passed
through zero, in the course of it increasing positively.

9.

The moving coil of a permanent-magnet voltmeter is m a d e


of copper and has a resistance of 5 R \vhen a t 20'C. T h e
instrument is connected in ierles wit11 ;I resistance. the value
of' which is 99512 at 20 C . Calculate the percentage error.
high or l o ~ v or'the
.
reading at 50 C, if the series resistance is
made c>i'(a)copper a n d ( b ) manganin. Take the temperature
coefficient of copper a s 0.00428 C at O:C a n d th'it ot'
mans,inln a j zero.

I0

,2 parallel clrcui: con.i15tsova branch A of reqlstance IOR.


induct;tncc 3 S m H and cap;lcit,inct. 312.8i1F. ;ind a br;~ncll 13
0 1 rlcgl~g~l~lc.
r . c 3 \ ~ \ ~ ; ~ l i C; tc~b l t l indt~c[ive r?;\c[ilncc ol' 1 7 0
I>er~\.eeraphicall!. the tot:~lcurrent and i t ? phnce angle. if
lllc \ \ \lc~iiI \ c ~ o ~ ~ r i c cto~ ~; I c4d4 0 V . 50fif s~11~111y.

1 1.

The field windings of a motor comprise 8 coils connected


In series. e;tch coil ha\.ing 1200 turns. The flux linked u,ith
each coil is 0.05Wb u h e n the current is 5 A . Calculate the
inductance of tlie field clrcuit and the value of the average
c.rn.1'. ~ n d u c e d 11'
. the current was cut off in 50ms

,4550kVA. 50Hz. slngle-phase transformer has 1875 a n d


75 t ~ t r n sin tlie p r l m a r Ltnd secondarb windings respecti\,elb
If the secondary voltage is 220V. calculate ( a ) primar!
\,olt:rye. ( b ) prirn;lr! and secondary currents. ( c ) n i a s i n ~ u ~ ~ ~
\;1lue OI'Illls:

13.

The armature and field resistances of a 220V shunt m o t o r


;Ire 0.25 and 110 o h m s respecti\,ely and. when running o n
n o load. the motor takes 6A. Calculate the losses attributtlble to Iron. friction a n d windage a n d , assuming this v:ilue to
rcni,~inconstant on all loads, determine tlic cliiciencq \\llcn
i l ~ ccurrent \upplied is 67A

1.1

1~l.lllc.ll3lc\I ~ I I ~ I C I ~ I C Illlc
I ~ ~ Tccc~\hll>1 0 1 . I I I C
L I I C Lcr-rll '1)o~vcrI;ICIOI.'
~IICIICOI~\I~CI.III~
< L C . ni;~cli~ncr!
An a11ern;ttor I \ \ u p p l > ~ n ga load ol' 560kU' ; I I ,I po,,\cr
1,tctc)r 01'0.7 (Iilgslny). Il'itpparatui 15 ~n\t:lliedl h ; ! t r;lI\c\ ilic
po\ter l i c t o r t(1 0 S ( l , ~ g g ~ ~calculate
ig).
the Increase In po\\,cr;1\;111at>lc
I'or (ilc \;lrnc I. L'il Io;rd~~lg.
1\1~l,l111

ll1('

I I I [ I O C ~ L I ~ [ I ~ I 0I 1I

15.

'

A battery consisting of 48 cells, having an internal resistance of 0.02 ohm per cell, is to be charged so that the e.m.f.
per cell is raised from 1.8 to 2.2V. Determine the maximum
number of carbon lamps of a parallel bank, which may be
switched on in series with'the circuit, so that the current from
the 200V mains does not exceed 10A at the commencement
of charging.
If the circuit remains unaltered, calculate the current
flowing through the battery when the charge is approaching
completion. Each lamp is marked 1 lOV, 32 c.p. and requlres
3.5 watts per candle power.

16.

A heater unit of negligible inductance has a resistance of


6.5R and is intended for use-with IOOV mains. For what
50Hz voltage would it be suitable when placed in series w ~ t h
an external apparatus, of negligible resistapce, having an
inductance of 0.01 H? If the frequency rises by 5 per cent and
this voltage remains constant, what would be the resulting
change of voltage at the heater terminals?

17.

The magnetic field in the air gap of a two-pole motor has


a,flux density of 0.8T. The armature is wound with 246
conductors, each of 400mm effective length, mounted a t
150mm effective radius, and at full load each conductor
carries a current of 20A. Assuming-that the actual torque
-produced is equivalent to that due to two-thirds of the
number of conductors cuttlng the l~nesof force at rightrrnglcs. find (a) tllc torqi~cin ncwlon mctrcs, und ( h ) thc
shaft power developed at 500 revjmin.

18.

State briefly, the meaning of the expressions 'st:tr-connected' and 'delta-connected' as applied to three-phase a.c.
practice. What is the ratio of the maximum line voltage to
the maximum phase voltage in each case.
Determine the line current taken. by a 440V, three-phase,
star-connected motor having an output of 45kW at 0.88
(lagging) power factor and an efficiency of 93 per cent.

19.

A battery is to consist of a number of' cells connected In


series. Each cell has an e.m.f. of 1.5V and an internal
resistance of 0.5R. The external load has a resistance of
lOOf2 and requires approximately 2W for satisfactory
operation. Determine how m a n y cells will be required.

20.

Three conductors, fitted side by side in the stator of a


salient-pole alternator. each ger,erate a maximum voltage o f
200V (sinusoidal). The angle subtended at the centre of the
stator between adjacent conductors is 20 electrical degrees
If the three conductors are connected in series, find : ( a ) the
r.m.s, value of the effective voltage and (b) the 'breadth
factor'. Using the theory that is the basis of this problem,
give one reason why three-phase current has been introduced.

21.

The earth-lamps on a main switchboard comprise two


240V 60W lamps connected in the usual manner. The
~x'tclirial JilTcrc~icc:I[ rlic busbars is 220V. 1)aniagc 17) b c a
water occurs to a distribution cable so that the insulation
resistance to earth is reduced to 16R and 6R for +ve : ~ n d
vc cablca ~.csl,ccrivcly.Find by ciilculation ( a ) whicli 01'
the two lamps burns the brighter; and (b) the addition211load
on the generators occasioned by the fault. The resistance o f
the cables and the ship's structure may be neglected, and that
of the lamps taken as constant at the value corresponding to
the 60W rating.

22.

If an alternator supplies the following loads: ( a ) 200kW


lighting load at unity power factor, (b) 400kW inductionmotor load at 0.8 (lagging), power factor, (c) 200kW
synchronous-motor load, find the power factor of the
synchronous-motor load, to give an overall power factor of
0.97 (laggins).

23.

A d.c. shunt-wound machine is run as a motor, being


supplied with 55kW at 220V when its speed is 500 revlmin.
Find the speed at which this machine should be driven to
generate an output of 55kW with a terminal potential
difference of 220V. The resistance of the armature is 0 4 2 Q
and that of the field, which is the same for each case, is 1 IOQ.

74.

I\ s~nglc-phase.
50Hz trunsl'orn~er,has a core with a squ;irc
cross-section, each side being 270mm. The transfhrmation
ri~tiois 35001440V. and the mitxi~numflux density in rhc
core is not to exceed 1.4T. Find the number o f t u r n s of the
windings required if the frequency is 50Hz.

25.

Explain the purpose of interpoles and state their magnetlc


polarity relative to the main poles of both generators and
motors.

A 200V, long-shunt compound-wound generator has a


full-load output of 20kW. The various resistances are a s
follows ; armature (including brush contact) 0. I
S,
series
field 0.0250, interpole field 0.02%2, shunt field (including
the field-regulator rcsistancc) 1 1 5 0 . Tllc iron losscs at full
load are 780W, and the friction and windage losses 5WW.
Calculnte the ellicicncy i ~ full
t
load.

26.

A 500V, three-phase alternator supplies a balanced


delta-connected load in parallel with a balanced starconnected load. The delta load is 30kW at a power factor of
0.92 (leading) and thestar load is 40kW at a power factor of
0.85 (lagging). Calculate the line current and the power
-factor of the supply.

27.

A 440V shunt motor takes an armature current of 30A


at 700 revjmin. The armature resistance is V7R. If the flux
is suddenly reduced 20 per cent. to what value will the
armature current rise momentarily? Assuming unchanged
resisting torque to motion, what will be the new steady
values of speed and armature current? Sketch graphs
showing armature current and speed as functions of time
during the transition from initial to final, steady-state
conditions.

28.

' A load takes 250A at 240V and a power factor of 0.8


(lagging) from a 50Hz supply. The supply cable is then
operating at its full rating. Find graphically, the additional
power which can be supplied, without the cuhlc cxcceding its
rating, when a 800pF capacitor is connected across the load.

29.

Two shunt generators work in pi~rallcl. Each has i\n


armature resistance of 0.0132 and a shunt-field resistance of
85Q. Machine A is excited so that its e.m.f. is 600V, while
the other machine B is excited so that its e.m.f. is 620V.
What is the output of each machine when they jointly
supply a load of 2500A and what is the busbar voltage?

30.

Illustrate what is meant by phase difference in an a.c


circuit, by both wave and phasor diagrams. Three currents
which can be represented by: i, = 19 sin 6 amperes

i,

35 sin 8 -

and i, = 42 sin (8

);

amperes

$)amperes

flow in the same conductor. Find graphically the-maximum


value of the resultant current, its r.m.s. value and its phase
position relative to the 42A (maximum value) current?
31,

A 230V motor. which normally develops lOkW at 1000


rev/min with an efficiency of 85 per cent, is to be used as a
generator. The armature resistance is 0.15R and the shuntfield resistance is 2 2 N . If it is driven at 1080 revlmin and
the field current is adjusted to l . l A , by means of the shunt
regulator, what output in kW could be expected as a
gener;Itor. it' the armature copper loss was kept down to
t11i1t ~ v l i c tr~i ~ t ~ ~:IS
~ :Ii ~tiiotor?
ls
I ' w o c o ~ l hi11.1: c o r ~ ~ ~ c c t icl lc lp;trallcl across

;I

2 2 0 V , (A)II/.

supply. At the supply frequency, their impedances are 16Q


and 25R respectively. and their resistances are 3R and 7R
respectively. Find the current in each coil, the total current
and the total power. Draw a complete phasor diagram for
the system.

33.

A shunt-wound generator has a magnetisation-curve given


by the figures below. The total resistance in the field circuit
is 2 0 0 and the armature resistance is 0.02R. With the
machine on load, estimate the e.m.f. generated and the
armature current when the terminal voltage of the machine
-is 140V.
Field current ( 1 , t a m p e r e s 1.2 2.8 5.0 7.0 7.7 9.0 '1 1.0
Generatede.m.f.(E+volts 46 88 126 149 154 162 168
4

34.

A fwelve-pole, three-phase, delta-connected alternator


runs at 600 revlmin and supplies a balanced star-connected
loud. E a c h phase 01' tlie load is a coil ol' resistance 35R a n d
inductive reactance 2 5 0 . The line terminal voltage of the
:rltcrnalor is 440V. Dctcrminc ( a ) frequency of supply, ( b )
current in each coil, (c) current in each phase of the alterriiltor, ((1) t o t ; ~ l p o w c r \ i ~ p p l i c dt o thc l o ; ~ d .

35.

A 400V shunt motor has an armature resistance of 0.45R


and a shunt-field resistance of 4 5 m . When run on no load,
the current taken from the supply is 2.8A. Calculate the
o l ~ t p u t po\vcs slid clliclcncy o f tlic ~nacliinc wlicn tlic
~.lirrcriti t t ; ~ k c sI'r.om ~ h cni;~iri\i \ 15/1

36.

Find the core cross-sectional area in n~ni', the number of


turiis and the position of the tapping point for a 5OkVA.
50Hz. single-phase, 3700/2500V auto-transformer, assuming
that the e.m.f. per turn is 7V and the nlaximum flux density
is 1.5T.

37.

Three d.c. generators connected in pi~rallel,ei~clisuppl! a


load of 640A ro a sct of ZL'OV busbars. Thc e.rn.1'. ot' onc
generator I S r;il\cd from 230V to 2 3 5 V . I f t h u load and thc
resistances are constant. dctcrminc the current suppllcd
from each generator and the wltage at the busbars.

38.

A balanced delta-connected load and a balanced st:irconnected load are connected in parallel to a 220V. threephase supply. The delta-connected load takes a total power
of 50kW at a power factor of 0.75 (lagging), and the starconnected load, 40kW at a power factor of 8.62 (leading).
Calculate the power. volt amperes and power factor of the
supply.

39.

A section of a supply cable AB Ikm long has a fault to


earth such that, when end B is disconnected, the resistance
measurement from end A to earth is 5R. When end A is
disconnected, the resistance reading from end B to earth is
3R. The length of the cable A B has a resistance of 4R when
intact. Find the distance of the fault from end A .

40.

A coil of 0.84H inductance and 50Q resistance is conncctcd In scrics with a ci~pacitor of 14pF cap;~cit;~ncc.
( a ) Find the frequency I'or rcbonance ~ I I K I the poten~~;rl
differences across the capacitor, across the coil and across
the two, when a current of 5A ot this frequency, is flowing,
( b ) find the three potential differences when the same current
flows at 60Hz.

41.

A series-connected d.c. motor has a field and armature


resistance of 0 . 1 0 and runs at 600 revlmin when taking a
full-load current of lOOA from a 210V supply. Calculate the
speed of the motor when the torque is reduced 75'per cent.

42.

Each phase of a star-connected load consists of a resistor


of 1452 in parallel with a 4 0 0 p F capacitor. Calculate the line
current, power and power factor when the above load 1s
connected to a 440V, 60Hz, three-phase supply. What
power would be dissipated in the load, if i t is reconnected In
delta?

43.

A ring-main, 900m long, is supplied at a point A at a p.d.


of 220V. At a point B, 2 4 b from A, a load of 45A is drawn
from the main, and at a' point C, 580m from A , measured in
the same-direction, a load of 78A is taken from the main. I f
the resistance of the main (lead and return) is 0.25R per
kilometre, calculate the current which will flow in each
direction round the main from the supply point A and the
potential difference across the main, at the load where it is
lowe~t.
7

44.

A non-inductive-coil of 612 resistance is connected in


p:~r:illc.l witti ;In indilctivt. coil of 3R resistilnce and 9R
impedance at 50Hz. If a potential difl'erence of 1 IOV is
applied to the terminals. find the current in each coil and in
the ~ n t ~ i n it'
s . ;I ci~pi~citor
01' (d)Opl: is conncctcd in pi~r;~llcl
with the c o ~ l sof the above example, calculate the total
curren t.

45.

A 400V shunt motor has an armature resistance of 0.3R


and a shunt-field circuit resistance of 300R. Calculate the
specd attained by tlic motor when the current taken from the
supply is ( a ) 30A and ( b ) 50A. The open-circuit characteristic of the machine, when driven as a generator at 1500
revlrnin, is given by the following figures:
0.25 0.5 0.75 1.0 1.25 1.5
Field current (A)
Open-circuit voltage ( E ) 220 398 51 5 598 638 662
Ncglcct t l ~ ccffccts of i~rnmaturcrei~ction.

46.

A three-phase transformer has 560 turns on the primary


and 42 turns on the secondary. The primary windings are
connected to a line voltage of 6.6kV. Calculate the secondary
line voltage when the transformer is connected (a) StarDelta, ( b ) Delta-Star.

47.

Two batteries, of e.m.f.s 220V and 225V and internal


resisti~nccso f O 22 ;ind 0.3R rcspcctively, are connected in
l~i~t.i~llcl~ L I I ~ I > :II ~IO:ILI rcsibtor o l ~100. l:i~id tllc cilrrcn[
h ~ p p l ~ c1))d c;1cI1 b;~ttcr);111d[lie terminill voltage.

48.

A kct~lc.~ I l c nconncctcd to :I 270V d.c. supply. I ~ o r l h


1 l ~ t r col'water f'rom l I C' In 3.5 min. Calculate the percentage time d~fkrcnccwhen the water is boiled, by connect~ng
the kettle to a 220V. 5 0 H r c1.c. supply. The inductance of the
element isL0.05H. T,tke I litre of water to have n mass ol'
I I\g and i ~ \\pec~licIlc;~tc a p ; ~ c ~as
t y 4 2h.1 k g ( ' .

49.

In a shunt motor the four field coils are connected in series.


Each coil 1s wound to give 750 ampere-turns, the length of
each turn being 450mm. At the safe working temperature,
there are 45 watts dissipated at each coil. If the supply
voltage is 220V. find ( a ) the ficld current, (b) the diameter ol'
the wire, and (c) the length of wire in each coil. T a k e the
resistivity of copper as 2.0 x lO-%n~.

50.

An eight-pole alternator running at a speed of 720 revlmin


supplies current to synchronous and induction motors with
forty-eight poles. Calculate the frequency and speed of
rotation of the motors if the induction motor runs with
2 per cent slip.

SOLUTIONS TO TYPICAL FIRST-CLASS


EXAMINATION QUESTIONS
I

~ n \ , o l v e d;is ~t ivould appear. bur


This problem I S not
r e q u ~ r e s;in ~ ~ n d e r s t ~ i n dof
i n gthe electrical connections o f a
W;ird-Lconiird s!stcm.
Armature-resistance \,oltage d r o p 8.5V R, = 0.0288Q

Slitint-licld current I,, = '! I

65

1 6 9 2 ~

Sllllll , l l l l c l - l l l l l I
I 0 0 - 000
I002
S c r ~ c arnpsrc-turns
j
pole
315 x 1 l = 3575
Total ampcre-turn\ pole = 1692 + 3575 = 5267
Plot the graph. F r o m this @ = 3!55!! kilo\vebers per pole.
From the g i ~ e ne\presslon :
T = 195 I5 x 3150 x
x 6.8 - 15
= 29.515 x 3.15 x 6 . 8 - 15
= 632.2 - 15 = 617.2Nrn Ans.
3rrZ.T
7 x 3.14 x 370 x 617.2
O u t p u t polrer = -- =
60
60
3.14 x 37 x-~
617.2 - 116.18 x 617.2
3
3
= h I!
x 0.617 kilowatts = 24kW Anh.
:

, , 7

T o t ; ~ appiircnt
l
po\\cr-ol'comblned load. .S =

, \ L ~ I I \ Cpo\\cr

o l ' o r i y ~ ~lo;ict.
~:~l I),

140
cos Cb

140
0.8

--

(IOkW
90 = 90
A p p a ~ e n tpower of origin;rl load. .TI =
cos
0.6
= ISU~VA
Reactlie poiver of original load. Q , = 150 x 0.8
= l2OkVAr
Tlluh [lie reiictlLc power i x reduced by 120 - 105 = 15
k\';\r : I I ~(111s
L ~ L\'t\r l j ~ t i r e1 5 t11;1t o l ' t l ~ c~ i i o ~ oQr 2, oper;tting
: I I:I lc;1(111ig
j ~ o \ \ , ~I '*; I rc I o ~ . Tli11\ r c \ ; ~ c ~ ip\ co ~ c Q~ 2. o S no for
I5kV!ll~ All\
:

-- ---

AISO

SOLUTIONS TO TYPICAL LXAMINATION QUESTIONS


--- - --- - -- -

5 19

.-

apparent power ~f motor, S , = J S O 2 n Z


= d2500 + 225
=

dm

=
=

I0 x 5.22
52.2kVA

--,

= 10,,Tz

P
50 = 0.96 (leading) Ans.
Power factor of motor. = 3
=
S2
52.1

3.

When cold and the 'bridge is balanced' B and D are at the


same potential. If R increases in resistance value. current in
branch ABC falls and the potential of point B falls below D
until, for the 'hot' condition being considered B is 2 x 80
x
= 160 x l o w 3= 0.16V below D.
Currents will flow as shown in the diagram and. a s a
solution by Kirchhoffs laws will be made for the hot condition, the values I , and I, are allocated.

When 'cold' and a t balance; value of R = 4 f i


When 'hot : consider mesh BCD
Then 41, - 20 ( I , - 0.08) (2 x 0.08) = 0
o r 41, - 2011 + 1.6 0.16 = 0
and 41, - 201,
1.76 = 0
whence 1, - 5Z1 + 0.44 = 0
. . . (a)
Consider circuit ADC and battery:
Then 201, 20 ( I , - 0.08) + 5 (I2 II - 0.08) = 12
9

520

REE D ' S BASIC ELECTROTECHNOLOGY

o r 201, + 201, - 1.6 + 51, + 51, - 0.4 = 12


and 451, + 51, - 2 = 12
whence 451, + 51, 1 14
. . (b)
S o l \ , ~ n p( a ) and ( b ) thcn: 451, + 51, = 14
and - 451,
91, = - 3.96
Adding 141, = 10.04
5.02
o r I, = - - = 0.717A
7

i\pplving Kirclllioff's law to circuit ABC and battery. T h e n :


R(O.717-0.08)+3(0.717)+5(0.717+0~2314-0.08) = 12
or. 1: ( 0 . 0 3 7 ) 1 7,HOX 1. 5 (O,HOR4) .: 12
a n d 0,637K 3- 2.868 1 - 4,342 = 12
whence 0,637R = 12 - 7.21
47.9
Thus 0.637R = 4.79 or R = ---- = 7 5 3 2
6.37
Let T, = the hot temperature and T, = the cold temperature = 15.55'C. The temperature coefficient, as given, is
assumcd to be for a base of 0C. Thus c i = 0.002 431C.
o r 7.52 ( 1 + 0.037 79) = 4 (1 + 0,002 43T2
whence 7.52 + 0,2842 = 4
0,009 72T,
7,8042 - 4 = 0,009 72T2

= 391.37'C

Ans.

Although the transformer has not been dealt with in any


dct;~ilin tliis book, [his problc~ncan bc workcd from first
pr~nc~plcs.
outpllt =
outpul
-l'llus cfliclcllcy (11) =
lnpi~t output + losses
-

can be wrltten as:


output ( k W 1
G u t ( k W )+copper loss fkW)+lron loss (kW)

expression

'

----

SOLUTIONS TO TYPICAL IeXAMIh'ATION QUFSTIONS


-

52 1

175000 = 1750
=26.51A
Now primary current = ------6600
66
26.5 l 2 x 0.4
Primary copper loss =
= 0.703 x 0 . 4
1000

1 7 5 0 0 --3 9 7 . 7 *
Similarly, secondary current = ' 7 5 o o o = 410
44
397.7' x 0.0015 = ,58
Secondary copper loss =
0,
1000
Thus q =

175 x 0.9

175 x 0.9

+ (0.2812 + v237.) + 2.75

Efficiency on full load = 98 per cent

5.

Ans.

The O.C. characterist~cis plotted as shown and a tangent


e
from this tangent is
drawn. The resistance v ~ ~ l uobtained
the critical resistance. Thus consider a field current of 1A
The voltage value for this current, reading off the critical-

I SO

yl

C
-1

0
>
W

"loo

Y
S

C
-8

50

2
J
FIELD CURRENT

- AM AM PERF^)

522

REED ' S BASIC ELECTROTECHNOLOGY

resistance line is 120V. The critical resistance is therefore

According to the glven data, the field resistance is


= 60R and the required O.C. voltage is obtained, by
2
taking any field-cnrrent value, say 2A, and finding the
voltage value for this current. Thus 2 x 60 = 120V. Plot
this value and join the point to the origin t o obtainthe field
voltage-drop line. The intersection with the O.C.C. gives the
required voltage = 146V Ans.
Apnin, from the O.C.C., an e.m.f. of 135V requires n field
current of 2.1A. I t must be remembered that the field resis1;111ccIias not b c e ~;~lterecl
~
and is 60R.'l'he volti~gcneccssil r y
to maintain 2.1A through this field resistance is the terminal
voltage.
:. Termlnal voltage = 2.1 x 60 = 126V Ans.
Armature voltage drop = 135 - 126 = 9V
Also 9 = I,R,
9
9
- -= 0,045Q
Ans.
.'.
R,=
200 + 2.1
202.1
Nore. I, = line current + shunt-field current

6. Using the expression E,, =


4

.M@2

- 0 1 )

60 [(0,4 x lo-') - (- 0.4 x


volts
1
40
= 60
0.8
1 0 4 x 40
o r E,, = 4.8 x 4 x l o - ' = 1.92V Ans.
Also E,, = L x average rate of change of current
I
12
Thus 1.92 = L x - volts
o r 1.92 = L x - .
I
1
--.

We have E,, =

80
.\'o/(n. The tirnc taken for the current to fall to zero h i ~ sbeen

taken.
The inductance of the coil is 2mH Ans.
1. c;\n also he round from first principles, since the inductance value can be detcrmineci frorn the flux-linkages per.111il)cre

= 0.002H

7.

Ans.

Let 1 bc the current in llle sllorl section ol' the ring. it* ill
the 4001n length. Therefore ( 1 10 - /) is the current in the
2000 - 400 = 1 600m lengt 11.
Resistance of 400m of cable (double conductor)
- 0'032 8oo = 0.032 x 4- = 0.0256Q
1000
5
Resistance of 1600m of cable
= 0.0256 x 4 = 0.1024Q
Since points X and Y are connected by both sectlons of t h e
ring, it follows that the voltage drop in the short-section
= the voltage drop in the longer section
or I x 0.0256 = (110 - I) x 0,1024.
and 0.02561 = 0.1024 x 110 - 0.10241
or 0.1281 = 0.1024 x 110 giving I = 1024 x 1 1
128
= 88A
Current in shorter section = 88A Ans.
Current in longer section = 110 - 88 = 22A Ans.
Voltage at load = 220 - (88 x 0.0256) = 220 - 2.25
= 21 7.75V Ans.
C

8. Cos 4 = 0.866. .: 4 = 30'. Thus current lags voltage by 30'.


Mslximu~nvaluc of' voltage - 'I1
- 34OV
0.707
70.7
Maximum value of current = -- = IOOA
0.77
1
Time for 1 cycle = second = 0.02s
50
Time for $ cycle = 0.0 1 s
Since voltage is sinusoidal, then v = 440 sin (2 x 180 x 50 x t )
When t = 0.001s, v = 440 sin (2 x 180 x 50 x 0.001 )
= 440 sin 18' = 440 x 0.309 = 136V
v
=
440 sin 36" = 440 x 0.588 = 259V
When t = 0.002s,
=
0.003s,
v = 440 sin 54' = 440 x 0.809 = 356V
,,
,,
= 0.004s, v = 440 sin 72' =;
= 418V
,>
= 0.005s, v = 440s sin 90' =
= 440V
,,
= 0.006s, v = 418V, erc, P I C .
The voltage wave is plotted to a time base t , as shown.

524

R E ED ' S BASIC ELECTROTECHNOLOGY

Similarly, for the current wave.


Since i = 100 sin (2 x 180 x 50 x r)
W h e n r = 0.001s. I = 100sin 1 8 ' = 1 0 0 ~ 0 , 3 0 =
9 30.9A
,,
= 0,002s, I = 100 sin 36' = 100 x 0.588 = 58.8A
= 80.9A
,,
=d0,003s, r =
= 95.1A
,
= 0QO4s, i =
= 0.005s. i =
.= lOOA
= 0.006s. I = 95. ] A , el(., rrc,.
Draw the current wave to a new scale with its zero diiplaced
from that of the voltage time scale by 0.001 66s. Use the
ordinates dcrived >ibovefor this new time scale.
Notc. If a time of 90' = 0.005s. 30' = 0,001 66s.
Read ofT the required answers from the original volti~getime scalc on both the voltage and current wavclorms.
Thus when
I = 3 x 10 .' = 0.0034, 1' -- 355V 2 n d i = 38A
Ans.
I = 0406s,
1, = 415V and
i = 96A Ans.
I = 0.01 1s.
I ' = - 140V and i = 25A
Ans.
1, = .- 260V and i = - 93A Ans
I = 0-018s,
1.

( a ) Assurnc rhe meter indicates correctly at 20'C when the


l~cblb~~lllcL~
I\
I O O O ~ ~1 . . ~ 1 I , I)C l I l C I ~ l C I C l ~~Llrl~cllt
~
L l l tI1is

525

SOLUTIONS TO TYPiCAL EXAMINATION QUESTIONS

temperature. Also let R2 and T 2 be the resistance and


temperature at 50C and let R, and T , be the resistance and
temperature at 20C.
Then from the resistance to temperature relation,

and R ,

1000 + 4.28 x 50
1 + 0.0856

Meter current I2 at 50C =

- 1214
--

18,20

I ,0856

mO

1 118.2 w h e ~ eI, =
amperes
The voltage across the meter can be assumed constant.

...

1 2 - 'Oo0
or I, = 0.8941, amperes
1, 1118.2
*
Now meter deflection is proportional to current

:. Percentage error =

diRrence in readings
true reading

Thus percenrage error

0.1061,
100
-

10.6 per cent (low) Ans.

100

I .1

(b) If the series resistor is made from manganin, its value


at 50C does not vary and the resistance of the 5S2 copper
coil is given by R 2
R2-- R,(1 + 0.004 28 x 50) - 1 0.214
R
R,(l
0.004 28 x 20)
1.0856
~

The resistance of the meter at


= 1000~58i2
New current I, =

--

1000.58

f2

is now 995

+ 5.58 ohms

V
o r as before I, = -- a m p e r e
I000

I2 - 'OO0
So new current 1, = 0.99941,
1, 1000.58
.: Percentage error, as deduced before,

..

o r percentage error = 0.0006 x 100

0.06 per cent (low)


Ans,

10. Branch A . induct~vereactance. X L = 2?JL


o r X L = 3 x 3.14 x 50 x 38 x
= 3.14 x 1.8 = 1 1.9320
1
C a p a c ~ t ~ vreactance.
e
X -- 2~fC
1 o2
1 O6
- or Y
C =
? x 3.14 x-50 x 312.8
3.14 x 3,128
= 10.160
Result:~ntreactance. ,\' = X L - X ,
= 11.932 - 10.16
or X = 1.772R (inductive)

cos 4,

cos 4,

=lo =

10- 1 7

--

z,

0.98 (lagging)

O sln 4,

= 1

Solving mathematically: I, = (43.2 x 0.98) + (36.66 x 0)


= 42.3 + 0 = 42.3A
I, = (43.2 x 0.174) + (36.66 x 1 )
= 7.52 + 36.66 = 44.18A
Resultant current
I = J42.3' -L 44,182
= loV'4.23' + 4.418* = 10 J17.81 + 19.5
= 1 0 v m 1 = 10 x 6.1 = 61A A n s .
42.3
Power I'i~cror,cos d, =
= 0.694 (lagging) Ans,
61
\ ' O I ( > . ' 1 ' 1 1 ~ p1.0hIc.111
1i:is k~e11solved ~ii;tllie~ii;tlic;~II~~.
~ L I
[lie grapliic;~l .;elution is made b ~ clioosing
.
a suit;tble
current scalc and. L V I ~ I I voltage as reference, rc, along the
horrzontal. drau I , lagging the voltage by an angle whose
coslne is 0.98. Newt draw I , lagging the voltage b), 90
Complete the p:rralleloram. Draw the longer diagonal,
measure this 10 sc~rleand :ilso the angle of lag. The cosine of
tills iingle will g ~ \ cthe power fiictor of the circuit. Thc
answers sllould chcck wlrh those obta~nedabove.
4

SOLUTIONS TO TYPICAL EXAMINATION QUESTIONS

527

1 1 . Using the exp_ression:


Inductance (in henries) = flux-linkages per ampere
NQ,
8 x 1200 x 50 x lo-'
Then L = -- =
I
5
A l s ~E,, = L x rate of cliangc: ol'currc~i[
5
- 96 5 2 = 96 x 102
= 96 x
50
104
5
IO= 9600V
t
Thus the value of induced e.m.f. is 9.6kV Ans.

12.

(a) Although no work has beei~done in this book on the


transformer, it can be stated that the voltages induced in the
r proportion to
primary and secondary windings are in direct
the turns.
Vl -N
Thus 1o r V , = V 2 x N
2

V2

N2

N2
1875
Hence V, = 220 x - = 220 x 25

75
and primary voltage = 5500V o r 5.5kV Ans.
(b) The kVA rating applies equally to the primary and
skcondary sides.
kVA rating x 1000
Thus primary .current =
V,

Secondary current =

kVA rating x 1OOO

v,

- 550 x lOO?l - 5 x lo00


220
2
= 2500A Ans.
(c) This part of the question is answered by a knowledge
of the e.m.f. formula for the transformer. This is developed
in Volume 7 but can be memorised.
Thus V , = 444@, fN,volts.
o r 5500 = 4.44 x @, x 50 x 1875
5560
- 55
10-~
Thus @, =
2.22 x 100 ~1875
- 2.22 x 1.875
-

55

lo-' = 13.2 x 10-' webers

4.163

Maximum value of flux = 13.2mWb Ans.

220
= 2A, then I, = I , - 1, = 6 - 2 = 4A
110
Input power = 220 x 6 = 1320W
1
Copper loss (armature), Z ~ R=, 4' x 0.25 = 16 x - = 4W
4
Copper loss (field), I:R, = 2' x 110 = 440W
Total copper loss = 444W
Rotational loss = 1320 - 444 = 876W. These are the losses
attributable to Iron, friction and windage. Ans.
CVhen the current is 62A, the input is 220 x 62 = 13 640W
The output = input - losses (all values in watts)
- 13 640 - copper losses - rotational Iosse.;
10.10 - (001 x 0.25 -1 2' x I 10) - 870
= 13 640 - (900 + 440) - 876
13 040 - 7710
1 1 424W

I?. Since I,

--

'2= 0 817 or 83 7 per cent Ani


13 640
\ate F O I 'I innre complete u n c l e ~ k t ~ ~ n of
d ~tnhp~ problem.
s
the reader should refer to Volunle 7, Chapter 1

So i.lfic~enc\ =

14.

A basic definition of power factor is that: it is the ratlo of


the 'active power' to the 'apparent power' being expended
In a circuit.
active power Thus power Ii~ctor.cos 4 =
iipparent
power
S
~.
or active power = apparent power x power fr-lctor
For theprohlem :
t'
S cos 4 or liW = kVA x cos 4
Hence 560 = kVA x 0.7
560
TIiu4 ;Ipparent power buppl~ed= - - = 800kVA,
0.7
\L'~th the power factor Increased to 0.8 and the apparent
power kept constant, the new 'active power' = 800 x 0.8 =
640kW
The Increase in active power would be:
(A0 - 560 = 80kW Ans.

15.

' 1 Ile c c l u ; ~ t i ofor


~ ~t l ~ ccllarpng circuit is V
E,-kIR, i I / (
where I:, is the battery c.m.f'., I the current, I<, thc batter)
jnternal resistance. R the resistance of the lamp bank and I/
the supply voltage.
A t commencement of charge:
200 = IOR + 10R, + I:', here R i= 48 x C.02 = 0.96R
E, = 48 x i . 8 = 86.4V
~2

SOLUTIONS TO TYPICAL EXAMINATION QUESTIONS

529

.: 200 = IOR
(10 x 0.96) + 86.4
or 10R = 200 - 9.6 - 86.4 = 200 - 96 = 104
:. R = 10.452 ohms.
Now a lamp is rated at 1 lOV and 32c.p. or 32 x 3.5 = 112W
V2
1102- 121 x 100o,,mr
Resistance of a lamp = - = -- = .
P
112
112
= 1.08 x 100 = 108R
If 'n' lamps are used in parallel
1
t h e n 1- = + 1- I+ . . 11 times
R
108 108 108'

Ll
--- 10.4, say 10
2.6
Thus number of lamps = 10 lamps in parallel at start. Ans.
At start of charge:
If R = 108
- = 10.8 ohms, the current is obtained
10
from 200 = (Il x 10.8) + (I, x 0.96) + 86.4
C

At end of charge:
If 10 lamps are in parallel, then R = 10.812.
So 200 = (I2 x 10.8)
(I2 x 0.96)
Eb . Here I, is the
final charging current value and Ebi is the final back e.m.f.
Then Eb = 48 x 2.2 = 105.6V
and 200 - 105.6X = 1 1 ~ 7 6 1 ~

Current at end of charge = 7.89 Ans.

16. Resistance of heater

6.5R Working voltage lOOV


100
:. Rated current = - = 15.38A
6.5
Reactance of choke, X, = 2nfL
o r X, = 2 x 3.14 x 50 x 0.01 = 3.14R
Impedance of circuit, Z = d6.52+ 3 . 1 4 ~= ~'42.25+9.86
or Z = 1= 7.22!2
Circuit voltage would be = 15.38 x 7 . 2 2 = 1 1 144V
Applied voltage should be 11 144V to give 1OOV on heater.
Ans.
=

530

REE D ' S BASIC ELECTROTECHNOLOGY

I f frequency rose to 50

5
+ (--100
x

50) = 52.5Hz. XL would

rlsc In proportion.
:.

New reactance X,,

3.14 x

52.5
50

---

= 3.14 x

1.05

V
111.04
15.23A
Z,
7.288
NCIVv01~1gc;\cross hcater tcnninals = 15.23 x 6.5

New heater current

= - = -=

= 98,995V
Ch;~npc01' volt;~pc= 100 - 98,995 = 1,005V Ans,

17. Force on one conductor:


F' = BII newtons
= 0.8 x 70 x 400 x
newtons
= 0 . 8 x 2 x 4 = 6.4N
Force d u e to all active conductors
=

23

lO5ON

x 246 x 6.4 = 164 x 6.4

( a ) Torque produced

=
=

1050 x 150 x
157.5Nm Ans.

( b ) Shaft power developed = 2''1'T- watts

60

3.14 x 500 x 157.5


60
- 3.14 x 50 x 157.5
3
= 3.14 x 50 x 52.5 = 8243 watts
= 8.24kW Ans
-

IS. Output I'rom motor

45kW

1000
100 w;,[[s r- 4X 387%'
93
= 48.4kW
V I cos Q,
But 3-phase power (watts) I
. . 48 387 =
x 440 x I x 0.88
11113\11

1 0 1110101.

45

SOLUTIONS TO TYPICAL EXAMINATION QUESTIONS

19. Let

53 1

= the no of cells in series.


Battery e.m.f. = n x 1.5 volts
Battery internal resistance = n x 0.5 ohms
Also since P = 1 2 R, then for the load: 2 .= 12 x 100
11

Also for the circuit, current I is given by:

giving n

I0 (approx) Ans.
1.5 x 10
_- -l 5 = 0.143A
Asacheck: I =
100 + (0.5 x 10) 105
So power dissi p ated = 0 . 1 4 3 ~x 100
= 0,143 x 14.3 =,2.045W
o r the given rating. 2W (approx)
20.

The conductors, being cut a t the same speed by the same


magnetic field, must each be generating the same maximum
value of e.m.f., but as they are spaced 20" apart, their e.m.f.s
must be 20' out of phase with each other. As the conductors
are in series, the e.m.f.s phasors can be added by a method
already described.
Resolving the phasors into hor~zontaland vertical com
ponents, we have :
Total horizontal component
= 200 cos 0" + 2 0 cos 20" + 200 coa 40"
= 200 ( 1 + 0.9397 + 0.766)
= 200 x 2.7057 = 541.14V
Total vertical component
= 200 sin 0"
200 sin 20" + 200 sin 40'
= 200 (0 + 0.342 + 0.6428) = 200 x 0.9848
= 196.96V
Resultant maximum e.m.f.

The maximum value of the resultant e.m.f. = 576V


(a) The r.m.s. value = 0.707 x 576 = 407.23V Ans.
actual e.m.f. generated - 576
( b ) Breadth factor =
total possible e.m.f.
600
Thus 'breadth factor = 0.96 Ans.
9

$'oft.. Maximum values have been used since the ratio of'
tlie r.m.s. values would give the same result. The total e.m.f.
would be the e.m.f. generated, if all conductors were in series
without phase displacement.
60
1
21 current taken by a lamp at full voltage = ---- = - = 0.25A
240 4
240
Resistance of I earth lamp = --- = 960R
0.25
Let R , be the resistance between the +ve line and earth.

Lcr li, bc tllc rcslbtance bet!\-een the

- ve

line and earth.

Total resistance between + ve and - ve line


= 15.74 + 5.96 = 21.7C2
Current flowing between the +ve and -ve line
:.

Voltage across +be line and earth = 10.14 x 15.74 volts


= 159.6V
,,
- ve .. ,. ,, = 10.14 x 5.96 volts
= 60.43v
d

( a ) The + ve lamp, io lamp connected between +ve line and


;.ns.
earth will burn the brighter.
220
220
- (b) Current passed by the fault =
15.74 + 5.96 21.7
= 10A (approx)
Additional load on generator = 220 x 10 = 2200W
= 2.2kW Ans.
22. Load (a) P , = 200kW cos 4, = 1 sin 4, = 0
S, = 200kVA Q, = S, sin 4, = OkVAr
,, ( b ) Pb = 400kW cos & = 0.8 sin & = 0.6

Qb = Sbsin

4,

= 500 x 0.6
= 300kVAr

,, (c) P C = 200kW
Total active power of loads, P = 200

+ 400 + 200 = 800kW

533

SOLUTIONS TO TYPICAL EXAMINATION QUESTIONS

Total apparent power of all loads, S = Plcos 4


- --800 = 824.74kVA
0.97
Also cos d, = 0.97. .: 4 = 14" 4 : and sin 4 = 0.243
So reactive power of all loads, Q = S sin d, = 824.74 x 0.243
= 200.4kVAr
Thus lagging kVAr value is reduced by 300 - 200.4 = 99.6
This must therefore, be the leading reactive power Q, of the
synchronous motor. Apparent power rating of motor
S, = J20o2 + 99.6' = ~ o o J+~0 .~9 9 6 ~
= 100d2= 100J;rW2
= 100 x 2.233 = 223.3kVA
200
Power factor of motor, cos 4, = ---- = 0.89(leading) Ans.
223.3
23. Motor condition.
Input power

c.

55 000
Line current ='= 250A
220

55kW

Field current =

2A

Armature current'= 250 - 2 = 248A


Back e.m.f. = 220 - (248 x 0.02) = 220
= 215.04V
Generator condition.

4.96

55 000 = 250A
Line current = 220

Output power = 55kW

220 = 2A
Field current = 110

Armature current

250

= 252A

+ (252 x 0.02)
+ 5.04 = 225.04V
325.04 - N2
As e.m.f. is proportional to speed, then =--215.04 500
Generated e.m.f. = 220
= 220

Thus generator speed

=
=

1.0464 x 500
523.2 revlmin

0.5232 x 1000
Ans.

24. If B, = 1.4T and area of core .= 0.27 x 0.27 = 0.0729rn2


Then 0, = 1.4 x 0.0729 = 0.102Wb
Substituting in the formula :
3500
V , = 4.44@JN1 o r N , =
turns
4.44 x 0.102 x 50

3500
3500 = 154.7 turns
2.22 x 10.2 22.64
or primar), turns = 155 (approx)
I,
Second'lrq turns = 440 = 3500
155
44
792
,155 x --- = -- = 19.5 turns
or I.=
350
35
Thus secondary turns = 30 (approx) Ans
:.

I., =

35 Load current of generator =

20 000
= l0OA
300

'00
1.74A
115
A ~ I ~ I ; I ~LI .IIVI IC. ~ C I 101
I ~ 7A
Copper lobs of armature circuit = 101 .7-L(0.15+ 0.025 +
0.028) = 101~72(0.203)
= 2099.6W = 2.1 kW
Copper loss in shunt field = 1.74 x 200 = 348W =
0.348kW
Total copper loss = 2.1 + 0.348 = 2.448kW
output output (k W)
Efficient) = -- ---Input
output ( k W ) + losses ( k W )

Shunt-field current

= 2=

20

copper 105s + Iron loss

Thus full-load efficiency = 83.9 per cent

rotational

105;

Ans.

26 Delta-connected load
Act~be pou rer, P, = 30kW at a power factor of 0.92
(leading)
30= 32 61 kVA
Apparent power. S , = -

0.02
c.o\ 4) = O 93
4 = 22 56, sin qb = 0 3896
Co reactl\e power, Q , = 32 61 x 0 3896 = 12 7kVAr
\ [ < I 1 - ~ o l l l l c ~IO'l<i
~~<l
A c t i ~ e power. P , = 40kW at a power factor of 0 85
( I,~gging)
Apparent pouer. S 2 = -40 - 47 ILVA
08 5

SOLUTIONS TO TYPICAL EXAMINATION QUffTlONS

535

Note. - ve sign g e e n to the lagging reactive power value


to distinguish it from the leading reactive power value of
the other load.
Total active power on alternator, P = P , + P2
= 30 + 4 0 = 70kW
Total reactive power loading, Q = Q , + Q 1
= 13.7 - 24.7 L-. - I3kVAr
So apparent power, loading on alternator is:

43Vl
gain three-phase kilovolt amperes is given by -

lo00
1.732 x 500 x I 71
.06
Hence
1000
a
71.06 x 2 - 142.12
or
I=
- -= 82.1A
1.732
r
,
1.732
Line current = 82.1A Ans.
P =70 = 0.98 (lagging) Ans
Supply power factor = S
71.06
The lagging corldition is determined from the resultant -ve
sign of the total reactive power value.
Original conditions:
Eb, = 440 - (30 x 0.7) = 440 - 21 = 419V
Original flux condition @,
Final flux condition :
ds, = O)Rds,.Assuming n o speed change then since generated
e.m.f. is proportional to flux
new E, value = 0.8 x 419 = 335.2V
and momentary current is given by I.' - Eb amperes
R.
If final torque condition T , = original torque T , and since
torque is proportlonal t o flux and armature current; then
T = k@I,
Thus we can write T , = T , or k ~ D ~=l , k@,In2

?!! = 1 3 7 . 5 ~
0.8@, 0.8
New armature current wili be 37.5A Ans.
New back e.m.f., Eb = 440 - 37.5 x 0.7
= 440 - 26.25 = 413.75V

and la, = %L

Also since Eb cc Oh'


E
k0,lY2
413.75 - 0.8@,hT2
we can write
= ---- or ------ - ----------Eb, X-@,N,
419
700
413.75 x 700 289
625
- ------- = 864.4 rev mln
whence N , =
419 x 0.8
335.2
Ans.
28. Original power transmitted by supply cable is given by:
240 x 250 x 0.8 = 48kW
P , = V l cos 4 =
1000
Current taken by a 800pF capacitor 1s given by;

and capacitor current =

240

--- =

60A

There are now two currents, I , = 250A, lagging the


voltage by 36" 44' (Note cos36' 44' = 0.8) and I c = 60A,
leading the voltage by 90c. The problem calls for a graphical
solution. Draw a voltage ordinate horizontally. Choose a
suitable current scale and, from the origin, ie left-hand point
of the voltage ordinate, draw I,, to scale, ve~ticallyupwards.
Next from the origin draw I, to scale below the voltage
ordinate by 36' 44'. Complete the parallelogram for the
current \,ectors. draw and measure the resultant current for
r i i , ~ g ~ i ~atn~$~ pliasc.'This
dc
part of the problem is wnrhcd
out hcrc rnathematicaily.
TIILISI = J I ~
+ Ic2
~ + 21L,ICcos 0
=
=

d250'
"'250~
2;~?

lo2\

--

+ 60' + (2
+

x 250 x 60 x cos 126 44')


602 - (2 x 250 x 60 x cos 5 3 16')
~ 7 7 3 ~ 7 3 x 9 3 7 ~ )
7
-

=
' 6 . 7 5 t 0.36 - 1,7943 = 10't'4.8157
= 10' x 2,194 = 219.4A Ans.
Assulning the same power of' 48kW was being supplied,

the new power factor of the supply would be obtained


from :
48 000
-- 200
/I2
-:I 'I cos 42 or cos 42=
240 x 219.4 - 219.4
= 0.91 (lagging)
71ie question is not cleiir as to the final loading condition5
b l ~ tI I is ;~ssumcci!hat this power factor condition 01' 0,OI

537

SOLUTIONS TO TYPICAL EXAMINATION QUESTIONS


-

(lagging) is maintained. The maximum current would be


limited to 250A and the power transmitted would be, P2.
240 x 250 x 0.91 kilowatts
Thus P2 =
1000
= 60 x 0.91 = 54.6kW
Additional power, Pz - P, = 54.6 - 48 = 6.6kW Ans.
29.

Altliough the parallel working of generators has not bccn


considered in this book, this- problem can be solved by a
direct application of Kirchhoff's laws. Thus, let I, be the
current output from machine A and IB the current output
from machine B. Let I/ = the common busbar o r terminal
voltage.
Then I , + I, = 2500A and two voltage equations can be
v
I.'
built up as follows, since I,, = I, -t - and I,, = I, + 85
85
Thus V = 600 - 0.0151,,
and V = 620 - 0.01 51,,

and

V=620-0.015

I,+-

giving V = 600 - 0.01 51,

gy5)

0.01
5V
--85
0.015
V
---85

V = 620 - 0.0 1 5 IB Subtracting (a) from (b),


O = 20--0~01510+0~0151,
or 0
and 0

--

20

0~015[1,,- . (2500

=
..

20-0.015(1,-I,)
I,)]

20 - 0.015[1, - 2500 + I,]


= 20 - 0.031, + 37.5
57.5 5750 = 1916.66A Ans
Thus I, = -0.03
3
I, = 2500 - 1916.66 = 583.34A Ans.
0.01 5 V
From (a) V = 600 - (583.34 x 0.015) - --85
=

Whence V
giving

85V

0 ' 0 1 5 v- 591.25
+85

+ 0.015V
85

= 59 1.25

. . . (b)

85 = 6.94 85
85.0 15
Thus
I' = 590.75 Volts Ans.
Output of machine 4 = 583.34 590'75 = 344,fkW A ~ . . ;
1 000
lf = 591'25

and

l o [ ( :ZItlio~igl1tilt' h W r'ltlng of B I S correct for tile ~ ' t l ~ i e s


g l e n in the question. ~t is high for the accepted slzes of d c
l~l,lcllll~c

30. The current can be written a s :


I,, -19 sin 0
1,

35 sln I,

180
)
- ,

or

i, = 35 sin (0 - 90)

A graphical solution is required. Use i, as reference.


Choose a suitable current scale and draw O A horizontally
from left to right to equal 19A. Draw OB vertically downwards. irl 90' behind OA and make a length to represent
!5A. Finally. draw OC. 42A long 150' behind A, i~ it will be
30 belo\\. the horizontal and should be drawn from rlght to
left Complete the parallelogram for QA and OB, draw the
resultant and call this OX. Now use O X with O C to complete
a ncw paralfelctgrarn and draw the resultant O Y . Measure
O Y to scale and this gives the required answer.
Multiply the answer by 0.707 and a further answer is
obtained. Finally measure the angle between OC and O Y .
The required phase position is thus obtained and O Y will
lead OC. A mathematical solution for the above problem is
set out below.
Kesolvlng into horizontal and vertical components we
11;1ve:
I,, = I9 c.05 0 1 35 cos 00 - 42 cos 30
= (19 x I ) + (35 x O) - (42 x 0,866)
I0 b 0
7 0 772
17.372A
/,. = 19 sin 0 - 35 sin 90 - 42 sin 30
= (19 x 0 ) - (35 x 1 ) - (42 x 0.5)
= 0 - 35 - 21 = - 56A
The resultant I = v/i7,3722 562 = 102/1.7372~
= 10t'3.017 + 3 ! . 3 6 ' = ' l o m 8
10 x 5,864 = 58.64A Ans.

+m

539

SOLUTIONS TO TYPICAL EXAMINATION QUESTIONS

The phase angle can be obtained from

The resultant IS thus (73' 8' - 30') = 43" 8' ahead o f thc
third current-of m a x i n ~ u ~valiic
n
= 4312 Ana.
The required r.m.s. value of' the resultnnt = 41.46,A .411s.
31. Motor output = IOkW
100
200
input = 10 x - = --- = 11.76kW = 1 1 760W
85
17
1 1 760
Current taken from supply = -= 5 1.13A
230
230
Shunt-field current. I f = --- = 1.045A

,,

220

Armature current, I, = 51.13 - 1.045 = 50.085A


Back e.m.f., Eb = 230 - (50.1 x 0.15)
= 230 - 7.51 = 222.5 volts
As a generator, speed is increased and flux is increased in
proportion to the shunt-field current.
1080
1.1
... E = 222.5 x ---- x -1000
1,045
= 222.5 x 1.137 = 252.98V
Also since armature copper loss is the same as for the motor,
armature current must be the same = 50.1A.
:. Terminal voltage, V =r 252398 - (50.1 x 0.1 5 )
= 252.98 - 7.51 = 245.47V
Output current = 50.1 - 1 . 1 = 49A

So output = 245'47 49 = 12.03kW Ans.


1000
220
16
= 13.75.A Ans.

32. Let IA = the current in the first coil, then I, =

cos

4,

.5
== 0.187

16

4,

= 79" 13'

sin

4,

--

0,9824

220

I, = the current in the second coil, then I , = ?C


i
.J

= 8.8A

cos

4,

= - = 0.28

25

6,= 73" 44'

sin

4,

0.96

Ans

Total active components. I,

(13.75 x 0.187) + ( 8 . 8 x 0.28)


amperes
= 2.57 + 2.464 = 5.034.4

Total reactive components.


I, = ( - 13.75 x 0,9824)- (8.8 x
0.96) amperes
= - 13.51 -8.448 = -21.96A
Resultant current.
I = h . 0 3 4 ~+ 21.962 amperes
= J25.34 + 482.24
= J5O758 = 10 d5.075
= 10 x 2.253
= 22.53A Ans.
5,034
cos 4 = ---- = 0,223 (lagging).
22.53
IJower, / J - 220--Y 22 53 r 0 223 k~lowatt\= 1 1 0 5 k W
1000
Ans
The ph~rsordiagram tor this problem. can be cnns~derecl
as one of the baslcs for parallel circu~t.and 1124 :~lre;\ti!
been illustrated seLeral tin-ies.
Tile rn,~gn~tucle
of the e

n1.f.generated

on open c ~ r c u i [is

541

SOLUTIONS TO TYPICAL EXAMINATION QUESTIONS

determined from the point where the field \.oltage-drop line


intersects the O.C.C.
Plot the O.C.C. and the field voltage-drop line b!' obtaining a typical value. Thuh. for a field current of4A. the \oltage
drop would be 4 x 20 = 80V. I'lot tliis ~ x ) i n t .1)1.;1w ;I lint
from tlie origln through this point and produce the line to
cut lllc O.C.C. Thc intluccd c.m.S, is rlle11 I55V.
The \.oltage drop iicros4 the field, 1s also the terminti1
voltage of tlie generator and from thc graph a tcrniin;ll
voltage of 140V is obtained when the field current is 7 A . 1 0
read horizontally for 140V on the voltazc scale to determine
tlie corresponding field current value on the field voltnse
line. This value is 7A. For this balue of field current Ilo\vever. the e.m.1'. generated 15 149V. Ans.
Also 1.' = E - I,R,, :. 140 = 149 - (I, x 0.02r o r
c
0.021, = 149 - 140
Giving an armature current, I,
34.

oCi?

450A

An,

A fundamental formula for tlie alternator and


supply is.
f'= P N wlierc P = tlie number of poles.
120

(a) Therefore/ =

,in

a.c

= 60Hr Ans
120
( b ) S ~ n c ethe load is balanced. t ! ~ e boltage krcros.; each

p h i l s ~ ,I,',,

= 44" V O I ~ S

\, 3
Impedance of one phase of lo;\d. Z,

--.

. . -, --.

ohmi

o r Z,, = 1 0 d m 2 ?
1 0 ~ / i 2 . 3 5+ 0 . 2 5
= 10dm = 10 x 4.301 = 43.01R
For a star-connected load, current in one phasc o f loaci.
I,,, = Line current, I
440
- 440
- 5.91A
o r I,, =
1.732 x 43.01
74.49
Thus current in a coil = 5 9 1 A Ans
(cl Current in each phase of alternator,
I
5.91 - 5.91
or I,, = -- o r I,, = - = 3.41A An5
't 3
3
1.732
R
35
( d ) Power factor of load = cos 4 = - = -Z
13.01
= 0.81 (lagging).
,

and total power of load =


-

=
=

3 VI cos 4
1.732 x 440 x 5.9 1 x 0.8 1

Ioo0
kilowatts
1 . 7 3 2 ~ 0 . 4 4 ~ 5 . ~9 01 . 8 1
3.65kW Ans.

3 5 , No load
I n ~ u to
t motor = 400 x 2.8 = 1120W
8
Shunt-field current. I - 400
= - = 0.89A
' - --450
9
/ \ I ~ I I I : \ I Ucttrrctit,
IT
la - 258 - 0*$9 1 - 9 l A
Copper losses = field copper loss +'armature opper
I(1ss
= (400 x 0.89) + (1.91 x 0.45)
= 356 + 1.64 = 357.64W
Rotational loss of machine = input - copper l o s e s
o r P, = 1120 - 357.64 = 762.36W
On load
Input to motor = 400 x 35 = 14 OOOW
Losses on load = copper loss + rotational loss
Copper losses. PC, = field copper loss + armature
copper loss
= (400 x 0.89) + [(35 - 0 . 8 9 1 ~
x 0.451 watts
Note. Load armature current = line current - field
current
Thus."Pc, = 356 + (34.1l 2 x 0.45)
= 356 + 523 = 879W
Total losses = 879 + 762.36 = 1641.36W = 1.6414k W
Output of motor = 14 - 1.6414 = 12.358kW
output - 12.358 - 0,882
Efficiency = 7--- -in~ut
14
o r = 88.2 per cent Ans.
1

20.

Tllc ~ ~ ~ I O - I I . ~ \ I Iutiliscs
S ~ ' O ~a I cottlmon
~CI
winding whicli
serves for both primary and secondary. The well-known
rr;~nsformcrrelationships and formulae hold well equally for
such an arrangement as for the double-wound type.
Since the e.m.f. per turn is 7V, then the primary turns
would be 3700 = 528.5. ie 528 turns
7

2500
The prlmarq tapping turns would be -7

357.1.
357 turns

SOLUTIONS TO TYPICAL EXAMINATION QUESTIONS

543

The transformer would have 528 turns and the tapping


point would be 357 turns from the common end of the
winding Ans.
Also since V = 4,44@,f;V
Then 3700 = 4.440, x 50 x 528
o r 37 = 2.22 x 5280,
37

TI]11s @,, =

webcrs
2.22 x 528
Tlie flux d e n s ~ t y(maximum) is to be limited to 1.5T.
37
square metres
S O area =
2.22 x 528 x 1.5
37 x lo6
square millimetreb
2 . 2 2 ~ 0 . 5 2 81 ~. 5 lo3
~
-

37

lo' = 21.02 x 10'


1.76
= 21 020mm 2 Ans.

37.

Before adjusting the e.m.f. of one of the machines, the


e.m.f. of each machine is considered to be 230V. The busbar
voltage is 220V. Voltage drop due to resistance R of one
machine and cables up to busbars = 230 - 220 = IOV.
Current = 640A.
Kesistance R of machine and cables

10

-- ohms
640
64
Let V = the busbar voltage under the new condition.

.:

= --- =

Then current supplied by generator N o I = 235-- amperes


I
64

64(235 - 1 ' )
Current supplied by generator No 2 = (230 - V)64 ampere\
.,
,
,
., 3 = (230 -. 1364 i\mpclc\
Power supplied by the three machines
= V(235 - V)64
V(230 - V)64 + V(230 - V)64 watt$
= 64(235V - V2) + 2 x 64(230V - V2)
Now the o r ~ g l n a lpower suppl~edby three machines
= 3 x 640 x 220 watts = 422 400W o r 422.4kW
. 3 x 640 x 220 = 64(235 V - V2) + 2 x 64(230V - LT2
a n d 6 6 0 0 = 2 3 5 V - V2 + 4 6 0 V - 2V 2
o r - 3 v 2 + 695V = 6600
Thus V2 - 23 1.66V + 2200 = 0
S o l v ~ n gfor V using the q u a d r a t ~ cformula.
. 231.66 f J 2 3 1 . 6 6 ~ - 4 x 2200
C =
2
=

. .

Thus busbar voltage


- would be 221.8V Ans.
Current of Machine No I = 235 - 221.78
1
-

64

= 13.22 x 64 = 846A

Current of Machine No 2 =

Ans.

230 - 221.78

I
64
= 8.22 x 64 = 526A

Ans.

64
= 8.22 x 64 = 526A Ans.
Check. Power supplied = (846 + 526 + 526)221.78
= 1898 x 221.78 watts
= 421 kW (approx).
Using the other root of the quadratic equation we have
19.76
v = 231.66 2- 21 1.9 --7-=
9.88V
Current of Machine No 1 = (235 - 9.88) x 64
= 225.12 x 64 = 14 408A
Current of Machine No 2 = (230 - 9.88) x 64
4
= 220.12 x 64 = 14088A
Current of Machine No 3 as for N o 2.
The above conditions though theoretical, would relate to
a busbar voltage of 9.88V and could be imagined as the
result of a 'short-circuit' at the busbars, where the power of
422kW could be assumed to be dissipated.

38. Delta load


Aclive power, P, = 50kW cos d, = 0.75
Hence 4 = 4 1 V 5 ' and sin 4 = 0.6615
50 - 50 x 4
Apparent power, S , = ---0.75 -- 3
= 66.66kVA
Reactive power, Q, = 66.66 x 0.66'15
= - 44.1 kVAr (lagging)

545

SOLUTIONS TO TYPICAL EXAMINATION QUESTIONS

Star load
Active power, P2 = 40kW
cos 4 = 0.62 4 = 5 1' 41' and sin 4 = 0.7846
40 = 64.5kVA
Apparent power, S2 = 0.62
Reactive power, Q2 = 64.5 x 0.7846 = 50.6kVAr
(leading)
Total power, P = P , P2 = 50 30
= 90kW Ans.
Total reactive power, Q = Q l + Q 2 = - 44.1 + 50.6
= 6.5kVAr (leading)
Apparent power, S =

= l o w 2 = 10 x 9.023
=

Power factor

= -=

90.23kVA Ans.

90 -- 0.988 (&ding)
90.23

Ans.

39. Let the resistance of the fault be R ohms


Since the cable resistance itself is 4 ohms,
then (Resistance of end A to earth - R) + (Resistance of
end B to earth - R) = 4
or5-R+3-R=4or8-2R=4
giving 2R = 4 o r R = 252
I t can be assumed that for a cable, the resistance is proportional to length.
Then from end A to fault = 5C2 and the fault resistance is 252
:,
cable length is such as to have il resistance of 3R.
3

Thus fault must be - x 1000 = 750 metres from end A Ans.


4

I
40. (a) At resonance X, = X, o r 2nfL = 2lcfC
Thus resonant frequency f =

(27Q2CL

1 O6
1 o6
( 2 ~ 3 . 1 4 ) 1' ~4 ~ 0 . 8 4 6.28' x 1 4 ~ 0 . 8 4
1o4
- 10 000
and f 2 =
3.94 x 1.18 - 4.65
1100
~ h u sJ' = -i
= J21505
4.65
or f = 10d21.505 = 10 x 4,638 = 46.38Hz Ans.

or f 2 =

Also

27~fL= 2 x 3.14 x 46.38 x 0.84


6.28 x 46.38 x 0.84
5.28 x 46.38 = 244,890
For a current flow of 5A
Voltage drop across the inductive reactance of the coil
= 5 x 244.89 = 1224.45V
Voltage drop across resistance of coil = 5 x 50
= 250V
Voltage drop across capacitance = 1224.45V Ans
Voltage drop across coil = d 5 o 2 + 1224.52
= lo2J2.52 + 12.245'
= IO'J?ZSTT~~~
= 1 0 2 m 2= 1o3drn2
= 103 x 1,25 = 1250V A n \ .
Voltage drop across coil and capacitor = voltage across
resistance only = 5 x 50 = 250V Ans.
,
'
A

=
=
=

( b ) At 60Hz. since k, is proportional to frequency

Then X , = 244.89 x

60
2448.9 x 6
-- 46.38
46.38

Voltage drop across inductive reactance = 5 x 316.8


= 1584V
Voltage drop across resistan~e= 250V
Again X, is inversely proportional to frequency
d
1o6
103
- lo3,
-- .:
' - 2 x x 60 x 1 4 - r r x 1.4 x 1.2 5.28
= 189.30
Voltage across capacitor = 5 x 189.3 = 946.5V Ans.
Voltage across resultant reactance = 1584 - 946.5
= 637.5V
Voltage across resistance = 250V
:. Supply voltage = J637.52 + 250'
= lo2J6.3752 i2,S2
= 1O2J4O.5 + 6.25 = 1 0 2 ~ 5
= lo2 x 6.845 = 684.5V A n s .
Voltage across coil = dl 584' + 2502
= lo24 1 5 . 8 4 ' + 2.52
= I 0' J250.9 + 6.25' = I 0' J257. I 5
= lo3Jm
= 103
1.604
= 1604V Ans.

SOLLITIONS TO TYPICAL EXAMINATION QUESTIONS

547

The torque of a motor T x @I, and since, for a series


motor. 0 rc I, and I, = I, = I, :hen 7 x IL2 o r 7 = XIL2
On full load:
E h = 1'- I n R
= 2 1 0 - (100 x 0.1) = 30UV
When load torque is reduced to 25 per cent of' full-lo.~ti
value, then T I = T , x whel.e. T 2 is the nc\y torcluc a n d
7, the original torque.
T = k12
Then 2
o r l 2 = -1
7-1
k12L,
I LI
4
I
100
. I
=
or I , ? =
= -- = ~ O A
4
2
2
On new load condition
Eb2 = 210 - (50 x 0 . I ) , = 205V
205
50 x N ,
Eb, -k@2N,
As E, -/: ON then ---k
@
,
N
,
Thus
=
100
x 600
Eb,
Note that field current I, = I, = I, is now substituted for
flux, since 0 K I,.
205
Thus N - - x 600 x 2 = 205 x 6 = 1230 revimin
- 200
Speed of motor on 75 per cent load torque = 1230 rev,min
Ans.
1
o6
1 42. Reactance X, of capacitor = 2 x 3.14 x 60 x 400
2rlfC
lo3
- -1000
150.72 = 6.630
6.28 x 24
Since the load is balanced;
440 - 254.5V
The voltage across a phase, V,, = 1.732
254.5
Current in the resistor I, = 14
= 18.18A, in phase with V,,
254.5
Current in the capacitor I , = 6.63
= 38.38A, leading V , , by 90'
Let I,, = the resultant of 18.18A and 38.38A which are in
quadrature.
: I,, = j 1 8 . 1 8 ~+ 38.3X2 = 10 J1.818' + 3.8382
= 10~1'3.305 + 14.73 = 10 Jm
= 10 x 4.245 = 42-45A. This is also the line current
s ~ n c ethe load is star-connected :. I = 42.45A Ans.
i8.18
=
Power factor of load = cos $ =
I,,
42.45
= 0.43 (leading) Ans.

41.

rn

548

REED 'S BASIC ELECTROTECHNOLOGY

Power of load, P =

\/3

VI cos

= 1.732 x 440 x 42.45 x 0.43 watts


= 1,732 x 4.4 x 4,245 x 0.43 kilowatts
= 13.9kW Ans.
If the load is in delta, the current per phase would rise in the

440 o r = 1.732 x original I,,


proportion of 2545
= 1.732 x 42.45 amperes
times this new phase current.
The line current would be
.: New I = 1.732 x 1.732 x 42.45 = 3 x 42.45 = 127.35A
The power factor of the load would remain the same, so new
power, P = fl V l cos d, = dT x 440 x 127535 x 0.43 watts
= 1,732 x 4.4 x 12,735 x 0.43 kilowatts
= 1.732 x 4.4 x 3 x 4.245 x 0.43
= 3 x 13.9 = 41.7kW Ans.

78 = 123A. Length BC is (580 - 240)


43. Total load = 45
= 340m. Let the current in the remaining 320m section AC
be I amperes.
Then current in section A C = I amperes
.,
,.
,,
AB = (123 - I) amperes
.. , .
BC = (123 - I - 45) amperes
= (78 - I) amperes
0'25 x 320 =
Resistance of section AC = -

-low

32 = 0 , 0 8 0
4 x 100

By Kirchhoffs laws, the voltage drops in either section of


the main feeding the load at C, are equal.
:. I x 0.08 = (123 - 0 0 . 0 6 + (78 - 40.085
o r 81 = 6(123 - I) + 8.5(78 - I)
giving 81 = 738 - 61 + 663 - 8:5/
o r 22.51 = 738
663 = 1401
1401
Thus current I in section AC = -= 62.27A Ans.
22.5
Current in section AB = 123 - 62.27 = 60.73A Anx.
Current in section BC = 78
62.27 = l5.73A Ans.
P.D. at point C = 220 - (62.27 x 0.08) = 220 - 4.981
= 215V
, .. . . R = 220 - (60.73 x 0.06) = 220 - 3.64
= 21 6.36V
1'.1). i l l loi~clC is IowehI = 21 5 V Ans.

SOLUTIONS TO TYPICAL EXAMINATION QLIESTIONS

549

44. Let non-inductive coil of 652 be designated branch A .


110
Then I, = - = 18.33A Ans. cos 4, = 1 sin 4, = 0
6
Let inductive coil of impedance 9R be designated branch R
110
Then IB = - = 12.22A Ans.

4" = -3 =

0.33 (lagging) sin 4" = 0 4 4 3


9
Resolving in active and reactive components
I, = IA cos 4, + IB cos 4,
= 18.33 x 1 + 12.22 x 0.33 = 18.33 + 4.033
= 22.363A
and I, = - IA.sin 4, - IB sin 4,
- - 18.33 x 0 - 12.22 x 0.943 = - 0 - 1 1 .52
- - 1 l.52A
:. I = J22.362 + 1 1 3 2 2 = 10 J2.2362 ? 1.152'
= 10J5.01 + 1.32 = I
O = I 0~ x 2.52
= 25.2A
Thus current taken from mains is 25.2A Ans.

cos

With capacitor of 600pF connected in parallel :


1 1 o6
10 2
C
'
=
- 2 x 3.14 x 50 x 606 - 3 1 4 x 6

WC

110
20.75A
5.3
Resolving iis bcforc, activc colnponcnl I, rcn1;lins rllc xtnic:
but reactive component I, = 20.75 - 11.52
9.23A and I S
now vertically upwards, ie leading V by 90'.
Resultant current I = J22-36l
9;F2
o r I = 10J2.2362
0.9232
= 10J5.01 + 0.85 = l o r n 6
= 10 x 2.42 = 24.2A Ans.

Current I,

-=

400
. ~ I,. = --

= 1.33A
300
I , = 30A. . . 1, = 30 - 1.33 = 28.67A
And for the first load condition:
E, = 400 - (28.67 x 0.3) = 400 - 8,601
= 391.4V
With the data given and the O.C.C. plotted. it will be seen
that with 1.33A field current, the e.m.f. generated at 1500

45. Here Rf = 3 0 M .

FI ELD

C U RR E N T - I ~ ( A M R R E S )

rev!rnin is 648V. Therefore for the same field current and for
391.4V to be generated, since E cc A' the speed
will be
391'4 = 906 revlmin Ans.
648
For tlie second load condition I, = 50 - 1.33 = 48.67A
The E, is now = 400 - (48.67 x 0.3) = 400 - 14.6
= 385.4V
1500 x 385.4
As before. tlie new speed condition is given by 648
= 892 revlmin Ans.
The turns ratio .per .phase of tlie transformer are 560 to 42 o r
560 13.33
- . Tlit \,oltngcs pcr plinsc will bc In tlic silrnc
43
I
proportion.
( a ) With [lie transformer connected; Primary in star and
Secondary in delta; then :
Primary voltage per phase =

volts

SOLUTIONS TO TYPICAL EXAMINATION QUESTIONS

55 1

But for a delta connection, line voltage = phase voltage


or V = V,, = 286V Ans.
(b) With the. transformer connected ; Primary in delta and
Secondary in star, then:
i't'imi~ry vcllti~pepct

1711;1sc=

6600V

6600 - 286 x 1.732


13.33
= 495.4v
~ u for
t a star connection. line voltage = J3 phase
voltage or V = d ~ ,
=,
x 495.4 o r 3 x 286
= 858V Ans.

Secondary

47.

,,

"

Let V = the terminal voltage, I, the current given bq the


220V battery and I, the current given by the 225V battery.
~ 1 1 efollowing equations can then be set down.
220 - 0 . 2 4 = I'
. . (a)
. . . (b)
225 - 0.31, = I.'
. . . (c)
and lO(1, + I,) = V
Subtracting ( a ) from (b) we 1lave:225 - 220 - 0.31, + 0.21, = 0
5 + 0.2I,
Thus 5 = 0.31, - 0.21, or I, =
n.2
\I
Usintz- . ( b.) and ( c ) we can w-ite:
225 - 0.31, = 101, + 101, and substituting for I,
J

..

Thus 225 - 5

0.21,

and (220 x 0.3) - 0.061, =

o r 66

50

31,

+ 50 +0.321,
31, + 50 + 21,

101,

+ 21, + 0.061,

Hence 16 = 5,061, 1, =

16
= 3.16A Ans.
5.06

---

and V = 10(I, + I,) = lO(3.16 + 18.77)


o r V = 10 x 21.93 = 219.3V Ans.

48. Heat received by the water

mass x temperature rise x


specific heat capacltq
= 1 x (100 - 1 1 ) x 4.2 kilo~oules
= 89 x 4.2
= 373.8kJ
:. Energy put into water = 373.8kJ and assuming an
efficiency of 100 per cent for the kettle, then;
=

Energy
taken from mains = 373 8005
-.
With a d.c. supply. the current taken would be

373 800
220 x 3.5 x 60

220
2719R
8.09
On a.c., the reactance X of the kettle becomes effective.
:. X = 2nfL = 2 x 3.14 x 50 x 0.05 = 3-14 x 5 = 15.7R
Hence Z = d27.19' + 1 5 . 7 ~= 10J2.719~ + 1.572
= 0ld-6
=
= 10 x 3.14 = 31.4C2
220
T k r ~ qon an ;i.c. supply, currcnt takcn w o i ~ l dbc
3 i.4

and resistance of kettle element =

---- =

10m

The input power would be given by P = I ~ R


o r P = 7.062 x 27.1 9 = 49.84 x 27.19 = 1355W
The time taken to produce 373 8005 is given by:
The percentage time difference between heating with a.c.
1100
as against d.c. would be 276 - 210 - 66
210
210
= 31.4 per cent Ans.
*

49. ( a ) Total power dissipatioi of coils = 4 x 45 = 180W


180
9
If applied voltage = 220V, then field current = -= 220 1 1
0.r I, = 0.818A Ans.
Ic) Number of turns per field coil =

Z-E=
916.86
0.818- 1.636

Length of a turn = 450mm


.: Length of wire = 450 x 916.86 = 412 587mm
= 412.6m Ans.

p l - 2 x lo-' x 412.6 - 2 x 412.6 -x- lo-'


Then A = - R
67.22
67.22
825'2
x
= 12.3 x lo-' x lo6
67.22
square millimetres

SO1,UTIONS TO TYPICAL EXAMINATION QUESTIONS


2

o r 12.3 x

, - 12.3

Then d -

=
X

@- where d

lo-'

TI

553

diameter of wire

49.2

=TI

2- 3.95
- 15.62
giving d - --8- o r d =
10
10,
- 10 metres
or diameter of wire = 3.95 x lo-' x lo3
= 3.95 x l o - ' = 0.395mm Ans.
2

50.

The frequency of the e.m.f. produced by an alternator 1s


proportional to the number of poles and the speed at which
they are rotated. The relat~onshipcan be expressed by:
f = -P N where f = the frequency in hertz. P = the
120 number of poles iind Y = speed in revjrnin.

.: frequency of the supply

=/=

720 = 48Hz

120

Ans.
The machine speed is also referred to as the synchronous
speed, and if a supply of 48Hz was fed back into the
alternator and it was allowed to 'motor', it would rotate a t
a synchronous speed of 720 rev/min.
Synchronous motors are constructed like alternators and
the relation between speed and frequency is directly
applicable. Thus for the motor in the question
N = - 120 x f - 120 x 48
P
48
or speed of synchronous motor = 120 revlmin Ans.
An inducrion motor hccausc 01' its c o ~ i s t r ~ ~ c l i;11ic1
o~l
principle of operation, does not run at synchronous speed
but 'slips' this speed by an amount determined by the
design and loading of the machine. The amount of 'slip' is
usually expressed as a fraction o r percentage.
syn. speed - rotor speed
x 100b
Thus percentage slip =
syn. speed
x 100, where s is the slip.
o r s = Ns N,
For the problem. N, = synchronous speed = 120 revlmin

N = the speed of the motor


and s = 2 per cent
120 - N
Thus 2 =
]iO Y 100

554

REED ' S B A S I C E LE C T R O TE C H N O L OG Y

g~ving2.4 = 120

2.4 = 117.6 rev,min


Ans.
The above could be deduced more easily thus:
With 2 per cent slip, if synchronous speed was 100 revlmin,
then rotor speed would be 98 revlmin. Accordingly, if
synchronous speed was 120 rev/min, then rotor speed
would be
-

98

- N

o r A' = 120

l2? = 98 x 1:2 = 117.6 revlmin


100

Ans.

INDEX
A
Acceleration
..
..
. . 20
Acceptor. . . .
. . 376
A . C , circuit
..
..
..
192
,. generator . . . .
. . 124
Accumulators
.
..
79
Active component current
24 8
. .
. . 247
power . .
~ d d i t i o nof phasors . .
144
Admittance
..
..
331.334
Alternating e.m.f.
,
125
..
112.121
Alternator
Ammeter
..
..
..
I1
Ampere
.
2.22. 100
,
hour
..
23.8 1
.. -turn . . . .
103
Amplitude
..
..
. . 140
Angular velocity
. . 140
Anode . .
..
. . 60,351,368
,, characteristic
..
. 353
,, resistance ( a x . ) . . . . 354
,,
( d c . ) . . . . 354
Apparent power
..
. . 247
..
..
. . 160
Armature
,,
core . .
..
. . 160
,,
reaction
..
. . 174
,,
windings
..
160, 163
..
88
Artificial magnets
Atom
..
..
. . 301
..
. .63,64
Atomic weight
Average value . .
.,
. . 149
~

B
Bdck e m f
Battery
Bedrings
B-H curve
B ~ a (forward)
s
,, (reverse)
Brushes

66,224
6.7. 29
163
170
378
378
124, 162

C
Calorie . . . .
..
..
18
Capacitance
..
. 207
Capacitive reactance
194,208
Capacitor
..
. . 194.313.314
current
3 17
,,
systems
..
315
Capacity . . . .
..
.
81
Cathode .
..
60.351
,,
-ray oscilloscope
. 367
. . . . tube
367

29.72
Cell
Chdracterist~cs,d c generator 169- 189
.. motor 232-239
vacuum diode
353
sem~conductor
d~ode
7x0
Chargc
59 301
Charg~ng
83
Chem~cdleffect
23.58
equ~\alent
63.65
~ho<e
194
Circu~t( d c ) R
198
L
200
.. R ~ n 1,d
202
.. R a n d s '
Id.C.1
Cold-cathode lamp
. . 366
Commutation . .
..
128. 162
Commutator
..
162
Compensating ieads . .
. . 345
Component of a phasor
. . 147
Compound-connection. generator 184
motor
229
condenser
:'.
, .
194
. ,
5,331,334
Conductance . .
Conduction control
..
. . 374
Conductors
..
37.306
Copper loss
290
.,
voltameter
.
..
61
Cosine rule
..
. 146
Coulomb
..
..
23. 305
C'o-vi~lenthontl~np
773
Criticul ~ C I I \ ( ~ I I I C C
!?(I. 1 x 0
Cumulative connection . .
185. 236
Current . . . .
1.4
c q u a t ~ o n nio[or
,
225
Cycle
..
.
I40

..

..

Daniel1 cell
75
D.C. generator
. ,127. i67.172
,, &chine
.
..
I58
,. motor
..
..
. . 223
Delta connection
..
265,269
Depolariser
..
..
74
Derived units . .
.
..
20
Dielectric
. . 312
Differential connection . .
185. 239
Diode characteristic, vacuum . . 357
semiconductor
380
.. vacuum . . . .
35 1
.. junction
..
378
,

. . 362
Discharge lamp . . . .
Donor . .
..
..
. . 376
. . 133
Drum winding . . .
78
Drycell .
..
Dynamic cliaracter~stic. diode
353.357
(vacuum)
..
..
Dynamic characteristic d ~ o d e
. .
382
(semiconductor)
..
Dynamic electricity
..
induction
..

Eddy-current coefficient
. . 294
loss
..
290. 293
Efiective value : . . .
149
..
..
27.81.82
Efficiency
Electric charge . . . . 58.59.30 1
..
circuit.-see Circuit
flux
..
31 3
field
.
. 304
flux intensity . .
. . 313
Electrical energy
..
. .27. 52
..
22
units . . . .
Electrochemical equivalent
. . 63
Electrode
..
..
. . 59
..
..
. . 59
Electrolysis
..
..
. . 59
Electrolyte
Electrolytic cell . . . .
. . 60
Electromagnet . .
..
..
98
Electromagnetic induction
..
111
force . . . . 100
..
..
93
Electromagnetism
Electromot~veforce
..
. 2. 6
series
..
73
Electron
..
..
30 1
ern~sslon~ . .
. . 350
..
..
. . 307
Electroscope
Electrostatics . .
..
301. 305
Element . . . .
..
. . 301
E.M.F. equation. d.c. generator 168
motor . . 225
Energy . .
..
..
21.27. 49
Energy in magnetic field
. . 292
electrostatic field
. . 21 8
Equivalent-ring winding
. . 166
Extrinsic conductivity . . . . 375

....
..

..

..

. . .

F
Farad
..
..
. . 208
62
Faraday's laws of electrolysis . .
~nduction . 1 1 2
Ferromagnct~cmaterials
..
88
Field coils
..
..
. . 160
electric
..
..
31 1
magnetic . . . .
89:94. 96
..
..
. . 158
Field system
Filament . . . .
..
. . 351
Fleming's hand rules . .
122. 224
..
. . 363
Fluorescent lamp

....

....

Flux. magnetic . . . . 90.91. 101


electric
..
..
311. 313
density
..
..
91. 101
linkages . .
..
112. 196
..
369
Focussing
. . 20 i00 223
Force
.electric . .
..
. . 31 1
.line of
..
..
. . 9 0
..
magnetising
. . 103
Form factor
..
..
. . 155
Frequency
..
. .
140
Friction losses . . . .
. . 290

....
..

. .

....
...

G
Galvanometer . . . .
General series circuit . .
Generator. see d.c. generator
a.c. generator
Germanium
..
..
Grurnmc-ring . .
..
Grid
..
..
..
..
Grouping of cells

..

338. 342
. . 216
. . 127
. . 124
48.371
. . 131
. . 368
. . 2 9

H
Heat energy
..
..
. . 49
..
4 .23. 9
Heating effect . .
Henry . .
..
..
. . 195
..
..
. . 140
Hertz
High tension
..
..
. . 353
Holes
..
..
..
. 375
Horizontal component . . . . 147
Hvsteresis coefficient . .
. . 293
loop . . . .
. . 290
loss . . .
290. 293

...

I
Impedance
..
..
192
Impedances in series . .213.2i5. 331
parallel . . 249.251. 332
Impurity conductivity . .
375
1nducede.m.f. . . . .114.116. 119
Inductance
..
. . 113.193. 200
Induction
..
. . 111
Inductive impedances (in'series) 21 3
and capacitive
(in
series)
..
. . 215
..
193. 196
reactance
~ndictor
..
..
. . 193
Instantaneous value
..
126. 139
Instrumentation . . . .
. . 346

..

..

..

Joule
..
..
Junction diode . .

..

K
Kirchhoffs Laws

. . 4.324. 329

. .20.
2 1

. . 378

Lag

..

..

142

Laws of electrolys~s
,
62
electromagnetic induction I 12
. . 164
Lap winding
Lead
. .
142
-acid cell . .
..
79
. . 288
Leakage coeficienr
Lcclanche cell
78
Lrl't-hand rule
224
Ill
Lenz's law
Line of flux (magnetic) . . . . 90
. . . . . . (electr~c)
31 1
.. ..l'ol-cr s e c ; ~ h o \ c
Load c h a r a c t e r ~ s t ~ c
173. 174. 180. 183. 186
.. l ~ n e
. ,357.358. 383

...

..

M
Magnet . . . .
..
..
87
.
102. 169
Magnetic circuit
.. series . . . . 283
,. parallel
. . 284
,
effect ofcurrent
. . 93
.. field . .
..
..
89
.. strength (intensity)
102. 105
..
Hux dcnslr)
.
91. 101
..
Ily"ler.c\~~
172
.. leakage
28 X
.. mater~als . .
88
..
s a t u r a t ~ o n . 92. 169. 279

Magnet~sm
..
87
..
. 124
Magnetodynamo
Magnetomotive force . .
. . 104
Majority carriers
..
. . 376
..
..
..
. .19.20
Mass
M a x ~ m u mvalue. .
. . 139
M~~xwcll's
rule
'1 5
,
circulatin g current
theorem 326,330
..
..
27
Mechanical energy
,,
losscs
..
290
,
units
..
20
..
. . 269
Mesh connection
Midordinate rule
..
150. 15 1
Minority carriers
..
. .
376
..
. 368
Modulator
. .59,92
Molecular theory
Molecule. . . .
..
. . 301
Motor (d.c.)
..
. . 223
,, back e.m.f
..
224
,
characterlst~cs
232
,
currrnt equation
. 225
,, speed equatlon .
2 26
,, controll~ngfactors
227
,
torque equation
23 1
,
voltage equatlon
225
..
. 115
Mutual induct~on

N
Neutral . . . .
Neutron
Newton
North pole
N-type germanium
Nucleu~

..
..

267
301
20
4
375
30 1

0
Ohm
. . 2.25
Ohm s I;t\r
3
O p c r l - c ~ ~ . c~u~~h(: ~ r . : ~ c ( c r l s r172.
~ c 177

P
Parallel circuit ( d . c . )
.. (a.c.)

..
..
..

4. 5

332
31
.. -plate c a p a c ~ t o .r . . . 320
.. resonance
..
25 2
..
..
. . I55
Peak factor
.. inverse voltage . . . 329
.. value
..
..
. . 140
Percentage p o m p w n d i n g
. . 187
Periodic time
.
..
140
. . 88.98. 173
Permanent magnet
Permeability. of free space
276
relative
274'
..
absolute
28 I
P e r r n ~ t t ~ v ~oft yfree
, space
3 19
..
relatl~e
319
319
absolute
~hase"d~fTerence
.
142, 266
I39
Phasor . .
d ~ a g r a m. . . .
. . 143
P-N Junction . .
.,
. .
377
Polarisat~on . .
..
. .68,74
Poles
..
..
..
87, 158
..
. . 264
Polyphase working
Potential (pressure) difference . 2.308
PC~ICIIII~I~ICIQ~
341
Power . .
..
. .21.70
,. single-phase
..
. . 198
,. , three-phase
..
. . 270
Powcr c q u ~ ~ t ~m
o on t, o r . .
. . 230
factor
..
..
I93
-factor improvement
223.262
Primary cell
..
".
. .70,75
coil
..
..
. . 114
Proton . .
..
..
. . 301
.
. . 376
P-type germanium
. . 294
Pull (of an electromagnet)

..

connection

..

..

..

..

R
Range reslstor . .
Reactance
Rcact~vecomponent
Reactor . . .

14
193
..
..

248

. . 193

Rect~fier
Regulation
Reluctance
Residual 1n;ignetlsm
Kes~st;rnce
-.3
.. ti1c;isurcmcnI
Res~sti~~ty
Resonance
Reverse bias
Right-hand rule . .
Root mean square
..

S
Saturat~oneffect. magnetic92.169.279
.. vacuum d ~ o d e 353
Second;lr! cell . . . .
.75.79
COII . .
. . 144
Self-excitation . .
..
176, 183
inductance
195
~~iducliori
IIS
Semiconductor . .
48.37 1
Separate exc~tation . .
. . 174
S e r ~ e clrcult
s
(a.c.)
. . 3.4. 213, 331
.. (d.c.1 . . . . 3 . 4
Series-connected generator
. . 183
..
motor
229
S e r ~ e sconnection
..
. .
30
,. -parallel circuit (a.c.) . . 337
,
(d.c.)
..
7
.. connection
32
Shaft
..
..
. .
162
Shell
302,372
Shunt
..
I2
., -connected generator
. . I75
.. motor
. . 228
S~emens . . . .
..
.5,324
Sinewave (slnusoidd) . .
139. 142
S.1. system
..
..
..
19
Slope res~stance
..
. . 354
Solenoid . . . .
..
. .
96
..
. . 371
Solid-state devices
Space charge
..
..
. . 351
Specific heat capacity . .
. . 50
,.
resistance
..
38
Speed control (field) . .
. . 241
,
(voltage) . . .
242
,. controlling factor. motor
227
.. eq~13t1on
226
S I ~ I VC O I ~ I I C C I I ~ ~
..
26 5
Startcrb notor or) . . .
240
Statlc charactcr~st~cs.
dlodc
(\IIL.II~II~I)
3 5 1 , 154
S I ~ Lcliar;ictrr~hl~c$,
IC
cllotlc
(sem~conductor)
38 2
Static electricity . . .
. . 305

..

..
..

. . 314
Strength of field . . . .
Superposition of current theorem 327
. .
. . 310
Surface density
Susceptance
..
331.334
Susceplibil~tj
..
. . 98

T
Temperature coefficient
41
,,
measurement
340,343
T e r m ~ n a o.d.
l
7
Tesla
..
..
..
..
92
Therm~onicdev~ces . .
. . 350
,,
emission
. . 351
..
..
. . 48
Thermistor
Thermocouple . . . .
. . 343
..
..
. . 370
Time-base
. . 23 1
Torque equation (motor)
controlling factors
. . 231
Transient
..
, .
. . 196
l'rigononlc~rici~l
rcprescnlulion
142

..

U
Unbalanced load
U~iits
..

..

..

268
18,'20,22

Vacuum diode . . . .
Valency . . . .
..
Vertical component
..
Virtual ~ a l u e . .
..
Volt
..
..
..
Voltage . . . .
..
equation, generator
,,
motor
Voltage resonance
..
..
..
Voltaic cell
Voltameter
..
..
Voltmeter
..
..

24
2,4.24
. . 168
. . 225
. . 217
. .60,70
. .60,61
..
I1

W
Water voltameter
..
..
Watt
Wave winding . .
Waveform
..
Weber . .
..
Weston standard cell
Wheatstone bridge
Windage losses . .
Work
..
.,

91,122
352
329.338
. . ' 290
. .
21

..

z
Zener effect

..

. . 351
. .63,64
. . 147
. . 150

..

..
..
..

..
.

..
.

..

..

380

You might also like